You are on page 1of 119

[G.R. No. L-26615. April 30, 1970.

]
REV. FATHER LUCIO V. GARCIA, ANTONIO JESUS DE PRAGA, MARIA
NATIVIDAD DE JESUS AND DR. JAIME ROSARIO, Petitioners, v. HON.
CONRADO M. VASQUEZ, as Judge of the Court of First Instance of Manila,
Branch and CONSUELO GONZALES VDA. DE PRECILLA,Respondents.

SYLLABUS

1. CIVIL LAW; SUCCESSION, WILLS; PROBATE OF WILLS; GROUND FOR


DISALLOWANCE; TESTATRIXS DEFECTIVE EYESIGHT AS UNABLING HER TO READ
THE PROVISIONS OF LATER WILL. The declarations in court of the opthalmologist as
to the condition of the testatrixs eyesight fully establish the fact that her vision remained
mainly for viewing distant objects and not for reading print; that she was, at the time of the
execution of the second will on December 29, 1960, incapable of reading and could not
have read the provisions of the will supposedly signed by her.
2. ID.; ID.; ID.; ID.; ID.; IRREGULARITIES IN THE EXECUTION OF THE WILL; CASE AT
BAR. Upon its face, the testamentary provisions, the attestation clause and
acknowledgment were crammed together into a single sheet of paper, apparently to save
on space. Plainly, the testament was not prepared with any regard for the defective vision
of Da. Gliceria, the typographical errors remained uncorrected thereby indicating that the
execution thereof must have been characterized by haste. It is difficult to understand that
so important a document containing the final disposition of ones worldly possessions
should be embodied in an informal and untidy written instrument; or that the glaring
spelling errors should have escaped her notice if she had actually retained the ability to
read the purported will and had done so.
3. ID.; ID.; ID.; EXECUTION OF WILLS; REQUISITES FOR VALIDITY; ART. 808, NEW
CIVIL CODE READING OF THE WILL TWICE TO A BLIND TESTATOR; PURPOSE.
The rationale behind the requirement of reading the will to the testator if he is blind or
incapable of reading the will himself is to make the provisions thereof known to him, so that
he may be able to object if they are not in accordance with his wishes.
4. ID.; ID.; ID.; ID.; ID.; ID.; NOT COMPLIED WITH IN INSTANT CASE. Where as in the
1960 will there is nothing in the record to show that the requisites of Art. 808 of the Civil
Code of the Philippines that "if the testator is blind, the will shall be read to him twice,"
have not been complied with, the said 1960 will suffer from infirmity that affects its due
execution.
5. REMEDIAL LAW; SETTLEMENT OF ESTATE OF DECEASED PERSONS;
ADMINISTRATORS; GROUNDS FOR REMOVAL; ACQUISITION OF INTEREST
ADVERSE TO THAT OF THE ESTATE MAKES THE ADMINISTRATOR UNSUITABLE TO
DISCHARGE THE TRUST; CASE AT BAR. Considering that the alleged deed of sale
was executed when Gliceria del Rosario was already practically blind and that the
consideration given seems unconscionably small for the properties, there was likelihood
that a case for annulment might be filed against the estate or heirs of Alfonso Precilla. And
the administratrix being the widow and heir of the alleged transferee, cannot be expected
to sue herself in an action to recover property that may turn out to belong to the estate.

1 | SUCCESSION

This, plus her conduct in securing new copies of the owners duplicate of titles without the
courts knowledge and authority and having the contract bind the land through issuance of
new titles in her husbands name, cannot but expose her to the charge of unfitness or
unsuitability to discharge the trust, justifying her removal from the administration of the
estate.
6. REMEDIAL LAW; NOTICE OF LIS PENDENS; ACTION MUST AFFECT "THE TITLE
OR THE RIGHT OF POSSESSION OF REAL PROPERTY." On the matter of lis
pendens, the provisions of the Rules of Court are clear: notice of the pendency of an action
may be recorded in the office of the register of deeds of the province in which the property
is situated, if the action affects "the title or the right of possession of (such) real
property."cralaw virtua1aw library
7. ID.; ID.; ID.; NOT APPLICABLE TO INSTANT CASE. The issue in controversy here is
simply the fitness or unfitness of said special administratrix to continue holding the trust, it
does not involve or affect at all the title to, or possession of, the properties covered by TCT
Nos. 81735, 81736 and 81737. Clearly, the pendency of such case (L-26615) is not an
action that can properly be annotated in the record of the titles to the properties.

DECISION

REYES, J.B.L., J.:

G.R. No. L-27200 is an appeal from the order of the Court of First Instance of Manila (in
Sp. Proc. No. 62618) admitting to probate the alleged last will an, testament of the late
Gliceria Avelino del Rosario dated 29 December 1960. G.R. Nos. L-26615 and L-2684 are
separate petitions for mandamus filed by certain alleged heirs of said decedent seeking (1)
to compel the probate court to remove Consuelo S. Gonzales-Precilla as special
administratrix of the estate, for conflict of interest, to appoint a new one in her stead; and
(2) to order the Register of Deeds of Manila to annotate notice of lis pendens in TCT Nos.
81735, 81736 ,and 81737, registered in the name of Alfonso Precilla, married to Consuelo
Gonzales y Narciso, and said to be properly belonging to the estate of the deceased
Gliceria A. del Rosario.
Insofar as pertinent to the issues involved herein, the facts of these cases may be stated
as follows:chanrob1es virtual 1aw library
Gliceria Avelino del Rosario died unmarried in the City of Manila on 2 September 1965,
leaving no descendents, ascendants, brother or sister. At the time of her death, she was
said to be 90 years old more or less, and possessed of an estate consisting mostly of real
properties.
On 17 September 1965, Consuelo S. Gonzales Vda. de Precilla, a niece of the deceased,
petitioned the Court of First Instance of Manila for probate of the alleged last will and
testament of Gliceria A. del Rosario, executed on 29 December 1960, and for her
appointment as special administratrix of the latters estate, said to be valued at about
P100,000.00, pending the appointment of a regular administrator thereof.

The petition was opposed separately by several groups of alleged heirs: (1) Rev. Fr. Lucio
V. Garcia, a legatee named in an earlier will executed by Gliceria A. del Rosario on 9 June
1956; (2) Jaime Rosario and children, relatives and legatees in both the 1956 and 1960
wills; Antonio Jesus de Praga and Marta Natividad de Jesus, wards of the deceased and
legatees in the 1956 and 1960 wills; (3) Remedios, Encarnacion, and Eduardo, all
surnamed Narciso; (4) Natividad del Rosario-Sarmiento; (5) Maria Narciso; (6) Pascuala
Narciso de Manahan; (7) Severina, Rosa and Josefa, surnamed Narciso, and Vicente and
Delfin, surnamed Mauricio, the latter five groups of persons all claiming to be relatives
of Doa Gliceria within the fifth civil degree. The oppositions invariably charged that the
instrument executed in 1960 was not intended by the deceased to be her true will; that the
signatures of the deceased appearing in the will was procured through undue and
improper pressure and influence the part of the beneficiaries and/or other persons; that the
testatrix did not know the object of her bounty; that the instrument itself reveals
irregularities in its execution, and that the formalities required by law for such execution
have not been complied with.
Oppositor Lucio V. Garcia, who also presented for probate the 1956 will of the deceased,
joined the group of Dr. Jaime Rosario in registering opposition to the appointment of
petitioner Consuelo S. Gonzales Vda. de Precilla as special administratrix, on the ground
that the latter possesses interest adverse to the estate. After the parties were duly heard,
the probate court, in its order of 2 October 1965, granted petitioners prayer and appointed
her special administratrix of the estate upon a bond for P30,000.00. The order was
premised on the fact the petitioner was managing the properties belonging to the estate
even during the lifetime of the deceased, and to appoint another person as administrator or
co administrator at that stage of the proceeding would only result in further confusion and
difficulties.
On 30 September 1965, oppositors Jaime Rosario, Et. Al. filed with the probate court an
urgent motion to require the Hongkong & Shanghai Bank to report all withdrawals made
against the funds of the deceased after 2 September 1965. The court denied this motion
on 22 October 1965 for being premature, it being unaware that such deposit in the name of
the deceased existed. 1
On 14 December 1965, the same sets of oppositors, Dr. Jaime Rosario and children,
Antonio Jesus de Praga, Natividad de Jesus and Fr. Lucio V. Garcia, petitioned the court
for the immediate removal of the special administratrix. It was their claim that the special
administratrix and her deceased husband, Alfonso Precilla, 2 had caused Gliceria A. del
Rosario to execute a simulated and fraudulent deed of absolute sale dated 10 January
1961 allegedly conveying unto said spouses for the paltry sum of P30,000.00 ownership of
3 parcels of land and the improvements thereon located on Quiapo and San Nicolas,
Manila, with a total assessed value of P334,050.00. Oppositors contended that since it is
the duty of the administrator to protect and conserve the properties of the estate, and it
may become necessary that, an action for the annulment of the deed of sale land for
recovery of the aforementioned parcels of land be filed against the special administratrix,
as wife and heir of Alfonso Precilla, the removal of the said administratrix was imperative.
On 17 December 1965, the same oppositors prayed the court for an order directing the
Special Administratrix to deposit with the Clerk of Court all certificates of title belonging to
the estate. It was alleged that on 22 October 1965, or after her appointment, petitioner
Consuelo Gonzales Vda. de Precilla, in her capacity as special administratrix of the estate

2 | SUCCESSION

of the deceased Gliceria A. del Rosario, filed with Branch IV of the Court of First Instance
of Manila a motion for the issuance of new copies of the owners duplicates of certain
certificates of title in the name of Gliceria del Rosario, supposedly needed by her "in the
preparation of the inventory" of the properties constituting the estate. The motion having
been granted, new copies of the owners duplicates of certificates appearing the name of
Gliceria del Rosario (among which were TCT Nos. 66201, 66202 and 66204) were issued
on 15 November 1965. On 8 December 1965, according to the oppositors, the same
special administratrix presented to the Register of Deeds the deed of sale involving
properties covered by TCT Nos. 66201, 66202 and 66204 supposedly executed by Gliceria
del Rosario on 10 January 1961 in favor of Alfonso Precilla, and, in consequence, said
certificates of title were cancelled and new certificates (Nos. 81735, 81736 and 81737)
were issued in the name of Alfonso Precilla, married to Consuelo S. Gonzales y Narciso.
On 25 August 1966, the Court issued an order admitting to probate the 1960 will of Gliceria
A. del Rosario (Exhibit "D"). In declaring the due execution of the will, the probate court
took note that no evidence had been presented to establish that the testatrix was not of
sound mind when the will was executed; that the fact that she had prepared an earlier will
did not, prevent her from executing another one thereafter; that the fact that the 1956 will
consisted of 12 pages whereas the 1960 testament was contained in one page does not
render the latter invalid; that, the erasures and alterations in the instrument were
insignificant to warrant rejection; that the inconsistencies in the testimonies of the
instrumental witnesses which were noted by the oppositors are even indicative of their
truthfulness. The probate court, also considering that petitioner had already shown
capacity to administer the properties of the estate and that from the provisions of the will
she stands as the person most concerned and interested therein, appointed said petitioner
regular administratrix with a bond for P50,000.00. From this order all the oppositors
appealed, the case being docketed in this Court as G.R. No. L-27200.
Then, on 13 September 1966, the probate court resolved the oppositors motion of 14
December 1965 for the removal of the then special administratrix, as
follows:jgc:chanrobles.com.ph
"It would seem that the main purpose of the motion to remove the special administratrix
and to appoint another one in her stead, is in order that an action may be filed against the
special administratrix for the annulment of the deed of sale executed by the decedent on
January 10, 1961. Under existing documents, the properties sold pursuant to the said deed
of absolute sale no longer forms part of the estate. The alleged conflict of interest is
accordingly not between different claimants of the same estate. If it is desired by the
movants that an action be filed by them to annul the aforesaid deed absolute sale, it is not
necessary that the special administratrix be removed and that another one be appointed to
file such action. Such a course of action would only produce confusion and difficulties in
the settlement of the estate. The movants may file the aforesaid proceedings, preferably in
an independent action, to secure the nullity of the deed of absolute even without leave of
this court:"
As regard the motion of 17 December 1965 asking for the deposit in court of the titles in
the name of the decedent, the same was also denied, for the reason that if the movants
were referring to the old titles, they could no longer be produced, and if they meant the
new duplicate copies thereof that were issued at the instance of the special administratrix,
there would be no necessity therefor, because they were already cancelled and other
certificates were issued in the name of Alfonso Precilla. This order precipitated the

oppositors filing in this Court of a petition for mandamus (G.R. No. L-26615, Rev. Fr. Lucio
V. Garcia, Et. Al. v. Hon. Judge Conrado M. Vasquez, Et. Al.), which was given due course
on 6 October 1966.
On 15 December 1965, with that motion for removal pending in the court, the oppositors
requested the Register of Deeds of Manila to annotate a notice of lis pendens in the
records of TCT Nos. 81735, 81736, and 81737 in the name of Alfonso Precilla. And when
said official refused to do so, they applied to the probate court (in Sp. Proc. No. 62618) for
an order to compel the Register of Deeds to annotate a lis pendens notice in the
aforementioned titles contending that the matter of removal and appointment of the
administratrix, involving TCT Nos. 81735, 81736, and 81737, was already before the
Supreme Court. Upon denial of this motion on 12 November 1966, oppositors filed another
mandamus action, this time against the probate court and the Register of Deeds. The case
was docketed and given due course in this Court as G.R. No. L-26864.
Foremost of the questions to be determined here concerns the correctness of the order
allowing the probate of the 1960 will.
The records of the probate proceeding fully establish the fact that the testatrix, Gliceria A.
del Rosario, during her lifetime, executed two wills: one on 9 June 1956 consisting of 12
pages and written in Spanish, a language that she knew and spoke, witnessed by Messrs.
Antonio Cabrera, Jesus Y. Ayala and Valentin Marquez, and acknowledged before notary
public Jose Ayala; and another dated 29 December 1960, consisting of 1 page and written
in Tagalog, witnessed by Messrs. Vicente Rosales, Francisco Decena, and Francisco
Lopez and acknowledged before notary public Remigio M. Tividad.
Called to testify on the due execution of the 1960 will, instrumental witnesses Decena,
Lopez and Rosales uniformly declared that they were individually requested by Alfonso
Precilla (the late husband of petitioner special administratrix) to witness the execution of
the last will of Doa Gliceria A. del Rosario; that they arrived at the house of the old lady at
No. 2074 Azcarraga, Manila, one after the other, in the afternoon of 29 December 1960;
that the testatrix at the time was apparently of clear and sound mind, although she was
being aided by Precilla when she walked; 3 that the will, which was already prepared, was
first read "silently" by the testatrix herself before she signed it; 4 that he three witnesses
thereafter signed the will in the presence of the testatrix and the notary public and of one
another. There is also testimony that after the testatrix and the witnesses to the will
acknowledged the instrument to be their voluntary act and deed, the notary public asked
for their respective residence certificates which were handed to him by Alfonso Precilla,
clipped together; 5 that after comparing them with the numbers already written on the will,
the notary public filled in the blanks in the instrument with the date, 29 January 1960,
before he affixed his signature and seal thereto. 6 They also testified that on that occasion
no pressure or influence has been exerted by any person upon the testatrix to execute the
will.
Of course, the interest and active participation of Alfonso Precilla in the signing of this 1960
will are evident from the records. The will appeared to have been prepared by one who is
not conversant with the spelling of Tagalog words, and it has been shown that Alfonso
Precilla is a Cebuano who speaks Tagalog with a Visayan accent. 7 The witnesses to the
will, two of whom are fellow Visayans, 8 admitted their relationship or closeness to Precilla.
9 It was Precilla who instructed them to go to the house of Gliceria del Rosario on 29
December 1960 to witness an important document, 10 and who took their residence

3 | SUCCESSION

certificates from them a few days before the will was signed. 11 Precilla had met the notary
public and witnesses Rosales and Lopez at the door of the residence of the old woman; he
ushered them to the room at the second floor where the signing of the document took
place; 12 then he fetched witness Decena from the latters haberdashery shop a few doors
away and brought him to, the house the testatrix. 13 And when the will was actually
executed Precilla was present. 14
The oppositors-appellants in the present case, however, challenging the correctness of the
probate courts ruling, maintain that on 29 December 1960 the eyesight of Gliceria del
Rosario was so poor and defective that she could not have read the provisions of the will,
contrary to the testimonies of witnesses Decena, Lopez and Rosales.
On this point, we find the declarations in court of Dr. Jesus V. Tamesis very material and
illuminating. Said ophthalmologist, whose expertise was admitted by both parties, testified,
among other things, that when Doa Gliceria del Rosario saw him for consultation on 11
March 1960 he found her left eye to have cataract (opaque lens), 15 and that it was "above
normal in pressure", denoting a possible glaucoma, a disease that leads to blindness 16
As to the conditions of her right eye, Dr. Tamesis declared:jgc:chanrobles.com.ph
"Q But is there anything here in the entry appearing in the other documents Exhibits 3-B,
3-C and 3-D from which you could inform the court as to the condition of the vision of the
patient as to the right eve?
"A Under date of August 30, 1960, is the record of refraction. that is setting of glass by
myself which showed that the right eye with my prescription of glasses had a vision of 2
over 60 (20/60) and for the left eye with her correction 20 over 300 (20/300).
"Q In laymans language, Doctor, what is the significance of that notation that the right had
a degree of 20 over 60 (20/60)?
"A It meant that eye at least would be able to recognize objects or persons at a minimum
distance of twenty feet.
"Q But would that grade enable the patient to read print?
"A Apparently that is only a record for distance vision, for distance sight, not for
near."cralaw virtua1aw library
(pages 20-21, t.s.n., hearing of 23 March 1966)
The records also show that although Dr. Tamesis operated of the left eye of the decedent
at the Lourdes Hospital on 8 August 1960; as of 23 August 1960, inspite of the glasses her
vision was only "counting fingers," 17 at five feet. The cross-examination of the doctor
further elicited the following responses:jgc:chanrobles.com.ph
"Q After she was discharged from the hospital you prescribed lenses for her, or glasses?
"A After her discharge from the hospital, she was coming to my clinic for further
examination and then sometime later glasses were prescribed.
x

"Q And the glasses prescribed by you enabled her to read, Doctor?
"A As far as my record is concerned, with the glasses for the left eye which I prescribed
the eye which I operated she could see only forms but not read. That is on the left eye.
"Q How about the right eye?
"A The same, although the vision on the right eye is even better than the left eye." (pages
34. 85. t.s.n., hearing of 23 March 1966).
Then, confronted with a medical certificate (Exhibit H) issued by him on 29 November 1965
certifying that Gliceria del Rosario was provided with aphakic lenses and "had been under
medical supervision up to 1963 with apparently good vision", the doctor had this to
say:jgc:chanrobles.com.ph
"Q When yon said that she had apparently good vision you mean that she was able to
read?
"A No, not necessarily, only able to go around, take care of herself and see. This I can tell
you, this report was made on pure recollections and I recall she was using her glasses
although I recall also that we have to give her medicines to improve her vision, some
medicines to improve her identification some more.
x

"Q What about the vision in the right eve, was that corrected by the glasses?
"A Yes, with the new prescription which I issued on 80 August 1960. It is in the clinical
record.
"Q The vision in the right eye was corrected?
"A Yes That is the vision for distant objects."cralaw virtua1aw library
(pages 38, 39, 40. t.s.n., hearing of 23 March 1966).
The foregoing testimony of the ophthalmologist who treated the deceased and, therefore,
has first hand knowledge of the actual condition of her eyesight from August, 1960 up to
1963, fully establish the fact that notwithstanding the operation and removal of the cataract
in her left eye and her being fitted with aphakic lens (used by cataract patients), her vision
remained mainly for viewing distant objects and not for reading print. Thus, the conclusion
is inescapable that with the condition of her eyesight in August, 1960, and there is no
evidence that it had improved by 29 December 1960, Gliceria del Rosario was incapable f
reading, and could not have read the provisions of the will supposedly signed by her on 29
December 1960. It is worth noting that the instrumental witnesses stated that she read the
instrument "silently" (t.s.n., pages 164-165). which is a conclusion and not a fact.
Against the background of defective eyesight of the alleged testatrix, the appearance of the

4 | SUCCESSION

will, Exhibit "D", acquires striking significance. Upon its face, the testamentary provisions,
the attestation clause and acknowledgment were crammed together into a single sheet of
paper, to much so that the words had to be written very close on the top, bottom and two
sides of the paper, leaving no margin whatsoever; the word "and" had to be written by the
symbol" &", apparently to save on space. Plainly, the testament was not prepared with any
regard for the defective vision of Doa Gliceria. Further, typographical errors like "HULINH"
for "HULING" (last), "Alfonsa" ;or "Alfonso", "MERCRDRS" for MERCEDES",
"instrumental" for "Instrumental", and "acknowledged" for "acknowledge, remained
uncorrected, thereby indicating that execution thereof must have been characterized by
haste. It is difficult to understand that so important a document containing the final
disposition of ones worldly possessions should be embodied in an informal and untidily
written instrument; or that the glaring spelling errors should have escaped her notice if she
had actually retained the ability to read the purported will and had done so. The record is
thus convincing that the supposed testatrix could not have physically read or understood
the alleged testament, Exhibit "D", and that its admission to probate was erroneous and
should be reversed.
That Doa Gliceria should be able to greet her guests on her birthday, arrange flowers and
attend to kitchen tasks shortly prior to the alleged execution of the testament Exhibit "D",
as appears from the photographs, Exhibits "E" to "E-1", in no way proves; that she was
able to read a closely typed page, since the acts shown do not require vision at close
range. It must be remembered that with the natural lenses removed, her eyes had lost the
power of adjustment to near vision, the substituted glass lenses being rigid and
uncontrollable by her. Neither is the signing of checks (Exhibits "G" to "G-3") by her
indicative of ability to see at normal reading distances. Writing or signing of ones name,
when sufficiently practiced, becomes automatic, so that one need only to have a rough
indication of the place where the signature is to be affixed in order to be able to write it.
Indeed, a close examination of the checks, amplified in the photograph, Exhibit "O", et
seq., reinforces the contention of oppositors that the alleged testatrix could not see at
normal reading distance: the signatures in the checks are written far above the printed
base, lines, and the names of the payees as well as the amounts written do not appear to
be in the handwriting of the alleged testatrix, being in a much firmer and more fluid hand
than hers.
Thus, for all intents and purpose of the rules on probate, the deceased Gliceria del Rosario
was, as appellant oppositors contend, not unlike a blind testator, and the due execution of
her will would have required observance of the provisions of Article 808 of the Civil Code.
"ART. 808. If the testator is blind, the will shall be read to him twice; once, by one of the
subscribing witnesses, and again, by the notary public before whom the will is
acknowledged."cralaw virtua1aw library
The rationale behind the requirement of reading the will to the testator if he is blind or
incapable of reading the will himself (as when he is illiterate), 18 is to make the provisions
thereof known to him, so that he may be able to object if they are not in accordance with
his wishes. That the aim of the law is to insure that the dispositions of the will are properly
communicated to and understood by the handicapped testator, thus making them truly
reflective of his desire, is evidenced by the requirement that the will should be read to the
latter, not only once but twice, by two different persons, and that the witnesses have to act
within the range of his (the testators) other senses. 19

In connection with the will here in question, there is nothing in the records to show that the
above requisites have been complied with. Clearly, as already stated, the 1960 will sought
to be probated suffers from infirmity that affects its due execution.

the Hongkong and Shanghai Bank to report all withdrawals made against the funds of the
deceased after 2 September 1965 and (2) the motion for annotation of a lis pendens notice
on TCT Nos. 81735, 81736 and 81737, the same are to be affirmed.

We also find merit in the complaint of oppositors Lucio V. Garcia, Et Al., against the denial
by the probate court of their petition for the removal of Consuelo Gonzales Vda. de Precilla
as special administratrix of the estate of the deceased Doa Gliceria (Petition, G.R. No. L26615, Annex "B").

The probate court pointed out in its order of 22 October 1965 (Annex "H") that it could not
have taken action on the complaint against the alleged withdrawals from the bank deposits
of the deceased, because as of that time the court had not yet been apprised that such
deposits exist. Furthermore, as explained by the special administratrix in her pleading of 30
October 1965, the withdrawals referred to by the oppositors could be those covered by
checks issued in the name of Gliceria del Rosario during her lifetime but cleared only after
her death. That explanation, which not only appears plausible but has not been rebutted by
the petitioners-oppositors, negates any charge of grave abuse in connection with the
issuance of the order here in question.

The oppositors petition was based allegedly on the existence in the special administratrix
of an interest adverse to that of the estate. It was their contention that through fraud her
husband had caused the deceased Gliceria del Rosario to execute a deed of sale, dated
10 January 1961, by virtue of which the latter purportedly conveyed unto said Alfonso D.
Precilla, married to Consuelo Gonzales y Narciso, the ownership of 3 parcels of land and
the improvements thereon, assessed at P334,050.00, for the sum of P30,000.00.
In denying the petition, the probate court, in its order of 13 September 1966 (Annex "P",
Petition) reasoned out that since the properties were already sold no longer form part of
the estate. The conflict of interest would not be between the estate and third parties, but
among the different claimants of said properties, in which case, according to the court, the
participation of the special administratrix in the action for annulment that may be brought
would not be necessary.
The error in this line of reasoning lies in the fact that what was being questioned was
precisely the validity of the conveyance or sale of the properties. In short, if proper, the
action for annulment would have to be undertaken on behalf of the estate by the special
administratrix, affecting as it does the property or rights of the deceased. 20 For the rule is
that only where there is no special proceeding for the settlement of the estate of the
deceased may the legal heirs commence an action arising out of a right belonging to their
ancestor. 21
There is no doubt that to settle the question of the due execution and validity of the deed of
sale, an ordinary and separate action would have to be instituted, the matter not falling
within the competence of the probate court. 22 Considering the facts then before it, i.e., the
alleged deed of sale having been executed by Gliceria del Rosario on 10 January 1961,
when she was already practically blind; and that the consideration of P30,000.00 seems to
be unconscionably small for properties with a total assessed value of P334,050.00, there
was likelihood that a case for annulment might indeed be filed against the estate or heirs of
Alfonso Precilla. And the administratrix, being the widow and heir of the alleged transferee,
cannot be expected to sue herself in an action to recover property that may turn out to
belong to the estate. 22 Not only this, but the conduct of the special administratrix in
securing new copies of the owners duplicates of TCT Nos. 66201, 66202, and 66204,
without the courts knowledge or authority, and on the pretext that she needed them in the
preparation of the inventory of the estate, when she must have already known by then that
the properties covered therein were already "conveyed" to her husband by the deceased,
being the latters successor, and having the contract bind the land through issuance of new
titles in her husbands name cannot but expose her to the charge of unfitness or
unsuitableness to discharge the trust, justifying her removal from the administration of the
estate.
With respect to the orders of the court a quo denying (1) the oppositors motion to require

5 | SUCCESSION

On the matter of lis pendens (G.R. No. L-26864), the provisions of the Rules of Court are
clear: notice of the pendency of an action may be recorded in the office of the register of
deeds of the province in which the property is situated, if the action affects "the title or the
right of possession of (such) real property." 23 In the case at bar, the pending action which
oppositors seek to annotate in the records of TCT Nos. 81735, 81736, and 81737 is the
mandamus proceeding filed in this Court (G.R. No. L-26615). As previously discussed in
this opinion, however, that case is concerned merely with the correctness of the denial by
the probate court of the motion for the removal of Consuelo Gonzales Vda. de Precilla as
special administratrix of the estate of the late Gliceria del Rosario. In short, the issue in
controversy there is simply the fitness or unfitness of said special administratrix to continue
holding the trust; it does not involve or affect at all the title to, or possession of, the
properties covered by said TCT Nos. 81735, 81736 and 81737. Clearly, the pendency of
such case (L-26615) is not an action that can properly be annotated in the record of the
titles to the properties.
FOR THE FOREGOING REASONS, the order of the court below allowing to probate the
alleged 1960 will of Gliceria A. del Rosario is hereby reversed and set aside. The petition in
G.R. No. L-26615 being meritorious, the appealed order is set aside and the court below is
ordered to remove the administratrix, Consuelo Gonzales Vda. de Precilla, and appoint
one of the heirs intestate of the deceased Doa Gliceria Avelino del Rosario as special
administrator for the purpose of instituting action on behalf of her estate to recover the
properties allegedly sold by her to the late Alfonso D. Precilla. And in Case G.R. No. L26864, petition is dismissed. No costs.

G.R. No. 74695 September 14, 1993


.

In the Matter of the Probate of the Last Will and Testament of the Deceased
Brigido Alvarado, CESAR ALVARADO, petitioner,
vs.
HON. RAMON G. GAVIOLA, JR., Presiding Justice, HON. MA. ROSARIO
QUETULIO LOSA and HON. LEONOR INES LUCIANO, Associate Justices,
Intermediate Appellate Court, First Division (Civil Cases), and BAYANI MA.
RINO, respondents

BELLOSILLO, J.:
Before us is an appeal from the Decision dated 11 April 1986 1 of the First Civil Cases
Division of the then Intermediate Appellate Court, now Court of Appeals, which affirmed the
Order dated 27 June 1983 2 of the Regional Trial Court of Sta. Cruz, Laguna, admitting to
probate the last will and testament 3 with codicil 4 of the late Brigido Alvarado.
On 5 November 1977, the 79-year old Brigido Alvarado executed a notarial will entitled
"Huling Habilin" wherein he disinherited an illegitimate son (petitioner) and expressly
revoked a previously executed holographic will at the time awaiting probate before Branch
4 of the Regional Trial Court of sta. Cruz, Laguna.
As testified to by the three instrumental witnesses, the notary public and by private
respondent who were present at the execution, the testator did not read the final draft of
the will himself. Instead, private respondent, as the lawyer who drafted the eight-paged
document, read the same aloud in the presence of the testator, the three instrumental
witnesses and the notary public. The latter four followed the reading with their own
respective copies previously furnished them.
Meanwhile, Brigido's holographic will was subsequently admitted to probate on 9
December 1977. On the 29th day of the same month, a codicil entitled "Kasulatan ng
Pagbabago sa Ilang Pagpapasiya na Nasasaad sa Huling Habilin na may Petsa
Nobiembre 5, 1977 ni Brigido Alvarado" was executed changing some dispositions in the
notarial will to generate cash for the testator's eye operation. Brigido was then suffering
from glaucoma. But the disinheritance and revocatory clauses were unchanged. As in the
case of the notarial will, the testator did not personally read the final draft of the codicil.
Instead, it was private respondent who read it aloud in his presence and in the presence of
the three instrumental witnesses (same as those of the notarial will) and the notary public
who followed the reading using their own copies.
A petition for the probate of the notarial will and codicil was filed upon the testator's death
on 3 January 1979 by private respondent as executor with the Court of First Instance, now
Regional Trial Court, of Siniloan, Laguna. 5Petitioner, in turn, filed an Opposition on the
following grounds: that the will sought to be probated was not executed and attested as
required by law; that the testator was insane or otherwise mentally incapacitated to make a
will at the time of its execution due to senility and old age; that the will was executed under
duress, or influence of fear and threats; that it was procured by undue and improper
pressure and influence on the part of the beneficiary who stands to get the lion's share of
the testator's estate; and lastly, that the signature of the testator was procured by fraud or
trick.
When the oppositor (petitioner) failed to substantiate the grounds relied upon in the
Opposition, a Probate Order was issued on 27 June 1983 from which an appeal was made

6 | SUCCESSION

to respondent court. The main thrust of the appeal was that the deceased was blind within
the meaning of the law at the time his "Huling Habilin" and the codicil attached thereto was
executed; that since the reading required by Art. 808 of the Civil Code was admittedly not
complied with, probate of the deceased's last will and codicil should have been denied.
On 11 April 1986, the Court of Appeals rendered the decision under review with the
following findings: that Brigido Alvarado was not blind at the time his last will and codicil
were executed; that assuming his blindness, the reading requirement of Art. 808 was
substantially complied with when both documents were read aloud to the testator with each
of the three instrumental witnesses and the notary public following the reading with their
respective copies of the instruments. The appellate court then concluded that although Art.
808 was not followed to the letter, there was substantial compliance since its purpose of
making known to the testator the contents of the drafted will was served.
The issues now before us can be stated thus: Was Brigido Alvarado blind for purpose of
Art, 808 at the time his "Huling Habilin" and its codicil were executed? If so, was the
double-reading requirement of said article complied with?
Regarding the first issue, there is no dispute on the following facts: Brigido Alvarado was
not totally blind at the time the will and codicil were executed. However, his vision on both
eyes was only of "counting fingers at three (3) feet" by reason of the glaucoma which he
had been suffering from for several years and even prior to his first consultation with an
eye
specialist
on
14 December 1977.
The point of dispute is whether the foregoing circumstances would qualify Brigido as a
"blind" testator under Art. 808 which reads:
Art. 808. If the testator is blind, the will shall be read to him twice; once,
by one of the subscribing witnesses, and again, by the notary public
before whom the will is acknowledged.
Petitioner contends that although his father was not totally blind when the will and codicil
were executed, he can be so considered within the scope of the term as it is used in Art.
808. To support his stand, petitioner presented before the trial court a medical certificate
issued by Dr. Salvador R. Salceda, Director of the Institute of Opthalmology (Philippine
Eye Research Institute), 6 the contents of which were interpreted in layman's terms by Dr.
Ruperto Roasa, whose expertise was admitted by private respondent. 7 Dr. Roasa
explained that although the testator could visualize fingers at three (3) feet, he could no
longer read either printed or handwritten matters as of 14 December 1977, the day of his
first consultation. 8

On the other hand, the Court of Appeals, contrary to the medical testimony, held that the
testator could still read on the day the will and the codicil were executed but chose not to
do so because of "poor eyesight." 9 Since the testator was still capable of reading at that
time, the court a quo concluded that Art. 808 need not be complied with.

Private respondent however insists that there was substantial compliance and that the
single reading suffices for purposes of the law. On the other hand, petitioner maintains that
the only valid compliance or compliance to the letter and since it is admitted that neither
the notary public nor an instrumental witness read the contents of the will and codicil to
Brigido, probate of the latter's will and codicil should have been disallowed.

We agree with petitioner in this respect.


We sustain private respondent's stand and necessarily, the petition must be denied.
Regardless of respondent's staunch contention that the testator was still capable of
reading at the time his will and codicil were prepared, the fact remains and this was
testified to by his witnesses, that Brigido did not do so because of his
"poor," 10 "defective," 11 or "blurred" 12 vision making it necessary for private respondent to
do the actual reading for him.
The following pronouncement in Garcia vs. Vasquez 13 provides an insight into the scope of
the term "blindness" as used in Art. 808, to wit:
The rationale behind the requirement of reading the will to the
testator if he is blind or incapable of reading the will himself (as when
he is illiterate), is to make the provisions thereof known to him, so that
he may be able to object if they are not in accordance with his
wishes . . .
Clear from the foregoing is that Art. 808 applies not only to blind testators but also to those
who, for one reason or another, are "incapable of reading the(ir) will(s)." Since Brigido
Alvarado was incapable of reading the final drafts of his will and codicil on the separate
occasions of their execution due to his "poor," "defective," or "blurred" vision, there can be
no other course for us but to conclude that Brigido Alvarado comes within the scope of the
term "blind" as it is used in Art. 808. Unless the contents were read to him, he had no way
of ascertaining whether or not the lawyer who drafted the will and codicil did so confortably
with his instructions. Hence, to consider his will as validly executed and entitled to probate,
it is essential that we ascertain whether Art. 808 had been complied with.
Article 808 requires that in case of testators like Brigido Alvarado, the will shall be read
twice; once, by one of the instrumental witnesses and, again, by the notary public before
whom the will was acknowledged. The purpose is to make known to the incapacitated
testator the contents of the document before signing and to give him an opportunity to
object if anything is contrary to his instructions.
That Art. 808 was not followed strictly is beyond cavil. Instead of the notary public and an
instrumental witness, it was the lawyer (private respondent) who drafted the eight-paged
will and the five-paged codicil who read the same aloud to the testator, and read them only
once, not twice as Art. 808 requires.

7 | SUCCESSION

This Court has held in a number of occasions that substantial compliance is acceptable
where the purpose of the law has been satisfied, the reason being that the solemnities
surrounding the execution of wills are intended to protect the testator from all kinds of fraud
and trickery but are never intended to be so rigid and inflexible as to destroy the
testamentary privilege. 14
In the case at bar, private respondent read the testator's will and codicil aloud in the
presence of the testator, his three instrumental witnesses, and the notary public. Prior and
subsequent thereto, the testator affirmed, upon being asked, that the contents read
corresponded with his instructions. Only then did the signing and acknowledgement take
place. There is no evidence, and petitioner does not so allege, that the contents of the will
and codicil were not sufficiently made known and communicated to the testator. On the
contrary, with respect to the "Huling Habilin," the day of the execution was not the first time
that Brigido had affirmed the truth and authenticity of the contents of the draft. The
uncontradicted testimony of Atty. Rino is that Brigido Alvarado already acknowledged that
the will was drafted in accordance with his expressed wishes even prior to 5 November
1977 when Atty. Rino went to the testator's residence precisely for the purpose of securing
his conformity to the draft. 15
Moreover, it was not only Atty. Rino who read the documents on
5 November and 29 December 1977. The notary public and the three instrumental
witnesses likewise read the will and codicil, albeit silently. Afterwards, Atty. Nonia de la
Pena (the notary public) and Dr. Crescente O. Evidente (one of the three instrumental
witnesses and the testator's physician) asked the testator whether the contents of the
document were of his own free will. Brigido answered in the affirmative. 16 With four
persons following the reading word for word with their own copies, it can be safely
concluded that the testator was reasonably assured that what was read to him (those
which he affirmed were in accordance with his instructions), were the terms actually
appearing on the typewritten documents. This is especially true when we consider the fact
that the three instrumental witnesses were persons known to the testator, one being his
physician (Dr. Evidente) and another (Potenciano C. Ranieses) being known to him since
childhood.
The spirit behind the law was served though the letter was not. Although there should be
strict compliance with the substantial requirements of the law in order to insure the

authenticity of the will, the formal imperfections should be brushed aside when they do not
affect its purpose and which, when taken into account, may only defeat the testator's will. 17
As a final word to convince petitioner of the propriety of the trial court's Probate Order and
its affirmance by the Court of Appeals, we quote the following pronouncement in Abangan
v. Abangan, 18 to wit:
The object of the solemnities surrounding the execution of wills is to
close the door against bad faith and fraud, to avoid the substitution of
wills and testaments and to guaranty their truth and authenticity.
Therefore the laws on the subject should be interpreted in such a way
as to attain these primordial ends. But, on the other hand, also one
must not lose sight of the fact that it is not the object of the law to
restrain and curtail the exercise of the right to make a will. So when an
interpretation already given assures such ends, any other interpretation
whatsoever, that adds nothing but demands more requisites entirely
unnecessary, useless and frustrative of the testator's will, must be
disregarded (emphasis supplied).
Brigido Alvarado had expressed his last wishes in clear and unmistakable terms in his
"Huling Habilin" and the codicil attached thereto. We are unwilling to cast these aside fro
the mere reason that a legal requirement intended for his protection was not followed
strictly when such compliance had been rendered unnecessary by the fact that the
purpose of the law, i.e., to make known to the incapacitated testator the contents of the
draft of his will, had already been accomplished. To reiterate, substantial compliance
suffices where the purpose has been served.
WHEREFORE, the petition is DENIED and the assailed Decision of respondent Court of
Appeals dated 11 April 1986 is AFFIRMED. Considering the length of time that this case
has remained pending, this decision is immediately executory. Costs against petitioner.

G.R. No. L-21151

February 25, 1924

In re will of Antonio Vergel de Dios, deceased.


RAMON J. FERNANDEZ, petitioner-appellant,
HERMELO VERGEL DE DIOS and SEVERINA JAVIER, legatees-appellants,
vs.
FERNANDO VERGEL DE DIOS, ET AL., opponents-appellees.
ROMUALDEZ, J.:

8 | SUCCESSION

The question in this case is as to the validity of the document Exhibit A as a will, which was
propounded by Ramon J. Fernandez for probate, and contested by Fernando Vergel de
Dios and Francisco, Ricardo and Virgilio Rustia, the court of First Instance of Manila
having denied its probate.
The applicant takes this appeal, assigning error to the action of the lower court in holding
the attestation fatally defective and in not finding Act No. 2645 void.
The defects attributed to the will by the contestants are as follows, to wit:
(a) It was not sufficiently proven that the testator knew the contents of the will.
(b) The testator did not sign all the pages of the will.
(c) He did not request anybody to attest the document as his last will.
(d) He did not sign it in the presence of any witness.
(e) The witnesses did not sign it in the presence of the testator, or of each other,
nor with knowledge on the part of the testator that they were signing his will.
(f ) The witnesses did not sign the attestation clause before the death of the
testator.
(g) This clause was written after the execution of the dispositive part of the will
and was attached to the will after the death of the testator.
(h) The signatures of the testator on page 3 of Exhibit A are not authentic.
The evidence sufficiently shows that when Attorney Lopez Lizo read the will to the testator,
the latter's mind was perfectly sane and he understood it: that he signed all the pages of
the will proper, although he did not sign the page containing the attestation clause; that
while he did not personally call the witnesses, yet the latter were invited by Attorney Lopez
Lizo to act as such in his presence. The law does not require that the testator precisely be
the person to request the witnesses to attest his will. It was also sufficiently established in
the record, beside being stated in the attestation clause, that the testator signed the will in
the presence of the three witnesses and that the latter, in turn, signed it in the presence of
the testator and of each other, the testator knowing that the witnesses were signing his will;
that the witnesses signed the attestation clause before the death of the testator; that this
clause, with the names of the witnesses in blank, was prepared before the testator signed
the will, and that the sheet containing said clause, just as those of the will proper, was a
loose sheet, and that all the four sheets of which the will Exhibit A was actually composed

were kept together and are the very ones presented in this case; and finally, that the
signatures of the testator on page 3 of said exhibit are authentic.
It thus appearing from the record that there are no such defects as those mentioned by the
opponents, and it having been proven that the testator executed said will in a language
known by him and consciously, freely and spontaneously, it would seen unnecessary to go
further, and the matter might be brought to a close right here, by holding the will in question
valid and allowable to probate, were it not for the fact that the trial court and the opponents
questioned the sufficiency and validity of the attestation clause because the sheet on
which it is written is not numbered, and it is not stated there that the testator signed on the
margin of each sheet of the will in the presence of the three witnesses, or that the latter
signed it is the presence of the testator and of each other, and specially because said
attestation clause is not signed by the testator either at the margin or the bottom thereof.
As to the numbering of the sheet containing the attestation clause, it is true that it does not
appeal on the upper part of the sheet, but it does not appear in its text, the pertinent part of
which is copied hereinafter, with the words, having reference to the number of sheets of
the will, underscored, including the page number of the attestation:

The underscored phrase "in the same manner" cannot in the instant case mean, and it in
fact means nothing, but that the testator and the witnesses signed on the left margin of
each sheet of the will "in the same manner" in which they signed at the bottom thereof, that
is, the testator in the presence of the witnesses and the latter in the presence of the
testator and of each other. This phrase in the same manner cannot, in view of the context
of the pertinent part, refer to another thing, and was used here as a suppletory phrase to
include everything and avoid the repetition of a long and difficult one, such as what is
meant by it. The same section 618 of the Code of Civil Procedure, in order to avoid the
repetition of the same long phrase about the testator having signed in the presence of the
witnesses and the latter in the presence of each other, resorts to a similar expression in the
second paragraph and says, "as aforesaid."
Concerning the absolute absence of the signature of the testator from the sheet containing
the attestation clause, this point was already decided in the above cited case of Abangan
vs. Abangan, where this court held that:
The testator's signature is not necessary in the attestation clause because this,
as its name implies, appertains only to the witnesses and not to the testator.

* * * We certify that the foregoing document written in Spanish, a language known


by the testator Antonino Vergel de Dios, consisting of three sheet actually used,
correlatively enumerated, besides this sheet . . . .

In that case of Abangan vs. Abangan it was held that the signature of the testator is not
necessary in the attestation clause, but the theory is not announced that such a clause is
unnecessary to the validity to the will.

If, as stated in this clause, the foregoing document consists of three sheets, besides that of
the clause itself, which is in singular, it is clear that such a sheet of the attestation clause is
the fourth and that the will, including said sheet, has four sheets. This description
contained in the clause in question constitutes substantial compliance with the
requirements prescribed by the law regarding the paging. So it was held by this Court in
the case ofAbangan vs. Abangan (40 Phil., 476), where the sheet containing the
attestation, as well as the preceding one, was also not paged. Furthermore the law, as we
shall see later on, does not require that the sheet containing nothing but the attestation
clause, wholly or in part, be numbered or paged. Consequently this lack of paging on the
attestation sheet does not take anything from the validity of the will.

For this reason such doctrine does not annul the judgment in the case of Uy Coque vs.
Navas L. Sioca (43 Phil., 405), where in effect the doctrine, among others, was laid down
that the attestation clause is necessary to the validity of the will. One of the points on which
greatest stress was laid in that case Uy Coque is that the requirements of the law
regarding the number of the pages used, the signing of the will and of each of its pages by
the testator in the presence of three witnesses, and the attestation and signing of the will
and of each of its pages by the witnesses in the presence of each other cannot be
proven aliunde but by the attestation clause itself which must express the complaince of
the will with such requirements. But it was not held in that case of Uy Coque that the
signature of the testator was necessary in the attestation clause, nor was such point
discussed there, which was the point at issue in the case of Abangan vs. Abangan, supra.

Turning now to the question whether or not in this clause it is stated that the testator
signed on the margin of each sheet of the will, in the presence of the witnesses and the
latter in the presence of each other, let us see what is said in said clause on this point, and
to this end its pertinent part is hereinafter transcribed and is as follows:
* * * and he (the testator) signed at the bottom of the aforesaid will in our
presence and we at his request did the same in his presence and in that of each
other as witnesses to the will, and lastly, the testator, as well as we, as witnesses,
signed in the same manner on the left margin of each sheet. (Emphasis ours.)

9 | SUCCESSION

The appellees, however, argue that such clause in the case of Abangan vs.
Abangan begins at the bottom and on the same sheet in which the testamentary provision
terminated, that is to say, the will properly speaking. Even then if it is intended to commit
misrepresentation or fraud, which are the things that with the requirements of the law for
the making and attesting of wills it is intended to avoid, it is just the same that the clause;
as in the case ofAbangan vs. Abangan, begins at the bottom of the will properly speaking,
as, like the case before us, it is wholly contained in a separate sheet. The fact is that this

separate sheet, containing the attestation clause wholly or in part, is not signed any place
by the testator in the case of Abangan vs. Abangan, as it is not in the present case.
Section 618 of the code of Civil Procedure, as amended by Act No. 2645, contains three
paragraphs, of which the first enumerates in general terms the requirements to be met by a
will executed after said Code took effect, to wit, that the language or dialect in which it is
written be known by the testator, that it be signed by the latter or by another person in the
name of the testator by his express direction and in his presence, and that it be attested
and signed by three or more credible witnesses in the presence of the testator and of each
other.
These general rules are amplified in the next two paragraphs as to the special
requirements for the execution of the will by the testator and the signing thereof by the
witnesses, with which the second paragraph of the section deals, and as to the attestation
clause treated in the third and last paragraph of said section 618.
For this reason the second paragraph of this section 618 says:
The testator or the person requested by him to write his name and the
instrumental witnesses of the will, shall also sign, as aforesaid, each and every
page thereof, on the left margin, and said pages shall be numbered correlatively
in letters placed on the upper part of each sheet.
These are the solemnities that must surround the execution of the will properly speaking,
without any reference whatsoever to the attestation clause not treated in this second
paragraph. It is in this second paragraph which deals only with the will (without including
the attestation clause), that the signature or name of the testator and those of the
witnesses are mentioned as necessary on the left margin of each and everyone of the
sheets of the will (not of the attestation clause), as well as the paging of said sheet (of the
will, and not of the attestation clause which is not yet spoken of).
Now, are the signatures of the testator and the paging of the will also necessary in the
attestation clause? Let us see the last paragraph of this section 618 of the Code which
already deals with the requirements for the attestation clause. This last paragraph reads
thus:
The attestation shall state the number of sheets or pages used, upon which the
will is written, and the fact that the testator signed the will and every page
thereof, or caused some other person to write his name, under his express
direction, in the presence of three witnesses, and the latter witnessed and signed
the will and all pages thereof in the presence of the testator and of each other.

10 | S U C C E S S I O N

As may be seen this last paragraph refers to the contents of the text of the attestation, not
the requirements or signatures thereof outside of its text. It does not require that the
attestation be signed by the testator or that the page or sheet containing it be numbered.
From this analysis of our law now in force it appears:
First. That the will must have an attestation clause as a complement, without
which it cannot be probate and with which only not aliunde (Uy Coque vs. Navas
L. Sioca , supra) may the requirements to be stated in its text be proven. The
attestation clause must be prepared and signed, as in the instant case, on the
same occasion on which the will is prepared and signed, in such a way that the
possibility of fraud, deceit or suppression of the will or the attestation clause be
reduced to a minimum; which possibility always exists, as experience shows, in
spite of the many precautions taken by the legislator to insure the true and free
expression of one's last will.
Second. That the will is distinct and different from the attestation, although both
are necessary to the validity of the will, similar, in our opinion, to
a document which is not public so long as it is not acknowledged before a notary,
the document being a distinct and different thing from the acknowledgment, each
of which must comply with different requisites, among which is the signature of
the maker which is necessary in the document but not in the acknowledgment
and both things being necessary to the existence of the public document.
Third. That the will proper must meet the requirements enumerated in the second
paragraph of section 618 of the Code of Civil Procedure.
Fourth. That the text of the attestation clause must express compliance with the
requirements prescribed for the will.
In the case at bar the attestation clause in question states that the requirements prescribed
for the will were complied with, and this is enough for it, as such attestation clause, to be
held as meeting the requirements prescribed by the law for it.
The fact that in said clause the signature of the testator does not appear does not affect its
validity, for, as above stated, the law does not require that it be signed by the testator.
We find no merit in the assignment of error raising the question as to the validity of Act No.
2645, which is valid. For the purposes of this decision, it is not necessary to reason out this
conclusion, it being sufficient for the adjudication of this case to hold the first error
assigned by the appellants to have been demonstrated.

The foregoing conclusions lead us to hold, as we do here by hold, that the documents
Exhibit A, as the last will and testament of the deceased Antonio Vergel de Dios, meets all
the requirements prescribed by the low now in force and therefore it must be allowed to
probate as prayed for by the petitioner.
The judgment appealed from is reversed, and it is ordered that the lower court proceed
with the probate of the will Exhibit A in accordance with law, without express
pronouncement as to costs. So ordered.

G.R. No. 103554 May 28, 1993


TEODORO CANEDA, LORENZA CANEDA, TERESA CANEDA, JUAN
CABALLERO, AUREA CABALLERO, OSCAR LAROSA, HELEN CABALLERO,
SANTOS CABALLERO, PABLO CABALLERO, VICTOR RAGA, MAURICIA RAGA,
QUIRICA RAGA, RUPERTO ABAPO, represented herein by his Attorney-in-Fact,
ARMSTICIA * ABAPO VELANO, and CONSESO CANEDA, represented herein by
his
heirs, JESUS
CANEDA,
NATIVIDAD CANEDA and ARTURO
CANEDA, petitioners,
vs.
HON. COURT OF APPEALS and WILLIAM CABRERA, as Special Administrator
REGALADO, J.:
Presented for resolution by this Court in the present petition for review on certiorari is the
issue of whether or not the attestation clause contained in the last will and testament of the
late Mateo Caballero complies with the requirements of Article 805, in relation to Article
809, of the Civil Code.
The records show that on December 5, 1978, Mateo Caballero, a widower without any
children and already in the twilight years of his life, executed a last will and testament at his
residence in Talisay, Cebu before three attesting witnesses, namely, Cipriano Labuca,
Gregorio Cabando and Flaviano Toregosa. The said testator was duly assisted by his
lawyer, Atty. Emilio Lumontad, and a notary public, Atty. Filoteo Manigos, in the preparation
of that last will. 1 It was declared therein, among other things, that the testator was leaving
by way of legacies and devises his real and personal properties to Presentacion Gaviola,
Angel Abatayo, Rogelio Abatayo, Isabelito Abatayo, Benoni G. Cabrera and Marcosa
Alcantara, all of whom do not appear to be related to the testator. 2
Four months later, or on April 4, 1979, Mateo Caballero himself filed a petition docketed as
Special Proceeding No. 3899-R before Branch II of the then Court of First Instance of
Cebu seeking the probate of his last will and testament. The probate court set the petition
for hearing on August 20, 1979 but the same and subsequent scheduled hearings were

11 | S U C C E S S I O N

postponed for one reason to another. On May 29, 1980, the testator passed away before
his petition could finally be heard by the probate court. 3 On February 25, 1981, Benoni
Cabrera, on of the legatees named in the will, sough his appointment as special
administrator of the testator's estate, the estimated value of which was P24,000.00, and he
was so appointed by the probate court in its order of March 6, 1981. 4
Thereafter, herein petitioners, claiming to be nephews and nieces of the testator, instituted
a second petition, entitled "In the Matter of the Intestate Estate of Mateo Caballero" and
docketed as Special Proceeding No. 3965-R, before Branch IX of the aforesaid Court of
First Instance of Cebu. On October 18, 1982, herein petitioners had their said petition
intestate proceeding consolidated with Special Proceeding No. 3899-R in Branch II of the
Court of First Instance of Cebu and opposed thereat the probate of the Testator's will and
the appointment of a special administrator for his estate. 5
Benoni Cabrera died on February 8, 1982 hence the probate court, now known as Branch
XV of the Regional Trial Court of Cebu, appointed William Cabrera as special administrator
on June 21, 1983. Thereafter, on July 20, 1983, it issued an order for the return of the
records of Special Proceeding No. 3965-R to the archives since the testate proceeding for
the probate of the will had to be heard and resolved first. On March 26, 1984 the case was
reraffled and eventually assigned to Branch XII of the Regional Trial Court of Cebu where it
remained until the conclusion of the probate proceedings. 6
In the course of the hearing in Special Proceeding No. 3899-R, herein petitioners appeared
as oppositors and objected to the allowance of the testator's will on the ground that on the
alleged date of its execution, the testator was already in the poor state of health such that
he could not have possibly executed the same. Petitioners likewise reiterated the issue as
to the genuineness of the signature of the testator therein. 7
On the other hand, one of the attesting witnesses, Cipriano Labuca, and the notary public
Atty. Filoteo Manigos, testified that the testator executed the will in question in their
presence while he was of sound and disposing mind and that, contrary to the assertions of
the oppositors, Mateo Caballero was in good health and was not unduly influenced in any
way in the execution of his will. Labuca also testified that he and the other witnesses
attested and signed the will in the presence of the testator and of each other. The other two
attesting witnesses were not presented in the probate hearing as the had died by then. 8
On April 5, 1988, the probate court rendered a decision declaring the will in question as the
last will and testament of the late Mateo Caballero, on the ratiocination that:
. . . The self-serving testimony of the two witnesses of the oppositors
cannot overcome the positive testimonies of Atty. Filoteo Manigos and
Cipriano Labuca who clearly told the Court that indeed Mateo Caballero
executed the Last Will and Testament now marked Exhibit "C" on

December 5, 1978. Moreover, the fact that it was Mateo Caballero who
initiated the probate of his Will during his lifetime when he caused the
filing of the original petition now marked Exhibit "D" clearly underscores
the fact that this was indeed his Last Will. At the start, counsel for the
oppositors manifested that he would want the signature of Mateo
Caballero in Exhibit "C" examined by a handwriting expert of the NBI
but it would seem that despite their avowal and intention for the
examination of this signature of Mateo Caballero in Exhibit "C", nothing
came out of it because they abandoned the idea and instead presented
Aurea Caballero and Helen Caballero Campo as witnesses for the
oppositors.
All told, it is the finding of this Court that Exhibit "C" is the Last Will and
Testament of Mateo Caballero and that it was executed in accordance
with all the requisites of the law. 9
Undaunted by the said judgment of the probate court, petitioners elevated the case in the
Court of Appeals in CA-G.R. CV No. 19669. They asserted therein that the will in question
is null and void for the reason that its attestation clause is fatally defective since it fails to
specifically state that the instrumental witnesses to the will witnessed the testator signing
the will in their presence and that they also signed the will and all the pages thereof in the
presence of the testator and of one another.
On October 15, 1991, respondent court promulgated its decision 10 affirming that of the trial
court, and ruling that the attestation clause in the last will of Mateo Caballero substantially
complies with Article 805 of the Civil Code, thus:
The question therefore is whether the attestation clause in question
may be considered as having substantialy complied with the
requirements of Art. 805 of the Civil Code. What appears in the
attestation clause which the oppositors claim to be defective is "we do
certify that the testament was read by him and the attestator, Mateo
Caballero, has published unto us the foregoing will consisting of
THREE PAGES, including the acknowledgment, each page numbered
correlatively in letters of the upper part of each page, as his Last Will
and Testament, and he has signed the same and every page thereof,
on the spaces provided for his signature and on the left hand margin in
the presence of the said testator and in the presence of each and all of
us (emphasis supplied).
To our thinking, this is sufficient compliance and no evidence need be
presented to indicate the meaning that the said will was signed by the
testator and by them (the witnesses) in the presence of all of them and

12 | S U C C E S S I O N

of one another. Or as the language of the law would have it that the
testator signed the will "in the presence of the instrumental witnesses,
and that the latter witnessed and signed the will and all the pages
thereof in the presence of the testator and of one another." If not
completely or ideally perfect in accordance with the wordings of Art. 805
but (sic) the phrase as formulated is in substantial compliance with the
requirement of the law." 11
Petitioners moved for the reconsideration of the said ruling of respondent court, but the
same was denied in the latter's resolution of January 14, 1992, 12 hence this appeal now
before us. Petitioners assert that respondent court has ruled upon said issue in a manner
not in accord with the law and settled jurisprudence on the matter and are now questioning
once more, on the same ground as that raised before respondent court, the validity of the
attestation clause in the last will of Mateo Caballero.
We find the present petition to be meritorious, as we shall shortly hereafter, after some
prefatory observations which we feel should be made in aid of the rationale for our
resolution of the controversy.
1. A will has been defined as a species of conveyance whereby a person is permitted, with
the formalities prescribed by law, to control to a certain degree the disposition of his estate
after his death. 13 Under the Civil Code, there are two kinds of wills which a testator may
execute. 14 the first kind is the ordinary or attested will, the execution of which is governed
by Articles 804 to 809 of the Code. Article 805 requires that:
Art. 805. Every will, other than a holographic will, must be subscribed at
the end thereof by the testator himself or by the testator's name written
by some other person in his presence, and by his express direction,
and attested and subscribed by three or more credible witnesses in the
presence of the testator and of one another.
The testator or the person requested by him to write his name and the
instrumental witnesses of the will, shall also sign, as aforesaid, each
and every page thereof, except the last, on the left margin, and all the
pages shall be numbered correlatively in letters placed on the upper
part of each page.
The attestation should state the number of pages used upon which the
will is written, and the fact that the testator signed the will and every
page thereof, or caused some other person to write his name, under his
express direction, in the presence of the instrumental witnesses, and
that the latter witnessed and signed the will and all the pages thereof in
the presence of the testator and of one another.

If the attestation clause is in a language not known to the witness, it


shall be interpreted to them.
In addition, the ordinary will must be acknowledged before a notary public by a testator and
the attesting witness.15 hence it is likewise known as notarial will. Where the attestator is
deaf or deaf-mute, Article 807 requires that he must personally read the will, if able to do
so. Otherwise, he should designate two persons who would read the will and communicate
its contents to him in a practicable manner. On the other hand, if the testator is blind, the
will should be read to him twice; once, by anyone of the witnesses thereto, and then again,
by the notary public before whom it is acknowledged. 16
The other kind of will is the holographic will, which Article 810 defines as one that is entirely
written, dated, and signed by the testator himself. This kind of will, unlike the ordinary type,
requires no attestation by witnesses. A common requirement in both kinds of will is that
they should be in writing and must have been executed in a language or dialect known to
the testator. 17
However, in the case of an ordinary or attested will, its attestation clause need not be
written in a language or dialect known to the testator since it does not form part of the
testamentary disposition. Furthermore, the language used in the attestation clause likewise
need not even be known to the attesting witnesses. 18 The last paragraph of Article 805
merely requires that, in such a case, the attestation clause shall be interpreted to said
witnesses.
An attestation clause refers to that part of an ordinary will whereby the attesting witnesses
certify that the instrument has been executed before them and to the manner of the
execution the same. 19 It is a separate memorandum or record of the facts surrounding the
conduct of execution and once signed by the witnesses, it gives affirmation to the fact that
compliance with the essential formalities required by law has been observed. 20 It is made
for the purpose of preserving in a permanent form a record of the facts that attended the
execution of a particular will, so that in case of failure of the memory of the attesting
witnesses, or other casualty, such facts may still be proved. 21
Under the third paragraph of Article 805, such a clause, the complete lack of which would
result in the invalidity of the will, 22 should state (1) the number of the pages used upon
which the will is written; (2) that the testator signed, or expressly caused another to sign,
the will and every page thereof in the presence of the attesting witnesses; and (3) that
theattesting witnesses witnessed the signing by the testator of the will and all its
pages, and that said witnesses also signed the will and every page thereof in the presence
of the testator and of one another.
The purpose of the law in requiring the clause to state the number of pages on which the
will is written is to safeguard against possible interpolation or omission of one or some of

13 | S U C C E S S I O N

its pages and to prevent any increase or decrease in the pages; 23 whereas the
subscription of the signature of the testator and the attesting witnesses is made for the
purpose of authentication and identification, and thus indicates that the will is the very
same instrument executed by the testator and attested to by the witnesses. 24
Further, by attesting and subscribing to the will, the witnesses thereby declare the due
execution of the will as embodied in the attestation clause. 25 The attestation clause,
therefore, provide strong legal guaranties for the due execution of a will and to insure the
authenticity thereof. 26 As it appertains only to the witnesses and not to the testator, it need
be signed only by them. 27 Where it is left unsigned, it would result in the invalidation of the
will as it would be possible and easy to add the clause on a subsequent occasion in the
absence of the testator and its witnesses. 28
In its report, the Code Commission commented on the reasons of the law for requiring the
formalities to be followed in the execution of wills, in the following manner:
The underlying and fundamental objectives permeating the provisions
on the law on wills in this Project consists in the liberalization of the
manner of their execution with the end in view of giving the testator
more freedom in expressing his last wishes, but with sufficient
safeguards and restrictions to prevent the commission of fraud and the
exercise of undue and improper pressure and influence upon the
testator.
This objective is in accord with the modern tendency with respect to the
formalities in the execution of wills. . . . 29
2. An examination of the last will and testament of Mateo Caballero shows that it is
comprised of three sheets all of which have been numbered correlatively, with the left
margin of each page thereof bearing the respective signatures of the testator and the three
attesting witnesses. The part of the will containing the testamentary dispositions is
expressed in the Cebuano-Visayan dialect and is signed at the foot thereof by the testator.
The attestation clause in question, on the other hand, is recited in the English language
and is likewise signed at the end thereof by the three attesting witnesses hereto. 30 Since it
is the proverbial bone of contention, we reproduce it again for facility of reference:
We, the undersigned attesting Witnesses, whose Residences and
postal addresses appear on the Opposite of our respective names, we
do hereby certify that the Testament was read by him and the testator,
MATEO CABALLERO; has published unto us the foregoing Will
consisting of THREE PAGES, including the Acknowledgment, each
page numbered correlatively in the letters on the upper part of each
page, as his Last Will and Testament and he has the same and every

page thereof, on the spaces provided for his signature and on the left
hand margin, in the presence of the said testator and in the presence of
each and all of us.
It will be noted that Article 805 requires that the witness should both attest and subscribe to
the will in the presence of the testator and of one another. "Attestation" and "subscription"
differ in meaning. Attestation is the act of senses, while subscription is the act of the hand.
The former is mental, the latter mechanical, and to attest a will is to know that it was
published as such, and to certify the facts required to constitute an actual and legal
publication; but to subscribe a paper published as a will is only to write on the same paper
the names of the witnesses, for the sole purpose of identification. 31
In Taboada vs. Rizal, 32 we clarified that attestation consists in witnessing the testator's
execution of the will in order to see and take note mentally that those things are done
which the statute requires for the execution of a will and that the signature of the testator
exists as a fact. On the other hand, subscription is the signing of the witnesses' names
upon the same paper for the purpose of identification of such paper as the will which was
executed by the testator. As it involves a mental act, there would be no means, therefore,
of ascertaining by a physical examination of the will whether the witnesses had indeed
signed in the presence of the testator and of each other unless this is substantially
expressed in the attestation.
It is contended by petitioners that the aforequoted attestation clause, in contravention of
the express requirements of the third paragraph of Article 805 of the Civil Code for
attestation clauses, fails to specifically state the fact that the attesting witnesses the
testator sign the will and all its pages in their presence and that they, the witnesses,
likewise signed the will and every page thereof in the presence of the testator and of each
other. We agree.
What is fairly apparent upon a careful reading of the attestation clause herein assailed is
the fact that while it recites that the testator indeed signed the will and all its pages in the
presence of the three attesting witnesses and states as well the number of pages that were
used, the same does not expressly state therein the circumstance that said witnesses
subscribed their respective signatures to the will in the presence of the testator and of each
other.
The phrase "and he has signed the same and every page thereof, on the spaces provided
for his signature and on the left hand margin," obviously refers to the testator and not the
instrumental witnesses as it is immediately preceded by the words "as his Last Will and
Testament." On the other hand, although the words "in the presence of the testator and in
the presence of each and all of us" may, at first blush, appear to likewise signify and refer
to the witnesses, it must, however, be interpreted as referring only to the testator signing in
the presence of the witnesses since said phrase immediately follows the words "he has

14 | S U C C E S S I O N

signed the same and every page thereof, on the spaces provided for his signature and on
the left hand margin." What is then clearly lacking, in the final logical analysis , is the
statement that the witnesses signed the will and every page thereof in the presence of the
testator and of one another.
It is our considered view that the absence of that statement required by law is a fatal defect
or imperfection which must necessarily result in the disallowance of the will that is here
sought to be admitted to probate. Petitioners are correct in pointing out that the aforestated
defect in the attestation clause obviously cannot be characterized as merely involving the
form of the will or the language used therein which would warrant the application of the
substantial compliance rule, as contemplated in the pertinent provision thereon in the Civil
Code, to wit:
Art. 809. In the absence of bad faith, forgery, or fraud, or undue and
improper pressure and influence, defects and imperfections in
the form of attestation or in the language used therein shall not render
the will invalid if it is not proved that the will was in fact executed and
attested in substantial compliance with all the requirements of article
805" (Emphasis supplied.)
While it may be true that the attestation clause is indeed subscribed at the end thereof and
at the left margin of each page by the three attesting witnesses, it certainly cannot be
conclusively inferred therefrom that the said witness affixed their respective signatures in
the presence of the testator and of each other since, as petitioners correctly observed, the
presence of said signatures only establishes the fact that it was indeed signed, but it does
not prove that the attesting witnesses did subscribe to the will in the presence of the
testator and of each other. The execution of a will is supposed to be one act so that where
the testator and the witnesses sign on various days or occasions and in various
combinations, the will cannot be stamped with the imprimatur of effectivity. 33
We believe that the further comment of former Justice J.B.L. Reyes 34 regarding Article
809, wherein he urged caution in the application of the substantial compliance rule therein,
is correct and should be applied in the case under consideration, as well as to future cases
with similar questions:
. . . The rule must be limited to disregarding those defects that can be
supplied by an examination of the will itself: whether all the pages are
consecutively numbered; whether the signatures appear in each and
every page; whether the subscribing witnesses are three or the will was
notarized. All theses are facts that the will itself can reveal, and defects
or even omissions concerning them in the attestation clause can be
safely disregarded. But the total number of pages, and whether all
persons required to sign did so in the presence of each other must

substantially appear in the attestation clause, being the only check


against perjury in the probate proceedings. (Emphasis ours.)

respecting said formalities found in Act. No. 190 and the amendment thereto were
practically reproduced and adopted in the Civil Code.

3. We stress once more that under Article 809, the defects and imperfections must only be
with respect to the form of the attestation or the language employed therein. Such defects
or imperfections would not render a will invalid should it be proved that the will was really
executed and attested in compliance with Article 805. In this regard, however, the manner
of proving the due execution and attestation has been held to be limited to merely an
examination of the will itself without resorting to evidence aliunde, whether oral or written.

One view advance the liberal or substantial compliance rule. This was first laid down in the
case of Abangan vs. Abangan, 36 where it was held that the object of the solemnities
surrounding the execution of wills is to close the door against bad faith and fraud, to avoid
substitution of wills and testaments and to guarantee their truth and authenticity. Therefore,
the laws on this subject should be interpreted in such a way as to attain these primordial
ends. Nonetheless, it was also emphasized that one must not lose sight of the fact that it is
not the object of the law to restrain and curtail the exercise of the right to make a will,
hence when an interpretation already given assures such ends, any other interpretation
whatsoever that adds nothing but demands more requisites entirely unnecessary, useless
and frustrative of the testator's last will, must be disregarded. The subsequent cases
of Avera vs. Garcia, 37 Aldaba vs. Roque, 38 Unson vs. Abella, 39Pecson vs.
Coronel, 40 Fernandez vs. Vergel de Dios, et al., 41 and Nayve vs. Mojal, et al. 42 all adhered
to this position.

The foregoing considerations do not apply where the attestation clause totally omits the
fact that the attesting witnesses signed each and every page of the will in the presence of
the testator and of each other. 35 In such a situation, the defect is not only in the form or
language of the attestation clause but the total absence of a specific element required by
Article 805 to be specifically stated in the attestation clause of a will. That is precisely the
defect complained of in the present case since there is no plausible way by which we can
read into the questioned attestation clause statement, or an implication thereof, that the
attesting witness did actually bear witness to the signing by the testator of the will and all of
its pages and that said instrumental witnesses also signed the will and every page thereof
in the presence of the testator and of one another.
Furthermore, the rule on substantial compliance in Article 809 cannot be revoked or relied
on by respondents since it presupposes that the defects in the attestation clause can be
cured or supplied by the text of the will or a consideration of matters apparent therefrom
which would provide the data not expressed in the attestation clause or from which it may
necessarily be gleaned or clearly inferred that the acts not stated in the omitted textual
requirements were actually complied within the execution of the will. In other words,
defects must be remedied by intrinsic evidence supplied by the will itself.
In the case at bar, contrarily, proof of the acts required to have been performed by the
attesting witnesses can be supplied by only extrinsic evidence thereof, since an overall
appreciation of the contents of the will yields no basis whatsoever from with such facts may
be plausibly deduced. What private respondent insists on are the testimonies of his
witnesses alleging that they saw the compliance with such requirements by the
instrumental witnesses, oblivious of the fact that he is thereby resorting to extrinsic
evidence to prove the same and would accordingly be doing by the indirection what in law
he cannot do directly.
4. Prior to the advent of the Civil Code on August 30, 1950, there was a divergence of
views as to which manner of interpretation should be followed in resolving issues centering
on compliance with the legal formalities required in the execution of wills. The formal
requirements were at that time embodied primarily in Section 618 of Act No. 190, the Code
of Civil Procedure. Said section was later amended by Act No. 2645, but the provisions

15 | S U C C E S S I O N

The other view which advocated the rule that statutes which prescribe the formalities that
should be observed in the execution of wills are mandatory in nature and are to be strictly
construed was followed in the subsequent cases of In the Matter of the Estate of
Saguinsin, 43 In re Will of Andrada, 44 Uy Coque vs. Sioca, 45 In re Estate of
Neumark, 46 and Sano vs. Quintana. 47
Gumban vs. Gorecho, et al., 48 provided the Court with the occasion to clarify the seemingly
conflicting decisions in the aforementioned cases. In said case of Gumban, the attestation
clause had failed to state that the witnesses signed the will and each and every page
thereof on the left margin in the presence of the testator. The will in question was
disallowed, with these reasons therefor:
In support of their argument on the assignment of error abovementioned, appellants rely on a series of cases of this court beginning
with (I)n the Matter of the (E)state of Saguinsin ([1920], 41 Phil., 875),
continuing with In re Will of Andrada [1921], 42 Phil., 180), Uy Coque
vs. Navas L. Sioca [1922], 43 Phil., 405), and In re Estate of Neumark
([1923], 46 Phil., 841), and ending with Sano vs. Quintana ([1925], 48
Phil., 506). Appellee counters with the citation of a series of cases
beginning with Abangan vs. Abangan ([1919], 40 Phil., 476), continuing
through Aldaba vs. Roque ([1922], 43 Phil., 378), and Fernandez vs.
Vergel de Dios ([1924], 46 Phil., 922), and culminating in Nayve vs.
Mojal and Aguilar ([1924], 47 Phil., 152). In its last analysis, our task is
to contrast and, if possible, conciliate the last two decisions cited by
opposing counsel, namely, those of Sano vs. Quintana,supra,
and Nayve vs. Mojal and Aguilar, supra.

In the case of Sano vs. Quintana, supra, it was decided that an


attestation clause which does not recite that the witnesses signed the
will and each and every page thereof on the left margin in the presence
of the testator is defective, and such a defect annuls the will. The case
of Uy Coque vs. Sioca, supra, was cited, but the case of Nayve vs.
Mojal and Aguilar, supra, was not mentioned. In contrast, is the
decision in Nayve vs. Mojal and Aguilar, supra, wherein it was held that
the attestation clause must estate the fact that the testator and the
witnesses reciprocally saw the signing of the will, for such an act cannot
be proved by the mere exhibition of the will, if it is not stated therein. It
was also held that the fact that the testator and the witnesses signed
each and every page of the will can be proved also by the mere
examination of the signatures appearing on the document itself, and the
omission to state such evident facts does not invalidate the will.
It is a habit of courts to reaffirm or distinguish previous cases; seldom
do they admit inconsistency in doctrine. Yet here, unless aided
impossible to reconcile the Mojal and Quintana decisions. They are
fundamentally at variance. If we rely on one, we affirm. If we rely on the
other, we reverse.
In resolving this puzzling question of authority, three outstanding points
may be mentioned. In the first place, the Mojal, decision was concurred
in by only four members of the court, less than a majority, with two
strong dissenting opinions; the Quintana decision was concurred in by
seven members of the court, a clear majority, with one formal dissent.
In the second place, the Mojal decision was promulgated in December,
1924, while the Quintana decision was promulgated in December,
1925; the Quintana decision was thus subsequent in point of time. And
in the third place, the Quintana decision is believed more nearly to
conform to the applicable provisions of the law.
The right to dispose of property by will is governed entirely by statute.
The law of the case is here found in section 61 of the Code of Civil
Procedure as amended by Act No. 2645, and in section 634 of the
same Code, as unamended. It is in part provided in section 61, as
amended that "No will . . .shall be valid . . . unless . . .." It is further
provided in the same section that "The attestation shallstate the number
of sheets or pages used, upon which the will is written, and the fact that
the testator signed the will and every page thereof, or caused some
other person to write his name, under his express direction, in the
presence of three witnesses, and the latter witnessed and signed the
will and all pages thereof in the presence of the testator and of each
other." Codal section 634 provides that "The will shall be disallowed in

16 | S U C C E S S I O N

either of the following case: 1. If not executed and attested as in this Act
provided." The law not alone carefully makes use of the imperative, but
cautiously goes further and makes use of the negative, to enforce
legislative intention. It is not within the province of the courts to
disregard the legislative purpose so emphatically and clearly
expressed.
We adopt and reaffirm the decision in the case of Sano vs.
Quintana, supra, and, to the extent necessary, modify the decision in
the case of Nayve vs. Mojal and Aguilar, supra. (Emphases in the
original text).
But after the Gumban clarificatory pronouncement, there were decisions of the Court that
once more appeared to revive the seeming diversity of views that was earlier threshed out
therein. The cases of Quinto vs. Morata, 49Rodriguez vs. Alcala, 50 Enchevarria vs.
Sarmiento, 51 and Testate Estate of Toray 52 went the way of the ruling as restated
in Gumban. But De Gala vs. Gonzales, et al., 53 Rey vs. Cartagena, 54 De Ticson vs. De
Gorostiza, 55 Sebastian vs. Panganiban, 56 Rodriguez vs. Yap, 57 Grey vs. Fabia, 58 Leynez
vs.
Leynez, 59 Martir
vs.
Martir, 60 Alcala
vs.
De
Villa, 61 Sabado
vs.
62
63
64
Fernandez, Mendoza vs. Pilapil, and Lopez vs. Liboro, veered away from the strict
interpretation rule and established a trend toward an application of the liberal view.
The Code Commission, cognizant of such a conflicting welter of views and of the
undeniable inclination towards a liberal construction, recommended the codification of the
substantial compliance rule, as it believed this rule to be in accord with the modern
tendency to give a liberal approach to the interpretation of wills. Said rule thus became
what is now Article 809 of the Civil Code, with this explanation of the Code Commission:
The present law provides for only one form of executing a will, and that
is, in accordance with the formalities prescribed by Section 618 of the
Code of Civil Procedure as amended by Act No. 2645. The Supreme
Court of the Philippines had previously upheld the strict compliance
with the legal formalities and had even said that the provisions of
Section 618 of the Code of Civil Procedure, as amended regarding the
contents of the attestation clause were mandatory, and non-compliance
therewith invalidated the will (Uy Coque vs. Sioca, 43 Phil. 405). These
decisions necessarily restrained the freedom of the testator in disposing
of his property.
However, in recent years the Supreme Court changed its attitude and
has become more liberal in the interpretation of the formalities in the
execution of wills. This liberal view is enunciated in the cases
ofRodriguez vs. Yap, G.R. No. 45924, May 18, 1939; Leynez vs.

Leynez, G.R. No. 46097, October 18, 1939; Martir vs. Martir, G.R. No.
46995, June 21, 1940; and Alcala vs. Villa, G.R. No. 47351, April 18,
1941.
In the above mentioned decisions of our Supreme Court, it has
practically gone back to the original provisions of Section 618 of the
Code of Civil Procedure before its amendment by Act No. 2645 in the
year 1916. To turn this attitude into a legislative declaration and to
attain the main objective of the proposed Code in the liberalization of
the manner of executing wills, article 829 of the Project is
recommended, which reads:
"Art. 829. In the absence of bad faith, forgery, or
fraud, or undue and improper pressure and
influence, defects and imperfections in the form of
attestation or in the language used therein shall not
render the will invalid if it is proved that the will was
in fact executed and attested in substantial
compliance with all the requirements of article
829."65
The so-called liberal rule, the Court said in Gil vs. Murciano, 66 "does not offer any puzzle or
difficulty, nor does it open the door to serious consequences. The later decisions do tell us
when and where to stop; they draw the dividing line with precision. They do not allow
evidence aliunde to fill a void in any part of the document or supply missing details that
should appear in the will itself. They only permit a probe into the will, an exploration into its
confines, to ascertain its meaning or to determine the existence or absence of the requisite
formalities of law. This clear, sharp limitation eliminates uncertainty and ought to banish
any fear of dire results."
It may thus be stated that the rule, as it now stands, is that omissions which can be
supplied by an examination of the will itself, without the need of resorting to extrinsic
evidence, will not be fatal and, correspondingly, would not obstruct the allowance to
probate of the will being assailed. However, those omissions which cannot be supplied
except by evidence aliunde would result in the invalidation of the attestation clause and
ultimately, of the will itself.67
WHEREFORE, the petition is hereby GRANTED and the impugned decision of respondent
court is hereby REVERSED and SET ASIDE. The court a quo is accordingly directed to
forthwith DISMISS its Special Proceeding No. 3899-R (Petition for the Probate of the Last
Will
and
Testament
Mateo Caballero)
G.R.
No.
L-38338ofJanuary
28, 1985 and to REVIVE Special Proceeding No. 3965-R (In
the matter of the Intestate Estate of Mateo Caballero) as an active case and thereafter duly
proceed
the settlement
the estate ofESTATE
the said decedent.
IN THEwith
MATTER
OF THEofINTESTATE
OF ANDRES G. DE JESUS AND

BIBIANA ROXAS DE JESUS, SIMEON R. ROXAS & PEDRO ROXAS DE


JESUS,
petitioners,
17
|SU
CCESSION
vs.

GUTIERREZ, JR., J.:


This is a petition for certiorari to set aside the order of respondent Hon. Jose C. Colayco,
Presiding Judge Court of First Instance of Manila, Branch XXI disallowing the probate of
the holographic Will of the deceased Bibiana Roxas de Jesus.
The antecedent facts which led to the filing of this petition are undisputed.
After the death of spouses Andres G. de Jesus and Bibiana Roxas de Jesus, Special
Proceeding No. 81503 entitled "In the Matter of the Intestate Estate of Andres G. de Jesus
and Bibiana Roxas de Jesus" was filed by petitioner Simeon R. Roxas, the brother of the
deceased Bibiana Roxas de Jesus.
On March 26, 1973, petitioner Simeon R. Roxas was appointed administrator. After Letters
of Administration had been granted to the petitioner, he delivered to the lower court a
document purporting to be the holographic Will of the deceased Bibiana Roxas de Jesus.
On May 26, 1973, respondent Judge Jose Colayco set the hearing of the probate of the
holographic Win on July 21, 1973.
Petitioner Simeon R. Roxas testified that after his appointment as administrator, he found a
notebook belonging to the deceased Bibiana R. de Jesus and that on pages 21, 22, 23 and
24 thereof, a letter-win addressed to her children and entirely written and signed in the
handwriting of the deceased Bibiana R. de Jesus was found. The will is dated "FEB./61 "
and states: "This is my win which I want to be respected although it is not written by a
lawyer. ...
The testimony of Simeon R. Roxas was corroborated by the testimonies of Pedro Roxas
de Jesus and Manuel Roxas de Jesus who likewise testified that the letter dated "FEB./61
" is the holographic Will of their deceased mother, Bibiana R. de Jesus. Both recognized
the handwriting of their mother and positively Identified her signature. They further testified
that their deceased mother understood English, the language in which the holographic Will
is written, and that the date "FEB./61 " was the date when said Will was executed by their
mother.

Respondent Luz R. Henson, another compulsory heir filed an "opposition to probate"


assailing the purported holographic Will of Bibiana R. de Jesus because a it was not
executed in accordance with law, (b) it was executed through force, intimidation and/or
under duress, undue influence and improper pressure, and (c) the alleged testatrix acted
by mistake and/or did not intend, nor could have intended the said Will to be her last Will
and testament at the time of its execution.
On August 24, 1973, respondent Judge Jose C. Colayco issued an order allowing the
probate of the holographic Will which he found to have been duly executed in accordance
with law.

Respondent Luz Henson on the other hand submits that the purported holographic Will is
void for non-compliance with Article 810 of the New Civil Code in that the date must
contain the year, month, and day of its execution. The respondent contends that Article 810
of the Civil Code was patterned after Section 1277 of the California Code and Section
1588 of the Louisiana Code whose Supreme Courts had consistently ruled that the
required date includes the year, month, and day, and that if any of these is wanting, the
holographic Will is invalid. The respondent further contends that the petitioner cannot plead
liberal construction of Article 810 of the Civil Code because statutes prescribing the
formalities to be observed in the execution of holographic Wills are strictly construed.
We agree with the petitioner.

Respondent Luz Roxas de Jesus filed a motion for reconsideration alleging inter alia that
the alleged holographic Will of the deceased Bibiana R. de Jesus was not dated as
required by Article 810 of the Civil Code. She contends that the law requires that the Will
should contain the day, month and year of its execution and that this should be strictly
complied with.
On December 10, 1973, respondent Judge Colayco reconsidered his earlier order and
disallowed the probate of the holographic Will on the ground that the word "dated" has
generally been held to include the month, day, and year. The dispositive portion of the
order reads:
WHEREFORE, the document purporting to be the holographic Will of
Bibiana Roxas de Jesus, is hereby disallowed for not having been
executed as required by the law. The order of August 24, 1973 is
hereby set aside.
The only issue is whether or not the date "FEB./61 " appearing on the holographic Will of
the deceased Bibiana Roxas de Jesus is a valid compliance with the Article 810 of the Civil
Code which reads:
ART. 810. A person may execute a holographic will which must be
entirely written, dated, and signed by the hand of the testator himself. It
is subject to no other form, and may be made in or out of the
Philippines, and need not be witnessed.
The petitioners contend that while Article 685 of the Spanish Civil Code and Article 688 of
the Old Civil Code require the testator to state in his holographic Win the "year, month, and
day of its execution," the present Civil Code omitted the phrase Ao mes y dia and simply
requires that the holographic Will should be dated. The petitioners submit that the liberal
construction of the holographic Will should prevail.

18 | S U C C E S S I O N

This will not be the first time that this Court departs from a strict and literal application of
the statutory requirements regarding the due execution of Wills. We should not overlook
the liberal trend of the Civil Code in the manner of execution of Wills, the purpose of which,
in case of doubt is to prevent intestacy
The underlying and fundamental objectives permeating the provisions
of the law on wigs in this Project consists in the liberalization of the
manner of their execution with the end in view of giving the testator
more freedom in expressing his last wishes, but with sufficien
safeguards and restrictions to prevent the commission of fraud and the
exercise of undue and improper pressure and influence upon the
testator.
This objective is in accord with the modem tendency with respect to the
formalities in the execution of wills. (Report of the Code Commission, p.
103)
In Justice Capistrano's concurring opinion in Heirs of Raymundo Castro v. Bustos (27
SCRA 327) he emphasized that:
xxx xxx xxx
... The law has a tender regard for the will of the testator expressed in
his last will and testament on the ground that any disposition made by
the testator is better than that which the law can make. For this reason,
intestate succession is nothing more than a disposition based upon the
presumed will of the decedent.
Thus, the prevailing policy is to require satisfaction of the legal requirements in order to
guard against fraud and bad faith but without undue or unnecessary curtailment of
testamentary privilege Icasiano v. Icasiano, 11 SCRA 422). If a Will has been executed in

substantial compliance with the formalities of the law, and the possibility of bad faith and
fraud in the exercise thereof is obviated, said Win should be admitted to probate (Rey v.
Cartagena 56 Phil. 282). Thus,
xxx xxx xxx
... More than anything else, the facts and circumstances of record are
to be considered in the application of any given rule. If the surrounding
circumstances point to a regular execution of the wilt and the
instrument appears to have been executed substantially in accordance
with the requirements of the law, the inclination should, in the absence
of any suggestion of bad faith, forgery or fraud, lean towards its
admission to probate, although the document may suffer from some
imperfection of language, or other non-essential defect. ... (Leynez v.
Leynez 68 Phil. 745).
If the testator, in executing his Will, attempts to comply with all the requisites, although
compliance is not literal, it is sufficient if the objective or purpose sought to be
accomplished by such requisite is actually attained by the form followed by the testator.

As a general rule, the "date" in a holographic Will should include the day, month, and year
of its execution. However, when as in the case at bar, there is no appearance of fraud, bad
faith, undue influence and pressure and the authenticity of the Will is established and the
only issue is whether or not the date "FEB./61" appearing on the holographic Will is a valid
compliance with Article 810 of the Civil Code, probate of the holographic Will should be
allowed under the principle of substantial compliance.
WHEREFORE, the instant petition is GRANTED. The order appealed from is REVERSED
and SET ASIDE and the order allowing the probate of the holographic Will of the deceased
Bibiana Roxas de Jesus is reinstated.

G.R. Nos. 83843-44

April 5, 1990

IN THE MATTER OF THE PETITION TO APPROVE THE WILL OF MELECIO


LABRADOR. SAGRADO LABRADOR (Deceased), substituted by ROSITA
LABRADOR, ENRICA LABRADOR, and CRISTOBAL LABRADOR, petitionersappellants,
vs.
COURT
OF
APPEALS, 1 GAUDENCIO
LABRADOR,
and
JESUS
PARAS, J.:

The purpose of the solemnities surrounding the execution of Wills has been expounded by
this Court in Abangan v. Abanga 40 Phil. 476, where we ruled that:
The object of the solemnities surrounding the execution of wills is to
close the door against bad faith and fraud, to avoid substitution of wills
and testaments and to guaranty their truth and authenticity. ...
In particular, a complete date is required to provide against such contingencies as that of
two competing Wills executed on the same day, or of a testator becoming insane on the
day on which a Will was executed (Velasco v. Lopez, 1 Phil. 720). There is no such
contingency in this case.
We have carefully reviewed the records of this case and found no evidence of bad faith
and fraud in its execution nor was there any substitution of Wins and Testaments. There is
no question that the holographic Will of the deceased Bibiana Roxas de Jesus was entirely
written, dated, and signed by the testatrix herself and in a language known to her. There is
also no question as to its genuineness and due execution. All the children of the testatrix
agree on the genuineness of the holographic Will of their mother and that she had the
testamentary capacity at the time of the execution of said Will. The objection interposed by
the oppositor-respondent Luz Henson is that the holographic Will is fatally defective
because the date "FEB./61 " appearing on the holographic Will is not sufficient compliance
with Article 810 of the Civil Code. This objection is too technical to be entertained.

19 | S U C C E S S I O N

The sole issue in this case is whether or not the alleged holographic will of one Melecio
Labrador is dated, as provided for in Article 8102 of the New Civil Code.
The antecedent and relevant facts are as follows: On June 10, 1972, Melecio Labrador
died in the Municipality of Iba, province of Zambales, where he was residing, leaving
behind a parcel of land designated as Lot No. 1916 under Original Certificate of Title No.
P-1652, and the following heirs, namely: Sagrado, Enrica, Cristobal, Jesus, Gaudencio,
Josefina, Juliana, Hilaria and Jovita, all surnamed Labrador, and a holographic will.
On July 28, 1975, Sagrado Labrador (now deceased but substituted by his heirs), Enrica
Labrador and Cristobal Labrador, filed in the court a quo a petition for the probate docketed
as Special Proceeding No. 922-I of the alleged holographic will of the late Melecio
Labrador.
Subsequently, on September 30, 1975, Jesus Labrador (now deceased but substituted by
his heirs), and Gaudencio Labrador filed an opposition to the petition on the ground that
the will has been extinguished or revoked by implication of law, alleging therein that on
September 30, 1971, that is, before Melecio's death, for the consideration of Six Thousand
(P6,000) Pesos, testator Melecio executed a Deed of Absolute Sale, selling, transferring
and conveying in favor of oppositors Jesus and Gaudencio Lot No. 1916 and that as a
matter of fact, O.C.T. No. P-1652 had been cancelled by T.C.T. No. T-21178. Earlier
however, in 1973, Jesus Labrador sold said parcel of land to Navat for only Five Thousand
(P5,000) Pesos. (Rollo, p. 37)

Sagrado thereupon filed, on November 28, 1975, against his brothers, Gaudencio and
Jesus, for the annulment of said purported Deed of Absolute Sale over a parcel of land
which Sagrado allegedly had already acquired by devise from their father Melecio
Labrador under a holographic will executed on March 17, 1968, the complaint for
annulment docketed as Civil Case No. 934-I, being premised on the fact that the aforesaid
Deed of Absolute Sale is fictitious.
After both parties had rested and submitted their respective evidence, the trial court
rendered a joint decision dated February 28, 1985, allowing the probate of the holographic
will and declaring null and void the Deed of Absolute sale. The court a quo had also
directed the respondents (the defendants in Civil Case No. 934-I) to reimburse to the
petitioners the sum of P5,000.00 representing the redemption price for the property paid by
the plaintiff-petitioner Sagrado with legal interest thereon from December 20, 1976, when it
was paid to vendee a retro.
Respondents appealed the joint decision to the Court of Appeals, which on March 10, 1988
modified said joint decision of the court a quo by denying the allowance of the probate of
the will for being undated and reversing the order of reimbursement. Petitioners' Motion for
Reconsideration of the aforesaid decision was denied by the Court of Appeals, in the
resolution of June 13, 1988. Hence, this petition.
Petitioners now assign the following errors committed by respondent court, to wit:
I
THE COURT OF APPEALS ERRED IN NOT ALLOWING AND APPROVING THE
PROBATE OF THE HOLOGRAPHIC WILL OF THE TESTATOR MELECIO
LABRADOR; and
II
THE COURT OF APPEALS ERRED IN FINDING THAT THE ORDER OF THE
LOWER COURT DIRECTING THE REIMBURSEMENT OF THE FIVE
THOUSAND PESOS REPRESENTING THE REDEMPTION PRICE WAS
ERRONEOUS.
The alleged undated holographic will written in Ilocano translated into English, is quoted as
follows:
ENGLISH
INTERPRETATION
OF
LATE
MELECIO
LABRADOR
BY ATTY. FIDENCIO L. FERNANDEZ

THE
WILL
WRITTEN
IN

OF
THE
ILOCANO

This is also where it appears in writing of the place which is assigned and shared
or the partition in favor of SAGRADO LABRADOR which is the fishpond located
and known place as Tagale.
And this place that is given as the share to him, there is a measurement of more
or less one hectare, and the boundary at the South is the property and
assignment share of ENRICA LABRADOR, also their sister, and the boundary in
the West is the sea, known as the SEA as it is, and the boundary on the NORTH
is assignment belonging to CRISTOBAL LABRADOR, who likewise is also their
brother. That because it is now the time for me being now ninety three (93) years,
then I feel it is the right time for me to partition the fishponds which were and had
been bought or acquired by us, meaning with their two mothers, hence there
shall be no differences among themselves, those among brothers and sisters, for
it is I myself their father who am making the apportionment and delivering to each
and everyone of them the said portion and assignment so that there shall not be
any cause of troubles or differences among the brothers and sisters.
II Second Page
And this is the day in which we agreed that we are making the partitioning and
assigning the respective assignment of the said fishpond, and this being in the
month of March, 17th day, in the year 1968, and this decision and or instruction
of mine is the matter to be followed. And the one who made this writing is no
other than MELECIO LABRADOR, their father.
Now, this is the final disposition that I am making in writing and it is this that
should be followed and complied with in order that any differences or troubles
may be forestalled and nothing will happen along these troubles among my
children, and that they will be in good relations among themselves, brothers and
sisters;
And those improvements and fruits of the land; mangoes, bamboos and all
coconut trees and all others like the other kind of bamboo by name of Bayog, it is
their right to get if they so need, in order that there shall be nothing that anyone
of them shall complain against the other, and against anyone of the brothers and
sisters.
III THIRD PAGE
And that referring to the other places of property, where the said property is
located, the same being the fruits of our earnings of the two mothers of my
children, there shall be equal portion of each share among themselves, and or to
be benefitted with all those property, which property we have been able to
acquire.

I First Page
That in order that there shall be basis of the truth of this writing (WILL) which I
am here hereof manifesting of the truth and of the fruits of our labor which their
two mothers, I am signing my signature below hereof, and that this is what

20 | S U C C E S S I O N

should be complied with, by all the brothers and sisters, the children of their two
mothers JULIANA QUINTERO PILARISA and CASIANA AQUINO
VILLANUEVA Your father who made this writing (WILL), and he is, MELECIO
LABRADOR y RALUTIN (p. 46, Rollo)
The petition, which principally alleges that the holographic will is really dated, although the
date is not in its usual place, is impressed with merit.
The will has been dated in the hand of the testator himself in perfect compliance with
Article 810.1wphi1 It is worthy of note to quote the first paragraph of the second page of
the holographic will, viz:
And this is the day in which we agreed that we are making the partitioning and
assigning the respective assignment of the said fishpond, and this being in the
month of March, 17th day, in the year 1968, and this decision and or instruction
of mine is the matter to be followed. And the one who made this writing is no
other than MELECIO LABRADOR, their father. (emphasis supplied) (p. 46, Rollo)
The law does not specify a particular location where the date should be placed in the will.
The only requirements are that the date be in the will itself and executed in the hand of the
testator. These requirements are present in the subject will.
Respondents claim that the date 17 March 1968 in the will was when the testator and his
beneficiaries entered into an agreement among themselves about "the partitioning and
assigning the respective assignments of the said fishpond," and was not the date of
execution of the holographic will; hence, the will is more of an "agreement" between the
testator and the beneficiaries thereof to the prejudice of other compulsory heirs like the
respondents. This was thus a failure to comply with Article 783 which defines a will as "an
act whereby a person is permitted, with the formalities prescribed by law, to control to a
certain degree the disposition of his estate, to take effect after his death."
Respondents are in error. The intention to show 17 March 1968 as the date of the
execution of the will is plain from the tenor of the succeeding words of the paragraph. As
aptly put by petitioner, the will was not an agreement but a unilateral act of Melecio
Labrador who plainly knew that what he was executing was a will. The act of partitioning
and the declaration that such partitioning as the testator's instruction or decision to be
followed reveal that Melecio Labrador was fully aware of the nature of the estate property
to be disposed of and of the character of the testamentary act as a means to control the
disposition of his estate.
Anent the second issue of finding the reimbursement of the P5,000 representing the
redemption price as erroneous, respondent court's conclusion is incorrect. When private
respondents sold the property (fishpond) with right to repurchase to Navat for P5,000, they
were actually selling property belonging to another and which they had no authority to sell,
rendering such sale null and void. Petitioners, thus "redeemed" the property from Navat for
P5,000, to immediately regain possession of the property for its disposition in accordance
with the will. Petitioners therefore deserve to be reimbursed the P5,000.

21 | S U C C E S S I O N

PREMISES CONSIDERED, the decision of the Court of Appeals dated March 10, 1988 is
hereby REVERSED. The holographic will of Melecio Labrador is APPROVED and
ALLOWED probate. The private respondents are directed to REIMBURSE the petitioners
the sum of Five Thousand Pesos (P5,000.00).
.

DY YIENG SEANGIO, G.R. Nos. 140371-72


BARBARA D. SEANGIO
and VIRGINIA D. SEANGIO,
Petitioners, Present:
versus - SANDOVAL-GUTIERREZ,
CORONA
AZCUNA, and
HON. AMOR A. REYES, in her GARCIA, JJ.
capacity as Presiding Judge,
DECISION
AZCUNA, J.:
This is a petition for certiorari[1] with application for the issuance of a writ of preliminary
injunction and/or temporary restraining order seeking the nullification of the orders, dated
August 10, 1999 and October 14, 1999, of the Regional Trial Court of Manila, Branch 21
(the RTC), dismissing the petition for probate on the ground of preterition, in the
consolidated cases, docketed as SP. Proc. No. 98-90870 and SP. Proc. No. 99-93396, and
entitled, In the Matter of the Intestate Estate of Segundo C. Seangio v. Alfredo D. Seangio,
et al. and In the Matter of the Probate of the Will of Segundo C. Seangio v. Dy Yieng
Seangio, Barbara D. Seangio and Virginia Seangio.
The facts of the cases are as follows:
On September 21, 1988, private respondents filed a petition for the settlement of the
intestate estate of the late Segundo Seangio, docketed as Sp. Proc. No. 9890870 of the
RTC, and praying for the appointment of private respondent Elisa D. SeangioSantos as
special administrator and guardian ad litem of petitioner Dy Yieng Seangio.
Petitioners Dy Yieng, Barbara and Virginia, all surnamed Seangio, opposed the
petition. They contended that: 1) Dy Yieng is still very healthy and in full command of her
faculties; 2) the deceased Segundo executed a general power of attorney in favor of
Virginia giving her the power to manage and exercise control and supervision over his
business in the Philippines; 3) Virginia is the most competent and qualified to serve as the
administrator of the estate of Segundo because she is a certified public accountant; and, 4)
Segundo left a holographic will, dated September 20, 1995, disinheriting one of the private
respondents, Alfredo Seangio, for cause. In view of the purported holographic will,
petitioners averred that in the event the decedent is found to have left a will, the intestate
proceedings are to be automatically suspended and replaced by the proceedings for the
probate of the will.
On April 7, 1999, a petition for the probate of the holographic will of Segundo, docketed as
SP. Proc. No. 9993396, was filed by petitioners before the RTC. They likewise reiterated

that the probate proceedings should take precedence over SP. Proc. No. 9890870
because testate proceedings take precedence and enjoy priority over intestate
proceedings.[2]
The document that petitioners refer to as Segundos holographic will is quoted, as follows:
Kasulatan sa pag-aalis ng mana
Tantunin ng sinuman
Ako si Segundo Seangio Filipino may asawa naninirahan sa 465-A
Flores St., Ermita, Manila at nagtatalay ng maiwanag na pag-iisip at
disposisyon ay tahasan at hayagang inaalisan ko ng lahatat anumang
mana ang paganay kong anak na si Alfredo Seangio dahil siya ay
naging lapastangan sa akin at isan beses siya ng sasalita ng masama
harapan ko at mga kapatid niya na si Virginia Seangio labis kong
kinasama ng loob ko at sasabe rin ni Alfredo sa akin na ako nasa
ibabaw gayon gunit daratin ang araw na ako nasa ilalim siya at siya
nasa ibabaw.
Labis kong ikinasama ng loob ko ang gamit ni Alfredo ng akin pagalan
para makapagutang na kuarta siya at kanya asawa na si Merna de los
Reyes sa China Bangking Corporation na millon pesos at hindi ng
babayad at hindi ng babayad ito ay nagdulot sa aking ng malaking
kahihiya sa mga may-ari at stockholders ng China Banking.
At ikinagalit ko pa rin ang pagkuha ni Alfredo at ng kanyang asawa na
mga custome[r] ng Travel Center of the Philippines na pinagasiwaan ko
at ng anak ko si Virginia.
Dito ako nagalit din kaya gayon ayoko na bilanin si Alfredo ng anak ko
at hayanan kong inaalisan ng lahat at anoman mana na si Alfredo at si
Alfredo Seangio ay hindi ko siya anak at hindi siya makoha mana.
Nila[g]daan ko ngayon ika 20 ng Setyembre 1995 sa longsod
ng Manila sa harap ng tatlong saksi. [3]
(signed)
Seg
undo Seangio
Nilagdaan sa harap namin
(signed)
Dy Yieng Seangio (signed)
Unang Saksi ikalawang saksi
(signed)
ikatlong saksi
On May 29, 1999, upon petitioners motion, SP. Proc. No. 9890870 and SP. Proc. No.
9993396 were consolidated.[4]
On July 1, 1999, private respondents moved for the dismissal of the probate
proceedings[5] primarily on the ground that the document purporting to be the holographic

22 | S U C C E S S I O N

will of Segundo does not contain any disposition of the estate of the deceased and thus
does not meet the definition of a will under Article 783 of the Civil Code. According to
private respondents, the will only shows an alleged act of disinheritance by the decedent of
his eldest son, Alfredo, and nothing else; that all other compulsory heirs were not named
nor instituted as heir, devisee or legatee, hence, there is preterition which would result to
intestacy. Such being the case, private respondents maintained that while procedurally the
court is called upon to rule only on the extrinsic validity of the will, it is not barred from
delving into the intrinsic validity of the same, and ordering the dismissal of the petition for
probate when on the face of the will it is clear that it contains no testamentary disposition of
the property of the decedent.
Petitioners filed their opposition to the motion to dismiss contending that: 1)
generally, the authority of the probate court is limited only to a determination of the
extrinsic validity of the will; 2) private respondents question the intrinsic and not the
extrinsic validity of the will; 3) disinheritance constitutes a disposition of the estate of a
decedent; and, 4) the rule on preterition does not apply because Segundos will does not
constitute a universal heir or heirs to the exclusion of one or more compulsory heirs.[6]
On August 10, 1999, the RTC issued its assailed order, dismissing the petition for
probate proceedings:
A perusal of the document termed as will by oppositors/petitioners Dy
Yieng Seangio, et al., clearly shows that there is preterition, as the only
heirs mentioned thereat are Alfredo and Virginia. [T]he other heirs being
omitted, Article 854 of the New Civil Code thus applies. However,
insofar as the widow Dy Yieng Seangio is concerned, Article 854 does
not apply, she not being a compulsory heir in the direct line.
As such, this Court is bound to dismiss this petition, for to do otherwise
would amount to an abuse of discretion. The Supreme Court in the
case of Acain v. Intermediate Appellate Court [155 SCRA 100 (1987)]
has made its position clear: for respondents to have tolerated the
probate of the will and allowed the case to progress when, on its face,
the will appears to be intrinsically void would have been an exercise in
futility. It would have meant a waste of time, effort, expense, plus added
futility. The trial court could have denied its probate outright or could
have passed upon the intrinsic validity of the testamentary provisions
before the extrinsic validity of the will was resolved (underscoring
supplied).
WHEREFORE, premises considered, the Motion to Suspend
Proceedings is hereby DENIED for lack of merit. Special Proceedings
No. 9993396 is hereby DISMISSED without pronouncement as to
costs.
SO ORDERED.[7]
Petitioners motion for reconsideration was denied by the RTC in its order
dated October 14, 1999.
Petitioners contend that:
THE RESPONDENT JUDGE ACTED IN EXCESS OF HER
JURISDICTION OR WITH GRAVE ABUSE OF DISCRETION
AMOUNTING TO LACK OR EXCESS OF JURISDICTIONAND
DECIDED A QUESTION OF LAW NOT IN ACCORD WITH LAW AND
JURISPRUDENCE IN ISSUING THE QUESTIONED ORDERS, DATED

10 AUGUST 1999 AND 14 OCTOBER 1999 (ATTACHMENTS A AND B


HEREOF) CONSIDERING THAT:
I
THE RESPONDENT JUDGE, WITHOUT EVEN COMPLYING WITH
SECTIONS 3 AND 4 OF RULE 76 OF THE RULES OF COURT ON
THE PROPER PROCEDURE FOR SETTING THE CASE FOR INITIAL
HEARING FOR THE ESTABLISHMENT OF THE JURISDICTIONAL
FACTS, DISMISSED THE TESTATE CASE ON THE ALLEGED
GROUND THAT THE TESTATORS WILL IS VOID ALLEGEDLY
BECAUSE OF THE EXISTENCE OF PRETERITION, WHICH GOES
INTO THE INTRINSIC VALIDITY OF THE WILL, DESPITE THE FACT
THAT IT IS A SETTLED RULE THAT THE AUTHORITY OF PROBATE
COURTS IS LIMITED ONLY TO A DETERMINATION OF THE
EXTRINSIC VALIDITY OF THE WILL, I.E., THE DUE EXECUTION
THEREOF, THE TESTATORS TESTAMENTARY CAPACITY AND THE
COMPLIANCE WITH THE REQUISITES OR SOLEMNITIES
PRESCRIBED BY LAW;
II
EVEN ASSUMING ARGUENDO THAT THE RESPONDENT JUDGE
HAS THE AUTHORITY TO RULE UPON THE INTRINSIC VALIDITY
OF THE WILL OF THE TESTATOR, IT IS INDUBITABLE FROM THE
FACE OF THE TESTATORS WILL THAT NO PRETERITON EXISTS
AND THAT THE WILL IS BOTH INTRINSICALLY AND
EXTRINSICALLY VALID; AND,

Fourth, inasmuch as it clearly appears from the face of the holographic will that it is both
intrinsically and extrinsically valid, respondent judge was mandated to proceed with the
hearing of the testate case; and,
Lastly, the continuation of the proceedings in the intestate case will work injustice to
petitioners, and will render nugatory the disinheritance of Alfredo.
The purported holographic will of Segundo that was presented by petitioners was dated,
signed and written by him in his own handwriting. Except on the ground of preterition,
private respondents did not raise any issue as regards the authenticity of the document.
The document, entitled Kasulatan ng Pag-Aalis ng Mana, unmistakably showed Segundos
intention of excluding his eldest son, Alfredo, as an heir to his estate for the reasons that
he cited therein. In effect, Alfredo was disinherited by Segundo.
For disinheritance to be valid, Article 916 of the Civil Code requires that the same must be
effected through a will wherein the legal cause therefor shall be specified. With regard to
the reasons for the disinheritance that were stated by Segundo in his document, the Court
believes that the incidents, taken as a whole, can be considered a form of maltreatment of
Segundo by his son, Alfredo, and that the matter presents a sufficient cause for the
disinheritance of a child or descendant under Article 919 of the Civil Code:
Article 919. The following shall be sufficient causes for the
disinheritance of children and descendants, legitimate as well as
illegitimate:
(1)

III
RESPONDENT JUDGE WAS DUTY BOUND TO SUSPEND THE
PROCEEDINGS IN THE INTESTATE CASE CONSIDERING THAT IT
IS A SETTLED RULE THAT TESTATE PROCEEDINGS TAKE
PRECEDENCE OVER INTESTATE PROCEEDINGS.

(2)
(3)

Petitioners argue, as follows:


(4)
First, respondent judge did not comply with Sections 3 and 4 of Rule 76 of the Rules of
Court which respectively mandate the court to: a) fix the time and place for proving the will
when all concerned may appear to contest the allowance thereof, and cause notice of such
time and place to be published three weeks successively previous to the appointed time in
a newspaper of general circulation; and, b) cause the mailing of said notice to the heirs,
legatees and devisees of the testator Segundo;

(5)
(6)
(7)

Second, the holographic will does not contain any institution of an heir, but rather, as its
title clearly states, Kasulatan ng Pag-Aalis ng Mana, simply contains a disinheritance of a
compulsory heir. Thus, there is no preterition in the decedents will and the holographic will
on its face is not intrinsically void;
Third, the testator intended all his compulsory heirs, petitioners and private respondents
alike, with the sole exception of Alfredo, to inherit his estate. None of the compulsory heirs
in the direct line of Segundo were preterited in the holographic will since there was no
institution of an heir;

23 | S U C C E S S I O N

(8)

When a child or descendant has been found guilty of an


attempt against the life of the testator, his or her spouse,
descendants, or ascendants;
When a child or descendant has accused the testator of a
crime for which the law prescribes imprisonment for six years
or more, if the accusation has been found groundless;
When a child or descendant has been convicted of adultery
or concubinage with the spouse of the testator;
When a child or descendant by fraud, violence,
intimidation, or undue influence causes the testator to make a
will or to change one already made;
A refusal without justifiable cause to support the parents or
ascendant who disinherit such child or descendant;
Maltreatment of the testator by word or deed, by the child
or descendant;[8]
When a child or descendant leads a dishonorable or
disgraceful life;
Conviction of a crime which carries with it the penalty of
civil interdiction.

Now, the critical issue to be determined is whether the document executed by Segundo
can be considered as a holographic will.

A holographic will, as provided under Article 810 of the Civil Code, must be entirely written,
dated, and signed by the hand of the testator himself. It is subject to no other form, and
may be made in or out of the Philippines, and need not be witnessed.
Segundos document, although it may initially come across as a mere disinheritance
instrument, conforms to the formalities of a holographic will prescribed by law. It is written,
dated and signed by the hand of Segundo himself. An intent to dispose mortis causa[9] can
be clearly deduced from the terms of the instrument, and while it does not make an
affirmative disposition of the latters property, the disinheritance of Alfredo, nonetheless, is
an act of disposition in itself. In other words, the disinheritance results in the disposition of
the property of the testator Segundo in favor of those who would succeed in the absence
of Alfredo.[10]
Moreover, it is a fundamental principle that the intent or the will of the testator, expressed in
the form and within the limits prescribed by law, must be recognized as the supreme law in
succession. All rules of construction are designed to ascertain and give effect to that
intention. It is only when the intention of the testator is contrary to law, morals, or public
policy that it cannot be given effect.[11]
Holographic wills, therefore, being usually prepared by one who is not learned in the law,
as illustrated in the present case, should be construed more liberally than the ones drawn
by an expert, taking into account the circumstances surrounding the execution of the
instrument and the intention of the testator.[12] In this regard, the Court is convinced that the
document, even if captioned as Kasulatan ng Pag-Aalis ng Mana, was intended by
Segundo to be his last testamentary act and was executed by him in accordance with law
in the form of a holographic will. Unless the will is probated, [13] the disinheritance cannot be
given effect.[14]

24 | S U C C E S S I O N

With regard to the issue on preterition, [15] the Court believes that the compulsory
heirs in the direct line were not preterited in the will. It was, in the Courts opinion,
Segundos last expression to bequeath his estate to all his compulsory heirs, with the sole
exception of Alfredo. Also, Segundo did not institute an heir [16] to the exclusion of his other
compulsory heirs. The mere mention of the name of one of the petitioners, Virginia, in the
document did not operate to institute her as the universal heir. Her name was included
plainly as a witness to the altercation between Segundo and his son, Alfredo.
Considering that the questioned document is Segundos holographic will, and that the law
favors testacy over intestacy, the probate of the will cannot be dispensed with. Article 838
of the Civil Code provides that no will shall pass either real or personal property unless it is
proved and allowed in accordance with the Rules of Court. Thus, unless the will is
probated, the right of a person to dispose of his property may be rendered nugatory.[17]
In view of the foregoing, the trial court, therefore, should have allowed the holographic will
to be probated. It is settled that testate proceedings for the settlement of the estate of the
decedent take precedence over intestate proceedings for the same purpose. [18]
WHEREFORE, the petition is GRANTED. The Orders of the Regional Trial Court of
Manila, Branch 21, dated August 10, 1999 and October 14, 1999, are set aside.
Respondent judge is directed to reinstate and hear SP Proc. No. 99-93396 for the
allowance of the holographic will of Segundo Seangio. The intestate case or SP. Proc. No.
98-90870 is hereby suspended until the termination of the aforesaid testate proceedings.
No costs.
SO ORDERED.

[G.R. No. L-12190. August 30, 1958.]


TESTATE ESTATE OF FELICIDAD ESGUERRA ALTO-YAP deceased.
FAUSTO E. GAN, Petitioner-Appellant, v. ILDEFONSO YAP, OppositorAppellee.

On March 17, 1952, Fausto E. Gan initiated these proceedings in the Manila court of first
instance with a petition for the probate of a holographic will allegedly executed by the
deceased, substantially in these words:jgc:chanrobles.com.ph
"Nobyembre 5, 1951
Ako, si Felicidad E. Alto-Yap, may asawa, at ganap na pagiisip, ay nagsasalaysay na ang
aking kayamanan sa bayan ng Pulilan, Bulacan ay aking ipinamamana sa aking mga
kamaganakang sumusunod:chanrob1es virtual 1aw library
Vicente Esguerra, Sr. 5 Bahagi
Fausto E. Gan 2 Bahagi
Rosario E. Gan 2 Bahagi
Filomena Alto 1 Bahagi
Beatriz Alto 1 Bahagi
At ang aking lahat ng ibang kayamanan sa Maynila at iba pang lugar ay aking
ipinamamana sa aking asawang si Ildefonso D. Yap sa kondisyong siyay magpapagawa
ng isang Health Center na nagkakahalaga ng di kukulangin sa halagang P60,000.00 sa
bayan ng Pulilan, Bulacan, na nakaukit ang aking pagalang Felicidad Esguerra-Alto. At
kung ito ay may kakulagan man ay bahala na ang aking asawa ang magpuno upang
matupad ang aking kagustuhan.
(Lagda) Felicidad E. Alto-Yap"
Opposing the petition, her surviving husband Ildefonso Yap asserted that the deceased
had not left any will, nor executed any testament during her lifetime.

SYLLABUS

1. HOLOGRAPHIC WILLS; PROBATE OF; EXECUTION AND CONTENTS OF WILL,


HOW PROVED. The execution and the contents of a lost or destroyed holographic will
may not be proved by the bare testimony of witnesses who have seen and/or read such
will. The will itself must be presented; otherwise, it shall produce no effect. The law regards
the document itself as material proof of authenticity.

DECISION

BENGZON, J.:

On November 20, 1951, Felicidad Esguerra Alto Yap died of heart failure in the University
of Santo Tomas Hospital, leaving properties in Pulilan, Bulacan, and in the City of Manila.

25 | S U C C E S S I O N

After hearing the parties and considering their evidence, the Hon. Ramon R. San Jose,
Judge, 1 refused to probate the alleged will. A seventy-page motion for reconsideration
failed. Hence this appeal.
The will itself was not presented. Petitioner tried to establish its contents and due
execution by the statements in open court of Felina Esguerra, Primitivo Reyes, Socorro
Olarte and Rosario Gan Jimenez, whose testimonies may be summarized as
follows:chanrob1es virtual 1aw library
Sometime in 1950 after her last trip abroad, Felicidad Esguerra mentioned to her first
cousin, Vicente Esguerra, her desire to make a will. She confided however that it would be
useless if her husband discovered or knew about it. Vicente consulted with Fausto E. Gan,
nephew of Felicidad, who was then preparing for the bar examinations. The latter replied it
could be done without any witness, provided the document was entirely in her handwriting,
signed and dated by her. Vicente Esguerra lost no time in transmitting the information, and
on the strength of it, in the morning of November 5, 1951, in her residence at Juan Luna
Street, Manila, Felicidad wrote, signed and dated a holographic will substantially of the
tenor above transcribed, in the presence of her niece, Felina Esguerra (daughter of
Vicente), who was invited to read it. In the afternoon of that day, Felicidad was visited by a

distant relative, Primitivo Reyes, and she allowed him to read the will in the presence of
Felina Esguerra, who again read it.
Nine days later, he had other visitors: Socorro Olarte a cousin, and Rosario Gan Jimenez,
a niece. To these she showed the will, again in the presence of Felina Esguerra, who read
it for the third time.
When on November 19, 1951, Felicidad was confined at the U.S.T. Hospital for her last
illness, she entrusted the said will, which was contained in a purse, to Felina Esguerra. But
a few hours later, Ildefonso Yap, her husband, asked Felina for the purse; and being afraid
of him by reason of his well-known violent temper, she- delivered it to him. Thereafter, in
the same day, Ildefonso Yap returned the purse to Felina, only to demand it the next day
shortly before the death of Felicidad. Again, Felina handed it to him but not before she had
taken the purse to the toilet, opened it and read the will for the last time. 2
From the oppositors proof it appears that Felicidad Esguerra had been suffering from
heart disease for several years before her death; that she had been treated by prominent
physicians, Dr. Agerico Sison, Dr. Agustin Liboro and others; that in May 1950 husband
and wife journeyed to the United States wherein for several weeks she was treated for the
disease; that thereafter she felt well and after visiting interesting places, the couple
returned to this country in August 1950. However, her ailment recurred, she suffered
several attacks, the most serious of which happened in the early morning of the first
Monday of November 1951 (Nov. 5). The whole household was surprised and alarmed,
even the teachers of the Harvardian Colleges occupying the lower floors and owned by the
Yap spouses. Physicians help was hurriedly called, and Dr. Tanjuaquio arrived at about
8:00 a.m., found the patient hardly breathing, lying in bed, her head held high by her
husband. Injections and oxygen were administered. Following the doctors advice the
patient stayed in bed, and did nothing the whole day, her husband and her personal
attendant, Mrs. Bantique, constantly at her side. These two persons swore that Mrs.
Felicidad Esguerra Yap made no will, and could have made no will on that day.
The trial judge refused to credit the petitioners evidence for several reasons, the most
important of which were these: (a) if according to his evidence, the decedent wanted to
keep her will a secret, so that her husband would not know it, it is strange she executed it
in the presence of Felina Esguerra, knowing as she did that witnesses were unnecessary;
(b) in the absence of a showing that Felina was a confidant of the decedent it is hard to
believe that the latter would have allowed the former to see and read the will several times;
(c) it is improbable that the decedent would have permitted Primitivo Reyes, Rosario Gan
Jimenez and Socorro Olarte to read her will, when she precisely wanted its contents to
remain a secret during her lifetime; (d) it is also improbable that her purpose being to
conceal the will from her husband she would carry it around, even to the hospital, in her
purse which could for one reason or another be opened by her husband; (e) if it is true that
the husband demanded the purse from Felina in the U.S.T. Hospital and that the will was
there, it is hard to believe that he returned it without destroying the will, the theory of the
petitioner being precisely that the will was executed behind his back for fear he will destroy
it.

oppositor and of his witnesses in a vigorous effort to discredit them. It appears that the
same arguments, or most of them, were presented in the motion to reconsider; but they
failed to induce the court a quo to change its mind. The oppositors brief, on the other
hand, aptly answers the criticisms. We deem it unnecessary to go over the same matters,
because in our opinion the case should be decided not on the weakness of the opposition
but on the strength of the evidence of the petitioner, who has the burden of proof.
The Spanish Civil Code permited the execution of holographic wills along with other forms.
The Code of Civil Procedure (Act 190) approved August 7, 1901, adopted only one form,
thereby repealing the other forms, including holographic wills.
The New Civil Code effective in 1950 revived holographic wills in its arts. 810-814. "A
person may execute a holographic will which must be entirely written, dated, and signed by
the hand of the testator himself. It is subject to no other form and may be made in or out of
the Philippines, and need not be witnessed."cralaw virtua1aw library
This is indeed a radical departure from the form and solemnities provided for wills under
Act 190, which for fifty years (from 1901 to 1950) required wills to be subscribed by the
testator and three credible witnesses in each and every page; such witnesses to attest to
the number of sheets used and to the fact that the testator signed in their presence and
that they signed in the presence of the testator and of each other.
The object of such requirements it has been said, is to close the door against bad faith and
fraud, to prevent substitution of wills, to guarantee their truth and authenticity (Abangan v.
Abangan, 40 Phil., 476) and to avoid that those who have no right to succeed the testator
would succeed him and be benefited with the probate of same. (Mendoza v. Pilapil, 40 off.
Gaz., 1855). However, formal imperfections may be brushed aside when authenticity of the
instrument is duly proved. (Rodriguez v. Yap, 40 Off. Gaz. Ist Supp. No. 3 p. 194.) .
Authenticity and due execution is the dominant requirement to be fulfilled when such will is
submitted to the courts for allowance. For that purpose the testimony of one of the
subscribing witnesses would be sufficient, if there is no opposition (Sec. 5, Rule 77). If
there is, the three must testify, if available. (Cabang v. Delfinado 34 Phil., 291; Tolentino v.
Francisco, 57 Phil., 742). From the testimony of such witnesses (and of other additional
witnesses) the court may form its opinion as to the genuineness and authenticity of the
testament, and the circumstances of its due execution.

In the face of these improbabilities, the trial judge had to accept the oppositors evidence
that Felicidad did not and could not have executed such holographic will.

Now, in the matter of holographic wills, no such guaranties of truth and veracity are
demanded, since as stated, they need no witnesses; provided however, that they are
"entirely written, dated, and signed by the hand of the testator himself." The law, it is
reasonable to suppose, regards the document itself as material proof of authenticity, and
as its own safeguard, since it could at any time, be demonstrated to be or not to be in
the hands of the testator himself. "In the probate of a holographic will" says the New Civil
Code, "it shall be necessary that at least one witness who knows the handwriting and
signature of the testator explicitly declare that the will and the signature are in the
handwriting of the testator. If the will is contested, at least three such witnesses shall be
required. In the absence of any such witnesses, (familiar with decedents handwriting) and
if the court deem it necessary, expert testimony may be resorted to."cralaw virtua1aw
library

In this appeal, the major portion of appellants brief discussed the testimony of the

The witnesses so presented do not need to have seen the execution of the holographic

26 | S U C C E S S I O N

will. They may be mistaken in their opinion of the handwriting, or they may deliberately lie
in affirming it is in the testators hand. However, the oppositor may present other witnesses
who also know the testators handwriting, or some expert witnesses, who after comparing
the will with other writings or letters of the deceased, have come to the conclusion that
such will has not been written by the hand of the deceased. (Sec. 50, Rule 123). And the
court, in view of such contradictory testimony may use its own visual sense, and decide in
the face of the document, whether the will submitted to it has indeed been written by the
testator.

Art. 692 bears the same implication, to a greater degree. It requires that the surviving
spouse and the legitimate ascendants and descendants be summoned so that they may
make "any statement they may desire to submit with respect to the authenticity of the will."
As it is universally admitted that the holographic will is usually done by the testator and by
himself alone, to prevent others from knowing either its execution or its contents, the above
article 692 could not have the idea of simply permitting such relatives to state whether they
know of the will, but whether in the face of the document itself they think the testator wrote
it. Obviously, this they cant do unless the will itself is presented to the Court and to them.

Obviously, when the will itself is not submitted, these means of opposition, and of
assessing the evidence are not available. And then the only guaranty of authenticity 3
the testators handwriting has disappeared.

Undoubtedly, the intention of the law is to give the near relatives the choice of either
complying with the will if they think it authentic, or to oppose it, if they think it spurious. 5
Such purpose is frustrated when the document is not presented for their examination. If it
be argued that such choice is not essential, because anyway the relatives may oppose, the
answer is that their opposition will be at a distinct disadvantage, and they have the right
and privilege to comply with the will, if genuine, a right which they should not be denied by
withholding inspection thereof from them.

Therefore, the question presents itself, may a holographic will be probated upon the
testimony of witnesses who have allegedly seen it and who declare that it was in the
handwriting of the testator? How can the oppositor prove that such document was not in
the testators handwriting? His witnesses who know testators handwriting have not
examined it. His experts can not testify, because there is no way to compare the alleged
testament with other documents admittedly, or proven to be, in the testators hand. The
oppositor will, therefore, be caught between the upper millstone of his lack of knowledge of
the will or the form thereof, and the nether millstone of his inability to prove its falsity. Again
the proponents witnesses may be honest and truthful; but they may have been shown a
faked document, and having no interest to check the authenticity thereof have taken no
pains to examine and compare. Or they may be perjurers boldly testifying, in the
knowledge that none could convict them of perjury, because no one could prove that they
have not "been shown" a document which they believed was in the handwriting of the
deceased. Of course, the competency of such perjured witnesses to testify as to the
handwriting could be tested by exhibiting to them other writings sufficiently similar to those
written by the deceased; but what witness or lawyer would not foresee such a move and
prepare for it? His knowledge of the handwriting established, the witness (or witnesses)
could simply stick to his statement: he has seen and read a document which he believed
was in the deceaseds handwriting. And the court and the oppositor would practically be at
the mercy of such witness (or witnesses) not only as to the execution, but also as to the
contents of the will. Does the law permit such a situation?
The Rules of Court, (Rule 77) approved in 1940, allow proof (and probate) of a lost or
destroyed will by secondary evidence the testimony of witnesses, in lieu of the original
document. Yet such Rules could not have contemplated holographic wills which could not
then be validly made here. (See also Sec. 46, Rule 123; Art. 830-New Civil Code.) .

We find confirmation of these ideas about exhibition of the document itself in the
decision of the Supreme Court of Spain of June 5, 1925, which denied protocolization or
probate to a document containing testamentary dispositions in the handwriting of the
deceased, but apparently mutilated, the signature and some words having been torn from
it. Even in the face of allegations and testimonial evidence (which was controverted),
ascribing the mutilation to the opponents of the will. The aforesaid tribunal declared that, in
accordance with the provision of the Civil Code (Spanish) the will itself, whole and
unmutilated, must be presented; otherwise, it shall produce no effect.
"Considerando que sentado lo anterior, y estableciendose en el parrafo segundo del
articulo 688 del Codigo civil, que para que sea valido el testamento olografo debera estar
escrito todo el y firmado por testador, con expression del ao, mes y dia en que se
otorque, resulta evidente que para la validez y eficacia de esos testamentos, no basta la
demostracion mas o menos cumplida de que cuando se otorgaron se llenaron todos esos
requisitos, sino que de la expresada redaccion el precepto legal, y por el tiempo en que el
verbo se emplea, se desprende la necesidad de que el documento se encuentre en dichas
condiciones en el momento de ser presentado a la Autoridad competente, para su
adveracion y protocolizacion; y como consecuencia ineludible de ello, forzoso es affirmar
que el de autos carece de validez y aficacia, por no estar firmado por el testador,
cualquiera que sea la causa de la falta de firma, y sin perjuicio de las acciones que
puedan ejercitar los perjudicados, bien para pedir indemnizacion por el perjuicio a la
persona culpable, si la hubiere, o su castigo en via criminal si procediere, por constituir
dicha omision un defecto insubsanable . . . ."cralaw virtua1aw library

Could Rule 77 be extended, by analogy, to holographic wills?


Spanish commentators agree that one of the greatest objections to the holographic will is
that it may be lost or stolen 4 an implied admission that such loss or theft renders it
useless.
This must be so, because the Civil Code requires it to be protocoled and presented to the
judge, (Art. 689) who shall subscribe it and require its identity to be established by the
three witnesses who depose that they have no reasonable doubt that the will was written
by the testator (Art. 691). And if the judge considers that the identity of the will has been
proven he shall order that it be filed (Art. 693). All these, imply presentation of the will itself.

27 | S U C C E S S I O N

This holding aligns with the ideas on holographic wills in the Fuero Juzgo, admittedly the
basis of the Spanish Civil Code provisions on the matter. 6
"PRECEDENTES LEGALES Fuero Juzgo, libro segundo, titulo V, ley 15 E depues
que los herederos e sus fijos ovieren esta manda, fasta . . . annos muestrenla al obispo de
la tierra, o al juez fasta Vl meses y el obispo o el juez tomen otros tales tres escritos, que
fuesen fechos por su mano daquel que fizo la manda; e por aquellos escriptos, si semjara
la letra de la manda, sea confirmada la manda. E depues que todo esto fuere connoscido,
el obispo o el juez, o otras testimonios confirmen el escripto de la manda otra vez, y en
esta manera vala la manda." (Art. 689, Scaevola - Codigo Civil.)

and teachers of Civil Law. 10


(According to the Fuero above, the will itself must be compared with specimens of the
testators handwriting.)
All of which can only mean: the courts will not distribute the property of the deceased in
accordance with his holographic will, unless they are shown his handwriting and signature.
7
Parenthetically, it may be added that even the French Civil Law considers the loss of the
holographic will to be fatal. (Planiol y Ripert, Derecho Civil Frances, traduccion por Diaz
Cruz, 1946, Tomo V, page 555).
Taking all the above circumstances together, we reach the conclusion that the execution
and the contents of a lost or destroyed holographic will may not be proved by the bare
testimony of witnesses who have seen and/or read such will. 8
Under the provisions of Art. 838 of the New Civil Code, we are empowered to adopt this
opinion as a Rule of Court for the allowance of such holographic wills. We hesitate,
however, to make this Rule decisive of this controversy, simultaneously with its
promulgation. Anyway, decision of the appeal may rest on the sufficiency, rather the
insufficiency, of the evidence presented by petitioner Fausto E. Gan.
At this point, before proceeding further, it might be convenient to explain why, unlike
holographic wills, ordinary wills may be proved by testimonial evidence when lost or
destroyed. The difference lies in the nature of the wills. In the first, the only guarantee of
authenticity is the handwriting itself; in the second, the testimony of the subscribing or
instrumental witnesses (and of the notary, now). The loss of the holographic will entails the
loss of the only medium of proof; if the ordinary will is lost, the subscribing witnesses are
available to authenticate.
In the case of ordinary wills, it is quite hard to convince three witnesses (four with the
notary) deliberately to lie. And then their lies could be checked and exposed, their
whereabouts and acts on the particular day, the likelihood that they would be called by the
testator, their intimacy with the testator, etc. And if they were intimates or trusted friends of
the testator they are not likely to lend themselves to any fraudulent scheme to distort his
wishes. Last but not least, they can not receive anything on account of the will.
Whereas in the case of holographic wills, if oral testimony were admissible 9 only one man
could engineer the whole fraud this way: after making a clever or passable imitation of the
handwriting and signature of the deceased, he may contrive to let three honest and
credible witnesses see and read the forgery; and the latter, having no interest, could easily
fall for it, and in court they would in all good faith affirm its genuineness and authenticity.
The will having been lost the forger may have purposely destroyed it in an "accident"
the oppositors have no way to expose the trick and the error, because the document itself
is not at hand. And considering that the holographic will may consist of two or three pages,
and only one of them need be signed, the substitution of the unsigned pages, which may
be the most important ones, may go undetected.
If testimonial evidence of holographic wills be permitted, one more objectionable feature
feasibility of forgery would be added to the several objections to this kind of wills listed
by Castan, Sanchez Roman and Valverde and other well-known Spanish Commentators

28 | S U C C E S S I O N

One more fundamental difference: in the case of a lost will, the three subscribing witnesses
would be testifying to a fact which they saw, namely the act of the testator of subscribing
the will; whereas in the case of a lost holographic will, the witnesses would testify as to
their opinion of the handwriting which they allegedly saw, an opinion which can not be
tested in court, nor directly contradicted by the oppositors, because the handwriting itself is
not at hand.
Turning now to the evidence presented by the petitioner, we find ourselves sharing the trial
judges disbelief. In addition to the dubious circumstances described in the appealed
decision, we find it hard to believe that the deceased should show her will precisely to
relatives who had received nothing from it: Socorro Olarte and Primitivo Reyes. These
could pester her into amending her will to give them a share, or threaten to reveal its
execution to her husband Ildefonso Yap. And this leads to another point: if she wanted so
much to conceal the will from her husband, why did she not entrust it to her beneficiaries?
Opportunity to do so was not lacking: for instance, her husbands trip to Davao, a few days
after the alleged execution of the will.
In fine, even if oral testimony were admissible to establish and probate a lost holographic
will, we think the evidence submitted by herein petitioner is so tainted with improbabilities
and inconsistencies that it fails to measure up to that "clear and distinct" proof required by
Rule 77, sec. 6. 11
Wherefore, the rejection of the alleged will must be sustained.
Judgment affirmed, with costs against petitioner.

G.R. No. L-58509 December 7, 1982


IN THE MATTER OF THE PETITION TO APPROVE THE WILL OF RICARDO
B. BONILLA deceased, MARCELA RODELAS, petitioner-appellant,
vs.
AMPARO ARANZA, ET AL., oppositors-appellees, ATTY. LORENZO
SUMULONG, intervenor.
RELOVA, J.:
This case was certified to this Tribunal by the Court of Appeals for final determination
pursuant to Section 3, Rule 50 of the Rules of Court.
As found by the Court of Appeals:
... On January 11, 1977, appellant filed a petition with the Court of First
Instance of Rizal for the probate of the holographic will of Ricardo B.
Bonilla and the issuance of letters testamentary in her favor. The

petition, docketed as Sp. Proc. No. 8432, was opposed by the


appellees Amparo Aranza Bonilla, Wilferine Bonilla Treyes Expedita
Bonilla Frias and Ephraim Bonilla on the following grounds:
(1) Appellant was estopped from claiming that the deceased left a will
by failing to produce the will within twenty days of the death of the
testator as required by Rule 75, section 2 of the Rules of Court;
(2) The alleged copy of the alleged holographic will did not contain a
disposition of property after death and was not intended to take effect
after death, and therefore it was not a will
(3) The alleged hollographic will itself,and not an alleged copy thereof,
must be produced, otherwise it would produce no effect, as held in
Gam v. Yap, 104 Phil. 509; and
(4 ) The deceased did not leave any will, holographic or otherwise,
executed and attested as required by law.
The appellees likewise moved for the consolidation of the case with
another case Sp. Proc. No, 8275). Their motion was granted by the
court in an order dated April 4, 1977.
On November 13, 1978, following the consolidation of the cases, the
appellees moved again to dismiss the petition for the probate of the will.
They argued that:

dismissed the petition for the probate of the will of Ricardo B. Bonilla.
The court said:
... It is our considered opinion that once the original copy of the
holographic will is lost, a copy thereof cannot stand in lieu of the
original.
In the case of Gam vs. Yap, 104 Phil. 509, 522, the Supreme Court held
that 'in the matter of holographic wills the law, it is reasonable to
suppose, regards the document itself as the material proof of
authenticity of said wills.
MOREOVER, this Court notes that the alleged holographic will was
executed on January 25, 1962 while Ricardo B. Bonilla died on May 13,
1976. In view of the lapse of more than 14 years from the time of the
execution of the will to the death of the decedent, the fact that the
original of the will could not be located shows to our mind that the
decedent had discarded before his death his allegedly missing
Holographic Will.
Appellant's motion for reconsideration was denied. Hence, an appeal to the Court of
Appeals in which it is contended that the dismissal of appellant's petition is contrary to law
and well-settled jurisprudence.
On July 7, 1980, appellees moved to forward the case to this Court on the ground that the
appeal does not involve question of fact and alleged that the trial court committed the
following assigned errors:

(1) The alleged holographic was not a last will but merely an instruction
as to the management and improvement of the schools and colleges
founded by decedent Ricardo B. Bonilla; and

I. THE LOWER COURT ERRED IN HOLDING THAT A LOST


HOLOGRAPHIC WILL MAY NOT BE PROVED BY A COPY THEREOF;

(2) Lost or destroyed holographic wills cannot be proved by secondary


evidence unlike ordinary wills.

II. THE LOWER COURT ERRED IN HOLDING THAT THE DECEDENT


HAS DISCARDED BEFORE HIS DEATH THE MISSING
HOLOGRAPHIC WILL;

Upon opposition of the appellant, the motion to dismiss was denied by


the court in its order of February 23, 1979.
The appellees then filed a motion for reconsideration on the ground that
the order was contrary to law and settled pronouncements and rulings
of the Supreme Court, to which the appellant in turn filed an opposition.
On July 23, 1979, the court set aside its order of February 23, 1979 and

29 | S U C C E S S I O N

III. THE LOWER COURT ERRED IN DISMISSING APPELLANT'S


WILL.
The only question here is whether a holographic will which was lost or cannot be found can
be proved by means of a photostatic copy. Pursuant to Article 811 of the Civil Code,
probate of holographic wills is the allowance of the will by the court after its due execution
has been proved. The probate may be uncontested or not. If uncontested, at least one

Identifying witness is required and, if no witness is available, experts may be resorted to. If
contested, at least three Identifying witnesses are required. However, if the holographic will
has been lost or destroyed and no other copy is available, the will can not be probated
because the best and only evidence is the handwriting of the testator in said will. It is
necessary that there be a comparison between sample handwritten statements of the
testator and the handwritten will. But, a photostatic copy or xerox copy of the holographic
will may be allowed because comparison can be made with the standard writings of the
testator. In the case of Gam vs. Yap, 104 PHIL. 509, the Court ruled that "the execution
and the contents of a lost or destroyed holographic will may not be proved by the bare
testimony of witnesses who have seen and/or read such will. The will itself must be
presented; otherwise, it shall produce no effect. The law regards the document itself as
material proof of authenticity." But, in Footnote 8 of said decision, it says that "Perhaps it
may be proved by a photographic or photostatic copy. Even a mimeographed or carbon
copy; or by other similar means, if any, whereby the authenticity of the handwriting of the
deceased may be exhibited and tested before the probate court," Evidently, the photostatic
or xerox copy of the lost or destroyed holographic will may be admitted because then the
authenticity of the handwriting of the deceased can be determined by the probate court.
WHEREFORE, the order of the lower court dated October 3, 1979, denying appellant's
motion for reconsideration dated August 9, 1979, of the Order dated July 23, 1979,
dismissing her petition to approve the will of the late Ricardo B. Bonilla, is hereby SET
ASIDE.
SO ORDERED.

G.R. No. L-14003

August 5, 1960

FEDERICO AZAOLA, petitioner-appellant,


vs.
REYES,
J.B.L.,
J.:
CESARIO
SINGSON,
oppositor-appellee.
This appeal, taken on points of law from a decision rendered on 15 January 1958 by the
Court of First Instance of Quezon City in its Special Proceedings No. Q-2640, involves the
determination of the quantity of evidence required for the probate of a holographic will.
The established facts are thus summarized in the decision appealed from (Rec. App. pp.
22-24):
"Briefly speaking, the following facts were established by the petitioner; that on
September 9, 1957, Fortunata S. Vda. de Yance died at 13 Luskot, Quezon City,
known to be the last residence of said testatrix; that Francisco Azaola, petitioner
herein for probate of the holographic will, submitted the said holographic will
(Exh. C) whereby Maria Milagros Azaola was made the sole heir as against the

30 | S U C C E S S I O N

nephew of deceased Cesario Singson; that witness Francisco Azaola testified


that he saw the holographic will (Exh. C) one month, more or less, before the
death of the testatrix, as the same was handed to him and his wife; that the
witness testified also that he recognized all the signatures appearing in the
holographic will (Exh. C) as the handwriting of the testatrix and to reinforce said
statement, witness presented the mortgage (Exh. E), the special power of the
attorney (Exh. F), and the general power of attorney (Exh. F-1), besides the
deeds of sale (Exhs. G and G-1) including an affidavit (Exh. G-2), and that there
were further exhibited in court two residence certificates (Exhs. H and H-1) to
show the signatures of the testatrix, for comparison purposes; that said witness,
Azaola, testified that the penmanship appearing in the aforesaid documentary
evidence is in the handwriting of the testatrix as well as the signatures appearing
in the aforesaid documentary evidence is in the handwriting of the testatrix as
well as the signatures appearing therein are the signatures of the testatrix; that
said witness, in answer to a question of his counsel admitted that the holographic
will was handed to him by the testatrix. "apparently it must have been written by
her" (t.s.n., p. 11). However, on page 16 on the same transcript of the
stenographic notes, when the same witness was asked by counsel if he was
familiar with the penmanship and handwriting of the deceased Fortunata Vda. de
Yance, he answered positively in the affirmative and when he was asked again
whether the penmanship referred to in the previous answer as appearing in the
holographic will (Exh. C) was hers (testatrix'), he answered, "I would definitely
say it is hers"; that it was also established in the proceedings that the assessed
value of the property of the deceased in Luskot, Quezon City, is in the amount of
P7,000.00.
The opposition to the probate was on the ground that (1) the execution of the will was
procured by undue and improper pressure and influence on the part of the petitioner and
his wife, and (2) that the testatrix did not seriously intend the instrument to be her last will,
and that the same was actually written either on the 5th or 6th day of August 1957 and not
on November 20, 1956 as appears on the will.
The probate was denied on the ground that under Article 811 of the Civil Code, the
proponent must present three witnesses who could declare that the will and the signature
are in the writing of the testatrix, the probate being contested; and because the lone
witness presented by the proponent "did not prove sufficiently that the body of the will was
written in the handwriting of the testatrix."
The proponent appealed, urging: first, that he was not bound to produce more than one
witness because the will's authenticity was not questioned; and second, that Article 811
does not mandatorily require the production of three witnesses to identify the handwriting
and signature of a holographic will, even if its authenticity should be denied by the adverse
party.

Article 811 of the Civil Code of the Philippines is to the following effect:
ART. 811. In the probate of a holographic will, it shall be necessary that at least
one witness who knows the handwriting and signature of the testator explicitly
declare that the will and the signature are in the handwriting of the testator. If the
will is contested, at least three of such witnesses shall be required.
In the absence of any competent witnesses referred to in the preceding
paragraph, and if the court deems it necessary, expert testimony may be resorted
to. (691a).
We agree with the appellant that since the authenticity of the will was not contested, he
was not required to produce more than one witness; but even if the genuineness of the
holographic will were contested, we are of the opinion that Article 811 of our present Civil
Code can not be interpreted as to require the compulsory presentation of three witnesses
to identify the handwriting of the testator, under penalty of having the probate denied.
Since no witness may have been present at the execution of a holographic will, none being
required by law (Art. 810, new Civil Code), it becomes obvious that the existence of
witness possessing the requisite qualifications is a matter beyond the control of the
proponent. For it is not merely a question of finding and producing any three witnesses;
they must be witnesses "who know the handwriting and signature of the testator" and who
can declare (truthfully, of course, even if the law does not so express) "that the will and the
signature are in the handwriting of the testator". There may be no available witness of the
testator's hand; or even if so familiarized, the witnesses may be unwilling to give a positive
opinion. Compliance with the rule of paragraph 1 of Article 811 may thus become an
impossibility. That is evidently the reason why the second paragraph of Article 811
prescribes that
in the absence of any competent witness referred to in the preceding paragraph,
and if the court deems it necessary, expert testimony may be resorted to.
As can be seen, the law foresees the possibility that no qualified witness may be found (or
what amounts to the same thing, that no competent witness may be willing to testify to the
authenticity of the will), and provides for resort to expert evidence to supply the deficiency.
It may be true that the rule of this article (requiring that three witnesses be presented if the
will is contested and only one if no contest is had) was derived from the rule established for
ordinary testaments (cf. Cabang vs. Delfinado, 45 Phil., 291; Tolentino vs. Francisco, 57
Phil., 742). But it can not be ignored that the requirement can be considered mandatory
only in the case of ordinary testaments, precisely because the presence of at least three
witnesses at the execution of ordinary wills is made by law essential to their validity (Art.
805). Where the will is holographic, no witness need be present (Art. 10), and the rule

31 | S U C C E S S I O N

requiring production of three witnesses must be deemed merely permissive if absurd


results are to be avoided.
Again, under Article 811, the resort to expert evidence is conditioned by the words "if the
Court deem it necessary", which reveal that what the law deems essential is that the Court
should be convinced of the will's authenticity. Where the prescribed number of witnesses is
produced and the court is convinced by their testimony that the ill is genuine, it may
consider it unnecessary to call for expert evidence. On the other hand, if no competent
witness is available, or none of those produced is convincing, the Court may still, and in
fact it should, resort to handwriting experts. The duty of the Court, in fine, is to exhaust all
available lines of inquiry, for the state is as much interested as the proponent that the true
intention of the testator be carried into effect.
Commenting on analogous provisions of Article 691 of the Spanish Civil Code of 1889, the
noted Commentator, Mucuis Scaevola (Vol. 12, 2nd Ed., p.421), sagely remarks:
La manera como esta concebida la redaccion del ultimo apartado de dicho
precepto induce la conclusion de que siempre o por lo menos, en la mayor parte
de los casos, el Juez debe acudir al criterio pericial para que le ilustre acerca de
la autenticidad del testamento olografo, aunque ya esten insertas en los autos
del expediente las declaraciones testificales. La prudencia con que el Juez debe
de proceder en resoluciones de transcendencia asi lo exige, y la indole delicada
y peligrosa del testamento olografo lo hace necesario para mayor garantia de
todos los interes comprometidos en aquel.
En efecto, el cotejo pericial de letras puede ser una confirmacion facultativa del
dicho profano de los testigos y un modo de desvanecer las ultimas dudas que
pudieran ocurrir al Juez acerca de la autenticidad que trata de averigaur y
declarar. Para eso se ha escrito la frase del citado ultimo apartado, (siempre que
el Juez lo estime conveniente), haya habido o no testigos y dudaran o no estos
respecto de los extremos por que son preguntados.
El arbitrio judicial en este caso debe formarse con independencia de los sucesos
y de su significacion, para responder debidamente de las resoluciones que haya
de dictar.
And because the law leaves it to the trial court if experts are still needed, no unfavourable
inference can be drawn from a party's failure to offer expert evidence, until and unless the
court expresses dissatisfaction with the testimony of the lay witnesses.
Our conclusion is that the rule of the first paragraph of Article 811 of the Civil Code is
merely directory and is not mandatory.

Considering, however, that this is the first occasion in which this Court has been called
upon to construe the import of said article, the interest of justice would be better served, in
our opinion, by giving the parties ample opportunity to adduce additional evidence,
including expert witnesses, should the Court deem them necessary.

On April 6, 1990, Evangeline Calugay, Josephine Salcedo and Eufemia Patigas, devisees
and legatees of the holographic will of the deceased Matilde Seo Vda. de Ramonal, filed
with the Regional Trial Court, Misamis Oriental, Branch 18, a petition 3 for probate of the
holographic will of the deceased, who died on January 16, 1990.

In view of the foregoing, the decision appealed from is set aside, and the records ordered
remanded to the Court of origin, with instructions to hold a new trial in conformity with this
opinion. But evidence already on record shall not be retaken. No costs.

In the petition, respondents claimed that the deceased Matilde Seo Vda. de Ramonal,
was of sound and disposing mind when she executed the will on August 30, 1978, that
there was no fraud, undue influence, and duress employed in the person of the testator,
and will was written voluntarily.

Bengzon, Padilla, Bautista Angelo, Labrador, Concepcion, Barrera and Gutierrez David,
JJ., concur.

G.R. No. 123486

The assessed value of the decedent's property, including all real and personal property
was about P400,000.00, at the time of her death.4

August 12, 1999

EUGENIA RAMONAL CODOY, and MANUEL RAMONAL, petitioners,


vs.
EVANGELINE R. CALUGAY, JOSEPHINE SALCEDO, and UEFEMIA
PATIGAS, respondents.
PARDO, J.:
Before us is a petition for review on certiorari of the decision of the Court of Appeals 1 and
its resolution denying reconsideration, ruling:
Upon the unrebutted testimony of appellant Evangeline Calugay and witness
Matilde Ramonal Binanay, the authenticity of testators holographic will has been
established and the handwriting and signature therein (exhibit S) are hers,
enough to probate said will. Reversal of the judgment appealed from and the
probate of the holographic will in question be called for. The rule is that after
plaintiff has completed presentation of his evidence and the defendant files a
motion for judgment on demurrer to evidence on the ground that upon the facts
and the law plaintiff has shown no right to relief, if the motion is granted and the
order to dismissal is reversed on appeal, the movant loses his right to present
evidence in his behalf (Sec, 1 Rule 35 Revised Rules of Court). Judgment may,
therefore, be rendered for appellant in the instant case.
Wherefore, the order appealed from is REVERSED and judgment rendered
allowing the probate of the holographic will of the testator Matilde Seo Vda. de
Ramonal.2
The facts are as follows:

32 | S U C C E S S I O N

On June 28, 1990, Eugenia Ramonal Codoy and Manuel Ramonal filed an opposition 5 to
the petition for probate, alleging that the holographic will was a forgery and that the same
is even illegible. This gives an impression that a "third hand" of an interested party other
than the "true hand" of Matilde Seo Vda. de Ramonal executed the holographic will.
Petitioners argued that the repeated dates incorporated or appearing on will after every
disposition is out of the ordinary. If the deceased was the one who executed the will, and
was not forced, the dates and the signature should appear at the bottom after the
dispositions, as regularly done and not after every disposition. And assuming that the
holographic will is in the handwriting of the deceased, it was procured by undue and
improper pressure and influence on the part of the beneficiaries, or through fraud and
trickery.1wphi1.nt
Respondents presented six (6) witnesses and various documentary evidence. Petitioners
instead of presenting their evidence, filed a demurrer 6 to evidence, claiming that
respondents failed to establish sufficient factual and legal basis for the probate of the
holographic will of the deceased Matilde Seo Vda. de Ramonal.
On November 26, 1990, the lower Court issued an order, the dispositive portion of which
reads:
WHEREFORE, in view of the foregoing consideration, the Demurrer to Evidence
having being well taken, same is granted, and the petition for probate of the
document (Exhibit "S") on the purported Holographic Will of the late Matilde Seo
Vda. de Ramonal, is denied for insufficiency of evidence and lack of merits. 7
On December 12, 1990, respondents filed a notice of appeal, 8 and in support of their
appeal, the respondents once again reiterated the testimony of the following witnesses,
namely: (1) Augusto Neri; (2) Generosa Senon; (3) Matilde Ramonal Binanay; (4) Teresita
Vedad; (5) Fiscal Rodolfo Waga; and (6) Evangeline Calugay.

To have a clear understanding of the testimonies of the witnesses, we recite an account of


their testimonies.
Augusto Neri, Clerk of Court, Court of First Instance of Misamis Oriental, where the special
proceedings for the probate of the holographic will of the deceased was filed. He produced
and identified the records of the case. The documents presented bear the signature of the
deceased, Matilde Seo Vda. de Ramonal, for the purpose of laying the basis for
comparison of the handwriting of the testatrix, with the writing treated or admitted as
genuine by the party against whom the evidence is offered.
Generosa Senon, election registrar of Cagayan de Oro, was presented to produced and
identify the voter's affidavit of the decedent. However, the voters' affidavit was not
produced for the same was already destroyed and no longer available.
Matilde Ramonal Binanay, testified that the deceased Matilde Seo Vda. de Ramonal was
her aunt, and that after the death of Matilde's husband, the latter lived with her in her
parent's house for eleven (11) years from 1958 to 1969. During those eleven (11) years of
close association the deceased, she acquired familiarity with her signature and handwriting
as she used to accompany her (deceased Matilde Seo Vda. de Ramonal) in collecting
rentals from her various tenants of commercial buildings, and deceased always issued
receipts. In addition to this, she (witness Matilde Binanay) assisted the deceased in posting
the records of the accounts, and carried personal letters of the deceased to her creditors.
Matilde Ramonal Binanay further testified that at the time of the death of Matilde Vda. de
Ramonal, she left a holographic will dated August 30, 1978, which was personally and
entirely written, dated and signed, by the deceased and that all the dispositions therein, the
dates, and the signatures in said will, were that of the deceased.

she became familiar with the signature of the deceased. She testified that the signature
appearing in the holographic will is the true and genuine signature of Matilde Seo Vda. de
Ramonal.
The holographic will which was written in Visayan, is translated in English as follows:
Instruction
August 30, 1978
1. My share at Cogon, Raminal Street, for Evangeline Calugay.
(Sgd) Matilde Vda de Ramonal
August 30, 1978
2. Josefina Salcedo must be given 1,500 square meters at Pinikan Street.
(Sgd) Matilde Vda de Ramonal
August 30, 1978
3. My jewelry's shall be divided among:
1. Eufemia Patigas
2. Josefina Salcedo

Fiscal Rodolfo Waga testified that before he was appointed City Fiscal of Cagayan de Oro,
he was a practicing lawyer, and handled all the pleadings and documents signed by the
deceased in connection with the proceedings of her late husband, as a result of which he
is familiar with the handwriting of the latter. He testified that the signature appearing in the
holographic will was similar to that of the deceased, Matilde Seo Vda. de Ramonal, but he
can not be sure.
The fifth witness presented was Mrs. Teresita Vedad, an employee of the Department of
Environment and Natural Resources, Region 10. She testified that she processed the
application of the deceased for pasture permit and was familiar with the signature of the
deceased, since the signed documents in her presence, when the latter was applying for
pasture permit.
Finally, Evangeline Calugay, one of the respondents, testified that she had lived with the
deceased since birth, and was in fact adopted by the latter. That after a long period of time

33 | S U C C E S S I O N

3. Evangeline Calugay
(Sgd) Matilde Vda de Ramonal
August 30, 1978
4. I bequeath my one (1) hectare land at Mandumol, Indahag to Evangeline R.
Calugay
(Sgd) Matilde Vda de Ramonal
August 30, 1978

5. Give the 2,500 Square Meters at Sta. Cruz Ramonal Village in favor of
Evangeline R. Calugay, Helen must continue with the Sta. Cruz, once I am no
longer around.
(Sgd) Matilde Vda de Ramonal
August 30, 1978
6. Bury me where my husband Justo is ever buried.
(Sgd) Matilde Vda de Ramonal
August 30, 1978
Gene and Manuel:
Follow my instruction in order that I will rest peacefully.
Mama
Matilde Vda de Ramonal
On October 9, 1995, the Court of Appeals, rendered decision 9 ruling that the appeal was
meritorious. Citing the decision in the case of Azaola vs. Singson, 109 Phil. 102, penned
by Mr. Justice J. B. L. Reyes, a recognized authority in civil law, the Court of Appeals held:
. . . even if the genuineness of the holographic will were contested, we are of the
opinion that Article 811 of our present civil code can not be interpreted as to
require the compulsory presentation of three witnesses to identify the handwriting
of the testator, under penalty of having the probate denied. Since no witness may
have been present at the execution of the holographic will, none being required
by law (art. 810, new civil code), it becomes obvious that the existence of
witnesses possessing the requisite qualifications is a matter beyond the control
of the proponent. For it is not merely a question of finding and producing any
three witnesses; they must be witnesses "who know the handwriting and
signature of the testator" and who can declare (truthfully, of course, even if the
law does not express) "that the will and the signature are in the handwriting of the
testator." There may be no available witness acquainted with the testator's hand;
or even if so familiarized, the witness maybe unwilling to give a positive opinion.
Compliance with the rule of paragraph 1 of article 811 may thus become an
impossibility. That is evidently the reason why the second paragraph of article
811 prescribes that

34 | S U C C E S S I O N

in the absence of any competent witness referred to in the preceding paragraph,


and if the court deems it necessary, expert testimony may be resorted to.
As can be see, the law foresees, the possibility that no qualified witness ma be
found (or what amounts to the same thing, that no competent witness may be
willing to testify to the authenticity of the will), and provides for resort to expert
evidence to supply the deficiency.
It may be true that the rule of this article (requiring that three witnesses be
presented if the will is contested and only one if no contest is had) was derived
from the rule established for ordinary testaments (CF Cabang vs. Delfianado, 45
PHIL 291; Tolentino v. Francisco, 57 PHIL 742). But it can not be ignored that the
requirement can be considered mandatory only in case of ordinary testaments,
precisely because the presence of at least three witnesses at the execution of
ordinary wills is made by law essential to their validity (Art. 805). Where the will is
holographic, no witness need be present (art. 10), and the rule requiring
production of three witnesses must be deemed merely permissive if absurd
results are to be avoided.
Again, under Art. 811, the resort to expert evidence is conditioned by the words
"if the court deem it necessary", which reveal that what the law deems essential
is that the court should be convinced of the will's authenticity. Where the
prescribed number of witnesses is produced and the court is convinced by their
testimony that the will is genuine, it may consider it unnecessary to call for expert
evidence. On the other hand, if no competent witness is available, or none of
those produced is convincing, the court may still, and in fact it should resort to
handwriting experts. The duty of the court, in fine, is to exhaust all available lines
of inquiry, for the state is as much interested as the proponent that the true
intention of the testator be carried into effect.
Paraphrasing Azaola vs. Singson, even if the genuineness of the holographic will
were contested, Article 811 of the civil code cannot be interpreted as to require
the compulsory presentation of three witnesses to identify the handwriting of the
testator, under penalty of the having the probate denied. No witness need be
present in the execution of the holographic will. And the rule requiring the
production of three witnesses is merely permissive. What the law deems
essential is that the court is convinced of the authenticity of the will. Its duty is to
exhaust all available lines of inquiry, for the state is as much interested in the
proponent that the true intention of the testator be carried into effect. And
because the law leaves it to the trial court to decide if experts are still needed, no
unfavorable inference can be drawn from a party's failure to offer expert
evidence, until and unless the court expresses dissatisfaction with the testimony
of the lay witnesses.10

According to the Court of Appeals, Evangeline Calugay, Matilde Ramonal Binanay and
other witnesses definitely and in no uncertain terms testified that the handwriting and
signature in the holographic will were those of the testator herself.
Thus, upon the unrebutted testimony of appellant Evangeline Calugay and witness Matilde
Ramonal Binanay, the Court of Appeals sustained the authenticity of the holographic will
and the handwriting and signature therein, and allowed the will to probate.

It will be noted that not all the witnesses presented by the respondents testified explicitly
that they were familiar with the handwriting of testator. In the case of Augusto Neri, clerk of
court, Court of First Instance, Misamis Oriental, he merely identified the record of Special
Proceedings No. 427 before said court. He was not presented to declare explicitly that the
signature appearing in the holographic was that of the deceased.

Hence, this petition.

Generosa E. Senon, the election registrar of Cagayan de Oro City, was presented to
identify the signature of the deceased in the voter's affidavit, which was not even produced
as it was no longer available.

The petitioners raise the following issues:

Matilde Ramonal Binanay, on the other hand, testified that:

(1) Whether or not the ruling of the case of Azaola vs. Singson, 109 Phil. 102,
relied upon by the respondent Court of Appeals, was applicable to the case.

Q. And you said for eleven (11) years Matilde Vda de Ramonal resided with
your parents at Pinikitan, Cagayan de Oro City. Would you tell the court what was
your occupation or how did Matilde Vda de Ramonal keep herself busy that time?

(2) Whether or not the Court of Appeals erred in holding that private respondents
had been able to present credible evidence to that the date, text, and signature
on the holographic will written entirely in the hand of the testatrix.

A. Collecting rentals.
Q. From where?

(3) Whether or not the Court of Appeals erred in not analyzing the signatures in
the holographic will of Matilde Seo Vda. de Ramonal.
In this petition, the petitioners ask whether the provisions of Article 811 of the Civil Code
are permissive or mandatory. The article provides, as a requirement for the probate of a
contested holographic will, that at least three witnesses explicitly declare that the signature
in the will is the genuine signature of the testator.1wphi1.nt
We are convinced, based on the language used, that Article 811 of the Civil Code is
mandatory. The word "shall" connotes a mandatory order. We have ruled that "shall" in a
statute commonly denotes an imperative obligation and is inconsistent with the idea of
discretion and that the presumption is that the word "shall," when used in a statute is
mandatory.11

A. From the land rentals and commercial buildings at Pabayo-Gomez streets. 12


xxx

xxx

Q. Who sometime accompany her?


A. I sometimes accompany her.
Q. In collecting rentals does she issue receipts?
A. Yes, sir.13

Laws are enacted to achieve a goal intended and to guide against an evil or mischief that
aims to prevent. In the case at bar, the goal to achieve is to give effect to the wishes of the
deceased and the evil to be prevented is the possibility that unscrupulous individuals who
for their benefit will employ means to defeat the wishes of the testator.

xxx

So, we believe that the paramount consideration in the present petition is to determine the
true intent of the deceased. An exhaustive and objective consideration of the evidence is
imperative to establish the true intent of the testator.

A. Yes, sir.

35 | S U C C E S S I O N

xxx

xxx

xxx

Q. Showing to you the receipt dated 23 October 1979, is this the one you are
referring to as one of the receipts which she issued to them?

Q. Now there is that signature of Matilde vda. De Ramonal, whose signature is


that Mrs. Binanay?

A. Matilde vda. De Ramonal.

xxx

xxx

xxx

Q. Why do you say that is the signature of Matilde Vda. De Ramonal?

Q. You testified that at time of her death she left a will. I am showing to you a
document with its title "tugon" is this the document you are referring to?

A. I am familiar with her signature.


A. Yes, sir.
Q. Now, you tell the court Mrs. Binanay, whether you know Matilde vda de
Ramonal kept records of the accounts of her tenants?

Q. Showing to you this exhibit "S", there is that handwritten "tugon", whose
handwriting is this?

A. Yes, sir.
A. My Aunt.
Q. Why do you say so?
Q. Why do you say this is the handwriting of your aunt?
A. Because we sometimes post a record of accounts in behalf of Matilde Vda.
De Ramonal.
Q. How is this record of accounts made? How is this reflected?
A. In handwritten.14
xxx

xxx

xxx

Q. In addition to collection of rentals, posting records of accounts of tenants and


deed of sale which you said what else did you do to acquire familiarity of the
signature of Matilde Vda De Ramonal?
A. Posting records.
Q. Aside from that?
A. Carrying letters.

What Ms. Binanay saw were pre-prepared receipts and letters of the deceased, which she
either mailed or gave to her tenants. She did not declare that she saw the deceased sign a
document or write a note.
Further, during the cross-examination, the counsel for petitioners elicited the fact that the
will was not found in the personal belongings of the deceased but was in the possession of
Ms. Binanay. She testified that:
Q. Mrs. Binanay, when you were asked by counsel for the petitioners if the late
Matilde Seno vda de Ramonal left a will you said, yes?
A. Yes, sir.
Q. Who was in possession of that will?
A. I.

Q. Letters of whom?
A. Matilde.

Q. Since when did you have the possession of the will?


A. It was in my mother's possession.

Q. To whom?
A. To her creditors.

A. Because I am familiar with her signature.16

Q. So, it was not in your possession?


15

36 | S U C C E S S I O N

A. Sorry, yes.

Q. And when did you come into possession since as you said this was originally
in the possession of your mother?

xxx

xxx

Q. Now, let us go to the third signature of Matilde Ramonal. Do you know that
there are retracings in the word Vda.?

A. 1985.17
xxx

xxx

xxx

xxx

Q. Now, Mrs. Binanay was there any particular reason why your mother left that
will to you and therefore you have that in your possession?

A. Yes, a little. The letter L is continuous.


Q. And also in Matilde the letter L is continued to letter D?
A. Yes, sir.

A. It was not given to me by my mother, I took that in the aparador when she
died.

Q. Again the third signature of Matilde Vda de Ramonal the letter L in Matilde is
continued towards letter D.

Q. After taking that document you kept it with you?


A. Yes, sir.
A. I presented it to the fiscal.
Q. And there is a retracing in the word Vda.?
Q. For what purpose?
A. Yes, sir.20
A. Just to seek advice.
xxx

xxx

xxx

Q. Advice of what?
A. About the will.18
In her testimony it was also evident that Ms. Binanay kept the fact about the will from
petitioners, the legally adopted children of the deceased. Such actions put in issue her
motive of keeping the will a secret to petitioners and revealing it only after the death of
Matilde Seo Vda. de Ramonal.
In the testimony of Ms. Binanay, the following were established:
Q. Now, in 1978 Matilde Seno Vda de Ramonal was not yet a sickly person is
that correct?
A. Yes, sir.
Q. She was up and about and was still uprightly and she could walk agilely and
she could go to her building to collect rentals, is that correct?
A. Yes, sir.19

37 | S U C C E S S I O N

Q. Now, that was 1979, remember one year after the alleged holographic will.
Now, you identified a document marked as Exhibit R. This is dated January 8,
1978 which is only about eight months from August 30, 1978. Do you notice that
the signature Matilde Vda de Ramonal is beautifully written and legible?
A. Yes, sir the handwriting shows that she was very exhausted.
Q. You just say that she was very exhausted while that in 1978 she was healthy
was not sickly and she was agile. Now, you said she was exhausted?
A. In writing.
Q. How did you know that she was exhausted when you were not present and
you just tried to explain yourself out because of the apparent inconsistencies?
A. That was I think. (sic).
Q. Now, you already observed this signature dated 1978, the same year as the
alleged holographic will. In exhibit I, you will notice that there is no retracing;

there is no hesitancy and the signature was written on a fluid movement. . . . And
in fact, the name Eufemia R. Patigas here refers to one of the petitioners?

Q. Now, I am showing to you Exhibit S which is captioned "tugon" dated Agosto


30, 1978 there is a signature here below item No. 1, will you tell this court whose
signature is this?

A. Yes, sir.
A. Yes, sir, that is her signature.
Q. You will also notice Mrs. Binanay that it is not only with the questioned
signature appearing in the alleged holographic will marked as Exhibit X but in the
handwriting themselves, here you will notice the hesitancy and tremors, do you
notice that?
A. Yes, sir.21
Evangeline Calugay declared that the holographic will was written, dated and signed in the
handwriting of the testator. She testified that:

Q. Why do you say that is her signature?


A. I am familiar with her signature.23
So, the only reason that Evangeline can give as to why she was familiar with the
handwriting of the deceased was because she lived with her since birth. She never
declared that she saw the deceased write a note or sign a document.
The former lawyer of the deceased, Fiscal Waga, testified that:

Q. You testified that you stayed with the house of the spouses Matilde and Justo
Ramonal for the period of 22 years. Could you tell the court the services if any
which you rendered to Matilde Ramonal?
A. During my stay I used to go with her to the church, to market and then to her
transactions.
Q. What else? What services that you rendered?
A. After my college days I assisted her in going to the bank, paying taxes and to
her lawyer.
Q. What was your purpose of going to her lawyer?
A. I used to be her personal driver.
Q. In the course of your stay for 22 years did you acquire familiarity of the
handwriting of Matilde Vda de Ramonal?
A. Yes, sir.

A. Because I lived with her since birth.22


xxx

A. Yes, sir I know her because she is my godmother the husband is my


godfather. Actually I am related to the husband by consanguinity.
Q. Can you tell the name of the husband?
A. The late husband is Justo Ramonal.24
xxx

xxx

xxx

Q. Can you tell this court whether the spouses Justo Ramonal and Matilde
Ramonal have legitimate children?
A. As far as I know they have no legitimate children.25
xxx

xxx

xxx

Q. You said after becoming a lawyer you practice your profession? Where?

Q. How come that you acquired familiarity?

xxx

Q. Do you know Matilde Vda de Ramonal?

xxx

38 | S U C C E S S I O N

A. Here in Cagayan de Oro City.


Q. Do you have services rendered with the deceased Matilde vda de Ramonal?

A. I assisted her in terminating the partition, of properties.


Q. When you said assisted, you acted as her counsel? Any sort of counsel as in
what case is that, Fiscal?
A. It is about the project partition to terminate the property, which was under the
court before.26
xxx

xxx

xxx

Q. Appearing in special proceeding no. 427 is the amended inventory which is


marked as exhibit N of the estate of Justo Ramonal and there appears a
signature over the type written word Matilde vda de Ramonal, whose signature is
this?

A. I think this signature here it seems to be the signature of Mrs. Matilde vda de
Ramonal.
Q. Now, in item No. 2 there is that signature here of Matilde Vda de Ramonal,
can you tell the court whose signature is this?
A. Well, that is similar to that signature appearing in the project of partition.
Q. Also in item no. 3 there is that signature Matilde Vda de Ramonal, can you
tell the court whose signature is that?
A. As I said, this signature also seems to be the signature of Matilde vda de
Ramonal.
Q. Why do you say that?

A. That is the signature of Matilde Vda de Ramonal.


A. Because there is a similarity in the way it is being written.
Q. Also in exhibit n-3, whose signature is this?
A. This one here that is the signature of Mrs. Matilde vda de Ramonal. 27
xxx

xxx

xxx

Q. Aside from attending as counsel in that Special Proceeding Case No. 427
what were the other assistance wherein you were rendering professional service
to the deceased Matilde Vda de Ramonal?

Q. How about this signature in item no. 4, can you tell the court whose signature
is this?
A. The same is true with the signature in item no. 4. It seems that they are
similar.29
xxx

xxx

xxx

A. I can not remember if I have assisted her in other matters but if there are
documents to show that I have assisted then I can recall.28

Q. Mr. Prosecutor, I heard you when you said that the signature of Matilde Vda
de Ramonal Appearing in exhibit S seems to be the signature of Matilde vda de
Ramonal?

xxx

A. Yes, it is similar to the project of partition.

xxx

xxx

Q. Now, I am showing to you exhibit S which is titled "tugon", kindly go over this
document, Fiscal Waga and tell the court whether you are familiar with the
handwriting contained in that document marked as exhibit "S"?

Q. So you are not definite that this is the signature of Matilde vda de Ramonal.
You are merely supposing that it seems to be her signature because it is similar
to the signature of the project of partition which you have made?

A. I am not familiar with the handwriting.

A. That is true.30

Q. This one, Matilde Vda de Ramonal, whose signature is this?

39 | S U C C E S S I O N

From the testimonies of these witnesses, the Court of Appeals allowed the will to probate
and disregard the requirement of three witnesses in case of contested holographic will,

citing the decision in Azaola vs. Singson,31ruling that the requirement is merely directory
and not mandatory.
In the case of Ajero vs. Court of Appeals,32 we said that "the object of the solemnities
surrounding the execution of wills is to close the door against bad faith and fraud, to avoid
substitution of wills and testaments and to guaranty their truth and authenticity. Therefore,
the laws on this subject should be interpreted in such a way as to attain these primordial
ends. But on the other hand, also one must not lose sight of the fact that it is not the object
of the law to restrain and curtail the exercise of the right to make a will.
However, we cannot eliminate the possibility of a false document being adjudged as the
will of the testator, which is why if the holographic will is contested, that law requires three
witnesses to declare that the will was in the handwriting of the deceased.
The will was found not in the personal belongings of the deceased but with one of the
respondents, who kept it even before the death of the deceased. In the testimony of Ms.
Binanay, she revealed that the will was in her possession as early as 1985, or five years
before the death of the deceased.
There was no opportunity for an expert to compare the signature and the handwriting of
the deceased with other documents signed and executed by her during her lifetime. The
only chance at comparison was during the cross-examination of Ms. Binanay when the
lawyer of petitioners asked Ms. Binanay to compare the documents which contained the
signature of the deceased with that of the holographic will and she is not a handwriting
expert. Even the former lawyer of the deceased expressed doubts as to the authenticity of
the signature in the holographic will.
A visual examination of the holographic will convince us that the strokes are different when
compared with other documents written by the testator. The signature of the testator in
some of the disposition is not readable. There were uneven strokes, retracing and
erasures on the will.
Comparing the signature in the holographic will dated August 30, 1978, 33 and the
signatures in several documents such as the application letter for pasture permit dated
December 30, 1980,34 and a letter dated June 16, 1978, 35 the strokes are different. In the
letters, there are continuous flows of the strokes, evidencing that there is no hesitation in
writing unlike that of the holographic will. We, therefore, cannot be certain that ruling
holographic will was in the handwriting by the deceased.
IN VIEW WHEREOF, the decision appealed from is SET ASIDE. The records are ordered
remanded to the court of origin with instructions to allow petitioners to adduce evidence in
support
their opposition
to the
G.R. No.of106720
September
15,probate
1994 of the holographic will of the deceased Matilde
Seo vda. de Ramonal.1wphi1.nt.

SPOUSES ROBERTO AND THELMA AJERO, petitioners,

40
vs. | S U C C E S S I O N

THE COURT OF APPEALS AND CLEMENTE SAND, respondents.

PUNO, J.:
This is an
appeal
by certiorari from
the
Decision
of the
Court of
Appeals 1 in CA-G.R. CV No. 22840, dated March 30, 1992, the dispositive portion of
which reads;
PREMISES CONSIDERED, the questioned decision of November 19,
1988 of the trial court is hereby REVERSED and SET ASIDE, and the
petition for probate is hereby DISMISSED. No costs.
The earlier Decision was rendered by the RTC of Quezon City, Branch 94, 2 in
Sp. Proc. No. Q-37171, and the instrument submitted for probate is the
holographic will of the late Annie Sand, who died on November 25, 1982.
In the will, decedent named as devisees, the following: petitioners Roberto and Thelma
Ajero, private respondent Clemente Sand, Meriam S. Arong, Leah Sand, Lilia Sand, Edgar
Sand, Fe Sand, Lisa S. Sand, and Dr. Jose Ajero, Sr., and their children.
On January 20, 1983, petitioners instituted Sp. Proc. No. Q-37171, for allowance of
decedent's holographic will. They alleged that at the time of its execution, she was of
sound and disposing mind, not acting under duress, fraud or undue influence, and was in
every respect capacitated to dispose of her estate by will.
Private respondent opposed the petition on the grounds that: neither the testament's body
nor the signature therein was in decedent's handwriting; it contained alterations and
corrections which were not duly signed by decedent; and, the will was procured by
petitioners through improper pressure and undue influence. The petition was likewise
opposed by Dr. Jose Ajero. He contested the disposition in the will of a house and lot
located in Cabadbaran, Agusan Del Norte. He claimed that said property could not be
conveyed by decedent in its entirety, as she was not its sole owner.
Notwithstanding the oppositions, the trial court admitted the decedent's holographic will to
probate. It found, inter alia:
Considering then that the probate proceedings herein must decide only
the question of identity of the will, its due execution and the

testamentary capacity of the testatrix, this probate court finds no reason


at all for the disallowance of the will for its failure to comply with the
formalities prescribed by law nor for lack of testamentary capacity of the
testatrix.
For one, no evidence was presented to show that the will in question is
different from the will actually executed by the testatrix. The only
objections raised by the oppositors . . . are that the will was not written
in the handwriting of the testatrix which properly refers to the question
of its due execution, and not to the question of identity of will. No other
will was alleged to have been executed by the testatrix other than the
will herein presented. Hence, in the light of the evidence adduced, the
identity of the will presented for probate must be accepted, i.e., the will
submitted in Court must be deemed to be the will actually executed by
the testatrix.
xxx xxx xxx
While the fact that it was entirely written, dated and signed in the
handwriting of the testatrix has been disputed, the petitioners, however,
have satisfactorily shown in Court that the holographic will in question
was indeed written entirely, dated and signed in the handwriting of the
testatrix. Three (3) witnesses who have convincingly shown knowledge
of the handwriting of the testatrix have been presented and have
explicitly and categorically identified the handwriting with which the
holographic will in question was written to be the genuine handwriting
and signature of the testatrix. Given then the aforesaid evidence, the
requirement of the law that the holographic will be entirely written,
dated and signed in the handwriting of the testatrix has been complied
with.
xxx xxx xxx
As to the question of the testamentary capacity of the testratix, (private
respondent) Clemente Sand himself has testified in Court that the
testatrix was completely in her sound mind when he visited her during
her birthday celebration in 1981, at or around which time the
holographic will in question was executed by the testatrix. To be of
sound mind, it is sufficient that the testatrix, at the time of making the
will, knew the value of the estate to be disposed of, the proper object of
her bounty, and thecharacter of the testamentary act . . . The will itself
shows that the testatrix even had detailed knowledge of the nature of
her estate. She even identified the lot number and square meters of the

41 | S U C C E S S I O N

lots she had conveyed by will. The objects of her bounty were likewise
identified explicitly. And considering that she had even written a nursing
book which contained the law and jurisprudence on will and
succession, there is more than sufficient showing that she knows the
character of the testamentary act.
In this wise, the question of identity of the will, its due execution and the
testamentary capacity of the testatrix has to be resolved in favor of the
allowance of probate of the will submitted herein.
Likewise, no evidence was presented to show sufficient reason for the
disallowance of herein holographic will. While it was alleged that the
said will was procured by undue and improper pressure and influence
on the part of the beneficiary or of some other person, the evidence
adduced have not shown any instance where improper pressure or
influence was exerted on the testatrix. (Private respondent) Clemente
Sand has testified that the testatrix was still alert at the time of the
execution of the will, i.e., at or around the time of her birth anniversary
celebration in 1981. It was also established that she is a very intelligent
person and has a mind of her own. Her independence of character and
to some extent, her sense of superiority, which has been testified to in
Court, all show the unlikelihood of her being unduly influenced or
improperly pressured to make the aforesaid will. It must be noted that
the undue influence or improper pressure in question herein only refer
to the making of a will and not as to the specific testamentary
provisions therein which is the proper subject of another proceeding.
Hence, under the circumstances, this Court cannot find convincing
reason for the disallowance of the will herein.
Considering then that it is a well-established doctrine in the law on
succession that in case of doubt, testate succession should be
preferred over intestate succession, and the fact that no convincing
grounds were presented and proven for the disallowance of the
holographic will of the late Annie Sand, the aforesaid will submitted
herein must be admitted to probate. 3 (Citations omitted.)
On appeal, said Decision was reversed, and the petition for probate of decedent's will was
dismissed. The Court of Appeals found that, "the holographic will fails to meet the
requirements for its validity." 4 It held that the decedent did not comply with Articles 813 and
814 of the New Civil Code, which read, as follows:
Art. 813: When a number of dispositions appearing in a holographic will
are signed without being dated, and the last disposition has a signature

and date, such date validates the dispositions preceding it, whatever be
the time of prior dispositions.

(3) If it was executed through force or under duress,


or the influence of fear, or threats;

Art. 814: In case of insertion, cancellation, erasure or alteration in a


holographic will, the testator must authenticate the same by his full
signature.

(4) If it was procured by undue and improper


pressure and influence, on the part of the
beneficiary or of some other person;

It alluded to certain dispositions in the will which were either unsigned and undated, or
signed but not dated. It also found that the erasures, alterations and cancellations made
thereon had not been authenticated by decedent.

(5) If the signature of the testator was procured by


fraud;

Thus, this appeal which is impressed with merit.


Section 9, Rule 76 of the Rules of Court provides that will shall be disallowed in any of the
following cases:
(a) If not executed and attested as required by law;
(b) If the testator was insane, or otherwise mentally incapable to make
a will, at the time of its execution;
(c) If it was executed under duress, or the influence of fear, or threats;
(d) If it was procured by undue and improper pressure and influence, on
the part of the beneficiary, or of some other person for his benefit;
(e) If the signature of the testator was procured by fraud or trick, and he
did not intend that the instrument should be his will at the time of fixing
his signature thereto.
In the same vein, Article 839 of the New Civil Code reads:
Art. 839: The will shall be disallowed in any of the following cases;
(1) If the formalities required by law have not been
complied with;
(2) If the testator was insane, or otherwise mentally
incapable of making a will, at the time of its
execution;

42 | S U C C E S S I O N

(6) If the testator acted by mistake or did not intend


that the instrument he signed should be his will at
the time of affixing his signature thereto.
These lists are exclusive; no other grounds can serve to disallow a will. 5 Thus, in a petition
to admit a holographic will to probate, the only issues to be resolved are: (1) whether the
instrument submitted is, indeed, the decedent's last will and testament; (2) whether said
will was executed in accordance with the formalities prescribed by law; (3) whether the
decedent had the necessary testamentary capacity at the time the will was executed; and,
(4) whether the execution of the will and its signing were the voluntary acts of the
decedent. 6
In the case at bench, respondent court held that the holographic will of Anne Sand was not
executed in accordance with the formalities prescribed by law. It held that Articles 813 and
814 of the New Civil Code, ante, were not complied with, hence, it disallowed the probate
of said will. This is erroneous.
We reiterate what we held in Abangan vs. Abangan, 40 Phil. 476, 479 (1919), that:
The object of the solemnities surrounding the execution of wills is to
close the door against bad faith and fraud, to avoid substitution of wills
and testaments and to guaranty their truth and authenticity. Therefore,
the laws on this subject should be interpreted in such a way as to attain
these primordial ends. But, on the other hand, also one must not lose
sight of the fact that it is not the object of the law to restrain and curtail
the exercise of the right to make a will. So when an interpretation
already given assures such ends, any other interpretation whatsoever,
that adds nothing but demands more requisites entirely unnecessary,
useless and frustrative of the testator's last will, must be disregarded.
For purposes of probating non-holographic wills, these formal solemnities include the
subscription, attestation, and acknowledgment requirements under Articles 805 and 806 of
the New Civil Code.

In the case of holographic wills, on the other hand, what assures authenticity is the
requirement that they be totally autographic or handwritten by the testator himself, 7 as
provided under Article 810 of the New Civil Code, thus:

Art. 688: Holographic wills may be executed only by persons of full age.
In order that the will be valid it must be drawn on stamped paper
corresponding to the year of its execution, written in its entirety by the
testator and signed by him, and must contain a statement of the year,
month and day of its execution.

A person may execute a holographic will which must be entirely written,


dated, and signed by the hand of the testator himself. It is subject to no
other form, and may be made in or out of the Philippines, and need not
be witnessed. (Emphasis supplied.)

If it should contain any erased, corrected, or interlined words, the


testator must identify them over his signature.

Failure to strictly observe other formalities will not result in the disallowance of a
holographic will that is unquestionably handwritten by the testator.
A reading of Article 813 of the New Civil Code shows that its requirement affects the
validity of the dispositions contained in the holographic will, but not its probate. If the
testator fails to sign and date some of the dispositions, the result is that these
dispositions cannot be effectuated. Such failure, however, does not render the whole
testament void.
Likewise, a holographic will can still be admitted to probate, notwithstanding noncompliance with the provisions of Article 814. In the case of Kalaw vs. Relova 132 SCRA
237 242 (1984), this Court held:
Ordinarily, when a number of erasures, corrections, and interlineations
made by the testator in a holographic Will have not been noted under
his signature, . . . the Will is not thereby invalidated as a whole, but at
most only as respects the particular words erased, corrected or
interlined. Manresa gave an identical commentary when he said "la
omission de la salvedad no anula el testamento, segun la regla de
jurisprudencia establecida en la sentencia de 4 de Abril de
1985." 8 (Citations omitted.)
Thus, unless the unauthenticated alterations, cancellations or insertions were made on the
date of the holographic will or on testator's signature, 9 their presence does not invalidate
the will itself. 10 The lack of authentication will only result in disallowance of such changes.
It is also proper to note that the requirements of authentication of changes and signing and
dating of dispositions appear in provisions (Articles 813 and 814) separate from that which
provides for the necessary conditions for the validity of the holographic will (Article 810).
The distinction can be traced to Articles 678 and 688 of the Spanish Civil Code, from which
the present provisions covering holographic wills are taken. They read as follows:

Foreigners may execute holographic wills in their own language.


This separation and distinction adds support to the interpretation that only the
requirements of Article 810 of the New Civil Code and not those found in Articles 813
and 814 of the same Code are essential to the probate of a holographic will.
The Court of Appeals further held that decedent Annie Sand could not validly dispose of
the house and lot located in Cabadbaran, Agusan del Norte, in its entirety. This is correct
and must be affirmed.
As a general rule, courts in probate proceedings are limited to pass only upon the extrinsic
validity of the will sought to be probated. However, in exceptional instances, courts are not
powerless to do what the situation constrains them to do, and pass upon certain provisions
of the will. 11 In the case at bench, decedent herself indubitably stated in her holographic
will that the Cabadbaran property is in the name of her late father, John H. Sand (which led
oppositor Dr. Jose Ajero to question her conveyance of the same in its entirety). Thus, as
correctly held by respondent court, she cannot validly dispose of the whole property, which
she shares with her father's other heirs.
IN VIEW WHEREOF, the instant petition is GRANTED. The Decision of the Court of
Appeals in CA-G.R. CV No. 22840, dated March 30, 1992, is REVERSED and SET
ASIDE, except with respect to the invalidity of the disposition of the entire house and lot in
Cabadbaran, Agusan del Norte. The Decision of the Regional Trial Court of Quezon City,
Branch 94 in Sp. Proc. No. Q-37171, dated November 19, 1988, admitting to probate the
holographic will of decedent Annie Sand, is hereby REINSTATED, with the above
qualification as regards the Cabadbaran property. No costs.
SO ORDERED.

G.R. No. L-40207 September 28, 1984


Art. 678: A will is called holographic when the testator writes it himself in
the form and with the requisites required in Article 688.

43 | S U C C E S S I O N

ROSA K. KALAW, petitioner,


vs.
HON. JUDGE BENJAMIN RELOVA, Presiding Judge of the CFI of

After trial, respondent Judge denied probate in an Order, dated September 3, 197 3,
reading in part:

MELENCIO-HERRERA, J.:
On September 1, 1971, private respondent GREGORIO K. KALAW, claiming to be the sole
heir of his deceased sister, Natividad K. Kalaw, filed a petition before the Court of First
Instance of Batangas, Branch VI, Lipa City, for the probate of her holographic Will
executed on December 24, 1968.
The holographic Will reads in full as follows:
My Last will and Testament
In the name of God, Amen.
I Natividad K. Kalaw Filipino 63years of age, single, and a resident of Lipa City, being of
sound and disposing mind and memory, do hereby declare thus to be my last will and
testament.
1. It is my will that I'll be burried in the cemetery of the catholic church of Lipa City. In
accordance with the rights of said Church, and that my executrix hereinafter named
provide and erect at the expose of my state a suitable monument to perpetuate my
memory.
xxx xxx xxx

The document Exhibit "C" was submitted to the National Bureau of


Investigation for examination. The NBI reported that the handwriting,
the signature, the insertions and/or additions and the initial were made
by one and the same person. Consequently, Exhibit "C" was the
handwriting of the decedent, Natividad K. Kalaw. The only question is
whether the win, Exhibit 'C', should be admitted to probate although the
alterations and/or insertions or additions above-mentioned were not
authenticated by the full signature of the testatrix pursuant to Art. 814 of
the Civil Code. The petitioner contends that the oppositors are
estopped to assert the provision of Art. 814 on the ground that they
themselves agreed thru their counsel to submit the Document to the
NBI FOR EXAMINATIONS. This is untenable. The parties did not
agree, nor was it impliedly understood, that the oppositors would be in
estoppel.
The Court finds, therefore, that the provision of Article 814 of the Civil
Code is applicable to Exhibit "C". Finding the insertions, alterations
and/or additions in Exhibit "C" not to be authenticated by the full
signature of the testatrix Natividad K. Kalaw, the Court will deny the
admission to probate of Exhibit "C".
WHEREFORE, the petition to probate Exhibit "C" as the holographic
will of Natividad K. Kalaw is hereby denied.
SO ORDERED.

The holographic Will, as first written, named ROSA K. Kalaw, a sister of the testatrix as her
sole heir. Hence, on November 10, 1971, petitioner ROSA K. Kalaw opposed probate
alleging, in substance, that the holographic Will contained alterations, corrections, and
insertions without the proper authentication by the full signature of the testatrix as required
by Article 814 of the Civil Code reading:

From that Order, GREGORIO moved for reconsideration arguing that since the alterations
and/or insertions were the testatrix, the denial to probate of her holographic Will would be
contrary to her right of testamentary disposition. Reconsideration was denied in an Order,
dated November 2, 1973, on the ground that "Article 814 of the Civil Code being , clear
and explicit, (it) requires no necessity for interpretation."

Art. 814. In case of any insertion, cancellation, erasure or alteration in a


holographic will the testator must authenticate the same by his full
signature.

From that Order, dated September 3, 1973, denying probate, and the Order dated
November 2, 1973 denying reconsideration, ROSA filed this Petition for Review on
certiorari on the sole legal question of whether or not theoriginal unaltered text after
subsequent alterations and insertions were voided by the Trial Court for lack of
authentication by the full signature of the testatrix, should be probated or not, with her as
sole heir.

ROSA's position was that the holographic Will, as first written, should be given effect and
probated so that she could be the sole heir thereunder.

44 | S U C C E S S I O N

Ordinarily, when a number of erasures, corrections, and interlineations made by the


testator in a holographic Will litem not been noted under his signature, ... the Will is not
thereby invalidated as a whole, but at most only as respects the particular words erased,
corrected or interlined.1 Manresa gave an Identical commentary when he said "la omision
de la salvedad no anula el testamento, segun la regla de jurisprudencia establecida en la
sentencia de 4 de Abril de 1895." 2
However, when as in this case, the holographic Will in dispute had only one substantial
provision, which was altered by substituting the original heir with another, but which
alteration did not carry the requisite of full authentication by the full signature of the
testator, the effect must be that the entire Will is voided or revoked for the simple reason
that nothing remains in the Will after that which could remain valid. To state that the Will as
first written should be given efficacy is to disregard the seeming change of mind of the
testatrix. But that change of mind can neither be given effect because she failed to
authenticate it in the manner required by law by affixing her full signature,
The ruling in Velasco, supra, must be held confined to such insertions, cancellations,
erasures or alterations in a holographic Will, which affect only the efficacy of the altered
words themselves but not the essence and validity of the Will itself. As it is, with the
erasures, cancellations and alterations made by the testatrix herein, her real intention
cannot be determined with certitude. As Manresa had stated in his commentary on Article
688 of the Spanish Civil Code, whence Article 814 of the new Civil Code was derived:
... No infringe lo dispuesto en este articulo del Codigo (el 688) la
sentencia que no declara la nulidad de un testamento olografo que
contenga palabras tachadas, enmendadas o entre renglones no
salvadas por el testador bajo su firnia segun previene el parrafo tercero
del mismo, porque, en realidad, tal omision solo puede afectar a la
validez o eficacia de tales palabras, y nunca al testamento mismo, ya
por estar esa disposicion en parrafo aparte de aquel que determine las
condiciones necesarias para la validez del testamento olografo, ya
porque, de admitir lo contrario, se Ilegaria al absurdo de que pequefias
enmiendas no salvadas, que en nada afectasen a la parte esencial y
respectiva del testamento, vinieran a anular este, y ya porque el
precepto contenido en dicho parrafo ha de entenderse en perfecta
armonia y congruencia con el art. 26 de la ley del Notariado que
declara nulas las adiciones apostillas entrerrenglonados, raspaduras y
tachados en las escrituras matrices, siempre que no se salven en la
forma prevenida, paro no el documento que las contenga, y con mayor
motivo cuando
las
palabras
enmendadas,
tachadas,
o
entrerrenglonadas no tengan importancia ni susciten duda alguna
acerca del pensamiento del testador, o constituyan meros accidentes
de ortografia o de purez escrituraria, sin trascendencia alguna(l).

45 | S U C C E S S I O N

Mas para que sea aplicable la doctrina de excepcion contenida en este


ultimo fallo, es preciso que las tachaduras, enmiendas o
entrerrenglonados sin salvar saan de pala bras que no afecter4 alteren
ni uarien de modo substancial la express voluntad del testador
manifiesta en el documento. Asi lo advierte la sentencia de 29 de
Noviembre de 1916, que declara nulo un testamento olografo por no
estar salvada por el testador la enmienda del guarismo ultimo del ao
en que fue extendido 3(Emphasis ours).
WHEREFORE, this Petition is hereby dismissed and the Decision of respondent Judge,
dated September 3, 1973, is hereby affirmed in toto. No costs.

G.R. No. 76714 June 2, 1994


SALUD TEODORO VDA. DE PEREZ, petitioner,
vs.
HON. ZOTICO A. TOLETE in his capacity as Presiding Judge, Branch 18,
RTC, Bulacan, respondent.

QUIASON, J.:

This is a petition for certiorari under Rule 65 of the Revised Rules of Court to set aside the
Order dated November 19, 1986 of the Regional Trial Court, Branch 18, Bulacan presided
by respondent Judge Zotico A. Tolete, in Special Proceedings No. 1793-M.
We grant the petition.
II
Dr. Jose F. Cunanan and his wife, Dr. Evelyn Perez-Cunanan, who became American
citizens, established a successful medical practice in New York, U.S.A. The Cunanans
lived at No. 2896 Citation Drive, Pompey, Syracuse, New York, with their children, Jocelyn,
18; Jacqueline, 16; and Josephine, 14.
On August 23, 1979, Dr. Cunanan executed a last will and testament, bequeathing to his
wife "all the remainder" of his real and personal property at the time of his death
"wheresoever situated" (Rollo, p. 35). In the event he would survive his wife, he
bequeathed all his property to his children and grandchildren with Dr. Rafael G. Cunanan,
Jr. as trustee. He appointed his wife as executrix of his last will and testament and Dr.
Rafael G. Cunanan, Jr. as substitute executor. Article VIII of his will states:
If my wife, EVELYN PEREZ-CUNANAN, and I shall die under such
circumstances that there is not sufficient evidence to determine the

order of our deaths, then it shall be presumed that I predeceased her,


and my estate shall be administered and distributed, in all respects, in
accordance with such presumption (Rollo, p. 41).

In a motion dated May 19, 1983, petitioner asked that Dr. Rafael Cunanan, Sr. be ordered
to deliver to her a Philippine Trust Company passbook with P25,594.00 in savings deposit,
and the Family Savings Bank time deposit certificates in the total amount of P12,412.52.

Four days later, on August 27, Dr. Evelyn P. Cunanan executed her own last will and
testament containing the same provisions as that of the will of her husband. Article VIII of
her will states:

On May 31, Atty. Federico Alday filed a notice of appearance as counsel for the heirs of Dr.
Jose F. Cunanan, namely, Dr. Rafael Cunanan, Sr., Priscilla Cunanan Bautista, Lydia
Cunanan Ignacio, Felipe F. Cunanan and Loreto Cunanan Concepcion (Cunanan heirs).
He also manifested that before receiving petitioner's motion of May 19, 1983, his clients
were unaware of the filing of the testate estate case and therefore, "in the interest of
simple fair play," they should be notified of the proceedings (Records, p. 110). He prayed
for deferment of the hearing on the motions of May 19, 1983.

If my husband, JOSE F. CUNANAN, and I shall die under such


circumstances that there is not sufficient evidence to determine the
order of our deaths, then it shall be presumed that he predeceased me,
and my estate shall be administered and distributed in all respects, in
accordance with such presumption. (Rollo, p. 31).
On January 9, 1982, Dr. Cunanan and his entire family perished when they were trapped
by fire that gutted their home. Thereafter, Dr. Rafael G. Cunanan, Jr. as trustee and
substitute executor of the two wills, filed separate proceedings for the probate thereof with
the Surrogate Court of the County of Onondaga, New York. On April 7, these two wills were
admitted to probate and letters testamentary were issued in his favor.
On February 21, 1983, Salud Teodoro Perez, the mother of Dr. Evelyn P. Cunanan, and
petitioner herein, filed with the Regional P. Cunanan, and petitioner herein, filed with the
Regional Trial Court, Malolos, Bulacan a petition for the reprobate of the two bills ancillary
to the probate proceedings in New York. She also asked that she be appointed the special
administratrix of the estate of the deceased couple consisting primarily of a farm land in
San Miguel, Bulacan.
On March 9, the Regional Trial Court, Branch 16, Malolos, Bulacan, presided by Judge
Gualberto J. de la Llana, issued an order, directing the issuance of letters of special
administration in favor of petitioner upon her filing of a P10,000.00 bond. The following day,
petitioner posted the bond and took her oath as special administration.
As her first act of administration, petitioner filed a motion, praying that the Philippine Life
Insurance Company be directed to deliver the proceeds in the amount of P50,000.00 of the
life insurance policy taken by Dr. Jose F. Cunanan with Dr. Evelyn Perez-Cunanan and
their daughter Jocelyn as beneficiaries. The trial court granted the motion.
Counsel for the Philippine American Life Insurance Company then filed a manifestation,
stating that said company then filed a manifestation, stating that said company had
delivered to petitioner the amount of P49,765.85, representing the proceeds of the life
insurance policy of Dr. Jose F. Cunanan.

46 | S U C C E S S I O N

Petitioner then filed a counter manifestation dated June 13, 1983, asserting: (1) that the
"Cunanan collaterals are neither heirs nor creditors of the late Dr. Jose F. Cunanan" and
therefore, they had "no legal or proprietary interests to protect" and "no right to intervene";
(2) that the wills of Dr. Jose F. Cunanan and Dr. Evelyn Perez-Cunanan, being American
citizens, were executed in accordance with the solemnities and formalities of New York
laws, and produced "effects in this jurisdiction in accordance with Art. 16 in relation to Art.
816 of the Civil Code"; (3) that under Article VIII of the two wills, it was presumed that the
husband predeceased the wife; and (4) that "the Cunanan collaterals are neither
distributees, legatees or beneficiaries, much less, heirs as heirship is only by institution"
under a will or by operation of the law of New York (Records, pp. 112-113).
On June 23, the probate court granted petitioner's motion of May 19, 1983. However, on
July 21, the Cunanan heirs filed a motion to nullify the proceedings and to set aside the
appointment of, or to disqualify, petitioner as special administratrix of the estates of Dr.
Jose F. Cunanan and Dr. Evelyn Perez-Cunanan. The motion stated: (1) that being the
"brothers and sisters and the legal and surviving heirs" of Dr. Jose F. Cunanan, they had
been "deliberately excluded" in the petition for the probate of the separate wills of the
Cunanan spouses thereby misleading the Bulacan court to believe that petitioner was the
sole heir of the spouses; that such "misrepresentation" deprived them of their right to "due
process in violation of Section 4, Rule 76 of the Revised Rules of Court; (2) that Dr. Rafael
G. Cunanan, Jr., the executor of the estate of the Cunanan spouses, was likewise not
notified of the hearings in the Bulacan court; (3) that the "misrepresentation and
concealment committed by" petitioner rendered her unfit to be a special administratrix; (4)
that Dr. Rafael G. Cunanan, Jr. had, by virtue of a verified power of attorney, authorized his
father,
Dr. Rafael Cunanan, Sr., to be his attorney-in-fact; and (5) that Dr. Rafael Cunanan, Sr. is
qualified to be a regular administrator "as practically all of the subject estate in the
Philippines belongs to their brother, Dr. Jose F. Cunanan" (Records, pp. 118-122). Hence,
they prayed: (1) that the proceedings in the case be declared null and void; (2) that the
appointment of petitioner as special administratrix be set aside; and (3) that Dr. Rafael
Cunanan, Sr. be appointed the regular administrator of the estate of the deceased
spouses.

Thereafter, the Cunanan heirs filed a motion requiring petitioner to submit an inventory or
accounting of all monies received by her in trust for the estate.
In her opposition, petitioner asserted: (1) that she was the "sole and only heir" of her
daughter, Dr. Evelyn Perez-Cunanan to the exclusion of the "Cunanan collaterals"; hence
they were complete strangers to the proceedings and were not entitled to notice; (2) that
she could not have "concealed" the name and address of Dr. Rafael G. Cunanan, Jr.
because his name was prominently mentioned not only in the two wills but also in the
decrees of the American surrogate court; (3) that the rule applicable to the case is Rule 77,
not Rule 76, because it involved the allowance of wills proved outside of the Philippines
and that nowhere in Section 2 of Rule 77 is there a mention of notice being given to the
executor who, by the same provision, should himself file the necessary ancillary
proceedings in this country; (4) that even if the Bulacan estate came from the "capital" of
Dr. Jose F. Cunanan, he had willed all his worldly goods to his wife and nothing to his
brothers and sisters; and (5) that Dr. Rafael G. Cunanan, Jr. had unlawfully disbursed
$215,000.00 to the Cunanan heirs, misappropriated $15,000.00 for himself and irregularly
assigned assets of the estates to his American lawyer (Records, pp. 151-160).
In their reply, the Cunanan heirs stressed that on November 24, 1982, petitioner and the
Cunanan heirs had entered into an agreement in the United States "to settle and divide
equally the estates," and that under Section 2 of Rule 77 the "court shall fix a time and
place for the hearing and cause notice thereof to be given as in case of an original will
presented for allowance" (Records, pp. 184-185).
Petitioner asked that Dr. Rafael G. Cunanan, Jr. be cited for contempt of court for failure to
comply with the Order of June 23, 1983 and for appropriating money of the estate for his
own benefit. She also alleged that she had impugned the agreement of November 24,
1982 before the Surrogate Court of Onondaga, New York which rendered a decision on
April 13, 1983, finding that "all assets are payable to Dr. Evelyn P. Cunanans executor to
be then distributed pursuant to EPTL4-1.1 subd [a] par [4]" (Rollo, p. 52).
On their part, the Cunanan heirs replied that petitioner was estopped from claiming that
they were heirs by the agreement to divide equally the estates. They asserted that by
virtue of Section 2 of Rule 77 of the Rules of Court, the provisions of Sections 3, 4 and 5 of
Rule 76 on the requirement of notice to all heirs, executors, devisees and legatees must be
complied with. They reiterated their prayer: (1) that the proceedings in the case be nullified;
(2) that petitioner be disqualified as special administratrix; (3) that she be ordered to submit
an inventory of all goods, chattels and monies which she had received and to surrender
the same to the court; and (4) that Dr. Rafael Cunanan, Sr. be appointed the regular
administrator.
Petitioner filed a rejoinder, stating that in violation of the April 13, 1983 decision of the
American court Dr. Rafael G. Cunanan, Jr. made "unauthorized disbursements from the

47 | S U C C E S S I O N

estates as early as July 7, 1982" (Records, p. 231). Thereafter, petitioner moved for the
suspension of the proceedings as she had "to attend to the settlement proceedings" of the
estate of the Cunanan spouses in New York (Records, p. 242). The Cunanans heirs
opposed this motion and filed a manifestation, stating that petitioner had received
$215,000.00 "from the Surrogates Court as part of legacy" based on the aforesaid
agreement of November 24, 1982 (Records, p. 248).
On February 21, 1984, Judge de la Llana issued an order, disallowing the reprobate of the
two wills, recalling the appointment of petitioner as special administratrix, requiring the
submission of petitioner of an inventory of the property received by her as special
administratrix and declaring all pending incidents moot and academic. Judge de la Llana
reasoned out that petitioner failed to prove the law of New York on procedure and
allowance of wills and the court had no way of telling whether the wills were executed in
accordance with the law of New York. In the absence of such evidence, the presumption is
that the law of succession of the foreign country is the same as the law of the Philippines.
However, he noted, that there were only two witnesses to the wills of the Cunanan spouses
and the Philippine law requires three witnesses and that the wills were not signed on each
and every page, a requirement of the Philippine law.
On August 27, 1985, petitioner filed a motion for reconsideration of the Order dated
February 21, 1984, where she had sufficiently proven the applicable laws of New York
governing the execution of last wills and testaments.
On the same day, Judge de la Llana issued another order, denying the motion of petitioner
for the suspension of the proceedings but gave her 15 days upon arrival in the country
within which to act on the other order issued that same day. Contending that the second
portion of the second order left its finality to the discretion of counsel for petitioner, the
Cunanans filed a motion for the reconsideration of the objectionable portion of the said
order so that it would conform with the pertinent provisions of the Judiciary Reorganization
Act of 1980 and the Interim Rules of Court.
On April 30, 1985, the respondent Judge of Branch 18 of the Regional Trial Court, Malolos,
to which the reprobate case was reassigned, issued an order stating that "(W)hen the last
will and testament . . . was denied probate," the case was terminated and therefore all
orders theretofore issued should be given finality. The same Order amended the February
21, 1984 Order by requiring petitioner to turn over to the estate the inventoried property. It
considered the proceedings for all intents and purposes, closed (Records,
p. 302).
On August 12, petitioner filed a motion to resume proceedings on account of the final
settlement and termination of the probate cases in New York. Three days later, petitioner
filed a motion praying for the reconsideration of the Order of April 30, 1985 on the strength
of the February 21, 1984 Order granting her a period of 15 days upon arrival in the country

within which to act on the denial of probate of the wills of the Cunanan spouses. On August
19, respondent Judge granted the motion and reconsidered the Order of April 30, 1985.
On August 29, counsel for petitioner, who happens to be her daughter, Natividad, filed a
motion praying that since petitioner was ailing in Fort Lee, New Jersey, U.S.A. and
therefore incapacitated to act as special administratrix, she (the counsel) should be named
substitute special administratrix. She also filed a motion for the reconsideration of the
Order of February 21, 1984, denying probate to the wills of the Cunanan spouses, alleging
that respondent Judge "failed to appreciate the significant probative value of the exhibits . .
. which all refer to the offer and admission to probate of the last wills of the Cunanan
spouses including all procedures undertaken and decrees issued in connection with the
said probate" (Records, pp. 313-323).
Thereafter, the Cunanans heirs filed a motion for reconsideration of the Order of August
19, 1985, alleging lack of notice to their counsel.
On March 31, 1986, respondent Judge to which the case was reassigned denied the
motion for reconsideration holding that the documents submitted by petitioner proved "that
the wills of the testator domiciled abroad were properly executed, genuine and sufficient to
possess real and personal property; that letters testamentary were issued; and that
proceedings were held on a foreign tribunal and proofs taken by a competent judge who
inquired into all the facts and circumstances and being satisfied with his findings issued a
decree admitting to probate the wills in question." However, respondent Judge said that the
documents did not establish the law of New York on the procedure and allowance of wills
(Records, p. 381).
On April 9, 1986, petitioner filed a motion to allow her to present further evidence on the
foreign law. After the hearing of the motion on April 25, 1986, respondent Judge issued an
order wherein he conceded that insufficiency of evidence to prove the foreign law was not
a fatal defect and was curable by adducing additional evidence. He granted petitioner 45
days to submit the evidence to that effect.

On July 18, respondent Judge denied the motion holding that to allow the probate of two
wills in a single proceeding "would be a departure from the typical and established mode of
probate where one petition takes care of one will." He pointed out that even in New York
"where the wills in question were first submitted for probate, they were dealt with in
separate proceedings" (Records, p. 395).
On August 13, 1986, petitioner filed a motion for the reconsideration of the Order of July
18, 1986, citing Section 3, Rule 2 of the Rules of Court, which provides that no party may
institute more than one suit for a single cause of action. She pointed out that separate
proceedings for the wills of the spouses which contain basically the same provisions as
they even named each other as a beneficiary in their respective wills, would go against
"the grain of inexpensive, just and speedy determination of the proceedings" (Records, pp.
405-407).
On September 11, 1986, petitioner filed a supplement to the motion for reconsideration,
citing Benigno
v. De
La
Pea,
57
Phil.
305
(1932)
(Records,
p. 411), but respondent Judge found that this pleading had been filed out of time and that
the adverse party had not been furnished with a copy thereof. In her compliance, petitioner
stated that she had furnished a copy of the motion to the counsel of the Cunanan heirs and
reiterated her motion for a "final ruling on her supplemental motion" (Records, p. 421).
On November 19, respondent Judge issued an order, denying the motion for
reconsideration filed by petitioner on the grounds that "the probate of separate wills of two
or more different persons even if they are husband and wife cannot be undertaken in a
single petition" (Records, pp. 376-378).
Hence, petitioner instituted the instant petition, arguing that the evidence offered at the
hearing of April 11, 1983 sufficiently proved the laws of the State of New York on the
allowance of wills, and that the separate wills of the Cunanan spouses need not be
probated in separate proceedings.
II

However, without waiting for petitioner to adduce the additional evidence, respondent
Judge ruled in his order dated June 20, 1986 that he found "no compelling reason to
disturb its ruling of March 31, 1986" but allowed petitioner to "file anew the appropriate
probate proceedings for each of the testator" (Records, p. 391).
The Order dated June 20, 1986 prompted petitioner to file a second motion for
reconsideration stating that she was "ready to submit further evidence on the law obtaining
in the State of New York" and praying that she be granted "the opportunity to present
evidence on what the law of the State of New York has on the probate and allowance of
wills" (Records, p. 393).

48 | S U C C E S S I O N

Petitioner contends that the following pieces of evidence she had submitted before
respondent Judge are sufficient to warrant the allowance of the wills:
(a) two certificates of authentication of the respective wills of Evelyn
and Jose by the Consulate General of the Philippines (Exhs. "F" and
"G");
(b) two certifications from the Secretary of State of New York and
Custodian of the Great Seal on the facts that Judge Bernard L. Reagan
is the Surrogate of the Country of Onondaga which is a court of record,

that his signature and seal of office are genuine, and that the Surrogate
is duly authorized to grant copy of the respective wills of Evelyn and
Jose
(Exhs. "F-1" and "G-1");
(c) two certificates of Judge Reagan and Chief Clerk Donald E. Moore
stating that they have in their records and files the said wills which were
recorded on April 7, 1982 (Exhs. "F-2" and "G-2");
(d) the respective wills of Evelyn and Jose (Exhs. "F-3", "F-6" and Exh.
"G-3" "G-6");

the decrees of probate, letters testamentary and proceedings held in


their court (Exhs. "H-6" and "I-6") (Rollo, pp. 13-16).
Petitioner adds that the wills had been admitted to probate in the Surrogate Courts
Decision of April 13, 1983 and that the proceedings were terminated on November 29,
1984.
The respective wills of the Cunanan spouses, who were American citizens, will only be
effective in this country upon compliance with the following provision of the Civil Code of
the Philippines:

(e) certificates of Judge Reagan and the Chief Clerk certifying to the
genuineness and authenticity of the exemplified copies of the two wills
(Exhs. "F-7" and "F-7");

Art. 816. The will of an alien who is abroad produces effect in the
Philippines if made with the formalities prescribed by the law of the
place in which he resides, or according to the formalities observed in
his country, or in conformity with those which this Code prescribes.

(f) two certificates of authentication from the Consulate General of the


Philippines in New York (Exh. "H" and "F").

Thus, proof that both wills conform with the formalities prescribed by New York laws or by
Philippine laws is imperative.

(g) certifications from the Secretary of State that Judge Reagan is duly
authorized to grant exemplified copies of the decree of probate, letters
testamentary and all proceedings had and proofs duly taken
(Exhs. "H-1" and "I-1");

The evidence necessary for the reprobate or allowance of wills which have been probated
outside of the Philippines are as follows: (1) the due execution of the will in accordance
with the foreign laws; (2) the testator has his domicile in the foreign country and not in the
Philippines; (3) the will has been admitted to probate in such country; (4) the fact that the
foreign tribunal is a probate court, and (5) the laws of a foreign country on procedure and
allowance of wills (III Moran Commentaries on the Rules of Court, 1970 ed., pp. 419-429;
Suntay v. Suntay, 95 Phil. 500 [1954]; Fluemer v. Hix, 54 Phil. 610 [1930]). Except for the
first and last requirements, the petitioner submitted all the needed evidence.

(h) certificates of Judge Reagan and the Chief Clerk that letters
testamentary were issued to Rafael G. Cunanan (Exhs. "H-2" and "I-2");
(i) certification to the effect that it was during the term of Judge Reagan
that a decree admitting the wills to probate had been issued and
appointing Rafael G. Cunanan as alternate executor (Exhs. "H-3" and
"I-10");
(j) the decrees on probate of the two wills specifying that proceedings
were held and proofs duly taken (Exhs. "H-4" and "I-5");
(k) decrees on probate of the two wills stating that they were properly
executed, genuine and valid and that the said instruments were
admitted to probate and established as wills valid to pass real and
personal property (Exhs. "H-5" and "I-5"); and
(l) certificates of Judge Reagan and the Chief Clerk on the genuineness
and authenticity of each others signatures in the exemplified copies of

49 | S U C C E S S I O N

The necessity of presenting evidence on the foreign laws upon which the probate in the
foreign country is based is impelled by the fact that our courts cannot take judicial notice of
them (Philippine Commercial and Industrial Bank v. Escolin, 56 SCRA 266 [1974]).
Petitioner must have perceived this omission as in fact she moved for more time to submit
the pertinent procedural and substantive New York laws but which request respondent
Judge just glossed over. While the probate of a will is a special proceeding wherein courts
should relax the rules on evidence, the goal is to receive the best evidence of which the
matter is susceptible before a purported will is probated or denied probate (Vda. de Ramos
v. Court of Appeals, 81 SCRA 393 [1978]).
There is merit in petitioners insistence that the separate wills of the Cunanan spouses
should be probated jointly. Respondent Judges view that the Rules on allowance of wills is
couched in singular terms and therefore should be interpreted to mean that there should
be separate probate proceedings for the wills of the Cunanan spouses is too literal and

simplistic an approach. Such view overlooks the provisions of Section 2, Rule 1 of the
Revised Rules of Court, which advise that the rules shall be "liberally construed in order to
promote their object and to assist the parties in obtaining just, speedy, and inexpensive
determination of every action and proceeding."

the wills of the Cunanan spouses and see to it that the brothers and sisters of Dr. Jose F.
Cunanan are given all notices and copies of all pleadings pertinent to the probate
proceedings.
SO ORDERED.

A literal application of the Rules should be avoided if they would only result in the delay in
the administration of justice (Acain v. Intermediate Appellate Court, 155 SCRA 100 [1987];
Roberts v. Leonidas, 129 SCRA 33 [1984]).
What the law expressly prohibits is the making of joint wills either for the testators
reciprocal benefit or for the benefit of a third person (Civil Code of the Philippines, Article
818). In the case at bench, the Cunanan spouses executed separate wills. Since the two
wills contain essentially the same provisions and pertain to property which in all probability
are conjugal in nature, practical considerations dictate their joint probate. As this Court has
held a number of times, it will always strive to settle the entire controversy in a single
proceeding leaving no root or branch to bear the seeds of future litigation (Motoomull v.
Dela Paz, 187 SCRA 743 [1990]).
This petition cannot be completely resolved without touching on a very glaring fact
petitioner
has
always
considered
herself
the
sole
heir
of
Dr. Evelyn Perez Cunanan and because she does not consider herself an heir of Dr. Jose
F. Cunanan, she noticeably failed to notify his heirs of the filing of the proceedings. Thus,
even in the instant petition, she only impleaded respondent Judge, forgetting that a judge
whose order is being assailed is merely a nominal or formal party (Calderon v. Solicitor
General, 215 SCRA 876 [1992]).
The rule that the court having jurisdiction over the reprobate of a will shall "cause notice
thereof to be given as in case of an original will presented for allowance" (Revised Rules of
Court, Rule 27, Section 2) means that with regard to notices, the will probated abroad
should be treated as if it were an "original will" or a will that is presented for probate for the
first time. Accordingly, compliance with Sections 3 and 4 of Rule 76, which require
publication and notice by mail or personally to the "known heirs, legatees, and devisees of
the testator resident in the Philippines" and to the executor, if he is not the petitioner, are
required.
The brothers and sisters of Dr. Jose F. Cunanan, contrary to petitioner's claim, are entitled
to notices of the time and place for proving the wills. Under Section 4 of Rule 76 of the
Revised Rules of Court, the "court shall also cause copies of the notice of the time and
place fixed for proving the will to be addressed to the designated or other known heirs,
legatees, and devisees of the testator, . . . "
WHEREFORE, the questioned Order is SET ASIDE. Respondent Judge shall allow
petitioner reasonable time within which to submit evidence needed for the joint probate of

50 | S U C C E S S I O N

IN RE: IN THE MATTER OF THE G.R. No. 169144


PETITION TO APPROVE THE WILL
OF RUPERTA PALAGANAS WITH
PRAYER FOR THE APPOINTMENT
OF SPECIAL ADMINISTRATOR,
MANUEL MIGUEL PALAGANAS and
BENJAMIN GREGORIO PALAGANAS,
Petitioners, Present:
CARPIO, J., Chairperson,
- versus - NACHURA,
ABAD,
DECISION
ABAD, J.:
This case is about the probate before Philippine court of a will executed abroad by a
foreigner although it has not been probated in its place of execution.

The Facts and the Case


On November 8, 2001 Ruperta C. Palaganas (Ruperta), a Filipino who became a
naturalized United States (U.S.) citizen, died single and childless. In the last will and
testament she executed in California, she designated her brother, Sergio C. Palaganas
(Sergio), as the executor of her will for she had left properties in the Philippines and in
the U.S.
On May 19, 2003 respondent Ernesto C. Palaganas (Ernesto), another brother of Ruperta,
filed with the Regional Trial Court (RTC) of Malolos, Bulacan, a petition for the probate of
Rupertas will and for his appointment as special administrator of her estate. [1] On October
15, 2003, however, petitioners Manuel Miguel Palaganas (Manuel) and Benjamin Gregorio
Palaganas (Benjamin), nephews of Ruperta, opposed the petition on the ground that
Rupertas will should not be probated in the Philippines but in the U.S.where she executed
it. Manuel and Benjamin added that, assuming Rupertas will could be probated in
the Philippines, it is invalid nonetheless for having been executed under duress and
without the testators full understanding of the consequences of such act. Ernesto, they
claimed, is also not qualified to act as administrator of the estate.
Meantime, since Rupertas foreign-based siblings, Gloria Villaluz and Sergio, were on
separate occasions in the Philippines for a short visit, respondent Ernesto filed a motion
with the RTC for leave to take their deposition, which it granted. On April, 13, 2004 the

RTC directed the parties to submit their memorandum on the issue of whether or not
Rupertas U.S. will may be probated in and allowed by a court in the Philippines.

court having jurisdiction to have the will allowed, whether the same be in his possession or
not, or is lost or destroyed.

On June 17, 2004 the RTC issued an order: [2] (a) admitting to probate Rupertas last will;
(b) appointing respondent Ernesto as special administrator at the request of Sergio, the
U.S.-based executor designated in the will; and (c) issuing the Letters of Special
Administration to Ernesto.

Our rules require merely that the petition for the allowance of a will must show, so far as
known to the petitioner: (a) the jurisdictional facts; (b) the names, ages, and residences of
the heirs, legatees, and devisees of the testator or decedent; (c) the probable value and
character of the property of the estate; (d) the name of the person for whom letters are
prayed; and (e) if the will has not been delivered to the court, the name of the person
having custody of it. Jurisdictional facts refer to the fact of death of the decedent, his
residence at the time of his death in the province where the probate court is sitting, or if he
is an inhabitant of a foreign country, the estate he left in such province. [7] The rules do not
require proof that the foreign will has already been allowed and probated in the country of
its execution.

Aggrieved by the RTCs order, petitioner nephews Manuel and Benjamin appealed to the
Court of Appeals (CA),[3] arguing that an unprobated will executed by an American citizen
in the U.S. cannot be probated for the first time in the Philippines.
On July 29, 2005 the CA rendered a decision, [4] affirming the assailed order of the RTC,
[5]
holding that the RTC properly allowed the probate of the will, subject to respondent
Ernestos submission of the authenticated copies of the documents specified in the order
and his posting of required bond. The CA pointed out that Section 2, Rule 76 of the Rules
of Court does not require prior probate and allowance of the will in the country of its
execution, before it can be probated in the Philippines. The present case, said the CA, is
different from reprobate, which refers to a will already probated and allowed
abroad. Reprobate is governed by different rules or procedures. Unsatisfied with the
decision, Manuel and Benjamin came to this Court.
The Issue Presented
The key issue presented in this case is whether or not a will executed by a foreigner
abroad may be probated in the Philippines although it has not been previously probated
and allowed in the country where it was executed.
The Courts Ruling
Petitioners Manuel and Benjamin maintain that wills executed by foreigners abroad must
first be probated and allowed in the country of its execution before it can be probated
here. This, they claim, ensures prior compliance with the legal formalities of the country of
its execution. They insist that local courts can only allow probate of such wills if the
proponent proves that: (a) the testator has been admitted for probate in such foreign
country, (b) the will has been admitted to probate there under its laws, (c) the probate court
has jurisdiction over the proceedings, (d) the law on probate procedure in that foreign
country and proof of compliance with the same, and (e) the legal requirements for the valid
execution of a will.
But our laws do not prohibit the probate of wills executed by foreigners abroad although
the same have not as yet been probated and allowed in the countries of their execution. A
foreign will can be given legal effects in our jurisdiction. Article 816 of the Civil Code states
that the will of an alien who is abroad produces effect in the Philippines if made in
accordance with the formalities prescribed by the law of the place where he resides, or
according to the formalities observed in his country.[6]
In this connection, Section 1, Rule 73 of the 1997 Rules of Civil Procedure provides that if
the decedent is an inhabitant of a foreign country, the RTC of the province where he has
an estate may take cognizance of the settlement of such estate. Sections 1 and 2 of Rule
76 further state that the executor, devisee, or legatee named in the will, or any other
person interested in the estate, may, at any time after the death of the testator, petition the

51 | S U C C E S S I O N

In insisting that Rupertas will should have been first probated and allowed by the court of
California, petitioners Manuel and Benjamin obviously have in mind the procedure for
the reprobate of will before admitting it here. But, reprobate or re-authentication of a will
already probated and allowed in a foreign country is different from that probate where the
will is presented for the first time before a competent court. Reprobate is specifically
governed by Rule 77 of the Rules of Court. Contrary to petitioners stance, since this latter
rule applies only to reprobate of a will, it cannot be made to apply to the present case. In
reprobate, the local court acknowledges as binding the findings of the foreign probate court
provided its jurisdiction over the matter can be established.
Besides, petitioners stand is fraught with impractically. If the instituted heirs do not have
the means to go abroad for the probate of the will, it is as good as depriving them outright
of their inheritance, since our law requires that no will shall pass either real or personal
property unless the will has been proved and allowed by the proper court. [8]
Notably, the assailed RTC order of June 17, 2004 is nothing more than an initial ruling that
the court can take cognizance of the petition for probate of Rupertas will and that, in the
meantime, it was designating Ernesto as special administrator of the estate. The parties
have yet to present evidence of the due execution of the will, i.e. the testators state of mind
at the time of the execution and compliance with the formalities required of wills by the
laws of California. This explains the trial courts directive for Ernesto to submit the duly
authenticated copy of Rupertas will and the certified copies of the Laws of Succession and
Probate of Will of California.
WHEREFORE, the Court DENIES the petition and AFFIRMS the Court of Appeals
decision in CA-G.R. CV 83564 dated July 29, 2005.
SO ORDERED.

G.R. No. L-12767

November 16, 1918

In the matter of the estate of EMIL H. JOHNSON. EBBA INGEBORG


JOHNSON, applicant-appellant,
STREET, J.:

On February 4, 1916, Emil H. Johnson, a native of Sweden and a naturalized citizen of the
United States, died in the city of Manila, leaving a will, dated September 9, 1915, by which
he disposed of an estate, the value of which, as estimated by him, was P231,800. This
document is an holographic instrument, being written in the testator's own handwriting, and
is signed by himself and two witnesses only, instead of three witnesses required by section
618 of the Code of Civil Procedure. This will, therefore, was not executed in conformity
with the provisions of law generally applicable to wills executed by inhabitants of these
Islands, and hence could not have been proved under section 618.
On February 9, 1916, however, a petition was presented in the Court of First Instance of
the city of Manila for the probate of this will, on the ground that Johnson was at the time of
his death a citizen of the State of Illinois, United States of America; that the will was duly
executed in accordance with the laws of that State; and hence could properly be probated
here pursuant to section 636 of the Code of Civil Procedure. This section reads as follows:
Will made here by alien. A will made within the Philippine Islands by a citizen
or subject of another state or country, which is executed in accordance with the
law of the state or country of which he is a citizen or subject, and which might be
proved and allowed by the law of his own state or country, may be proved,
allowed, and recorded in the Philippine Islands, and shall have the same effect
as if executed according to the laws of these Islands.
The hearing on said application was set for March 6, 1916, and three weeks publication of
notice was ordered in the "Manila Daily Bulletin." Due publication was made pursuant to
this order of the court. On March 6, 1916, witnesses were examined relative to the
execution of the will; and upon March 16th thereafter the document was declared to be
legal and was admitted to probate. At the same time an order was made nominating Victor
Johnson and John T. Pickett as administrators of the estate, with the sill annexed. Shortly
thereafter Pickett signified his desire not to serve, and Victor Johnson was appointed sole
administrator.
By the will in question the testator gives to his brother Victor one hundred shares of the
corporate stock in the Johnson-Pickett Rope Company; to his father and mother in
Sweden, the sum of P20,000; to his daughter Ebba Ingeborg, the sum of P5,000; to his
wife, Alejandra Ibaez, the sum of P75 per month, if she remains single; to Simeona
Ibaez, spinster, P65 per month, if she remains single. The rest of the property is left to the
testator's five children Mercedes, Encarnacion, Victor, Eleonor and Alberto.
The biographical facts relative to the deceased necessary to an understanding of the case
are these: Emil H. Johnson was born in Sweden, May 25, 1877, from which country he
emigrated to the United States and lived in Chicago, Illinois, from 1893 to 1898. On May 9,
1898, at Chicago, he was married to Rosalie Ackeson, and immediately thereafter
embarked for the Philippine Islands as a soldier in the Army of the United States. As a

52 | S U C C E S S I O N

result of relations between Johnson and Rosalie Ackeson a daughter, named Ebba
Ingeborg, was born a few months after their marriage. This child was christened in Chicago
by a pastor of the Swedish Lutheran Church upon October 16, 1898.
After Johnson was discharged as a soldier from the service of the United States he
continued to live in the Philippine Islands, and on November 20, 1902, the wife, Rosalie
Johnson, was granted a decree of divorce from him in the Circuit Court of Cook County,
Illinois, on the ground of desertion. A little later Johnson appeared in the United States on a
visit and on January 10, 1903, procured a certificate of naturalization at Chicago. From
Chicago he appears to have gone to Sweden, where a photograph, exhibited in evidence
in this case, was taken in which he appeared in a group with his father, mother, and the
little daughter, Ebba Ingeborg, who was then living with her grandparents in Sweden.
When this visit was concluded, the deceased returned to Manila, where he prospered in
business and continued to live until his death.
In this city he appears to have entered into marital relations with Alejandra Ibaez, by
whom he had three children, to wit, Mercedes, baptized May 31, 1903; Encarnacion,
baptized April 29, 1906; and Victor, baptized December 9, 1907. The other two children
mentioned in the will were borne to the deceased by Simeona Ibaez.
On June 12, 1916, or about three months after the will had been probated, the attorneys
for Ebba Ingeborg Johnson entered an appearance in her behalf and noted an exception to
the other admitting the will to probate. On October 31, 1916, the same attorneys moved
the court to vacate the order of March 16 and also various other orders in the case. On
February 20, 1917, this motion was denied, and from this action of the trial court the
present appeal has been perfected.
As will be discerned, the purpose of the proceeding on behalf of the petitioner is to annul
the decree of probate and put the estate into intestate administration, thus preparing the
way for the establishment of the claim of the petitioner as the sole legitimate heir of her
father.
The grounds upon which the petitioner seeks to avoid the probate are four in number and
may be stated, in the same sequence in which they are set forth in the petition, as follows:
(1) Emil H. Johnson was a resident of the city of Manila and not a resident of the State of
Illinois at the time the will in question was executed;
(2) The will is invalid and inadequate to pass real and personal property in the State of
Illinois;
(3) The order admitting the will to probate was made without notice to the petitioner; and

(4) The order in question was beyond the jurisdiction of the court.
It cannot of course be maintained that a court of first instance lacks essential jurisdiction
over the probate of wills. The fourth proposition above stated must, accordingly, be
interpreted in relation with the third and must be considered as a corollary deduced from
the latter. Moreover, both the third and fourth grounds stated take precedence, by reason
of their more fundamental implications, over the first two; and a logical exposition of the
contentions of the petitioner is expressed in the two following propositions:
(I) The order admitting the will to probate was beyond the jurisdiction of the court
and void because made without notice to the petitioner;
(II) The judgment from which the petitioner seeks relief should be set aside
because the testator was not a resident of the State of Illinois and the will was
not in conformity with the laws of that State.
In the discussion which is to follow we shall consider the problems arising in this cae in the
order last above indicated. Upon the question, then, of the jurisdiction of the court, it is
apparent from an inspection of the record of the proceedings in the court below that all the
steps prescribed by law as prerequisites to the probate of a will were complied with in
every respect and that the probate was effected in external conformity with all legal
requirements. This much is unquestioned. It is, however, pointed out in the argument
submitted in behalf of the petitioner, that, at the time the court made the order of
publication, it was apprised of the fact that the petitioner lived in the United States and that
as daughter and heir she was necessarily interested in the probate of the will. It is,
therefore, insisted that the court should have appointed a date for the probate of the will
sufficiently far in the future to permit the petitioner to be present either in person or by
representation; and it is said that the failure of the court thus to postpone the probate of the
will constitutes an infringement of that provision of the Philippine Bill which declared that
property shall not be taken without due process of law.
On this point we are of the opinion that the proceedings for the probate of the will were
regular and that the publication was sufficient to give the court jurisdiction to entertain the
proceeding and to allow the will to be probated.
As was said in the case of In re Davis (136 Cal., 590, 596), "the proceeding as to the
probate of a will is essentially one in rem, and in the very nature of things the state is
allowed a wide latitude in determining the character of the constructive notice to be given
to the world in a proceeding where it has absolute possession of the res. It would be an
exceptional case where a court would declare a statute void, as depriving a party of his
property without due process of law, the proceeding being strictly in rem, and the res within
the state, upon the ground that the constructive notice prescribed by the statute was
unreasonably short."

53 | S U C C E S S I O N

In that case the petitioner had been domiciled in the Hawaiian Islands at the time of the
testator's death; and it was impossible, in view of the distance and means of
communication then existing, for the petitioner to appear and oppose the probate on the
day set for the hearing in California. It was nevertheless held that publication in the manner
prescribed by statute constituted due process of law. (See Estate of Davis, 151 Cal., 318;
Tracy vs.Muir, 151 Cal., 363.)
In the Davis case (136 Cal., 590) the court commented upon the fact that, under the laws
of California, the petitioner had a full year within which she might have instituted a
proceeding to contest the will; and this was stated as one of the reasons for holding that
publication in the manner provided by statute was sufficient. The same circumstance was
commented upon in O'Callaghan vs. O'Brien (199 U. S., 89), decided in the Supreme
Court of the United States. This case arose under the laws of the State of Washington, and
it was alleged that a will had been there probated without the notice of application for
probate having been given as required by law. It was insisted that this was an infringement
of the Fourteenth Amendment of the Constitution of the United States. This contention
was, however, rejected and it was held that the statutory right to contest the will within a
year was a complete refutation of the argument founded on the idea of a violation of the
due process provision.
The laws of these Islands, in contrast with the laws in force in perhaps all of the States of
the American Union, contain no special provision, other than that allowing an appeal in the
probate proceedings, under which relief of any sort can be obtained from an order of a
court of first instance improperly allowing or disallowing a will. We do, however, have a
provision of a general nature authorizing a court under certain circumstances to set aside
any judgment, order, or other proceeding whatever. This provision is found in section 113
of the Code of Civil Procedure, which reads as follows:
Upon such terms as may be just the court may relieve a party or his legal
representative from a judgment, order or other proceeding taken against him
through his mistake, inadvertence, surprise or excusable neglect; Provided, That
application therefor be made within a reasonable time, but in no case exceeding
six months after such judgment, order, or proceeding was taken.
The use of the word "judgment, order or other proceeding" in this section indicates an
intention on the part of the Legislature to give a wide latitude to the remedy here provided,
and in our opinion its operation is not to be restricted to judgments or orders entered in
ordinary contentious litigation where a plaintiff impleads a defendant and brings him into
court by personal service of process. In other words the utility of the provision is not limited
to actions proper but extends to all sorts of judicial proceedings.
In the second section of the Code of Civil Procedure it is declared that the provisions of
this Code shall be liberally construed to promote its object and to assist the parties in

obtaining speedy justice. We think that the intention thus exhibited should be applied in the
interpretation of section 113; and we hold that the word "party," used in this section, means
any person having an interest in the subject matter of the proceeding who is in a position to
be concluded by the judgment, order, to other proceeding taken.
The petitioner, therefore, in this case could have applied, under the section cited, at any
time within six months for March 16, 1916, and upon showing that she had been precluded
from appearing in the probate proceedings by conditions over which she had no control
and that the order admitting the will to probate had been erroneously entered upon
insufficient proof or upon a supposed state of facts contrary to the truth, the court would
have been authorized to set the probate aside and grant a rehearing. It is no doubt true
that six months was, under the circumstances, a very short period of time within which to
expect the petitioner to appear and be prepared to contest the probate with the proof which
she might have desired to collect from remote countries. Nevertheless, although the time
allowed for the making of such application was inconveniently short, the remedy existed;
and the possibility of its use is proved in this case by the circumstance that on June 12,
1916, she in fact here appeared in court by her attorneys and excepted to the order
admitting the will to probate.
It results that, in conformity with the doctrine announced in the Davis case, above cited, the
proceedings in the court below were conducted in such manner as to constitute due
process of law. The law supplied a remedy by which the petitioner might have gotten a
hearing and have obtained relief from the order by which she is supposed to have been
injured; and though the period within which the application should have been made was
short, the remedy was both possible and practicable.
From what has been said it follows that the order of March 16, 1916, admitting the will of
Emil H. Johnson to probate cannot be declared null and void merely because the petitioner
was unavoidably prevented from appearing at the original hearing upon the matter of the
probate of the will in question. Whether the result would have been the same if our system
of procedure had contained no such provision as that expressed in section 113 is a matter
which we need not here consider.
Intimately connected with the question of the jurisdiction of the court, is another matter
which may be properly discussed at this juncture. This relates to the interpretation to be
placed upon section 636 of the Code of Civil Procedure. The position is taken by the
appellant that this section is applicable only to wills of liens; and in this connection attention
is directed to the fact that the epigraph of this section speaks only of the will made here by
an alien and to the further fact that the word "state" in the body of the section is not
capitalized. From this it is argued that section 636 is not applicable to the will of a citizen of
the United States residing in these Islands.lawphil.net

We consider these suggestions of little weight and are of the opinion that, by the most
reasonable interpretation of the language used in the statute, the words "another state or
country" include the United States and the States of the American Union, and that the
operation of the statute is not limited to wills of aliens. It is a rule of hermeneutics that
punctuation and capitalization are aids of low degree in interpreting the language of a
statute and can never control against the intelligible meaning of the written words.
Furthermore, the epigraph, or heading,, of a section, being nothing more than a convenient
index to the contents of the provision, cannot have the effect of limiting the operative words
contained in the body of the text. It results that if Emil H. Johnson was at the time of his
death a citizen of the United States and of the State of Illinois, his will was provable under
this section in the courts of the Philippine Islands, provided the instrument was so
executed as to be admissible to probate under the laws of the State of Illinois.
We are thus brought to consider the second principal proposition stated at the outset of
this discussion, which raises the question whether the order f probate can be set aside in
this proceeding on the other ground stated in the petition, namely, that the testator was not
a resident of the State of Illinois and that the will was not made in conformity with the laws
of that State.
The order of the Court of First Instance admitting the will to probate recites, among other
things:
That upon the date when the will in question was executed Emil H. Johnson was
a citizen of the United States, naturalized in the State of Illinois, County of Cook,
and that the will in question was executed in conformity with the dispositions of
the law f the State of Illinois.
We consider this equivalent to a finding that upon the date of the execution of the will the
testator was a citizen of the State of Illinois and that the will was executed in conformity
with the laws of that State. Upon the last point the finding is express; and in our opinion the
statement that the testator was a citizen of the United States, naturalized in the State of
Illinois, should be taken to imply that he was a citizen of the State of Illinois, as well as of
the United States.
The naturalization laws of the United States require, as a condition precedent to the
granting of the certificate of naturalization, that the applicant should have resided at least
five years in the United States and for one year within the State or territory where the court
granting the naturalization papers is held; and in the absence of clear proof to the contrary
it should be presumed that a person naturalized in a court of a certain State thereby
becomes a citizen of that State as well as of the United States.
In this connection it should be remembered that the Fourteenth Amendment to the
Constitution of the United States declares, in its opening words, that all persons

54 | S U C C E S S I O N

naturalized in the United States, and subject to the jurisdiction thereof, are citizens of the
United States and of the State wherein they reside.
It is noteworthy that the petition by which it is sought to annul the probate of this will does
not assert that the testator was not a citizen of Illinois at the date when the will was
executed. The most that is said on this point is he was "never a resident of the State of
Illinois after the year 1898, but became and was a resident of the city of Manila," etc. But
residence in the Philippine Islands is compatible with citizenship in Illinois; and it must be
considered that the allegations of the petition on this point are, considered in their bearing
as an attempt to refute citizenship in Illinois, wholly insufficient.
As the Court of First Instance found that the testator was a citizen of the State of Illinois
and that the will was executed in conformity with the laws of that State, the will was
necessarily and properly admitted to probate. And how is it possible to evade the effect of
these findings?
In Section 625 of the Code of Civil Procedure it is declared that "the allowance by the court
of a will of real or personal property shall be conclusive as to its due execution."
The due execution of a will involves conditions relating to a number of matters, such as the
age and mental capacity of the testator, the signing of the document by the testator, or by
someone in his behalf, and the acknowledgment of the instrument by him in the presence
of the required number of witnesses who affix their signatures to the will to attest the act.
The proof of all these requisites is involved in the probate; and as to each and all of them
the probate is conclusive. (Castaeda vs. Alemany, 3 Phil. Rep., 426;
Pimentel vs. Palanca, 5 Phil. Rep., 436; Chiong Joc-Soy vs. Vao, 8 Phil. Rep., 119;
Sanchez vs. Pascual, 11 Phil. Rep., 395; Montaano vs.Suesa, 14 Phil. Rep., 676.)
Our reported cases do not contain the slightest intimation that a will which has been
probated according to law, and without fraud, can be annulled, in any other proceeding
whatever, on account of any supposed irregularity or defect in the execution of the will or
on account of any error in the action of the court upon the proof adduced before it. This
court has never been called upon to decide whether, in case the probate of a will should be
procured by fraud, relief could be granted in some other proceeding; and no such question
is now presented. But it is readily seen that if fraud were alleged, this would introduce an
entirely different factor in the cae. In Austruavs. Ventenilla (21 Phil. Rep., 180, 184), it was
suggested but not decided that relief might be granted in case the probate of a will were
procured by fraud.
The circumstance that the judgment of the trial court recites that the will was executed in
conformity with the law of Illinois and also, in effect, that the testator was a citizen of that
State places the judgment upon an unassailable basis so far as any supposed error
apparent upon the fact of the judgment is concerned. It is, however, probable that even if

55 | S U C C E S S I O N

the judgment had not contained these recitals, there would have been a presumption from
the admission of the will to probate as the will of a citizen of Illinois that the facts were as
recited in the order of probate.
As was said by this court in the case of Banco Espaol-Filipino vs. Palanca (37 Phil. Rep.,
921), "There is no principle of law better settled than that after jurisdiction has once been
acquired, every act of a court of general jurisdiction shall be presumed to have been rightly
done. This rule is applied to every judgment or decree rendered in the various stages of
the proceedings from their initiation to their completion (Voorhees vs. United States Bank,
10 Pet., 314; 35 U. S., 449); and if the record is silent with respect to any fact which must
have established before the court could have rightly acted, it will be presumed that such
fact was properly brought to its knowledge."
The Court of First Instance is a court of original and general jurisdiction; and there is no
difference in its faculties in this respect whether exercised in matters of probate or exerted
in ordinary contentious litigation. The trial court therefore necessarily had the power to
determine the facts upon which the propriety of admitting the will to probate depended; and
the recital of those facts in the judgment was probably not essential to its validity. No
express ruling is, however, necessary on this point.
What has been said effectually disposes of the petition considered in its aspect as an
attack upon the order of probate for error apparent on the face of the record. But the
petitioner seeks to have the judgment reviewed, it being asserted that the findings of the
trial court especially on the question of the citizenship of the testator are not
supported by the evidence. It needs but a moment's reflection, however, to show that in
such a proceeding as this it is not possible to reverse the original order on the ground that
the findings of the trial court are unsupported by the proof adduced before that court. The
only proceeding in which a review of the evidence can be secured is by appeal, and the
case is not before us upon appeal from the original order admitting the will to probate. The
present proceedings by petition to set aside the order of probate, and the appeal herein is
from the order denying this relief. It is obvious that on appeal from an order refusing to
vacate a judgment it is not possible to review the evidence upon which the original
judgment was based. To permit this would operate unduly to protract the right of appeal.
However, for the purpose of arriving at a just conception of the case from the point of view
of the petitioner, we propose to examine the evidence submitted upon the original hearing,
in connection with the allegations of the petition, in order to see, first, whether the evidence
submitted to the trial court was sufficient to justify its findings, and, secondly, whether the
petition contains any matter which would justify the court in setting the judgment, aside. In
this connection we shall for a moment ignore the circumstance that the petition was filed
after the expiration of the six months allowed by section 113 of the Code of Civil
Procedure.

The principal controversy is over the citizenship of the testator. The evidence adduced
upon this point in the trial court consists of the certificate of naturalization granted upon
January 10, 1903, in the Circuit Court of Cook County, Illinois, in connection with certain
biographical facts contained in the oral evidence. The certificate of naturalization supplies
incontrovertible proof that upon the date stated the testator became a citizen of the United
States, and inferentially also a citizen of said State. In the testimony submitted to the trial
court it appears that, when Johnson first came to the United States as a boy, he took up
his abode in the State of Illinois and there remained until he came as a soldier in the
United States Army to the Philippine Islands. Although he remained in these Islands for
sometime after receiving his discharge, no evidence was adduced showing that at the time
he returned to the United States, in the autumn of 1902, he had then abandoned Illinois as
the State of his permanent domicile, and on the contrary the certificate of naturalization
itself recites that at that time he claimed to be a resident of Illinois.
Now, if upon January 10, 1903, the testator became a citizen of the United States and of
the State of Illinois, how has he lost the character of citizen with respect to either of these
jurisdictions? There is no law in force by virtue of which any person of foreign nativity can
become a naturalized citizen of the Philippine Islands; and it was, therefore, impossible for
the testator, even if he had so desired, to expatriate himself from the United States and
change his political status from a citizen of the United States to a citizen of these Islands.
This being true, it is to be presumed that he retained his citizenship in the State of Illinois
along with his status as a citizen of the United States. It would be novel doctrine to
Americans living in the Philippine Islands to be told that by living here they lose their
citizenship in the State of their naturalization or nativity.
We are not unmindful of the fact that when a citizen of one State leaves it and takes up his
abode in another State with no intention of returning, he immediately acquires citizenship
in the State of his new domicile. This is in accordance with that provision of the Fourteenth
Amendment to the Constitution of the United States which says that every citizen of the
United States is a citizen of the State where in he resides. The effect of this provision
necessarily is that a person transferring his domicile from one State to another loses his
citizenship in the State of his original above upon acquiring citizenship in the State of his
new abode. The acquisition of the new State citizenship extinguishes the old. That
situation, in our opinion, has no analogy to that which arises when a citizen of an American
State comes to reside in the Philippine Islands. Here he cannot acquire a new citizenship;
nor by the mere change of domicile does he lose that which he brought with him.
The proof adduced before the trial court must therefore be taken as showing that, at the
time the will was executed, the testator was, as stated in the order of probate, a citizen of
the State of Illinois. This, in connection with the circumstance that the petition does not
even so much as deny such citizenship but only asserts that the testator was a resident of
the Philippine Islands, demonstrates the impossibility of setting the probate aside for lack
of the necessary citizenship on the part of the testator. As already observed, the allegation
of the petition on this point is wholly insufficient to justify any relief whatever.

56 | S U C C E S S I O N

Upon the other point as to whether the will was executed in conformity with the statutes
of the State of Illinois we note that it does not affirmatively appear from the transaction
of the testimony adduced in the trial court that any witness was examined with reference to
the law of Illinois on the subject of the execution of will. The trial judge no doubt was
satisfied that the will was properly executed by examining section 1874 of the Revised
Statutes of Illinois, as exhibited in volume 3 of Starr & Curtis's Annotated Illinois Statutes,
2nd ed., p. 426; and he may have assumed that he could take judicial notice of the laws of
Illinois under section 275 of the Code of Civil Procedure. If so, he was in our opinion
mistaken. that section authorizes the courts here to take judicial notice, among other
things, of the acts of the legislative department of the United States. These words clearly
have reference to Acts of the Congress of the United States; and we would hesitate to hold
that our courts can, under this provision, take judicial notice of the multifarious laws of the
various American States. Nor do we think that any such authority can be derived from the
broader language, used in the same action, where it is said that our courts may take
judicial notice of matters of public knowledge "similar" to those therein enumerated. The
proper rule we think is to require proof of the statutes of the States of the American Union
whenever their provisions are determinative of the issues in any action litigated in the
Philippine courts.
Nevertheless, even supposing that the trial court may have erred in taking judicial notice of
the law of Illinois on the point in question, such error is not now available to the petitioner,
first, because the petition does not state any fact from which it would appear that the law of
Illinois is different from what the court found, and, secondly, because the assignment of
error and argument for the appellant in this court raises no question based on such
supposed error. Though the trial court may have acted upon pure conjecture as to the law
prevailing in the State of Illinois, its judgment could not be set aside, even upon application
made within six months under section 113 of the Code of Civil procedure, unless it should
be made to appear affirmatively that the conjecture was wrong. The petitioner, it is true,
states in general terms that the will in question is invalid and inadequate to pass real and
personal property in the State of Illinois, but this is merely a conclusion of law. The
affidavits by which the petition is accompanied contain no reference to the subject, and we
are cited to no authority in the appellant's brief which might tent to raise a doubt as to the
correctness of the conclusion of the trial court. It is very clear, therefore, that this point
cannot be urged as of serious moment.
But it is insisted in the brief for the appellant that the will in question was not properly
admissible to probate because it contains provisions which cannot be given effect
consistently with the laws of the Philippine Islands; and it is suggested that as the
petitioner is a legitimate heir of the testator she cannot be deprived of the legitime to which
she is entitled under the law governing testamentary successions in these Islands. Upon
this point it is sufficient to say that the probate of the will does not affect the intrinsic validity
of its provisions, the decree of probate being conclusive only as regards the due execution
of the will. (Code of Civil Procedure, secs. 625, 614; Sahagun vs. De Gorostiza, 7 Phil.

Rep., 347, 349; Chiong Joc-Soy vs. Vao, 8 Phil. Rep., 119, 121; Limjuco vs.Ganara, 11
Phil. Rep., 393, 395.)
If, therefore, upon the distribution of this estate, it should appear that any legacy given by
the will or other disposition made therein is contrary to the law applicable in such case, the
will must necessarily yield upon that point and the law must prevail. Nevertheless, it should
not be forgotten that the intrinsic validity of the provisions of this will must be determined by
the law of Illinois and not, as the appellant apparently assumes, by the general provisions
here applicable in such matters; for in the second paragraph of article 10 of the Civil Code
it is declared that "legal and testamentary successions, with regard to the order of
succession, as well as to the amount of the successional rights and to the intrinsic validity
of their provisions, shall be regulated by the laws of the nation of the person whose
succession is in question, whatever may be the nature of the property and the country
where it may be situate."
From what has been said, it is, we think, manifest that the petition submitted to the court
below on October 31, 1916, was entirely insufficient to warrant the setting aside of the
other probating the will in question, whether said petition be considered as an attack on the
validity of the decree for error apparent, or whether it be considered as an application for a
rehearing based upon the new evidence submitted in the affidavits which accompany the
petition. And in this latter aspect the petition is subject to the further fatal defect that it was
not presented within the time allowed by law.
It follows that the trial court committed no error in denying the relief sought. The order
appealed from is accordingly affirmed with costs. So ordered.
Torres, Johnson, Malcolm, Avancea and Fisher, JJ., concur.

G.R. No. L-2200

August 2, 1950

In re Will of Victor Bilbao. RAMON N. BILBAO, petitioner-appellant,


vs.
DALMACIO BILBAO, CLEOFAS BILBAO, EUSEBIA BILBAO, CATALINA
BILBAO, FILEMON ABRINGE and FRANCISCO ACADEMIA, oppositorsappellee.
MONTEMAYOR, J.:
This is an appeal from a decision of the Court of First Instance of Negros Oriental denying
the petition for admission to probate of the last will and testament of Victor S. Bilbao who
died on July 13, 1943, which petition was filed by his widow and cotestator Ramona M.
Navarro.

57 | S U C C E S S I O N

The will in question was executed on October 6, 1931, on a single page or sheet by the
deceased Victor Bilbao jointly with his wife Ramona M. Navarro. The two testators in their
testament directed that "all of our respective private properties both real and personal, and
all of our conjugal properties, and any other property belonging to either or both of us, be
given and transmitted to anyone or either of us, who may survive the other, or who may
remain the surviving spouse of the other."
The petition for probate was opposed by one Filemon Abringe, a near relative of the
deceased, among other grounds, that the alleged will was executed by the husband and
wife for their reciprocal benefit and therefore not valid, and that it was not executed and
attested to as required by law. After hearing, the trial court found the will to have been
executed conjointly by the deceased husband and wife for their reciprocal benefit, and that
a will of that kind is neither contemplated by Act No. 190, known as the Code of Civil
Procedure nor permitted by article 669 of the Civil Code which provides:
Two or more persons cannot make a will conjointly or in the same instrument,
either for their reciprocal benefit or for the benefit of a third person.
The only assignment of error made in the appeal is that "the lower court erred in not finding
that a joint and reciprocal will particularly between husband and wife is valid under the
present law." The thesis of the appellant is, that "Chapter XXXI, particularly sections 614,
618, Act 190, appears to be a complete enactment on the subject of execution of wills and
may thus be regarded as the expression of the whole law thereon, and that it must be
deemed to have impliedly repealed the provision of the Civil Code (Title III, Chapter I) on
the matter ;" that inasmuch as the present law on wills as embodied in the Code of Civil
Procedure has been taken from American law, it should be interpreted in accordance with
the said law, and because joint and reciprocal wills are neither regarded as invalid nor on
the contrary they are allowed, then article 669 of the Civil Code prohibiting the execution of
joint wills whether reciprocal or for the benefit of a third party should be considered as
having been repealed and superseded by the new law.
We have made a rather extensive study of the cases decided by our Supreme Court
covering the field of wills, with particular attention to any reference to or ruling on article
669 of the Civil Code but we have failed to find any case wherein that particular codal
provision has been discussed or applied, declaring it either repealed or still in force. The
sole question and issue squarely raised in this appeal is, therefore one of first impression
and naturally we are constrained to act and to proceed with care and caution, realizing the
importance and far-reaching effects of any doctrine to be laid down by us in the present
case.
We cannot agree to the contention of the appellant that the provisions of the Code of Civil
Procedure on wills have completely superseded Chapter I, Title III of the Civil Code on the
same subject matter, resulting in the complete repeal of said Civil Code provisions. In the

study we have made of this subject, we have found a number of cases decided by this
court wherein several articles of the Civil Code regarding wills have not only been referred
to but have also been applied side by side with the provisions of the Code of Civil
Procedure.

inherited by his heirs on intestate succession when not covered by the will. As a rule this
Tribunal does not pass upon the legality, enforceability, or applicability of a law unless that
the point is raised and put in issue, and it is necessary to rule upon it in order to determine
the case.

In the case of in the matter of the will Kabigting (14 Phil., 463), where the will was executed
in the year 1908, articles 662 and 663 of the Civil Code regarding capacity and incapacity
of persons to dispose by will, have been cited and applied together with section 618 of the
Code of Civil Procedure regarding requisites of wills.

The provision of article 669 of the Civil Code prohibiting the execution of a will by two or
more persons conjointly or in the same instrument either for their reciprocal benefit or for
the benefit of a third person, is not unwise and is not against public policy. The reason for
this provision, especially as regards husbands and wife is that when a will is made jointly
or in the same instrument, the spouse who is more aggressive, stronger in will or character
and dominant is liable to dictate the terms of the will for his or her own benefit or for that of
third persons whom he or she desires to favor. And, where the will is not only joint but
reciprocal, either one of the spouses who may happen to be unscrupulous, wicked,
faithless, or desperate, knowing as he or she does the terms of the will whereby the whole
property of the spouses both conjugal and paraphernal goes to the survivor, may be
tempted to kill or dispose of the other.

In the case of Torres and Lopez De Bueno vs. Lopez (48 Phil., 772), article 666 of the Civil
Code regarding mental capacity of the testator has been cited and applied together with
section 614 and 634 of the Code of Civil Procedure regarding a will executed in 1924.
In the case of Marin vs. Nacianceno (19 Phil., 238), article 667 of the Civil Code was cited
in the dissenting opinion of Mr. Justice Torres.
In the cases of Postigo vs. Borjal (13 Phil., 240); In re Estate of Calderon (26 Phil., 333);
Natividad vs. Gabino (36 Phil., 663) wherein the wills involved had been executed after the
enactment of the Code of Civil Procedure, particularly the sections regarding wills, article
675 of the Civil Code regarding interpretation of wills was cited and applied.
In the case of Samson vs. Naval (41 Phil., 838), article 739 of the Civil Code regarding
revocation of wills has been applied in harmony with section 623 of the Code of Civil
Procedure. The will involved was executed in 1915 when the Code of Civil Procedure was
already in force.
The above-cited authorities all go to show that it is not exactly correct to say that the
provisions of the Code of Civil Procedure regarding wills completely cover the subject
matter and therefore have superseded the provisions of the Civil Code on the point.
It is also contended that in the case of Macrohon Ong Ham vs. Saavedra (51 Phil., 267) a
will executed in the year 1923, which was made jointly by husband and wife in the same
instrument, was admitted to probate by the Court of First Instance of Zamboanga and the
decision was affirmed by this court, thereby proving that this tribunal has disregarded the
prohibition regarding the execution of wills conjointly under article 669 of the Civil Code,
meaning that said article has already been repealed. After examining said case we find the
contention untenable. It is true that the will already described was allowed probate by the
trial court, but there was no appeal from the order approving the will on the ground of its
validity, but only on the manner the properties involved were to be distributed or otherwise
disposed of. The Supreme Court never touched this point of invalidity nor the applicability
of article 669 of the Civil Code, but merely ruled that a testator may die both testate and
intestate, depending upon the properties sought to be disposed of by him and those to be

58 | S U C C E S S I O N

Considering the wisdom of the provisions of this article 669 and the fact that it has not
been repealed, at least not expressly, as well as the consideration that its provisions are
not incompatible with those of the Code of Civil Procedure on the subject of wills, we
believe and rule that said article 669 of the Civil Code is still in force. And we are not alone
in this opinion. Mr. Justice Willard as shown by his notes on the Civil Code, on page 18
believes that this article 669 is still in force. Sinco and Capistrano in their work on the Civil
Code, Vol. II, page 33, favorably cite Justice Willard's opinion that this article is still in
force. Judge Camus in his book on the Civil Code does not include this article among
those he considers repealed. Lastly, we find that this article 669 has been reproduced word
for word in article 818 of the New Civil Code (Republic Act No. 386). The implication is that
the Philippine Legislature that passed this Act and approved the New Civil Code, including
the members of the Code Commission who prepared it, are of the opinion that the
provisions of article 669 of the old Civil Code are not incompatible with those of the Code
of Civil Procedure.
In the case of Testate estate of the late Bernabe Rodriguez (CA-G.R. No. 1627 -R, July 1,
1948; 46 Off. Gaz., reference to this article 669 of the Civil Code, though indirectly. In the
will involved therein, the testator Rodriguez instituted his wife his universal heir and the
latter in her separate will equally instituted her husband Rodriguez as her universal heir; in
other words they were reciprocal beneficiaries in their respective separate wills. Opposition
to the probate of the will of Rodriguez was base on the prohibition contained in article 669
of the Civil Code. The Court of Appeals said that what the law prohibits under said article is
two or more persons making a will conjointly or in the same instrument and not reciprocity
in separate wills.
In conclusion, we believe and hold that the provision of the Code of Civil procedure
regarding wills have not repealed all the articles of the old Civil Code on the same subject

matter, and that article 669 of the Civil Code is not incompatible or inconsistent with said
provision of the Article 669 of the Civil Code is still in force.
In view of the foregoing, the decision appealed form, is hereby affirmed, with costs.

G.R. No. L-2071

September 19, 1950

Testate estate of Isabel V. Florendo, deceased. TIRSO


DACANAY, petitioner-appellant,
vs.
PEDRO V. FLORENDO, ET AL., oppositor-appellees.
OZAETA, J.:
This is a special proceeding commenced in the Court of First Instance of La Union to
probate a joint and reciprocal will executed by the spouses Isabel V. Florendo and Tirso
Dacanay on October 20, 1940. Isabel V. Florendo having died, her surviving spouse Tirso
Dacanay is seeking to probate said joint and reciprocal will, which provides in substance
that whoever of the spouses, joint testators, shall survive the other, shall inherit all the
properties of the latter, with an agreement as to how the surviving spouse shall dispose of
the properties in case of his or her demise.
The relatives of the deceased Isabel V. Florendo opposed the probate of said will on
various statutory grounds.
Before hearing the evidence the trial court, after requiring and receiving from counsel for
both parties written arguments on the question of whether or not the said joint and
reciprocal will may be probated in view of article 669 of the Civil Code, issued an order
dismissing the petition for probate on the ground that said will is null and void ab initio as
having been executed in violation of article 669 of the Civil Code. From that order the
proponent of the will has appealed.
Article 669 of the Civil Code reads as follows:
ART. 669. Two or more persons cannot make a will conjointly or in the same
instrument, either for their reciprocal benefit or for the benefit of a third person.

59 | S U C C E S S I O N

We agree with appellant's view, supported by eminent commentators, that the prohibition
of article 669 of the Civil Code is directed against the execution of a joint will, or the
expression by two or more testators of their wills in a single document and by one act,
rather than against mutual or reciprocal wills, which may be separately executed. Upon this
premise, however, appellant argues that article 669 of the Civil Code has been repealed by
Act. No. 190, which he claims provides for and regulates the extrinsic formalities of wills,
contending that whether two wills should be executed conjointly or separately is but a
matter of extrinsic formality.
The question now raised by appellant has recently been decided by this court adversely to
him in In re Will of Victor Bilbao, supra, p. 144. It appears in that case that on October 6,
1931, the spouses Victor Bilbao and Ramona M. Navarro executed a will conjointly,
whereby they directed that "all of our respective private properties both real and personal,
and all of our conjugal properties, and any other property belonging to either or both of us,
be given and transmitted to anyone or either of us, who may survive the other, or who may
remain the surviving spouse of the other." That will was denied probate by the Court of
First Instance of Negros Oriental on the ground that it was prohibited by article 669 of the
Civil Code. The surviving spouse as proponent of the joint will also contended that said
article of the Civil Code has been repealed by sections 614 and 618 of the Code of Civil
Procedure, Act No. 190. In deciding that question this court, speaking through Mr. Justice
Montemayor, said:
We cannot agree to the contention of the appellant that the provisions of the
Code of Civil Procedure on wills have completely superseded Chapter I, Title III
of the Civil Code on the same subject matter, resulting in the complete repeal of
said Civil Code provisions. In the study we have made of this subject, we have
found a number of cases decided by this court wherein several articles of the
Civil Code regarding wills have not only been referred to but have also been
applied side by side with the provisions of the Code of Civil Procedure.
xxx

xxx

xxx

The provision of article 669 of the Civil Code prohibiting the execution of a will by
two or more persons conjointly or in the same instrument either for their
reciprocal benefit or for the benefit of a third person, is not unwise and is not
against public policy. The reason for this provision, especially as regards
husband and wife, is that when a will is made jointly or in the same instrument,
the spouse who is more aggressive, stronger in will or character and dominant is
liable to dictate the terms of the will for his or her own benefit or for that of third
persons whom he or she desires to favor. And, where the will is not only joint but
reciprocal, either one of the spouses who may happen to be unscrupulous,
wicked, faithless or desperate, knowing as he or she does the terms of the will
whereby the whole property of the spouses both conjugal and paraphernal goes
to the survivor, may be tempted to kill or dispose of the other.

Considering the wisdom of the provisions of this article 669 and the fact that it
has not been repealed, at least not expressly, as well as the consideration that its
provisions are not incompatible with those of the Code of Civil Procedure on the
subject of wills, we believe and rule that said article 669 of the Civil Code is still in
force. And we are not alone in this opinion. Mr. Justice Willard as shown by his
Notes on the Civil Code, on page 48 believes that this article 669 is still in force.
Sinco and Capistrano in their work on the Civil Code, Vol. II, page 33, favorably
cite Justice Willard's opinion that this article is still in force. Judge Camus in his
book on the Civil Code does not include this article among those he considers
repealed. Lastly, we find that this article 669 has been reproduced word for word
in article 818 of the New Civil Code (Republic Act No. 386). The implication is
that the Philippine Legislature that passed this Act and approved the New Civil
Code, including the members of the Code Commission who prepared it, are of
the opinion that the provisions of article 669 of the old Civil Code are not
incompatible with those of the Code of Civil Procedure.
In view of the foregoing, the order appealed from is affirmed, with costs against the
appellant.

G.R. No. L-37453 May 25, 1979


RIZALINA GABRIEL GONZALES, petitioner,
vs.
. HONORABLE COURT OF APPEALS and LUTGARDA
SANTIAGO, respondents

GUERRERO, J.:
This is a petition for review of the decision of the Court of Appeals, First
Division, 1 promulgated on May 4, 1973 in CA G.R. No. 36523-R which reversed the
decision of the Court of First Instance of Rizal dated December 15, 1964 and allowed the
probate of the last will and testament of the deceased Isabel Gabriel. *
It appears that on June 24, 1961, herein private respondent Lutgarda Santiago filed a
petition with the Court of First Instance of Rizal docketed as Special Proceedings No.
3617, for the probate of a will alleged to have been executed by the deceased Isabel
Gabriel and designating therein petitioner as the principal beneficiary and executrix.
There is no dispute in the records that the late Isabel Andres Gabriel died as a widow and
without issue in the municipality of Navotas, province of Rizal her place of residence, on
June 7, 1961 at the age of eighty-five (85), having been born in 1876. It is likewise not

60 | S U C C E S S I O N

controverted that herein private respondent Lutgarda Santiago and petitioner Rizalina
Gabriel Gonzales are nieces of the deceased, and that private respondent, with her
husband and children, lived with the deceased at the latters residence prior an- d up to the
time of her death.
The will submitted for probate, Exhibit "F", which is typewritten and in Tagalog, appears to
have been executed in Manila on the 15th day of April, 1961, or barely two (2) months prior
to the death of Isabel Gabriel. It consists of five (5) pages, including the pages whereon the
attestation clause and the acknowledgment of the notary public were written. The
signatures of the deceased Isabel Gabriel appear at the end of the will on page four and at
the left margin of all the pages. The attestation clause, which is found on page four, reads
as follows:
PATUNAY NG MGA SAKSI
Kaming mga nakalagdang mga saksi o testigo na ang aming mga
tinitirahan ay nakasulat sa gawing kanan at kahilira ng aming mga
pangalan sa ibaba nito, ay pagpapatutuo na ipinakilala ipinaalam at
ipinahayag sa amin ni Isabel Gabriel na ang kasulatang ito na binubuo
ng Limang Dahon (Five Pages) pati na ang dahong ito, na siya niyang
TESTAMENTO AT HULING HABILIN, ngayong ika 15 ng Abril, 1961, ay
nilagdaan ng nasabing testadora na si Isabel Gabriel ang nasabing
testamento sa ibaba o ilalim ng kasulatan na nasa ika apat na dahon
(page four) at nasa itaas ng patunay naming ito, at sa kaliwang panig
ng lahat at bawat dahon (and on the left hand margin of each and every
page), sa harap ng lahat at bawat isa sa amin, at kami namang mga
saksi ay lumagda sa harap ng nasabing testadora, at sa harap ng lahat
at bawat isa sa amin, sa ilalim ng patunay ng mga saksi at sa kaliwang
panig ng lahat at bawa't dahon ng testamentong ito.
At the bottom thereof, under the heading "Pangalan", are written the signatures of Matilde
D. Orobia, Celso D. Gimpaya and Maria R. Gimpaya, and opposite the same, under the
heading "Tirahan", are their respective places of residence, 961 Highway 54, Philamlife, for
Miss Orobia, and 12 Dagala St., Navotas, Rizal, for the two Gimpayas. Their signatures
also appear on the left margin of all the other pages. The WW is paged by typewritten
words as follows: "Unang Dahon" and underneath "(Page One)", "Ikalawang Dahon" and
underneath "(Page Two)", etc., appearing at the top of each page.
The will itself provides that the testatrix desired to be buried in the Catholic Cemetery of
Navotas, Rizal in accordance with the rites of the Roman Catholic Church, all expenses to
be paid from her estate; that all her obligations, if any, be paid; that legacies in specified
amounts be given to her sister, Praxides Gabriel Vda. de Santiago, her brother Santiago
Gabriel, and her nephews and nieces, Benjamin, Salud, Rizalina (herein petitioner),

Victoria, Ester, Andres, all surnamed Gabriel, and Evangeline, Rudyardo Rosa, Andrea,
Marcial, Numancia, Verena an surnamed Santiago. To herein private respondent Lutgarda
Santiago, who was described in the will by the testatrix as "aking mahal na pamangkin na
aking pinalaki, inalagaan at minahal na katulad ng isang tunay na anak" and named as
universal heir and executor, were bequeathed all properties and estate, real or personal
already acquired, or to be acquired, in her testatrix name, after satisfying the expenses,
debts and legacies as aforementioned.
The petition was opposed by Rizalina Gabriel Gonzales, herein petitioner, assailing the
document purporting to be the will of the deceased on the following grounds:
1. that the same is not genuine; and in the alternative
2. that the same was not executed and attested as required by law;
3. that, at the time of the alleged execution of the purported wilt the
decedent lacked testamentary capacity due to old age and sickness;
and in the second alternative
4. That the purported WW was procured through undue and improper
pressure and influence on the part of the principal beneficiary, and/or of
some other person for her benefit.
Lutgarda Santiago filed her Answer to the Opposition on February 1, 1962. After trial, the
court a quo rendered judgment, the summary and dispositive portions of which read:
Passing in summary upon the grounds advanced by the oppositor, this
Court finds:
1. That there is no iota of evidence to support the contentio that the
purported will of the deceased was procured through undue and
improper pressure and influence on the part of the petitioner, or of
some other person for her benefit;
2. That there is insufficient evidence to sustain the contention that at the
time of the alleged execution of the purported will, the deceased lacked
testamentary capacity due to old age and sickness;
3. That sufficient and abundant evidence warrants conclusively the fact
that the purported will of the deceased was not executed and attested
as required by law;

61 | S U C C E S S I O N

4. That the evidence is likewise conclusive that the document


presented for probate, Exhibit 'F' is not the purported win allegedly
dictated by the deceased, executed and signed by her, and attested by
her three attesting witnesses on April 15, 1961.
WHEREFORE, Exhibit "F", the document presented for probate as the
last wig and testament of the deceased Isabel Gabriel is here by
DISALLOWED.
From this judgment of disallowance, Lutgarda Santiago appealed to respondent Court,
hence, the only issue decided on appeal was whether or not the will in question was
executed and attested as required by law. The Court of Appeals, upon consideration of the
evidence adduced by both parties, rendered the decision now under review, holding that
the will in question was signed and executed by the deceased Isabel Gabriel on April 15,
1961 in the presence of the three attesting witnesses, Matilde Orobia, Celso Gimpaya and
Maria Gimpaya, signing and witnessing the document in the presence of the deceased and
of each other as required by law, hence allow ed probate.
Oppositor Rizalina Gabriel Gonzales moved for reconsideration 3 of the aforesaid decision
and such motion was opposed 4 by petitioner-appellant Lutgarda Santiago. Thereafter.
parties submitted their respective Memoranda, 5 and on August 28, 1973, respondent
Court, Former Special First Division, by Resolution 6 denied the motion for reconsideration
stating that:
The oppositor-appellee contends that the preponderance of evidence
shows that the supposed last wig and testament of Isabel Gabriel was
not executed in accordance with law because the same was signed on
several occasions, that the testatrix did not sign the will in the presence
of all the instrumental witnesses did not sign the will in the presence of
each other.
The resolution of the factual issue raised in the motion for
reconsideration hinges on the appreciation of the evidence. We have
carefully re-examined the oral and documentary evidence of record,
There is no reason to alter the findings of fact in the decision of this
Court sought to be set aside. 7
In her petition before this Court, oppositor Rizalina Gabriel Gonzales contends that
respondent Court abused its discretion and/or acted without or in excess of its jurisdiction
in reverssing the findings of fact and conclusions of the trial court. The Court, after
deliberating on the petition but without giving due course resolved, in the Resolution dated
Oct. 11, 1973 to require the respondents to comment thereon, which comment was filed on
Nov. 14, 1973. Upon consideration of the allegations, the issues raised and the arguments

adduced in the petition, as well as the Comment 8 of private respondent thereon, We


denied the petition by Resolution on November 26, 1973, 9 the question raised being
factual and for insufficient showing that the findings of fact by respondent Court were
unsupported by substantial evidence.
Subsequently, or on December 17, 1973, petitioner Rim Gabriel Goes fried a Motion for
Reconsideration 10 which private respondent answered by way of her Comment or
Opposition 11 filed on January 15, 1974. A Reply and Rejoinder to Reply followed. Finally,
on March 27, 1974, We resolved to give due course to the petition.
The petitioner in her brief makes the following assignment of errors:
I. The respondent Court of Appeals erred in holding that the document, Exhibit "F" was
executed and attested as required by law when there was absolutely no proof that the
three instrumental witnesses were credible witness
II. The Court of Appeals erred in reversing the finding of the lower court that the
preparation and execution of the win Exhibit "F", was unexpected and coincidental.
III. The Court of Appeals erred in finding that Atty, Paraiso was not previously furnished
with the names and residence certificates of the witnesses as to enable him to type such
data into the document Exhibit "F".
IV. The Court of Appeals erred in holding that the fact that the three typewritten lines under
the typewritten words "Pangalan" and "Tinitirahan" were left blank shows beyond cavil that
the three attesting witnesses were all present in the same occasion.
V. The Court of Appeals erred in reversing the trial court's finding that it was incredible that
Isabel Gabriel could have dictated the wilt Exhibit "F , without any note or document, to
Atty. Paraiso.
VI. The Court of Appeals erred in reversing the finding of the trial court that Matilde Orobia
was not physically present when the Will Exhibit "F" was allegedly signed on April 15, 1961
by the deceased Isabel Gabriel and the other witnesses Celso Gimpaya and Maria
Gimpaya.
VII. The Court of Appeals erred in holding that the trial court gave undue importance to the
picture takings as proof that the win was improperly executed.
VIII. The Court of Appeals erred in holding that the grave contradictions, evasions, and
misrepresentations of witnesses (subscribing and notary) presented by the petitioner had
been explained away, and that the trial court erred in rejecting said testimonies.

62 | S U C C E S S I O N

IX. The Court of Appeals acted in excess of its appellate jurisdiction or has so far departed
from the accepted and usual course of judicial proceedings, as to call for an exercise of the
power of supervision.
X. The Court of Appeals erred in reversing the decision of the trial court and admitting to
probate Exhibit "F", the alleged last will and testament of the deceased Isabel Gabriel.
It will be noted from the above assignments of errors that the same are substantially
factual in character and content. Hence, at the very outset, We must again state the oftrepeated and well-established rule that in this jurisdiction, the factual findings of the Court
of Appeals are not reviewable, the same being binding and conclusive on this Court. This
rule has been stated and reiterated in a long line of cases enumerated in Chan vs. CA (L27488, June 30, 1970, 33 SCRA 737, 743) 12 and Tapas vs. CA (L-22202, February 27;
1976, 69 SCRA 393), 13 and in the more recent cases of Baptisia vs. Carillo and
CA (L32192, July 30, 1976, 72 SCRA 214, 217) and Vda. de Catindig vs. Heirs of Catalina
Roque (L-25777, November 26, 1976, 74 SCRA 83, 88). In the case of Chan vs. CA, this
Court said:
... from Guico v. Mayuga, a 1936 decision, the opinion being penned by the then Justice
Recto, it has been well-settled that the jurisdiction of tills Court in cases brought to us from
the Court of Appeals is limited to reviewing and revising the errors of law imputed to it, its
findings of fact being conclusive. More specifically, in a decision exactly a month later, this
Court, speaking through the then Justice Laurel, it was held that the same principle is
applicable, even if the Court of Appeals was in disagreement with the lower court as to the
weight of the evidence with a consequent reversal of its findings of fact ...
Stated otherwise, findings of facts by the Court of Appeals, when supported by substantive
evidence are not reviewable on appeal by certiorari. Said findings of the appellate court are
final and cannot be disturbed by Us particularly because its premises are borne out by the
record or based upon substantial evidence and what is more, when such findings are
correct. Assignments of errors involving factual issues cannot be ventilated in a review of
the decision of the Court of Appeals because only legal questions may be raised. The
Supreme Court is not at liberty to alter or modify the facts as set forth in the decision of the
Court of Appeals sought to be reversed. Where the findings of the Court of Appeals are
contrary to those of the trial court, a minute scrutiny by the Supreme Court is in order, and
resort to duly-proven evidence becomes necessary. The general rule We have thus stated
above is not without some recognized exceptions.
Having laid down the above legal precepts as Our foundation, We now proceed to consider
petitioner's assignments of errors.
Petitioner, in her first assignment, contends that the respondent Court of Appeals erred in
holding that the document, Exhibit "F", was executed and attested as required by law when

there was absolutely no proof that the three instrumental witnesses were credible
witnesses. She argues that the require. ment in Article 806, Civil Code, that the witnesses
must be credible is an absolute requirement which must be complied with before an
alleged last will and testament may be admitted to probate and that to be a credible
witness, there must be evidence on record that the witness has a good standing in his
community, or that he is honest and upright, or reputed to be trustworthy and reliable.
According to petitioner, unless the qualifications of the witness are first established, his
testimony may not be favorably considered. Petitioner contends that the term "credible" is
not synonymous with "competent" for a witness may be competent under Article 820 and
821 of the Civil Code and still not be credible as required by Article 805 of the same Code.
It is further urged that the term "credible" as used in the Civil Code should receive the
same settled and well- known meaning it has under the Naturalization Law, the latter being
a kindred legislation with the Civil Code provisions on wigs with respect to the
qualifications of witnesses.

uprightness, because such attributes are presumed of the witness unless the contrary is
proved otherwise by the opposing party.

We find no merit to petitioner's first assignment of error. Article 820 of the Civil Code
provides the qualifications of a witness to the execution of wills while Article 821 sets forth
the disqualification from being a witness to a win. These Articles state:

In probate proceedings, the instrumental witnesses are not character witnesses for they
merely attest the execution of a will or testament and affirm the formalities attendant to
said execution. And We agree with the respondent that the rulings laid down in the cases
cited by petitioner concerning character witnesses in naturalization proceedings are not
applicable to instrumental witnesses to wills executed under the Civil Code of the
Philippines.

Art. 820. Any person of sound mind and of the age of eighteen years or
more, and not blind, deaf or dumb, and able to read and write, may be
a witness to the execution of a will mentioned in article 806 of this
Code. "Art. 821. The following are disqualified from being witnesses to
a will:
(1) Any person not domiciled in the Philippines,
(2) Those who have been convicted of falsification of a document,
perjury or false testimony.
Under the law, there is no mandatory requirement that the witness testify initially or at any
time during the trial as to his good standing in the community, his reputation for
trustworthythiness and reliableness, his honesty and uprightness in order that his
testimony may be believed and accepted by the trial court. It is enough that the
qualifications enumerated in Article 820 of the Civil Code are complied with, such that the
soundness of his mind can be shown by or deduced from his answers to the questions
propounded to him, that his age (18 years or more) is shown from his appearance,
testimony , or competently proved otherwise, as well as the fact that he is not blind, deaf or
dumb and that he is able to read and write to the satisfaction of the Court, and that he has
none of the disqualifications under Article 821 of the Civil Code. We reject petitioner's
contention that it must first be established in the record the good standing of the witness in
the community, his reputation for trustworthiness and reliableness, his honesty and

63 | S U C C E S S I O N

We also reject as without merit petitioner's contention that the term "credible" as used in
the Civil Code should be given the same meaning it has under the Naturalization Law
where the law is mandatory that the petition for naturalization must be supported by two
character witnesses who must prove their good standing in the community, reputation for
trustworthiness and reliableness, their honesty and uprightness. The two witnesses in a
petition for naturalization are character witnesses in that being citizens of the Philippines,
they personally know the petitioner to be a resident of the Philippines for the period of time
required by the Act and a person of good repute and morally irreproachable and that said
petitioner has in their opinion all the qualifications necessary to become a citizen of the
Philippines and is not in any way disqualified under the provisions of the Naturalization
Law (Section 7, Commonwealth Act No. 473 as amended).

In the case at bar, the finding that each and everyone of the three instrumental witnesses,
namely, Matilde Orobia, Celso Gimpaya and Maria Gimpaya, are competent and credible
is satisfactorily supported by the evidence as found by the respondent Court of Appeals,
which findings of fact this Tribunal is bound to accept and rely upon. Moreover, petitioner
has not pointed to any disqualification of any of the said witnesses, much less has it been
shown that anyone of them is below 18 years of age, of unsound mind, deaf or dumb, or
cannot read or write.
It is true that under Article 805 of the New Civil Code, every will, other than a holographic
will, must be subscribed at the end thereof by the testator himself or by the testator's name
written by some other person in his presence, and by his express direction, and attested
and subscribed by three or more credible witnesses in the presence of the testator and of
one another, While the petitioner submits that Article 820 and 821 of the New Civil Code
speak of the competency of a witness due to his qualifications under the first Article and
none of the disqualifications under the second Article, whereas Article 805 requires the
attestation of three or more credible witnesses, petitioner concludes that the
term credible requires something more than just being competent and, therefore, a witness
in addition to being competent under Articles 820 and 821 must also be a credible witness
under Article 805.

Petitioner cites American authorities that competency and credibility of a witness are not
synonymous terms and one may be a competent witness and yet not a credible one. She
exacerbates that there is no evidence on record to show that the instrumental witnesses
are credible in themselves, that is, that they are of good standing in the community since
one was a family driver by profession and the second the wife of the driver, a housekeeper.
It is true that Celso Gimpaya was the driver of the testatrix and his wife Maria Gimpaya,
merely a housekeeper, and that Matilde Orobia was a piano teacher to a grandchild of the
testatrix But the relation of employer and employee much less the humble or financial
position of a person do not disqualify him to be a competent testamentary witness. (Molo
Pekson and Perez Nable vs. Tanchuco, et al., 100 Phil. 344; Testate Estate of Raymundo,
Off. Gaz., March 18,1941, p. 788).
Private respondent maintains that the qualifications of the three or more credible witnesses
mentioned in Article 805 of the Civil Code are those mentioned in Article 820 of the same
Code, this being obvious from that portion of Article 820 which says "may be Q witness to
the execution of a will mentioned in Article 805 of this Code," and cites authorities that the
word "credible" insofar as witnesses to a will are concerned simply means " competent."
Thus, in the case of Suntay vs. Suntay, 95 Phil. 500, the Supreme Court held that
"Granting that a will was duly executed and that it was in existence at the time of, and not
revoked before, the death of the testator, still the provisions of the lost wig must be clearly
and distinctly proved by at least two credible witnesses. 'Credible witnesses' mean
competent witnesses and not those who testify to facts from or upon hearsay.
" emphasissupplied).
In Molo Pekson and Perez Nable vs. Tanchuco, et al., 100 Phil. 344, the Supreme Court
held that "Section 620 of the same Code of Civil Procedure provides that any person of
sound mind, and of the age of eighteen years or more, and not blind, deaf, or dumb and
able to read and write, may be a witness to the execution of a will. This same provision is
reproduced in our New Civil Code of 1950, under Art. 820. The relation of employer and
employee, or being a relative to the beneficiary in a win, does not disqualify one to be a
witness to a will. The main qualification of a witness in the attestation of wills, if other
qualifications as to age, mental capacity and literacy are present, is that said witness must
be credible, that is to say, his testimony may be entitled to credence. There is a long line of
authorities on this point, a few of which we may cite:
A 'credible witness is one who is not is not to testify by mental
incapacity, crime, or other cause. Historical Soc of Dauphin County vs.
Kelker 74 A. 619, 226 Pix 16, 134 Am. St. Rep. 1010. (Words and
Phrases, Vol. 10, p. 340).
As construed by the common law, a 'credible witness' to a will means a
'competent witness.' Appeal of Clark, 95 A. 517, 114 Me. 105, Ann. Cas.
1917A, 837. (lbid, p. 341).

64 | S U C C E S S I O N

Expression 'credible witness' in relation to attestation of wins means


'competent witness that is, one competent under the law to testify to
fact of execution of will. Vernon's Ann. Civ St. art. 8283. Moos vs. First
State Bank of Uvalde, Tex . Civ. App. 60 S.W. 2nd 888, 889. (Ibid, p.
342)
The term 'credible', used in the statute of wills requiring that a will shall
be attested by two credible witnesses means competent; witnesses
who, at the time of attesting the will, are legally competent to testify, in a
court of justice, to the facts attested by subscribing the will, the
competency being determined as of the date of the execution of the will
and not of the timr it is offered for probate,Smith vs. Goodell 101 N.E.
255, 256, 258 111. 145. (Ibid.)
Credible witnesses as used in the statute relating to wills, means
competent witnesses that is, such persons as are not legally
disqualified from testifying in courts of justice, by reason of mental
incapacity, interest, or the commission of crimes, or other cause
excluding them from testifying generally, or rendering them incompetent
in respect of the particular subject matter or in the particular suit. Hill vs.
Chicago Title & Trust co 152 N.E. 545, 546, 322 111. 42. (Ibid. p, 343)
In the strict sense, the competency of a person to be an instrumental witness to a will is
determined by the statute, that is Art. 820 and 821, Civil Code, whereas his credibility
depends On the appreciation of his testimony and arises from the belief and conclusion of
the Court that said witness is telling the truth. Thus, in the case ofVda. de Aroyo v. El
Beaterio del Santissimo Rosario de Molo, No. L-22005, May 3, 1968, the Supreme Court
held and ruled that: "Competency as a witness is one thing, and it is another to be a
credible witness, so credible that the Court must accept what he says. Trial courts may
allow a person to testify as a witness upon a given matter because he is competent, but
may thereafter decide whether to believe or not to believe his testimony." In fine, We state
the rule that the instrumental witnesses in Order to be competent must be shown to have
the qualifications under Article 820 of the Civil Code and none of the disqualifications under
Article 821 and for their testimony to be credible, that is worthy of belief and entitled to
credence, it is not mandatory that evidence be first established on record that the
witnesses have a good standing in the community or that they are honest and upright or
reputed to be trustworthy and reliable, for a person is presumed to be such unless the
contrary is established otherwise. In other words, the instrumental witnesses must be
competent and their testimonies must be credible before the court allows the probate of the
will they have attested. We, therefore, reject petitioner's position that it was fatal for
respondent not to have introduced prior and independent proof of the fact that the
witnesses were "credible witnesses that is, that they have a good standing in the
community and reputed to be trustworthy and reliable.

Under the second, third, fourth, fifth, sixth, seventh and eighth assignments of errors,
petitioner disputes the findings of fact of the respondent court in finding that the
preparation and execution of the will was expected and not coincidental, in finding that Atty.
Paraiso was not previously furnished with the names and residence certificates of the
witnesses as to enable him to type such data into the document Exhibit "F", in holding that
the fact that the three typewritten lines under the typewritten words "pangalan" and
"tinitirahan" were left blank shows beyond cavil that the three attesting witnesses were all
present in the same occasion, in holding credible that Isabel Gabriel could have dictated
the will without note or document to Atty. Paraiso, in holding that Matilde Orobia was
physically present when the will was signed on April 15, 1961 by the deceased Isabel
Gabriel and the other witnesses Celso Gimpaya and Maria Gimpaya, in holding that the
trial court gave undue importance to the picture takings as proof that the will was
improperly executed, and in holding that the grave contradictions, evasions and
misrepresentations of the witnesses (subscribing and notary) presented by the petitioner
had been explained away.
Since the above errors are factual We must repeat what We have previously laid down that
the findings of fact of the appellate court are binding and controlling which We cannot
review, subject to certain exceptions which We win consider and discuss hereinafter. We
are convinced that the appellate court's findings are sufficiently justified and supported by
the evidence on record. Thus, the alleged unnaturalness characterizing the trip of the
testatrix to the office of Atty. Paraiso and bringing all the witnesses without previous
appointment for the preparation and execution of the win and that it was coincidental that
Atty. Paraiso was available at the moment impugns the finding of the Court of Appeals that
although Atty. Paraiso admitted the visit of Isabel Gabriel and of her companions to his
office on April 15, 1961 was unexpected as there was no prior appointment with him, but
he explained that he was available for any business transaction on that day and that Isabel
Gabriel had earlier requested him to help her prepare her will. The finding of the appellate
court is amply based on the testimony of Celso Gimpaya that he was not only informed on
the morning of the day that he witnessed the will but that it was the third time when Isabel
Gabriel told him that he was going to witness the making of her will, as well as the
testimony of Maria Gimpaya that she was called by her husband Celso Gimpaya to
proceed to Isabel Gabriel's house which was nearby and from said house, they left in a car
to the lawyer's office, which testimonies are recited in the respondent Court's decision.
The respondent Court further found the following facts: that Celso Gimpaya and his wife
Maria Gimpaya obtained residence certificates a few days before Exhibit "F" was
executed. Celso Gimpaya's residence certificate No. A-5114942 was issued at Navotas,
Rizal on April 13, 1961 while Maria Gimpaya's residence certificate No. A-5114974 was
issued also at Navotas, Rizal on April 14, 1961. The respondent Court correctly observed
that there was nothing surprising in these facts and that the securing of these residence
certificates two days and one day, respectively, before the execution of the will on April 15,
1961, far from showing an amazing coincidence, reveals that the spouses were earlier
notified that they would be witnesses to the execution of Isabel Gabriel's will.

65 | S U C C E S S I O N

We also agree with the respondent Court's conclusion that the excursion to the office of
Atty. Paraiso was planned by the deceased, which conclusion was correctly drawn from
the testimony of the Gimpaya spouses that they started from the Navotas residence of the
deceased with a photographer and Isabel Gabriel herself, then they proceeded by car to
Matilde Orobia's house in Philamlife, Quezon City to fetch her and from there, all the three
witnesses (the Gimpayas and Orobia) passed by a place where Isabel Gabriel stayed for
about ten to fifteen minutes at the clinic of Dr. Chikiamco before they proceeded to Atty.
Cipriano Paraiso's office.
It is also evident from the records, as testified to by Atty. Paraiso, that previous to the day
that. the will was executed on April 15, 1961, Isabel Gabriel had requested him to help her
in the execution of her will and that he told her that if she really wanted to execute her will,
she should bring with her at least the Mayor of Navotas, Rizal and a Councilor to be her
witnesses and that he (Atty. Paraiso) wanted a medical certificate from a physician
notwithstanding the fact that he believed her to be of sound and disposition mind. From
this evidence, the appellate court rightly concluded, thus: "It is, therefore, clear that the
presence of Isabel Gabriel and her witnesses Matilde Orobia, Celso Gimpaya and Maria
Gimpaya including the photographer in the law office of Atty. Paraiso was not coincidental
as their gathering was pre-arranged by Isabel Gabriel herself."
As to the appellate court's finding that Atty. Paraiso was not previously furnished with the
names and residence certificates of the witnesses as to enable him to type such data into
the document Exhibit ' L which the petitioner assails as contradictory and irreconcilable
with the statement of the Court that Atty. Paraiso was handed a list (containing the names
of the witnesses and their respective residence certificates) immediately upon their arrival
in the law office by Isabel Gabriel and this was corroborated by Atty. Paraiso himself who
testified that it was only on said occasion that he received such list from Isabel Gabriel, We
cannot agree with petitioner's contention. We find no contradiction for the, respondent
Court held that on the occasion of the will making on April 15, 1961, the list was given
immediately to Atty. Paraiso and that no such list was given the lawyer in any previous
occasion or date prior to April 15, 1961.
But whether Atty. Paraiso was previously furnished with the names and residence
certificates of the witnesses on a prior occasion or on the very occasion and date in April
15, 1961 when the will was executed, is of no moment for such data appear in the notarial
acknowledgment of Notary Public Cipriano Paraiso, subscribed and sworn to by the
witnesses on April 15, 1961 following the attestation clause duly executed and signed on
the same occasion, April 15, 1961. And since Exhibit "F" is a notarial will duly
acknowledged by the testatrix and the witnesses before a notary public, the same is a
public document executed and attested through the intervention of the notary public and as
such public document is evidence of the facts in clear, unequivocal manner therein
expressed. It has in its favor the presumption of regularity. To contradict all these, there
must be evidence that is clear, convincing and more than merely preponderant. (Yturalde

vs. Azurin, 28 SCRA 407). We find no such evidence pointed by petitioner in the case at
bar.
Likewise, the conclusion of the Court of Appeals in holding that the fact that the three
typewritten lines under the typewritten words "pangalan ' and "tinitirahan" were left blank
shows beyond cavil that the three attesting witnesses were all present in the same
occasion merits Our approval because tills conclusion is supported and borne out by the
evidence found by the appellate court, thus: "On page 5 of Exhibit "F", beneath the
typewritten words "names", "Res. Tax Cert. date issued" and place issued the only name of
Isabel Gabriel with Residence Tax certificate No. A-5113274 issued on February 24, 1961
at Navotas Rizal appears to be in typewritten form while the names, residence tax
certificate numbers, dates and places of issuance of said certificates pertaining to the three
(3) witnesses were personally handwritten by Atty. Paraiso. Again, this coincides with Atty.
Paraiso's even the sale must be made to close relatives; and the seventh was the
appointment of the appellant Santiago as executrix of the will without bond. The technical
description of the properties in paragraph 5 of Exhibit F was not given and the numbers of
the certificates of title were only supplied by Atty. Paraiso. "
It is true that in one disposition, the numbers of the Torrens titles of the properties disposed
and the docket number of a special proceeding are indicated which Atty. Paraiso candidly
admitted were supplied by him, whereupon petitioner contends that it was incredible that
Isabel Gabriel could have dictated the will Exhibit "F" without any note or document to Atty.
Paraiso, considering that Isabel Gabriel was an old and sickly woman more than eightyone years old and had been suffering from a brain injury caused by two severe blows at
her head and died of terminal cancer a few weeks after the execution of Exhibit "F". While
we can rule that this is a finding of fact which is within the competency of the respondent
appellate court in determining the testamentary capacity of the testatrix and is, therefore,
beyond Our power to revise and review, We nevertheless hold that the conclusion reached
by the Court of Appeals that the testatrix dictated her will without any note or memorandum
appears to be fully supported by the following facts or evidence appearing on record. Thus,
Isabel Gabriel, despite her age, was particularly active in her business affairs as she
actively managed the affairs of the movie business ISABELITA Theater, paying the
aparatistas herself until June 4, 1961, 3 days before her death. She was the widow of the
late Eligio Naval, former Governor of Rizal Province and acted as coadministratrix in the
Intestate Estate of her deceased husband Eligio Naval. The text of the win was in Tagalog,
a dialect known and understood by her and in the light of all the circumstances, We agree
with the respondent Court that the testatrix dictated her will without any note or
memorandum, a fact unanimously testified to by the three attesting witnesses and the
notary public himself.
Petitioner's sixth assignment of error is also bereft of merit. The evidence, both testimonial
and documentary is, according to the respondent court, overwhelming that Matilde Orobia
was physically present when the will was signed on April 15, 1961 by the testatrix and the
other two witnesses, Celso Gimpaya and Maria Gimpaya. Such factual finding of the

66 | S U C C E S S I O N

appellate court is very clear, thus: "On the contrary, the record is replete with proof that
Matilde Orobia was physically present when the will was signed by Isabel Gabriel on April
'15, 1961 along with her co-witnesses Celso Gimpaya and Maria Gimpaya. The trial court's
conclusion that Orobia's admission that she gave piano lessons to the child of the
appellant on Wednesdays and Saturdays and that April 15, 1961 happened to be a
Saturday for which reason Orobia could not have been present to witness the will on that
day is purely conjectural. Witness Orobia did not admit having given piano lessons to
the appellant's child every Wednesday and Saturday without fail. It is highly probable that
even if April 15, 1961 were a Saturday, she gave no piano lessons on that day for which
reason she could have witnessed the execution of the will. Orobia spoke of occasions
when she missed giving piano lessons and had to make up for the same. Anyway, her
presence at the law office of Atty. Paraiso was in the morning of April 15, 1961 and there
was nothing to preclude her from giving piano lessons on the afternoon of the same day in
Navotas, Rizal."
In addition to the testimony of Matilde Orobia, Celso Gimpaya and Maria Gimpaya that
Matilde was present on April 15, 1961 and that she signed the attestation clause to the will
and on the left-hand margin of each of the pages of the will, the documentary evidence
which is the will itself, the attestation clause and the notarial acknowledgment
overwhelmingly and convincingly prove such fact that Matilde Orobia was present on that
day of April 15, 1961 and that she witnessed the will by signing her name thereon and
acknowledged the same before the notary public, Atty. Cipriano P. Paraiso. The attestation
clause which Matilde Orobia signed is the best evidence as to the date of signing because
it preserves in permanent form a recital of all the material facts attending the execution of
the will. This is the very purpose of the attestation clause which is made for the purpose of
preserving in permanent form a record of the facts attending the execution of the will, so
that in case of failure in the memory of the subscribing witnesses, or other casualty they
may still be proved. (Thompson on Wills, 2nd ed., Sec. 132; Leynez vs. Leynez, 68 Phil.
745).
As to the seventh error assigned by petitioner faulting the Court of Appeals in holding that
the trial court gave undue importance to the picture-takings as proof that the win was
improperly executed, We agree with the reasoning of the respondent court that: "Matilde
Orobia's Identification of the photographer as "Cesar Mendoza", contrary to what the other
two witnesses (Celso and Maria Gimpaya) and Atty. Paraiso said that the photographer
was Benjamin Cifra, Jr., is at worst a minor mistake attributable to lapse of time. The law
does not require a photographer for the execution and attestation of the will. The fact that
Miss Orobia mistakenly Identified the photographer as Cesar Mendoza scarcely detracts
from her testimony that she was present when the will was signed because what matters
here is not the photographer but the photograph taken which clearly portrays Matilde
Orobia herself, her co-witnesses Celso Gimpaya. " Further, the respondent Court correctly
held: "The trial court gave undue importance to the picture takings, jumping therefrom to
the conclusion that the will was improperly executed. The evidence however, heavily points
to only one occasion of the execution of the will on April 15, 1961 which was witnessed by

Matilde Orobia, Celso Gimpaya and Maria Gimpaya. These witnesses were quite emphatic
and positive when they spoke of this occasion. Hence, their Identification of some
photographs wherein they all appeared along with Isabel Gabriel and Atty. Paraiso was
superfluous."
Continuing, the respondent Court declared: "It is true that the second picture-taking was
disclosed at the cross examination of Celso Gimpaya. But this was explained by Atty.
Paraiso as a reenactment of the first incident upon the insistence of Isabel Gabriel. Such
reenactment where Matilde Orobia was admittedly no longer present was wholly
unnecessary if not pointless. What was important was that the will was duly executed and
witnessed on the first occasion on April 15, 1961 , " and We agree with the Court's
rationalization in conformity with logic, law and jurisprudence which do not require picturetaking as one of the legal requisites for the execution or probate of a will.
Petitioner points to alleged grave contradictions, evasions and misrepresentations of
witnesses in their respective testimonies before the trial court. On the other hand, the
respondent Court of Appeals held that said contradictions, evasions and
misrepresentations had been explained away. Such discrepancies as in the description of
the typewriter used by Atty. Paraiso which he described as "elite" which to him meant big
letters which are of the type in which the will was typewritten but which was Identified by
witness Jolly Bugarin of the N.B.I. as pica the mistake in mentioning the name of the
photographer by Matilde Orobia to be Cesar Mendoza when actually it was Benjamin Cifra,
Jr. these are indeed unimportant details which could have been affected by the lapse of
time and the treachery of human memory such that by themselves would not alter the
probative value of their testimonies on the true execution of the will, (Pascual vs. dela
Cruz, 28 SCRA 421, 424) for it cannot be expected that the testimony of every person win
be Identical and coinciding with each other with regard to details of an incident and that
witnesses are not expected to remember all details. Human experience teach us "that
contradictions of witnesses generally occur in the details of certain incidents, after a long
series of questionings, and far from being an evidence of falsehood constitute a
demonstration of good faith. In as much as not all those who witness an incident are
impressed in like manner, it is but natural that in relating their impressions, they should not
agree in the minor details; hence the contradictions in their testimony." (Lopez vs. Liboro,
81 Phil. 429).
It is urged of Us by the petitioner that the findings of the trial court should not have been
disturbed by the respondent appellate court because the trial court was in a better position
to weigh and evaluate the evidence presented in the course of the trial. As a general rule,
petitioner is correct but it is subject to well-established exceptions. The right of the Court of
Appeals to review, alter and reverse the findings of the trial court where the appellate court,
in reviewing the evidence has found that facts and circumstances of weight and influence
have been ignored and overlooked and the significance of which have been misinterpreted
by the trial court, cannot be disputed. Findings of facts made by trial courts particularly
when they are based on conflicting evidence whose evaluation hinges on questions of

67 | S U C C E S S I O N

credibility of contending witnesses hes peculiarly within the province of trial courts and
generally, the appellate court should not interfere with the same. In the instant case,
however, the Court of Appeals found that the trial court had overlooked and misinterpreted
the facts and circumstances established in the record. Whereas the appellate court said
that "Nothing in the record supports the trial court's unbelief that Isabel Gabriel dictated her
will without any note or document to Atty. Paraiso;" that the trial court's conclusion that
Matilde Orobia could not have witnessed anybody signing the alleged will or that she could
not have witnessed Celso Gimpaya and Maria Gimpaya sign the same or that she
witnessed only the deceased signing it, is a conclusion based not on facts but on
inferences; that the trial court gave undue importance to the picture-takings, jumping
therefrom to the conclusion that the will was improperly executed and that there is nothing
in the entire record to support the conclusion of the court a quo that the will signing
occasion was a mere coincidence and that Isabel Gabriel made an appointment only with
Matilde Orobia to witness the signing of her will, then it becomes the duty of the appellate
court to reverse findings of fact of the trial court in the exercise of its appellate jurisdiction
over the lower courts.
Still the petitioner insists that the case at bar is an exception to the rule that the judgment
of the Court of Appeals is conclusive as to the facts and cannot be reviewed by the
Supreme Court. Again We agree with the petitioner that among the exceptions are: (1)
when the conclusion is a finding grounded entirely on speculations, surmises or
conjectures; (2) when the inference is manifestly mistaken, absurd or impossible; (3) when
there is a grave abuse of discretion; (4) when the presence of each other as required by
law. " Specifically, We affirm that on April 15, 1961 the testatrix Isabel Gabriel, together
with Matilde Orobia, Celso Gimpaya and his wife Maria Gimpaya, and a photographer
proceeded in a car to the office of Atty. Cipriano Paraiso at the Bank of P.I. Building, Manila
in the morning of that day; that on the way, Isabel Gabriel obtained a medical certificate
from one Dr. Chikiamko which she gave to Atty. Paraiso upon arriving at the latter's office
and told the lawyer that she wanted her will to be made; that Atty. Paraiso asked Isabel
Gabriel to dictate what she wanted to be written in the will and the attorney wrote down the
dictation of Isabel Gabriel in Tagalog, a language known to and spoken by her; that Atty.
Paraiso read back to her what he wrote as dictated and she affirmed their correctness; the
lawyer then typed the will and after finishing the document, he read it to her and she told
him that it was alright; that thereafter, Isabel Gabriel signed her name at the end of the will
in the presence of the three witnesses Matilde Orobia, Celso Gimpaya and Maria Gimpaya
and also at the left-hand margin of each and every page of the document in the presence
also of the said three witnesses; that thereafter Matilde Orobia attested the will by signing
her name at the end of the attestation clause and at the left-hand margin of pages 1, 2, 3
and 5 of the document in the presence of Isabel Gabriel and the other two witnesses,
Celso Gimpaya and Maria Gimpaya; then, Celso Gimpaya signed also the will at the
bottom of the attestation clause and at the left-hand margin of the other pages of the
document in the presence of Isabel Gabriel, Matilde Orobia and Maria Gimpaya; that Maria
Gimpaya followed suit, signing her name at the foot of the attestation clause and at the lefthand margin of every page in the presence of Isabel Gabriel, Matilde Orobia and Celso

Gimpaya; that thereafter, Atty. Paraiso notarized the will as Page No. 94, Book No. IV,
Series of 1961, in his Notarial Register. On the occasion of the execution and attestation of
the will, a photographer took pictures, one Exhibit "G", depicting Matilde Orobia, the
testatrix Isabel Gabriel, Celso Gimpaya, Maria Gimpaya and Atty. Paraiso, taken on said
occasion of the signing of the will, and another, Exhibit "H", showing Matilde Orobia signing
testimony that he had earlier advised Isabel Gabriel to bring with her at least the Mayor
and a Councilor of Navotas, Rizal to be her witnesses for he did not know beforehand the
Identities of the three attesting witnesses until the latter showed up at his law office with
Isabel Gabriel on April 15, 1961. Atty. Paraiso's claim which was not controverted that he
wrote down in his own hand the date appearing on page 5 of Exhibit "F" dissipates any
lingering doubt that he prepared and ratified the will on the date in question."
It is also a factual finding of the Court of Appeals in holding that it was credible that Isabel
Gabriel could have dictated the will, Exhibit "F", without any note or document to Atty.
Paraiso as against the contention of petitioner that it was incredible. This ruling of the
respondent court is fully supported by the evidence on record as stated in the decision
under review, thus: "Nothing in the record supports the trial court's unbelief that Isabel
Gabriel dictated her will without any note or document to Atty. Paraiso. On the contrary, all
the three attesting witnesses uniformly testified that Isabel Gabriel dictated her will to Atty.
Paraiso and that other than the piece of paper that she handed to said lawyer she had no
note or document. This fact jibes with the evidence which the trial court itself believed
was unshaken that Isabel Gabriel was of sound disposing memory when she executed
her will.
Exhibit "F" reveals only seven (7) dispositions which are not complicated but quite simple.
The first was Isabel Gabriel's wish to be interred according to Catholic rites the second
was a general directive to pay her debts if any; the third provided for P1,000.00 for her
sister Praxides Gabriel Vda. de Santiago and P2,000.00 for her brother Santiago Gabriel;
the fourth was a listing of her 13 nephews and nieces including oppositor-appellee Rizalina
Gabriel and the amount for each legatee the fifth was the institution of the petitionerappellant, Lutgarda Santiago as the principal heir mentioning in general terms seven (7)
types of properties; the sixth disposed of the remainder of her estate which she willed in
favor of appellant Lutgarda Santiago but prohibiting the sale of such properties to anyone
except in extreme situations in which judgment is based on a misapprehension of facts; (5)
when the findings of fact are conflicting, (6) when the Court of Appeals, in making its
findings, went beyond the issues of the case and the same is contrary to the admissions of
both appellant and appellee. (Roque vs. Buan, et al., G.R. No. L-22459, Oct. 31, 1967;
Ramos vs. Pepsi Cola Bottling Co., G.R. No. L-22533, Feb. 9, 1967; Hilarion Jr. vs. City of
Manila, G.R. No. L-19570; Sept. 14, 1967).
Petitioner's insistence is without merit. We hold that the case at bar does not fall within any
of the exceptions enumerated above. We likewise hold that the findings of fact of the
respondent appellate court are fully supported by the evidence on record. The conclusions
are fully sustained by substantial evidence. We find no abuse of discretion and We discern

68 | S U C C E S S I O N

no misapprehension of facts. The respondent Court's findings of fact are not conflicting.
Hence, the well-established rule that the decision of the Court of Appeals and its findings of
fact are binding and conclusive and should not be disturbed by this Tribunal and it must be
applied in the case at bar in its full force and effect, without qualification or reservation. The
above holding simply synthesize the resolutions we have heretofore made in respect ' to
petitioner's previous assignments of error and to which We have disagreed and, therefore,
rejected.
The last assignments of error of petitioner must necessarily be rejected by Us as We find
the respondent Court acted properly and correctly and has not departed from the accepted
and usual course of judicial proceedings as to call for the exercise of the power of
supervision by the Supreme Court, and as We find that the Court of Appeals did not err in
reversing the decision of the trial court and admitting to probate Exhibit "F", the last will and
testament of the deceased Isabel Gabriel.
We rule that the respondent Court's factual findings upon its summation and evaluation of
the evidence on record is unassailable that: "From the welter of evidence presented, we
are convinced that the will in question was executed on April 15, 1961 in the presence of
Matilde Orobia, Celso Gimpaya and Maria Gimpaya signing and witnessing the same in
the the will on a table with Isabel Gabriel, Celso Gimpaya and Maria Gimpaya sitting
around the table. Atty. Paraiso, after finishing the notarial act, then delivered the original to
Isabel Gabriel and retained the other copies for his file and notarial register. A few days
following the signing of the will, Isabel Gabriel, Celso Gimpaya and another photographer
arrived at the office of Atty. Paraiso and told the lawyer that she wanted another picture
taken because the first picture did not turn out good. The lawyer told her that this cannot
be done because the will was already signed but Isabel Gabriel insisted that a picture be
taken, so a simulated signing was performed during which incident Matilde Orobia was not
present.
Petitioner's exacerbation centers on the supposed incredibility of the testimonies of the
witnesses for the proponent of the will, their alleged evasions, inconsistencies and
contradictions. But in the case at bar, the three instrumental witnesses who constitute the
best evidence of the will making have testified in favor of the probate of the will. So has the
lawyer who prepared it, one learned in the law and long in the practice thereof, who
thereafter notarized it. All of them are disinterested witnesses who stand to receive no
benefit from the testament. The signatures of the witnesses and the testatrix have been
identified on the will and there is no claim whatsoever and by anyone, much less the
petitioner, that they were not genuine. In the last and final analysis, the herein conflict is
factual and we go back to the rule that the Supreme Court cannot review and revise the
findings of facts of the respondent Court of Appeals.
WHEREFORE, IN VIEW OF THE FOREGOING, the judgment appealed from is hereby
AFFIRMED, with costs against the petitioner.

SO ORDERED.

G.R. No. 76464 February 29, 1988


TESTATE ESTATE OF THE LATE ADRIANA MALOTO, ALDINA MALOTO
CASIANO, CONSTANCIO MALOTO, PURIFICACION MIRAFLOR, ROMAN
CATHOLIC CHURCH OF MOLO, AND ASILO DE MOLO,petitioners,
. vs.
COURT OF APPEALS, PANFILO MALOTO AND FELINO
MALOTO, respondents
SARMIENTO, J.:
This is not the first time that the parties to this case come to us. In fact, two other cases
directly related to the present one and involving the same parties had already been
decided by us in the past. In G.R. No. L-30479, 1which was a petition for certiorari and
mandamus instituted by the petitioners herein, we dismissed the petition ruling that the
more appropriate remedy of the petitioners is a separate proceeding for the probate of the
will in question. Pursuant to the said ruling, the petitioners commenced in the then Court of
First Instance of Iloilo, Special Proceeding No. 2176, for the probate of the disputed will,
which was opposed by the private respondents presently, Panfilo and Felino both
surnamed Maloto. The trial court dismissed the petition on April 30, 1970. Complaining
against the dismissal, again, the petitioners came to this Court on a petition for review by
certiorari. 2 Acting on the said petition, we set aside the trial court's order and directed it to
proceed to hear the case on the merits. The trial court, after hearing, found the will to have
already been revoked by the testatrix. Adriana Maloto, and thus, denied the petition. The
petitioners appealed the trial court's decision to the Intermediate Appellate Court which, on
June 7, 1985, affirmed the order. The petitioners' motion for reconsideration of the adverse
decision proved to be of no avail, hence, this petition.
For a better understanding of the controversy, a factual account would be a great help.
On October 20, 1963, Adriana Maloto died leaving as heirs her niece and nephews, the
petitioners Aldina Maloto-Casiano and Constancio, Maloto, and the private respondents
Panfilo Maloto and Felino Maloto. Believing that the deceased did not leave behind a last
will and testament, these four heirs commenced on November 4, 1963 an intestate
proceeding for the settlement of their aunt's estate. The case was instituted in the then
Court of First Instance of Iloilo and was docketed as Special Proceeding No. 1736.
However, while the case was still in progress, or to be exact on February 1, 1964, the
parties Aldina, Constancio, Panfilo, and Felino executed an agreement of
extrajudicial settlement of Adriana's estate. The agreement provided for the division of the
estate into four equal parts among the parties. The Malotos then presented the
extrajudicial settlement agreement to the trial court for approval which the court did on

69 | S U C C E S S I O N

March 21, 1964. That should have signalled the end of the controversy, but, unfortunately,
it had not.
Three years later, or sometime in March 1967, Atty. Sulpicio Palma, a former associate of
Adriana's counsel, the late Atty. Eliseo Hervas, discovered a document entitled
"KATAPUSAN NGA PAGBUBULAT-AN (Testamento)," dated January 3,1940, and
purporting to be the last will and testament of Adriana. Atty. Palma claimed to have found
the testament, the original copy, while he was going through some materials inside the
cabinet drawer formerly used by Atty. Hervas. The document was submitted to the office of
the clerk of the Court of First Instance of Iloilo on April 1, 1967. Incidentally, while Panfilo
and Felino are still named as heirs in the said will, Aldina and Constancio are bequeathed
much bigger and more valuable shares in the estate of Adriana than what they received by
virtue of the agreement of extrajudicial settlement they had earlier signed. The will likewise
gives devises and legacies to other parties, among them being the petitioners Asilo de
Molo, the Roman Catholic Church of Molo, and Purificacion Miraflor.
Thus, on May 24, 1967, Aldina and Constancio, joined by the other devisees and legatees
named in the will, filed in Special Proceeding No. 1736 a motion for reconsideration and
annulment of the proceedings therein and for the allowance of the will When the trial court
denied their motion, the petitioner came to us by way of a petition for certiorari and
mandamus assailing the orders of the trial court . 3 As we stated earlier, we dismissed that
petition and advised that a separate proceeding for the probate of the alleged will would be
the appropriate vehicle to thresh out the matters raised by the petitioners.
Significantly, the appellate court while finding as inconclusive the matter on whether or not
the document or papers allegedly burned by the househelp of Adriana, Guadalupe Maloto
Vda. de Coral, upon instructions of the testatrix, was indeed the will, contradicted itself and
found that the will had been revoked. The respondent court stated that the presence
of animus revocandi in the destruction of the will had, nevertheless, been sufficiently
proven. The appellate court based its finding on the facts that the document was not in the
two safes in Adriana's residence, by the testatrix going to the residence of Atty. Hervas to
retrieve a copy of the will left in the latter's possession, and, her seeking the services of
Atty. Palma in order to have a new will drawn up. For reasons shortly to be explained, we
do not view such facts, even considered collectively, as sufficient bases for the conclusion
that Adriana Maloto's will had been effectively revoked.
There is no doubt as to the testamentary capacity of the testatrix and the due execution of
the will. The heart of the case lies on the issue as to whether or not the will was revoked by
Adriana.
The provisions of the new Civil Code pertinent to the issue can be found in Article 830.
Art. 830. No will shall be revoked except in the following cases:

(1) By implication of law; or


(2) By some will, codicil, or other writing executed as provided in case
of wills: or
(3) By burning, tearing, cancelling, or obliterating the will with the
intention of revoking it, by the testator himself, or by some other person
in his presence, and by his express direction. If burned, torn cancelled,
or obliterated by some other person, without the express direction of
the testator, the will may still be established, and the estate distributed
in accordance therewith, if its contents, and due execution, and the fact
of its unauthorized destruction, cancellation, or obliteration are
established according to the Rules of Court. (Emphasis Supplied.)
It is clear that the physical act of destruction of a will, like burning in this case, does not per
se constitute an effective revocation, unless the destruction is coupled with animus
revocandi on the part of the testator. It is not imperative that the physical destruction be
done by the testator himself. It may be performed by another person but under the express
direction and in the presence of the testator. Of course, it goes without saying that the
document destroyed must be the will itself.
In this case, while animus revocandi or the intention to revoke, may be conceded, for that
is a state of mind, yet that requisite alone would not suffice. "Animus revocandi is only one
of the necessary elements for the effective revocation of a last will and testament. The
intention to revoke must be accompanied by the overt physical act of burning, tearing,
obliterating, or cancelling the will carried out by the testator or by another person in his
presence and under his express direction. There is paucity of evidence to show
compliance with these requirements. For one, the document or papers burned by Adriana's
maid, Guadalupe, was not satisfactorily established to be a will at all, much less the will of
Adriana Maloto. For another, the burning was not proven to have been done under the
express direction of Adriana. And then, the burning was not in her presence. Both
witnesses, Guadalupe and Eladio, were one in stating that they were the only ones present
at the place where the stove (presumably in the kitchen) was located in which the papers
proffered as a will were burned.
The respondent appellate court in assessing the evidence presented by the private
respondents as oppositors in the trial court, concluded that the testimony of the two
witnesses who testified in favor of the will's revocation appear "inconclusive." We share the
same view. Nowhere in the records before us does it appear that the two witnesses,
Guadalupe Vda. de Corral and Eladio Itchon, both illiterates, were unequivocably positive
that the document burned was indeed Adriana's will. Guadalupe, we think, believed that
the papers she destroyed was the will only because, according to her, Adriana told her so.

70 | S U C C E S S I O N

Eladio, on the other hand, obtained his information that the burned document was the will
because Guadalupe told him so, thus, his testimony on this point is double hearsay.
At this juncture, we reiterate that "(it) is an important matter of public interest that a
purported win is not denied legalization on dubious grounds. Otherwise, the very institution
of testamentary succession will be shaken to its very foundations ...." 4
The private respondents in their bid for the dismissal of the present action for probate
instituted by the petitioners argue that the same is already barred by res adjudicata. They
claim that this bar was brought about by the petitioners' failure to appeal timely from the
order dated November 16, 1968 of the trial court in the intestate proceeding (Special
Proceeding No. 1736) denying their (petitioners') motion to reopen the case, and their
prayer to annul the previous proceedings therein and to allow the last will and testament of
the late Adriana Maloto. This is untenable.
The doctrine of res adjudicata finds no application in the present controversy. For a
judgment to be a bar to a subsequent case, the following requisites must concur: (1) the
presence of a final former judgment; (2) the former judgment was rendered by a court
having jurisdiction over the subject matter and the parties; (3) the former judgment is a
judgment on the merits; and (4) there is, between the first and the second action, Identity
of parties, of subject matter, and of cause of action. 5 We do not find here the presence of
all the enumerated requisites.
For one, there is yet, strictly speaking, no final judgment rendered insofar as the probate of
Adriana Maloto's will is concerned. The decision of the trial court in Special Proceeding No.
1736, although final, involved only the intestate settlement of the estate of Adriana. As
such, that judgment could not in any manner be construed to be final with respect to the
probate of the subsequently discovered will of the decedent. Neither is it a judgment on the
merits of the action for probate. This is understandably so because the trial court, in the
intestate proceeding, was without jurisdiction to rule on the probate of the contested
will . 6 After all, an action for probate, as it implies, is founded on the presence of a will and
with the objective of proving its due execution and validity, something which can not be
properly done in an intestate settlement of estate proceeding which is predicated on the
assumption that the decedent left no will. Thus, there is likewise no Identity between the
cause of action in intestate proceeding and that in an action for probate. Be that as it may,
it would be remembered that it was precisely because of our ruling in G.R. No. L-30479
that the petitioners instituted this separate action for the probate of the late Adriana
Maloto's will. Hence, on these grounds alone, the position of the private respondents on
this score can not be sustained.
One last note. The private respondents point out that revocation could be inferred from the
fact that "(a) major and substantial bulk of the properties mentioned in the will had been
disposed of: while an insignificant portion of the properties remained at the time of death

(of the testatrix); and, furthermore, more valuable properties have been acquired after the
execution of the will on January 3,1940." 7 Suffice it to state here that as these additional
matters raised by the private respondents are extraneous to this special proceeding, they
could only be appropriately taken up after the will has been duly probated and a certificate
of its allowance issued.

Upon the issue thus presented, the Honorable Anastacio R. Teodoro, judge, after hearing
the respective parties, denied the probation of said will of April 16, 1919, upon the ground
that the same had been cancelled and revoked in the year 1920. Judge Teodoro, after
examining the evidence adduced, found that the following facts had been satisfactorily
proved:

WHEREFORE, judgment is hereby rendered REVERSING and SETTING ASIDE the


Decision dated June 7, 1985 and the Resolution dated October 22, 1986, of the
respondent Court of Appeals, and a new one ENTERED for the allowance of Adriana
Maloto's last will and testament. Costs against the private respondents.

That Exhibit A is a mere carbon of its original which remained in the possession
of the deceased testator Miguel Mamuyac, who revoked it before his death as
per testimony of witness Jose Fenoy, who typed the will of the testator on April
16, 1919, and Carlos Bejar, who saw on December 30, 1920, the original Exhibit
A (will of 1919) actually cancelled by the testator Miguel Mamuyac, who assured
Carlos Bejar that inasmuch as he had sold him a house and the land where the
house was built, he had to cancel it (the will of 1919), executing thereby a new
testament. Narcisa Gago in a way corroborates the testimony of Jose Fenoy,
admitting that the will executed by the deceased (Miguel Mamuyac) in 1919 was
found in the possession of father Miguel Mamuyac. The opponents have
successfully established the fact that father Miguel Mamuyac had executed in
1920 another will. The same Narcisa Gago, the sister of the deceased, who was
living in the house with him, when cross-examined by attorney for the opponents,
testified that the original Exhibit A could not be found. For the foregoing
consideration and for the reason that the original of Exhibit A has been cancelled
by the deceased father Miguel Mamuyac, the court disallows the probate of
Exhibit A for the applicant." From that order the petitioner appealed.

This Decision is IMMEDIATELY EXECUTORY.


SO ORDERED.

G.R. No. L-26317

January 29, 1927

Estate of Miguel Mamuyac, deceased.


FRANCISCO GAGO, petitioner-appellant,
vs.
CORNELIO MAMUYAC, AMBROSIO LARIOSA,
FELICIANA BAUZON, and CATALINA MAMUYAC, opponents-appellees.

The purpose of this action was to obtain the probation of a last will and testament of Miguel
Mamuyac, who died on the 2d day of January, 1922, in the municipality of Agoo of the
Province of La Union. It appears from the record that on or about the 27th day of July,
1918, the said Miguel Mamuyac executed a last will and testament (Exhibit A). In the
month of January, 1922, the said Francisco Gago presented a petition in the Court of First
Instance of the Province of La Union for the probation of that will. The probation of the
same was opposed by Cornelio Mamuyac, Ambrosio Lariosa, Feliciana Bauzon, and
Catalina Mamuyac (civil cause No. 1144, Province of La Union). After hearing all of the
parties the petition for the probation of said will was denied by the Honorable C. M.
Villareal on the 2d day of November, 1923, upon the ground that the deceased had on the
16th day of April, 1919, executed a new will and testament.
On the 21st day of February, 1925, the present action was commenced. Its purpose was to
secure the probation of the said will of the 16th day of April, 1919 (Exhibit 1). To said
petition Cornelio Mamuyac, Ambrosio Lariosa, Feliciana Bauzon, and Catalina Mamuyac
presented their oppositions, alleging (a) that the said will is a copy of the second will and
testament executed by the said Miguel Mamuyac; (b) that the same had been cancelled
and revoked during the lifetime of Miguel Mamuyac and (c) that the said will was not the
last will and testament of the deceased Miguel Mamuyac.

71 | S U C C E S S I O N

The appellant contends that the lower court committed an error in not finding from the
evidence that the will in question had been executed with all the formalities required by the
law; that the same had been revoked and cancelled in 1920 before his death; that the said
will was a mere carbon copy and that the oppositors were not estopped from alleging that
fact.
With reference to the said cancellation, it may be stated that there is positive proof, not
denied, which was accepted by the lower court, that will in question had been cancelled in
1920. The law does not require any evidence of the revocation or cancellation of a will to
be preserved. It therefore becomes difficult at times to prove the revocation or cancellation
of wills. The fact that such cancellation or revocation has taken place must either remain
unproved of be inferred from evidence showing that after due search the original will
cannot be found. Where a will which cannot be found is shown to have been in the
possession of the testator, when last seen, the presumption is, in the absence of other
competent evidence, that the same was cancelled or destroyed. The same presumption
arises where it is shown that the testator had ready access to the will and it cannot be
found after his death. It will not be presumed that such will has been destroyed by any
other person without the knowledge or authority of the testator. The force of the
presumption of cancellation or revocation by the testator, while varying greatly, being weak

or strong according to the circumstances, is never conclusive, but may be overcome by


proof that the will was not destroyed by the testator with intent to revoke it.
In view of the fat that the original will of 1919 could not be found after the death of the
testator Miguel Mamuyac and in view of the positive proof that the same had been
cancelled, we are forced to the conclusion that the conclusions of the lower court are in
accordance with the weight of the evidence. In a proceeding to probate a will the burden of
proofs is upon the proponent clearly to establish not only its execution but its existence.
Having proved its execution by the proponents, the burden is on the contestant to show
that it has been revoked. In a great majority of instances in which wills are destroyed for
the purpose of revoking them there is no witness to the act of cancellation or destruction
and all evidence of its cancellation perishes with the testator. Copies of wills should be
admitted by the courts with great caution. When it is proven, however, by proper testimony
that a will was executed in duplicate and each copy was executed with all the formalities
and requirements of the law, then the duplicate may be admitted in evidence when it is
made to appear that the original has been lost and was not cancelled or destroyed by the
testator. (Borromeo vs. Casquijo, G.R. No. L-26063.)1
After a careful examination of the entire record, we are fully persuaded that the will
presented for probate had been cancelled by the testator in 1920. Therefore the judgment
appealed from is hereby affirmed. And without any finding as to costs, it is so ordered.
Street, Malcolm, Villamor, Ostrand, Romualdez and Villa-Real, JJ., concur.

G.R. No. L-11823

February 11, 1918

CRISTINA SAMSON, DELFINA NAVAL, and SOR CONSOLACION


EUGENIO, petitioners-appellants,
vs.
MONICA NAVAL, ROSA NAVAL, and CELESTINA NAVAL, objectorsappellants.
ARAULLO, J.:
On September 20, 1915, attorney Perfecto Gabriel presented in the Court of First Instance
of the city of Manila for allowance as the will of Simeona F. Naval, who died in said city two
days previously, a document executed by her of February 13, 1915, and in which he was
appointed executor. The case was recorded as No. 13386 and, after hearing the petition
for allowance filed by said executor, it was denied on the ground that said document was
not duly executed by the deceased as her last will and testament, inasmuch as she did not
sign it in the presence of three witness and the two witnesses did not sign it in the
presence of each other. Thereafter the nieces and legatees of the same deceased filed in

72 | S U C C E S S I O N

the same court for allowance as her will, another document executed by her on October
31, 1914, and, consequently, the case was registered under another number, which was
No. 13579. The petition for allowance was opposed by Monica Naval, Rosa Naval, and
Cristina Naval on the ground that the will, the allowance of which is asked, could not be
allowed, because of the existence of another will of subsequent date, executed during her
lifetime by the same Simeona F. Naval, and because said will has been revoked by
another executed subsequently by her during her lifetime, and further, because sail will has
not been executed with the formalities required by existing laws. Trial having taken place,
at which evidence was adduced, the court on February 8, 1916, issued an order, admitting
said second document and ordering its allowance as the last will and testament o said
deceased. From said order the opponents appealed to this court and transmitted to us the
corresponding declarations. Tow of the opponents, that is, Rosa and Cristina Naval,
assigned, as errors committed by the court, the following:
1. The finding of the court that the will of October 31, 1914, has not been revoked by that
of February 13, 1915;
2. The act of the court in permitting the petitioner to institute and proceed with the
proceedings relative to the last case for the allowance of the will, No. 13579,
notwithstanding that proceedings had already been had in the other case No. 13386 and
final judgment rendered therein; and
3. The act of the court in denying the motion for continuance of the trial on the allowance of
the will of October 31, 1914, which motion was presented for the sole purpose of
introducing evidence to show the falsity of the signature appearing in said will and
submitting said signature to the Bureau of Science for analysis.
The other opponent, Monica Naval, assigned, besides the first two errors already
mentioned, the finding of the court that the disallowance of the will of said deceased, dated
February 13, 1915, on the ground that is was not executed in such form that it could
transmit real and personal property, according to section 618 of the Code of Civil
Procedure, also had the effect of annulling the revocatory clause in said will.
From the evidence it appears, as we have already stated, that the trial court declared that
the first document presented by the executor of the deceased, Simeona F. Naval, as a will
executed by her on February 13, 1915, and which was the subject-matter of case No.
13386 of said court, could not be allowed, on the ground that it was not executed with the
requisites and formalities prescribed by law. Article 739 of the Civil Code provides that a
former will is by operation of law revoked by another valid subsequent will, if the testator
does not state in the later will his desire that the former should subsist wholly or partly. In
harmony with this provision of substantive law, we find section 623 of the Code of Civil
Procedure, which provides that no will shall be revoked, except by implication of law,
otherwise than by some will, codicil, or other writing executed as provided in case of wills.

Therefore, according to the legal provisions, in order that the will of February 13, 1915, that
is, the first document presented as the will of the deceased Simeona F. Naval, could have
the effect of revoking that which was presented afterwards by the petitioners as executed
by the same deceased on October 31, 1914, that is, on a date previous to the execution of
the first, it was necessary and indispensable that the later will, that is, that first presented
for allowance, should be perfect or valid, that it, executed as provided by lay in case of
wills.
It also appears from the record that the opponents themselves maintained that said later
will, that is, that of February 13, 1915, was not perfect, or executed as provided by law in
case of wills, and the Court of First Instance of Manila has so held in disallowing said
documents as the will of the deceased. So that it very evident that the second will
presented, that is, that of October 31, 1914, was not and could not have been revoked by
the first, and the court was not in error in so holding in the order appealed from. We deem
it unnecessary to add a single word mere or cite well-known doctrines and opinions of
jurists in support of what has already been stated.
As to the second error assigned by the opponents, we believe it sufficient to refer to what
the court below stated in the judgment appealed from. It is as follows:
The court finds no incongruency in the presentation of a prior will when another
will of subsequent date has been disallowed. Disregarding the fact that the
petitioners in this case were not those who presented the will in No. 13386, in
which the petition was presented by the same D. Perfecto Gabriel as executor, it
is proper to take into account that the object of a petition for allowance is to ask
for an order declaring that a will has been executed in accordance with the
requisites and formalities required by law. This is a question for the court to
decide and is out of the control of the party who presents the will. The allowance
or disallowance of a will by a competent court depends upon whether the
evidence adduced at the trial shows or does not show that the formalities
required by law have been complied with, and this cannot be determined in
advance, as a general rule, by the person who presents the testament. for he has
not always concurred in or seen the execution of the will.
If, therefore, the personal who presents a will and asks that if be allowed does
not secure its allowance, and he has in his possession another will, or has
information that another exists, he does not contradict himself by asking for the
allowance of the will of earlier date merely because the later will was declared
invalid by the proper court. If in this case there is any who adopts a contradictory
position, it is the respondent himself, inasmuch as in case No. 13386 he alleged,
as a ground for the disallowance of the will then presented, that it was not
executed in accordance with the law, and now he maintains the contrary, for he
claims that said will revoked that which is now presented.

73 | S U C C E S S I O N

With respect to the third error, it is beyond doubt that the court did not commit it, for it
appears that when the examination of the witness, Cristina Samson, was finished and the
court told Attorney Lualhati, counsel for the respondents, to continue adducing his
evidence, he said he had no more proof, although he added that he would ask the court to
grant him permission to send the will of 1914 to the Bureau of Science, which petition was
objected to by the attorney for the proponents and denied by the court. Immediately
thereafter the attorney for the opponents asked for the continuance of the trial, which was
also denied by the court, after objection was made by the proponents. The attorney for the
opponents excepted to said ruling.
Therefore, the petition of said attorney for the remission of said will to the Bureau of
Science, in the terms in which it was made to the court, after ha had stated that he had no
more evidence to present, signified that he left it to the discretion of the court to grant it or
not. Furthermore, no exception was taken to the order to the order denying this motion,
and although the attorney for the opponents excepted to the order denying the motion for
continuance of the trial, such exception was completely useless and ineffective for the
purpose of alleging before this court that the trial court erred in that respect, for said
resolution, being one of those left to the discretion of the court in the exercise of it
functions, according to section 141 of the Code of Civil Procedure, it could not be the
subject of an exception, unless the court, in denying said motion, abused its discretional
power and thereby prejudiced the essential rights of the respondents, which is not the case
here.
The error which, in addition to the first two already mentioned, has been assigned by the
opponent and appellant, Monica Naval, and refers, according to her, to the court's action in
declaring that the disallowance of the will of the deceased Simeona F. Naval, dated
February 13, 1915, for the reason that it was not executed in such manner and from that it
could transmit real and personal property, according to the provisions of section 618 of the
Code of Civil Procedure, also had the effect of annulling the revocatory clause of said will.
First of all, it is not true that the court made such statement in the terms given in said
assignment of error, that is, it is not true that the court declared that, because said will was
not executed in the form required by law in order that it may transmit real and personal
property, according to the provisions of section 618, the disallowance of said will also had
the effect of annulling the revocatory clause therein contained. In the order appealed from
there is no declaration or conclusion made in these terms. The court did not say that the
annulment of the revocatory clause in said will was the effect or consequence of the fact
that it was not allowed on the ground that it was not executed in the form required by law in
order that it may transmit real and personal property. Referring to the construction, given
by the respondent to sections 618 and 623 of the Code of Civil Procedure, to the effect that
a subsequent will may revoke a previous will, although the later will has not been allowed
by the competent court, it being sufficient that the intention of the testator to revoke the
previous will should be clearly expressed, and that, while the requisite of allowance is
necessary in order that it may transmit property from one person to another, it is not

necessary in order that it might procedure other effects, for example, the effect of a
revocatory clause, or a clause of aknowledgment of a child, what the court declared, we
repeat, was that although the revocation of a will should have been effected, not by means
of another will or codicil, but by mans of a document, as authorized by said section 623,
which document should have the requisites and conditions fixed in section 618, the
presentation of the document to the court was necessary in order that the latter might allow
it, by declaring that it was executed with the formalities required by law for the execution of
a will, and finally concluding that, just as to, is to be proved that the requisites of section
618 have been complied with in order that a will may be of value through its allowance, so
without such allowance the revocatory clause like the other provisions of the will, has no
value or effect except to show extraneous matters, as, for example, the acknowledgment
of natural children, of some debt or obligation. In such case, the document could produce
effect, but not as will, but simply as a written admission made by the person executing it.
And It is beyond doubt that the revocatory clause contained in a document, like the
present, which contains provisions proper of a will, as those relating to legacies and
distribution of the properties of the testator after his death as well as the appointment of
executors, is not matter extraneous to the will, but merely a part thereof, intimately
connected with it as well as with the will or wills, the revocation of which is declared in said
clause; in short, the desire of the testator declared in the revocatory clause is related to the
desire of the same testator expressed in the provisions of the testament in which said
clause is found and to that which he might have expressed in the testaments which he may
have previously executed. There is such relation between the revocatory clause and the
will which contains it, that if the will does not produce legal effects, because it has not been
executed in accordance with the provisions of the law, neither would the revocatory clause
therein produce legal effects. And if, in the present case, the so-called will of the deceased,
Simeona F. Naval, dated February 13, 1915, was not duly executed by her as her last will
and testament, ad declared by the court in its decision of November 19, 1915, in case No.
13386, for which reason its allowance was denied, neither may it be maintained that the
revocatory clause contained in said will is the expression of the last will of said deceased.
The disallowance of the ill, therefore, produced the effect of annulling the revocatory
clause, not exactly because said will was not executed in such from that it could transmit
real and personal property, as inaccurately alleged by the appellant, Monica Naval, to be
the court's finding, upon which said assignment of error is based, but because it was
proved that said will was not executed or signed with the formalities and requisites required
by section 618 of the Code of Civil Procedure, a cause which also produces the nullity of
the same will, according to section 634 of said law; and of course what is invalid in law can
produce no effect whatever.
If the instrument propounded as a revocation be in form a will, it must be perfect
as such, and be subscribed and attested as is required by the statute. An
instrument intended to be a will, but filing of its effect as such on account of some
imperfection in its structure or for want of due execution, cannot be set up for the
purpose of revoking a former will. (40 Cyc., p. 1177, and cases cited therein.)

74 | S U C C E S S I O N

A subsequent will containing a clause revoking an earlier will must, as a general


rule, be admitted to probate before the clause of revocation can have any effect,
and the same kind, quality, and method of proof is required for the establishment
of the subsequent will as was required for the establishment of the former will.
(40 Cyc., p. 1178, and cases cited therein.)
But admitting that the will said to have been executed by the deceased Simeona F. Naval
on February 13, 1915, notwithstanding its inefficacy to transmit property for the reason that
it has not been executed, according to the provisions of said section 618 of the Code of
Civil Procedure, should be considered as executed by her in order to express her desire,
appearing in one of its clauses, to revoke and annul any previous will of hers, as stated in
clause 13, this being the argument adduced by the appellant, Monica naval, in support of
said assignment of error neither could it be maintained that, the allowance of said will
having been denied by the court on November 11, 1915, said revocatory clause subsists
and the intention expressed by the testratrix therein is valid and legally effective, for the
simple reason that, in order that a will may be revoked by a document, it is necessary,
according to the conclusive provisions of section 623 of said procedural law, that such
documents be executed according to the provisions relating to will in section 618, and the
will in question, or, according to the respondent, the so-called document, was not executed
according to the provisions of said section, according to the express finding of the trial
court in its order of November 11, 1915, acquiesced in by the opponent herself, and which
is now final and executory. Therefore, the disallowance of said will and the declaration that
it was not executed according to the provisions of law as to wills, produced the effect of
annulling said revocatory clause.
In support of the argument advanced in her brief said appellant, Monica Naval, cites the
declaration made by the Supreme Court of Massachusetts in Wallis vs. Wallis (114 Mass.,
510, 512)m which, according to the appellant herself, was in the following terms:
If it be shown that a later will was duly executed and attested, containing a clause
expressly revoking former will nothing else appearing as to its contents, it is
nevertheless good as a revocation, but it can only be made available by setting it
up in opposition to the probate of the earlier will.
In the decision of said case the finding referred to be by the appellant appears not to have
been made by the Supreme Court of Massachusetts.
The syllabus of said decision says:
When a will revoking a former will is in existence, it must be established in the
Probate Court; but when it has been lost or destroyed, and its contents cannot be
sufficiently proved to admit it to probate, it may nevertheless be availed of as a
revocation in opposition to the probate of the will revoked by it.:

And in the body of the decision there is a declaration, to which the appellant must have
desired to refer in her brief, which declaration says:
If it can be proved that a later will was duly executed, attested and subscribed,
and that it contained a clause expressly revoking all former wills, but evidence of
the rest of its contents cannot be obtained, it is nevertheless a good revocation;
and it can be made available only by allowing it to be set up in opposition to the
probate of the earlier will,. . .
The facts of the case in which this decision was rendered are different from the facts of the
case at bar. That was a case concerning a will filed by one of the children of the testatrix,
Mary Wallis, as her last will, to the allowance of which another son objected, alleging that
said will had been revoked by another executed by the same deceased subsequent to the
will that was filed, and that it had been fraudulently destroyed or taken by his brother, the
proponent and his wife, or by one of them, in order to deprive him of the rights conferred
upon him by said will. Therefore, the will said to have been subsequently executed by the
testatrix and in which, according to the oppositor, the clause revocatory of the former will
appeared, was not presented by said oppositor, while the previous will was, in the contrary,
filed for allowance by the son of the testratrix, who appeared to be favored therein, said
oppositor having alleged that the subsequent will, that is, that containing the revocatory
clause, had been drawn, subscribed and executed in accordance with the provisions of the
law, a fact which he was ready to prove just as he was ready to prove that it had been
destroyed or suppressed by the proponent, his brother and his wife, or one of them. In the
case at bar, the subsequent will containing the revocatory clause of the previous will
executed by the deceased Simeona F. Naval was presented to the court for allowance and
it was disallowed a fact which gave opportunity to the legatees of said deceased to
present a previous will executed by her on October 31, 1914, and said two wills having
been successively presented, evidence as to them was also successively adduced for their
allowance by the court.
Therefore, the declaration made by the Supreme Court of Massachusetts in Wallis vs.
Wallis (supra), to the effect that a subsequent will containing a revocatory clause of
previous wills, constitutes a valid revocation and may be used in objecting to the allowance
of the previous will, even when it is not possible to obtain proof of the remainder of the
contents of said subsequent will, refers to the case in which the latter had been taken
away, destroyed or suppressed, and it was impossible to present it for allowance, but
requires for that purpose that it be proved that said subsequent will has been executed,
attested, and subscribed in due form and that it contained, furthermore, that revocatory
clause. This is what said declaration and, in relation thereto, also what the syllabus of the
decision thereof clearly says. The court, through Chief Justice Gray, in giving its opinion,
thus began by saying:
By our law, no will can be revoked by any subsequent instrument, other than a
"will, codicil or writing, signed, attested and subscribed in the manner provided

75 | S U C C E S S I O N

for making a will." And when an instrument of revocation is in existence and


capable of being propounded for probate, its validity should be tried by a direct
proceeding instituted for the purpose in the Probate Court. (Loughton vs. Atkins,
1 Pick., 535.)
It results, therefore, that while perfect parity does not exist between the case decided by
the Supreme Court of Massachusetts, to which the appellant Monica Naval refers, and that
which is not before us, it is wholly unquestionable that, whether the case deals with a
subsequent will revocatory of a previous will, which may possibly be presented to a
probate court for allowance, or of a subsequent will, also revocatory of a previous will,
which could not be presented for allowance, because it has been taken or hidding, or
mislaid in order that such will may constitute a valid revocation and be utilized in the
second case, although the remaining provisions may not be proven, in opposition to the
allowance of the previous will, it is necessary to prove that it was executed, attested, and
subscribed in due form, and, of course, also that it contained a clause expressly revoking
the previous will, or, what is the same thing, that said subsequent will has been executed
according to the provisions relating to wills, as expressed in section 623 of the procedural
law in force. There can be no doubt whatever that this applies when the revocation had
been made to appear in a writing or document susceptible of presentation for allowance,
like the so-called will of the deceased Simeona F. Naval, dated February 13, 1915, and
considered by said respondent and appellant as a mere document of revocation, for, as
already seen in said decision invoked by her, the requisite as to signing, attesting, and
subscribing in the form, required by law for the execution of wills in order that it may revoke
a previous will, is also required in a will as well as in a codicil, or in a writing, and in
referring to a document of revocation, it is also expressed that its validity should be proved
in a direct proceeding, instituted for the purpose in a probate court. In the case at bar, the
document, executed by the deceased, Simeona F. Naval, as her last will and testament,
dated February 13, 1915, has been presented for allowance; it validity has been proved by
means of said procedure in the Court of Probate of Manila, and that court denied its
allowance, on the ground that the document in question had not been duly executed by the
deceased, as her last will and testament, because she did not sign in the presence of three
witnesses, and two of these witnesses did not sign in the presence of each other, or what
is the same thing, that said document has not be attested and subscribed in the manner
established by law for the execution of will, or, in other words, as provided by law in case of
wills, as stated by section 623 of said procedural law, and this resolution was acquiesced
in, as already stated, by the respondents in this case, and is, therefore, final and executory.
In conclusions, the doctrine laid down in the decision of the Supreme Court of
Massachusetts, invoked by the appellant, Monica Naval, is in conformity with the provision
of said section 623 of our procedural law and article 739 of the Civil Code, and the will
executed by the deceased Simeona F. Naval on October 31, 1914, not having been
revoked, according to these provisions, by the will presented and alleged as executed by
the same deceased subsequently on February 13, 1915, the allowance of which was
denied by the Court of First Instance of Manila, the court below was not in error in ordering

the allowance of said will, that is, of that of October 31, 1914, as the last will and testament
of said deceased.
Wherefore, the order appealed from is affirmed, with the costs of this instance against the
appellants. So ordered.
Arellano, C.J., Torres, Carson, Streets and Malcolm, JJ., concur.

G.R. No. L-2538

September 21, 1951

Testate Estate of the Deceased MARIANO MOLO Y LEGASPI. JUANA


JUAN VDA. DE MOLO, petitioner-appellee,
vs.
LUZ, GLICERIA and CORNELIO MOLO, oppositors-appellants.
BAUTISTA ANGELO, J.:
This is an appeal from an order of the Court of First Instance of Rizal admitting to probate
the last will and testament of the deceased Mariano Molo y Legaspi executed on August
17, 1918. The oppositors-appellants brought the case on appeal to this Court for the
reason that the value of the properties involved exceeds P50,000.
Mariano Molo y Legaspi died on January 24, 1941, in the municipality of Pasay, province of
Rizal, without leaving any forced heir either in the descending or ascending line. He was
survived, however, by his wife, the herein petitioner Juana Juan Vda. de Molo, and by his
nieces and nephew, the oppositors-appellants, Luz Gliceria and Cornelio, all surnamed
Molo, who were the legitimate children of Candido Molo y Legaspi, deceased brother of the
testator. Mariano Molo y Legaspi left two wills, one executed on August 17, 1918, (Exhibit
A) and another executed on June 20, 1939. (Exhibit I). The later will executed in 1918.
On February 7, 1941, Juana Juan Vda. de Molo, filed in the Court of First Instance of Rizal
a petition, which was docketed as special proceeding No. 8022 seeking the probate of the
will executed by the deceased on June 20, 1939. There being no opposition, the will was
probated. However, upon petition filed by the herein oppositors, the order of the court
admitting the will to probate was set aside and the case was reopened. After hearing, at
which both parties presented their evidence, the court rendered decision denying the
probate of said will on the ground that the petitioner failed to prove that the same was
executed in accordance with law.
In view of the disallowance of the will executed on June 20, 1939, the widow on February
24, 1944, filed another petition for the probate of the will executed by the deceased on
August 17, 1918, which was docketed as special proceeding No. 56, in the same court.
Again, the same oppositors filed an opposition to the petition based on three grounds: (1)

76 | S U C C E S S I O N

that petitioner is now estopped from seeking the probate of the will of 1918; (2) that said
will has not been executed in the manner required by law and (3) that the will has been
subsequently revoked. But before the second petition could be heard, the battle for
liberation came and the records of the case were destroyed. Consequently, a petition for
reconstitution was filed, but the same was found to be impossible because neither
petitioner nor oppositors could produce the copies required for its reconstitution. As a
result, petitioner filed a new petition on September 14, 1946, similar to the one destroyed,
to which the oppositors filed an opposition based on the same grounds as those contained
in their former opposition. Then, the case was set for trial, and on May 28, 1948, the court
issued an order admitting the will to probate already stated in the early part of this decision.
From this order the oppositors appealed assigning six errors, to wit.
I. The probate court erred in not holding that the present petitioner voluntarily and
deliberately frustrated the probate of the will dated June 20, 1939, in special
proceeding No. 8022, in order to enable her to obtain the probate of another
alleged will of Molo dated 191.
II. The court a quo erred in not holding that the petitioner is now estopped from
seeking the probate of Molo's alleged will of 1918.
III. The lower court erred in not holding that petitioner herein has come to court
with "unclean hands" and as such is not entitled to relief.
IV. The probate court erred in not holding that Molo's alleged will of August 17,
1918 was not executed in the manner required by law.
V. The probate court erred in not holding that the alleged will of 1918 was
deliberately revoked by Molo himself.
VI. The lower court erred in not holding that Molo's will of 1918 was subsequently
revoked by the decedent's will of 1939.
In their first assignment of error, counsel for oppositors contend that the probate court
erred in not holding that the petitioner voluntarily and deliberately frustrated the probate of
the will dated June 20, 1939, in order to enable her to obtain the probate of the will
executed by the deceased on August 17, 1918, pointing out certain facts and
circumstances with their opinion indicate that petitioner connived with the witness Canuto
Perez in an effort to defeat and frustrate the probate of the 1939 will because of her
knowledge that said will intrinsically defective in that "the one and only testamentory
disposition thereof was a "disposicion captatoria". These circumstances, counsel for the
appellants contend, constitute a series of steps deliberately taken by petitioner with a view
to insuring the realization of her plan of securing the probate of the 1918 will which she
believed would better safeguard her right to inherit from the decease.

These imputations of fraud and bad faith allegedly committed in connection with special
proceedings No. 8022, now closed and terminated, are vigorously met by counsel for
petitioner who contends that to raise them in these proceedings which are entirely new and
distinct and completely independent from the other is improper and unfair as they find no
support whatsoever in any evidence submitted by the parties in this case. They are merely
based on the presumptions and conjectures not supported by any proof. For this reason,
counsel, contends, the lower court was justified in disregarding them and in passing them
sub silentio in its decision.
A careful examination of the evidence available in this case seems to justify this
contention. There is indeed no evidence which may justify the insinuation that petitioner
had deliberately intended to frustrate the probate of the 1939 will of the deceased to
enable her to seek the probate of another will other than a mere conjecture drawn from the
apparently unexpected testimony of Canuto Perez that he went out of the room to answer
an urgent call of nature when Artemio Reyes was signing the will and the failure of
petitioner later to impeach the character of said witness in spite of the opportunity given
her by the court to do so. Apart from this insufficiency of evidence, the record discloses
that this failure has been explained by petitioner when she informed the court that she was
unable to impeach the character of her witness Canuto Perez because of her inability to
find witnesses who may impeach him, and this explanation stands uncontradicted.
Whether this explanation is satisfactory or not, it is not now, for us to determine. It is an
incident that comes within the province of the former case. The failure of petitioner to
present the testimony of Artemio Reyes at the hearing has also been explained, and it
appears that petitioner has filed because his whereabouts could not be found. Whether this
is true or not is also for this Court to determine. It is likewise within the province and
function of the court in the former case. And the unfairness of this imputation becomes
more glaring when we stock of the developments that had taken place in these
proceedings which show in bold relief the true nature of the conduct, behavior and
character of the petitioner so bitterly assailed and held in disrepute by the oppositors.
It should be recalled that the first petition for the probate of the will executed on June 20,
1939, was filed on February 7, 1941, by the petitioner. There being no opposition, the will
was probated. Subsequently, however, upon petition of the herein oppositors, the order of
the court admitting said will to probate was set aside, over the vigorous opposition of the
herein petitioner, and the case was reopened. The reopening was ordered because of the
strong opposition of the oppositors who contended that he will had not been executed as
required by law. After the evidence of both parties had been presented, the oppositors filed
an extensive memorandum wherein they reiterated their view that the will should be denied
probate. And on the strenght of this opposition, the court disallowed the will.
If petitioner then knew that the 1939 will was inherently defective and would make the
testamentary disposition in her favor invalid and ineffective, because it is a "disposicion
captatoria", which knowledge she may easily acquire through consultation with a lawyer,
there was no need her to go through the order of filing the petition for the probate of the

77 | S U C C E S S I O N

will. She could accomplish her desire by merely suppressing the will or tearing or
destroying it, and then take steps leading to the probate of the will executed in 1918. But
for her conscience was clear and bade her to take the only proper step possible under the
circumstances, which is to institute the necessary proceedings for the probate of the 1939
will. This she did and the will was admitted to probate. But then the unexpected happened.
Over her vigorous opposition, the herein appellants filed a petition for reopening, and over
her vigorous objection, the same was granted and the case was reopened. Her motion for
reconsideration was denied. Is it her fault that the case was reopened? Is it her fault that
the order admitting the will to probate was set aside? That was a contingency which
petitioner never expected. Had appellants not filed their opposition to the probate of the will
and had they limited their objection to the intrinsic validity of said will, their plan to defeat
the will and secure the intestacy of the deceased would have perhaps been accomplished.
But they failed in their strategy. If said will was denied probate it is due to their own effort. It
is now unfair to impute bad faith petitioner simply because she exerted every effort to
protect her own interest and prevent the intestacy of the deceased to happen.
Having reached the foregoing conclusions, it is obvious that the court did not commit the
second and third errors imputed to it by the counsel for appellants. Indeed, petitioner
cannot be considered guilty or estoppel which would prevent her from seeking the probate
of the 1918 will simply because of her effort to obtain the allowance of the 1939 will has
failed considering that in both the 1918 and 1939 wills she was in by her husband as his
universal heir. Nor can she be charged with bad faith far having done so because of her
desire to prevent the intestacy of her husband. She cannot be blamed being zealous in
protecting her interest.
The next contention of appellants refers to the revocatory clause contained in 1939 will of
the deceased which was denied probate. They contend that, notwithstanding the
disallowance of said will, the revocatory clause is valid and still has the effect of nullifying
the prior of 1918.
Counsel for petitioner meets this argument by invoking the doctrine laid down in the case
of Samson vs. Naval, (41 Phil., 838). He contends that the facts involved in that case are
on all fours with the facts of this case. Hence, the doctrine is that case is here controlling.
There is merit in this contention. We have carefully read the facts involved in the Samson
case we are indeed impressed by their striking similarity with the facts of this case. We do
not need to recite here what those facts are; it is enough to point out that they contain
many points and circumstances in common. No reason, therefore, is seen by the doctrine
laid down in that case (which we quote hereunder) should not apply and control the
present case.
A subsequent will, containing a clause revoking a previous will, having been
disallowed, for the reason that it was not executed in conformity with the

provisions of section 618 of the Code of Civil Procedure as to the making of wills,
cannot produce the effect of annulling the previous will, inasmuch as said
revocatory clause is void. (41 Phil., 838.)
Apropos of this question, counsel for oppositors make the remark that, while they do not
disagree with the soundness of the ruling laid down in the Samson case, there is reason to
abandon said ruling because it is archaic or antiquated and runs counter to the modern
trend prevailing in American jurisprudence. They maintain that said ruling is no longer
controlling but merely represents the point of view of the minority and should, therefore, be
abandoned, more so if we consider the fact that section 623 of our Code of Civil
Procedure, which governs the revocation of wills, is of American origin and as such should
follow the prevailing trend of the majority view in the United States. A long line of
authorities is cited in support of this contention. And these authorities hold the view, that
"an express revocation is immediately effective upon the execution of the subsequent will,
and does not require that it first undergo the formality of a probate proceeding". (p. 63,
appellants' brief .
While they are many cases which uphold the view entertained by counsel for oppositors,
and that view appears to be in controlling the states where the decisions had been
promulgated, however, we are reluctant to fall in line with the assertion that is now the
prevailing view in the United States. In the search we have made of American authorities
on the subject, we found ourselves in a pool of conflicting opinions perhaps because of the
peculiar provisions contained in the statutes adopted by each State in the subject of
revocation of wills. But the impression we gathered from a review and the study of the
pertinent authorities is that the doctrine laid down in the Samson case is still a good law.
On page 328 of the American Jurisprudence Vol. 57, which is a revision Published in 1948,
we found the following passages which in our opinion truly reflect the present trend of
American jurisprudence on this matter affecting the revocation of wills:
SEC. 471. Observance of Formalities in Execution of Instrument. Ordinarily,
statutes which permit the revocation of a will by another writing provide that to be
effective as a revocation, the writing must be executed with the same formalities
which are required to be observed in the execution of a will. Accordingly, where,
under the statutes, attestation is necessary to the making of a valid will, an
unattested non testamentary writing is not effective to revoke a prior will. It has
been held that a writing fails as a revoking instrument where it is not executed
with the formalities requisite for the execution of a will, even though it is inscribed
on the will itself, although it may effect a revocation by cancellation or obliteration
of the words of the will. A testator cannot reserve to himself the power to modify a
will by a written instrument subsequently prepared but not executed in the
manner required for a will.
SEC, 472. Subsequent Unexecuted, Invalid, or Ineffective Will or Codicil. A will
which is invalid because of the incapacity of the testator, or of undue influence

78 | S U C C E S S I O N

can have no effect whatever as a revoking will. Moreover, a will is not revoked by
the unexecuted draft of a later one. Nor is a will revoked by a defectively
executed will or codicil, even though the latter contains a clause expressly
revoking the former will, in a jurisdiction where it is provided by a controlling
statute that no writing other than a testamentary instrument is sufficient to revoke
a will, for the simple reason that there is no revoking will. Similarly where the
statute provides that a will may be revoked by a subsequent will or other writing
executed with the same formalities as are required in the execution of wills, a
defectively executed will does not revoke a prior will, since it cannot be said that
there is a writing which complies with the statute. Moreover, a will or codicil
which, on account of the manner in which it is executed, is sufficient to pass only
personally does not affect dispositions of real estate made by a former will, even
though it may expressly purport to do so. The intent of the testator to revoke is
immaterial, if he has not complied with the statute. (57 Am. Jur., 328, 329.)
We find the same opinion in the American Law Reports, Annotated, edited in 1939. On
page 1400, Volume 123, there appear many authorities on the "application of rules where
second will is invalid", among which a typical one is the following:
It is universally agreed that where the second will is invalid on account of not
being executed in accordance with the provisions of the statute, or where the
testator who has not sufficient mental capacity to make a will or the will is
procured through undue influence, or the such, in other words, where the second
will is really no will, it does not revoke the first will or affect it in any manner. Mort
vs. Baker University (193-5) 229 Mo. App., 632, 78 S.W. (2d), 498.
These treaties cannot be mistaken. They uphold the view on which the ruling in the
Samson case is predicated. They reflect the opinion that this ruling is sound and good and
for this reason, we see no justification for abondoning it as now suggested by counsel for
the oppositors.
It is true that our law on the matter (sec. 623, Code Civil Procedure) provides that a will
may be some will, codicil, or other writing executed as proved in case of wills" but it cannot
be said that the 1939 will should be regarded, not as a will within the meaning of said word,
but as "other writing executed as provided in the case of wills", simply because it was
denied probate. And even if it be regarded as any other writing within the meaning of said
clause, there is authority for holding that unless said writing is admitted to probate, it
cannot have the effect of revocation. (See 57 Am. Jur. pp. 329-330).
But counsel for oppositors contemned that, regardless of said revocatory clause, said will
of 1918 cannot still be given effect because of the presumption that it was deliberately
revoked by the testator himself. The oppositors contend that the testator, after executing
the 1939 will, and with full knowledge of the recovatory clause contained said will, himself

deliberately destroyed the original of the 1918 will, and for that reason the will submitted by
petitioner for probate in these proceedings is only a duplicate of said original.
There is no evidence which may directly indicate that the testator deliberately destroyed
the original of the 1918 will because of his knowledge of the revocatory clause contained in
the will he executed in 1939. The only evidence we have is that when the first will was
executed in 1918, Juan Salcedo, who prepared it, gave the original and copies to the
testator himself and apparently they remained in his possession until he executed his
second will in 1939. And when the 1939 will was denied probate on November 29, 1943,
and petitioner was asked by her attorney to look for another will, she found the duplicate
copy (Exhibit A) among the papers or files of the testator. She did not find the original.
If it can be inferred that the testator deliberately destroyed the 1918 will because of his
knowledge of the revocatory clause of the 1939 will, and it is true that he gave a duplicate
copy thereof to his wife, the herein petitioner, the most logical step for the testator to take is
to recall said duplicate copy in order that it may likewise be destroyed. But this was not
done as shown by the fact that said duplicate copy remained in the possession of
petitioner. It is possible that because of the long lapse of twenty-one (21) years since the
first will was executed, the original of the will had been misplaced or lost, and forgetting
that there was a copy, the testator deemed it wise to execute another will containing
exactly the same testamentary dispositions. Whatever may be the conclusion we may
draw from this chain of circumstances, the stubborn fact is that there is no direct evidence
of voluntary or deliberate destruction of the first will by the testator. This matter cannot be
inference or conjectur.
Granting for the sake of argument that the earlier will was voluntarily destroyed by the
testator after the execution of the second will, which revoked the first, could there be any
doubt, under this theory, that said earlier will was destroyed by the testator in the honest
belief that it was no longer necessary because he had expressly revoked it in his will of
1939? In other words, can we not say that the destruction of the earlier will was but the
necessary consequence of the testator's belief that the revocatory clause contained in the
subsequent will was valid and the latter would be given effect? If such is the case, then it is
our opinion that the earlier will can still be admitted to probate under the principle of
"dependent relative revocation".
This doctrine is known as that of dependent relative revocation, and is usually
applied where the testator cancels or destroys a will or executes an instrument
intended to revoke a will with a present intention to make a new testamentary
disposition as a substitute for the old, and the new disposition is not made or, if
made, fails of effect for same reason. The doctrine is n limited to the existence of
some other document, however, and has been applied where a will was
destroyed as a consequence of a mistake of law. . . . (68 C.J.P. 799).

The rule is established that where the act of destruction is connected with the
making of another will so as fairly to raise the inference that the testator meant
the revocation of the old to depend upon the efficacy of a new disposition
intended to be substituted, the revocation will be conditional and dependent upon
the efficacy of the new disposition; and if, for any reason, the new will intended to
be made as a substitute is inoperative, the revocation fails and the original will
remains in full force. (Gardner, pp. 232, 233.)
This is the doctrine of dependent relative revocation. The failure of a new
testamentary disposition upon whose validity the revocation depends, is
equivalent to the non-fulfillment of a suspensive conditions, and hence prevents
the revocation of the original will. But a mere intent to make at some time a will in
the place of that destroyed will not render the destruction conditional. It must
appear that the revocation is dependent upon the valid execution of a new will. (1
Alexander, p. 751; Gardner, p. 253.)
We hold therefore, that even in the supposition that the destruction of the original will by
the testator could be presumed from the failure of the petitioner to produce it in court, such
destruction cannot have the effect of defeating the prior will of 1918 because of the fact
that it is founded on the mistaken belief that the will of 1939 has been validly executed and
would be given due effect. The theory on which this principle is predicated is that the
testator did not intend to die intestate. And this intention is clearly manifest when he
executed two wills on two different occasion and instituted his wife as his universal heir.
There can therefore be no mistake as to his intention of dying testate.
The remaining question to be determined refers to the sufficiency of the evidence to prove
the due execution of the will.
The will in question was attested, as required by law, by three witnesses, Lorenzo Morales,
Rufino Enriquez, and Angel Cuenca. The first two witnesses died before the
commencement of the present proceedings. So the only instrumental witness available
was Angel Cuenca and under our law and precedents, his testimony is sufficient to prove
the due execution of the will. However, petitioner presented not only the testimony of
Cuenca but placed on the witness stand Juan Salcedo, the notary public who prepared
and notarized the will upon the express desire and instruction of the testator, The
testimony of these witnesses shows that the will had been executed in the manner
required by law. We have read their testimony and we were impressed by their readiness
and sincerity. We are convinced that they told the truth.
Wherefore, the order appealed from is hereby affirmed, with costs against the
appellants.1wphl.nt
G.R. No. 17714
May 31, 1922
Paras,
C.J.
Feria,
Bengzon,
Tuasonde
and
Jugo JJ., concur.
In the
mater
ofPablo
the estate
of Jesus
Leon.

79 | S U C C E S S I O N

IGNACIA DIAZ, petitioner-appellant,


vs.
ANA DE LEON, opponent-appellee

.
STREET, J.:
ROMUALDEZ, J.:
The only question raised in this case is whether or to the will executed by Jesus de Leon,
now, was revoked by him.
The petitioner denies such revocation, while the contestant affirms the same by alleging
that the testator revoked his will by destroying it, and by executing another will expressly
revoking the former.
We find that the second will Exhibit 1 executed by the deceased is not cloth with all the
necessary requisites to constitute a sufficient revocation.
But according to the statute governing the subject in this jurisdiction, the destruction of a
will animo revocandiconstitutes, in itself, a sufficient revocation. (Sec. 623, Code of Civil
Procedure.)lvvph1n+
From the evidence submitted in this case, it appears that the testator, shortly after the
execution of the first will in question, asked that the same be returned to him. The
instrument was returned to the testator who ordered his servant to tear the document. This
was done in his presence and before a nurse who testified to this effect. After some time,
the testator, being asked by Dr. Cornelio Mapa about the will, said that it had been
destroyed.
The intention of revoking the will is manifest from the established fact that the testator was
anxious to withdraw or change the provisions he had made in his first will. This fact is
disclosed by the testator's own statements to the witnesses Canto and the Mother Superior
of the Hospital where he was confined.
The original will herein presented for probate having been destroyed with animo revocandi,
cannot now be probated as the will and last testament of Jesus de Leon.
Judgement is affirmed with costs against the petitioner. So ordered.

G.R. No. L-35993

December 19, 1932

In re Estate of the deceased Gregorio Tolentino. ADELAIDA


vs. NATALIA FRANCISCO, ET
80TOLENTINO,petitioner-appellee,
| SUCCESSION
AL., oppositors-appellants.

This petition was filed in the Court of First Instance of Manila by Adelaida Tolentino de
Concepcion, for the purpose of procuring probate of the will of Gregorio Tolentino,
deceased, who died at the hand of an assassin, in his home, No. 2541 Lico Street, in the
District of Santa Cruz, Manila, on November 9, 1930. In the inception of the proceedings
Eugene de Mitkiewicz was appointed special coadministrator, and he joined as coplaintiff
in the petition. Opposition was made to the probate of the will by Ciriaco Francisco, Natalia
Francisco, and Gervasia Francisco, all cousins of the deceased and residents of the City of
Manila. Upon hearing the cause the trial court overruled the opposition, declared the will to
have been properly executed, and allowed the probate thereof. From this order the three
opponents appealed.chanroblesvirtualawlibrary chanrobles virtual law library
At the time of his death on November 9, 1930, Gregorio Tolentino was sixty-six years of
age. During the more vigorous years of his life he had been married to Benita Francisco,
but she predeceased him years ago. By their industry and frugality the two had
accumulated a very considerable estate which does not appear to have suffered any
material diminution in the years of Tolentino's widowhood. The pair had no children, and
the generous instincts of the survivor prompted him to gather around him in his
comfortable and commodious home a number of his wife's kin; and by him various younger
members of the connection were supported and educated. At one time Tolentino
contemplated leaving his property mainly to these kin of his wife, of the surname
Francisco; and for several years prior to his death, he had kept a will indicating this desire.
However, in October, 1930, strained relations, resulting from grave disagreements,
developed between Tolentino and the Francisco relations and he determined to make a
new will in which, apart from certain legacies in favor of a few individuals, the bulk of his
estate, worth probably about P150,000, should be given to Adelaida Tolentino de
Concepcion, as his universal heir.chanroblesvirtualawlibrary chanrobles virtual law library
To this end, on October 17, 1930, Tolentino went to the office of Eduardo Gutierrez Repide,
an attorney at 97 General Luna, Manila, and informed him that he wanted to make a new
will and desired Repide to draft it for him. After the necessary preliminary inquiries had
been made, the attorney suggested to Tolentino to return later, bringing a copy of the will
previously made. Accordingly, on the second day thereafter, Tolentino again appeared in
Repide's office with the prior will; and the attorney proceeded to reduce the new will to
proper form. As the instrument was taking shape Tolentino stated that he wanted the will to
be signed in Repide's office, with Repide himself as one of the attesting witnesses. For the

other two witnesses Tolentino requested that two attorneys attached to the office, namely,
Leoncio B. Monzon and Ramon L. Sunico, should serve. For this reason, in the draft of the
will, as it at first stood, the names of the three above mentioned were inserted as the
names of the three attesting witnesses.chanroblesvirtualawlibrary chanrobles virtual law
library
When the instrument had been reduced to proper form it was placed in the hands of
Tolentino, the testator, in order that he might take it home to reflect over its provisions and
consider whether it conformed in all respects to his wishes. On the morning of October 21
he again appeared in Repide's office and returned to him the draft of the will with certain
corrections. Among the changes thus made was the suppression of the names of Monzon,
Sunico, and Repide as attesting witnesses, these names being substituted by the names
of Jose Syyap, Agustin Vergel de Dios, and Vicente Legarda. The explanation given by the
testator for desiring this change was that he had met Jose Syyap on the Escolta, the day
before, and had committed the indiscretion of communicating the fact that he (Tolentino)
was having a new will made in which Monzon, Sunico, and Repide would appear as the
attesting witnesses. Now Syyap had been the draftsman of the former will of Tolentino, and
in this same will the name of Syyap appeared as one of the attesting witnesses, the other
two being Vicente Legarda and Vergel de Dios. When, therefore, Syyap learned that a new
will was being drawn up without his intervention, he showed profound disappointment,
saying to Tolentino that he considered it a gross offense that he, Legarda, and Vergel de
Dios should be eliminated as witnesses to the new will. Upon this manifestation of feeling
by Syyap, Tolentino decided to avail himself of Syyap, Legarda, and Vergel de Dios as
witnesses to this will also, and he therefore requested Repide to change the names of the
attesting witnesses. After this point had been settled Tolentino stated that he would request
Syyap, Legarda, and Vergel de Dios to appear at the office of Repide for the purpose of
signing the will. To this end Tolentino went away but returned later saying that he had
spoken to Syyap about it and that the latter strenuously objected, observing that the will
should be signed at a chop-suey restaurant ( panciteria). Tolentino further stated to his
attorney in this conversation that he had arranged with Syyap and the other two intending
witnesses to meet at five o'clock in the afternoon of the next day, which was October 22,
for the purpose of executing the will.chanroblesvirtualawlibrary chanrobles virtual law
library
Pursuant to these instructions Repide made the desired changes in the will; and just before
twelve o'clock noon of the next day Tolentino returned to Repide's office and received from
him the criminal document with a carbon copy thereof. Repide advised the testator that the
copy should be executed with the same formality as the original in order that the intention
of the testator should not be frustrated by the possible loss or destruction of the
original.chanroblesvirtualawlibrary chanrobles virtual law library
It is a custom in the office of Repide not to number the consecutive pages of a will, on the
typewriting machine, the duty of numbering the pages being left to the testator himself.
This precaution appears to have been born of experience, and has been adopted by

81 | S U C C E S S I O N

Repide to prevent the possible destruction of a will by the mere erasure of the figures or
letters indicating the pagination, - a disaster which, in Repide's experience, had occurred in
at least one case. Accordingly, upon delivering the completed will and carbon copy to the
testator, Repide took particular pains to instruct the testator to write the consecutive paging
of
both
original
and
duplicate
before
signing
the
instrument.chanroblesvirtualawlibrary chanrobles virtual law library
At his interview the testator suggested to Repide that the latter should also go to the place
where the will was to be executed, so that he might be present at the formality. The
attorney replied that it was impossible for him to do so as he had another engagement for
the
hour
indicated,
which
would
prevent
his
attendance.chanroblesvirtualawlibrary chanrobles virtual law library
At about 4:30 p. m. on the same day, which was October 22, Tolentino started in his car to
pick up Syyap and Vergel de Dios at their respective homes on Antipolo and Benavides
streets. He then caused his chauffeur to drive with the three to La Previsora Filipina, on
Rizal Avenue, where Vicente Legarda, the third intending witness was to be found. Arriving
at this place, the three entered the office of Legarda, who was manager of the
establishment, and they were invited to take seats, which they did. Tolentino then
suggested that the three should go as his guests to a panciteria, where they could take
refreshments and the will could be executed. Legarda replied that he must decline the
invitation for he had an engagement to go to the Cosmos Club the same afternoon. Upon
this Tolentino asked Legarda to permit the will to be signed in his office, and to this request
Legarda acceded.chanroblesvirtualawlibrary chanrobles virtual law library
Tolentino thereupon drew two documents from his pocket saving that it was his last will and
testament, done in duplicate, and he proceeded to read the original to the witnesses. After
this had been completed, Legarda himself took the will in hand and read it himself. He then
returned it to Tolentino, who thereupon proceeded, with pen and ink, to number the pages
of the will thus, "Pagina Primera", "Pagina Segunda", etc. He then paged the duplicate
copy of the will in the same way. He next proceeded to sign the original will and each of its
pages by writing his name "G. Tolentino" in the proper places. Following this, each of the
three witnesses signed their own respective names at the end of the will, at the end of the
attesting clause, and in the left margin of each page of the instrument. During this
ceremony all of the persons concerned in the act of attestation were present together, and
all
fully
advertent
to
the
solemnity
that
engaged
their
attention.chanroblesvirtualawlibrary chanrobles virtual law library
After the original of the will had been executed in the manner just stated, the testator
expressed his desire that the duplicate should be executed in the same manner. To this
Syyap objected, on the ground that it was unnecessary; and in this view he was supported
by Vergel de Dios, with the result that the wishes of the testator on this point could not be
carried out. As the party was about to break up Tolentino used these words: "For God's
sake, as a favor, I request you not to let any one know the contents of this will." The

meeting then broke up and Tolentino returned Syyap and Vergel de Dios to their homes in
his car. He then proceeded to the law office of Repide, arriving about 6:15 p. m. After
preliminary explanations had been made, Tolentino requested Repide to keep the will
overnight in his safe, as it was already too late to place it in the compartment which
Tolentino was then renting in the Oriental Safe Deposit, in the Kneedler Building. In this
connection the testator stated that he did not wish to take the will to his home, as he knew
that his relatives were watching him and would take advantage of any carelessness on his
part to pry into his papers. Also, in this conversation Tolentino informed Repide of the
refusal of Syyap to execute the duplicate of the will.chanroblesvirtualawlibrary chanrobles
virtual law library
After a good part of an hour had thus been spent at Repide's office by the testator and
after the original of the will had been deposited in Repide's safe, Tolentino took the
attorney to the latter's residence in Ermita, and then returned to his own home, where he
remained without again going out that night. But promptly at nine o'clock the next morning
Tolentino presented himself at Repide's office for the purpose of securing the will. Repide
happened to be out and Tolentino went away, but again returned the next day and received
the will. With the instrument thus in his possession he proceeded at once to the Oriental
Safe Deposit and there left the instrument in his private compartment, No. 333, in which
place it remained until withdrawn some two weeks later by order of the
court.chanroblesvirtualawlibrary chanrobles virtual law library
On the morning of November 9, 1930, Gregorio Tolentino was found dead in his bed,
having perished by the hands of an assassin.chanroblesvirtualawlibrary chanrobles virtual
law library
The peculiarity of this case is that, upon the trial of this proceeding for the probate of the
will of the decedent, two of the attesting witnesses, Jose Syyap and Vergel de Dios,
repudiated their participation in the execution of the will at the time and place stated; and
while admitting the genuineness of their signatures to the will, pretended that they had
severally signed the instrument, at the request of the testator, at different places. Thus
Syyap, testifying as a witness, claimed that the testator brought the will to Syyap's house
on the afternoon of October 21 - a time, be it remembered, when the will had not yet left
the hands of the draftsman - and upon learning that Syyap could not be present at the time
and place then being arranged for the execution of the will, he requested Syyap, as a mere
matter of complaisance, to sign the will then, which Syyap did. Vergel de Dios has another
story to tell of isolated action, claiming that he signed the will in the evening of October 22
at the Hospital of San Juan de Dios in Intramuros.chanroblesvirtualawlibrary chanrobles
virtual law library
We are unable to give any credence to the testimony of these two witnesses on this point,
the same being an evident fabrication designed for the purpose of defeating the will. In the
first place, the affirmative proof showing that the will was properly executed is adequate,
consistent, and convincing, consisting of the testimony of the third attesting witness,

82 | S U C C E S S I O N

Vicente Legarda, corroborated by Miguel Legarda and Urbana Rivera, two disinterested
individuals, employees of La Previsora Filipina, who were present in Legarda's office when
the will was executed and who lent a discerning attention to what was being done. In the
second place, each of the seven signatures affixed to his will by Syyap appear to the
natural eye to have been made by using the same pen and ink that was used by Legarda
in signing the will. The same is also probably true of the seven signatures made by Vergel
de Dios. This could hardly have happened if the signatures of Syyap and Vergel de Dios
had been affixed, as they now pretend, at different times and places. In the third place,
Both Syyap and Vergel de Dios are impeached by proof of contradictory statements made
by them on different occasions prior to their appearance as witnesses in this case. In this
connection we note that, after the murder of Gregorio Tolentino, and while the police
authorities were investigating his death, Nemesio Alferez, a detective, sent for Syyap and
questioned him concerning his relations with the deceased. Upon this occasion Syyap
stated that Gregorio Tolentino had lately made a will, that it had been executed at the office
of La Previsora Filipina under the circumstances already stated, and that he himself had
served as one of the attesting witnesses.chanroblesvirtualawlibrary chanrobles virtual law
library
With respect to Vergel de Dios we have the following fact: On the day that Gregorio
Tolentino was buried, Ramon Llorente, a member of the city police force, was sent out to
the cemetery in order that he might be present and observe the demeanor on that
occasion of such Tolentino's kin as might be present. Llorente arrived before the funeral
cortege, having been taken out to the cemetery by Repide. While the two were waiting at
the cemetery, Llorente noted the presence of Vergel de Dios, he requested the policeman
to introduce him. In the conversation that ensued Vergel de Dios stated with considerable
detail that Gregorio Tolentino had made a will just before his death, that it was executed at
La Previsora Filipina, and that he was one of the witnesses who attested the instrument at
that time and place.chanroblesvirtualawlibrary chanrobles virtual law library
Again, on a certain occasion subsequent to the death of Gregorio Tolentino, Juan
Concepcion the husband of Adelaida Tolentino, accompanied by Genoveva de Mendoza,
called upon Vergel de Dios, and in the conversation that resulted Vergel de Dios told them
that the will was properly executed, that he was one of the attesting witnesses, and that it
had been signed by all of them in the office of La Previsora
Filipina.chanroblesvirtualawlibrary chanrobles virtual law library
These circumstances and other incidents revealed in the proof leave no room for doubt in
our mind that Syyap and Vergel de Dios have entered into a conspiracy between
themselves, and in concert with the opponents, to defeat the will of Gregorio Tolentino
although they are well aware that said will was in all respects properly executed; and the
trial court, in our opinion, committed no error in admitting the will to
probate.chanroblesvirtualawlibrary chanrobles virtual law library

When a will is contested it is the duty of the proponent to call all of the attesting witnesses,
if available but the validity of the will in no wise depends upon the united support of the will
by all of those witnesses. A will may be admitted to probate notwithstanding the fact that
one or more of the subscribing witnesses do not unite with the other, or others, in proving
all the facts upon which the validity of the will rests. (Fernandez vs. Tantoco, 49 Phil., 380.)
It is sufficient if the court is satisfied from all the proof that the will was executed and
attested in the manner required by law. In this case we feel well assured that the contested
will was properly executed and the order admitting to it probate was entirely
proper.chanroblesvirtualawlibrary chanrobles virtual law library
The order appealed from will therefore be affirmed, with costs against the appellants. So
ordered.chanroblesvirtualawlibrary chanrobles virtual law library

G.R. No. 45629


ANTILANO G. MERCADO, petitioner,
vs.
ALFONSO SANTOS, Judge of First Instance of Pampanga, respondent.

LAUREL, J.:
On May 28, 1931, the petitioner herein filed in the Court of First Instance of Pampanga a
petition for the probate of the will of his deceased wife, Ines Basa. Without any opposition,
and upon the testimony of Benigno F. Gabino, one of the attesting witnesses, the probate
court, on June 27,1931, admitted the will to probate. Almost three years later, on April 11,
1934, the five intervenors herein moved ex parte to reopen the proceedings, alleging lack
of jurisdiction of the court to probate the will and to close the proceedings. Because
filed ex parte, the motion was denied. The same motion was filed a second time, but with
notice to the adverse party. The motion was nevertheless denied by the probate court on
May 24, 1934. On appeal to this court, the order of denial was affirmed on July 26, 1935.
(Basa vs. Mercado, 33 Off. Gaz., 2521.)

83 | S U C C E S S I O N

It appears that on October 27, 1932, i. e., sixteen months after the probate of the will of
Ines Basa, intervenor Rosario Basa de Leon filed with the justice of the peace court of San
Fernando, Pampanga, a complaint against the petitioner herein, for falsification or forgery
of the will probated as above indicated. The petitioner was arrested. He put up a bond in
the sum of P4,000 and engaged the services of an attorney to undertake his defense.
Preliminary investigation of the case was continued twice upon petition of the complainant.
The complaint was finally dismissed, at the instance of the complainant herself, in an order
dated December 8, 1932. Three months later, or on March 2, 1933, the same intervenor
charged the petitioner for the second time with the same offense, presenting the complaint
this time in the justice of the peace court of Mexico, Pampanga. The petitioner was again
arrested, again put up a bond in the sum of P4,000, and engaged the services of counsel
to defend him. This second complaint, after investigation, was also dismissed, again at the
instance of the complainant herself who alleged that the petitioner was in poor health. That
was on April 27, 1933. Some nine months later, on February 2, 1934, to be exact, the
same intervenor accused the same petitioner for the third time of the same offense. The
information was filed by the provincial fiscal of Pampanga in the justice of the peace court
of Mexico. The petitioner was again arrested, again put up a bond of P4,000, and engaged
the services of defense counsel. The case was dismissed on April 24, 1934, after due
investigation, on the ground that the will alleged to have been falsified had already been
probated and there was no evidence that the petitioner had forged the signature of the
testatrix appearing thereon, but that, on the contrary, the evidence satisfactorily
established the authenticity of the signature aforesaid. Dissatisfied with the result, the
provincial fiscal, on May 9, 1934, moved in the Court of First Instance of Pampanga for
reinvestigation of the case. The motion was granted on May 23, 1934, and, for the fourth
time, the petitioner was arrested, filed a bond and engaged the services of counsel to
handle his defense. The reinvestigation dragged on for almost a year until February 18,
1934, when the Court of First Instance ordered that the case be tried on the merits. The
petitioner interposed a demurrer on November 25, 1935, on the ground that the will alleged
to have been forged had already been probated. This demurrer was overruled on
December 24, 1935, whereupon an exception was taken and a motion for reconsideration
and notice of appeal were filed. The motion for reconsideration and the proposed appeal
were denied on January 14, 1936. The case proceeded to trial, and forthwith petitioner
moved to dismiss the case claiming again that the will alleged to have been forged had
already been probated and, further, that the order probating the will is conclusive as to the
authenticity and due execution thereof. The motion was overruled and the petitioner filed
with the Court of Appeals a petition for certiorari with preliminary injunction to enjoin the
trial court from further proceedings in the matter. The injunction was issued and thereafter,
on June 19, 1937, the Court of Appeals denied the petition for certiorari, and dissolved the
writ of preliminary injunction. Three justices dissented in a separate opinion. The case is
now before this court for review on certiorari.
Petitioner contends (1) that the probate of the will of his deceased wife is a bar to his
criminal prosecution for the alleged forgery of the said will; and, (2) that he has been
denied the constitutional right to a speedy trial.

1. Section 306 of our Code of Civil Procedure provides as to the effect of judgments.
SEC. 306. Effect of judgment. The effect of a judgment or final order in an action or
special proceeding before a court or judge of the Philippine Islands or of the United States,
or of any State or Territory of the United States, having jurisdiction to pronounce the
judgment or order, may be as follows.
1. In case of a judgment or order against a specific thing, or in respect to the probate of a
will, or the administration of the estate of a deceased person, or in respect to the personal,
political, or legal condition or relation of a particular person, the judgment or order is
conclusive upon the title of the thing, the will or administration, or the condition or relation
of the person Provided, That the probate of a will or granting of letters of administration
shall only be prima facie evidence of the death of the testator or intestate.
xxx

xxx

xxx

(Emphasis ours.)
Section 625 of the same Code is more explicit as to the conclusiveness of the due
execution of a probate will. It says.
SEC. 625. Allowance Necessary, and Conclusive as to Execution. No will shall pass
either the real or personal estate, unless it is proved and allowed in the Court of First
Instance, or by appeal to the Supreme Court; and the allowance by the court of a will of
real and personal estate shall be conclusive as to its due execution. (Emphasis ours.)
In Manahan vs. Manahan (58 Phil., 448, 451), we held:
. . . The decree of probate is conclusive with respect to the due execution thereof and it
cannot be impugned on any of the grounds authorized by law, except that of fraud, in any
separate or independent action or proceeding. Sec. 625, Code of Civil
Procedure; Castaeda vs. Alemany, 3 Phil., 426; Pimentel vs. Palanca, 5 Phil.,
436;Sahagun vs. De
Gorostiza,
7
Phil.,
347; Limjuco vs. Ganara,
11
Phil.,
393; Montaanovs. Suesa, 14 Phil., 676; in re Estate of Johnson, 39 Phil.,
156; Riera vs. Palmaroli,
40
Phil.,
105; Austria vs. Ventenilla,
21
Phil.,
180; Ramirez vs. Gmur, 42 Phil., 855; andChiong Jocsoy vs. Vano, 8 Phil., 119.
In 28 R. C. L., p. 377, section 378, it is said.

The probate of a will by the probate court having jurisdiction thereof is usually considered
as conclusive as to its due execution and validity, and is also conclusive that the testator
was of sound and disposing mind at the time when he executed the will, and was not
acting under duress, menace, fraud, or undue influence, and that the will is genuine and
not a forgery. (Emphasis ours.)
As our law on wills, particularly section 625 of our Code of Civil Procedureaforequoted,
was taken almost bodily from the Statutes of Vermont, the decisions of the Supreme Court
of the State relative to the effect of the probate of a will are of persuasive authority in this
jurisdiction. The Vermont statute as to the conclusiveness of the due execution of a
probated will reads as follows.
SEC. 2356. No will shall pass either real or personal estate, unless it is proved and allowed
in the probate court, or by appeal in the county or supreme court; and the probate of a will
of real or personal estate shall be conclusive as to its due execution. (Vermont Statutes, p.
451.)
Said the Supreme Court of Vermont in the case of Missionary Society vs. Eells (68 Vt.,
497, 504): "The probate of a will by the probate court having jurisdiction thereof, upon the
due notice, is conclusive as to its due execution against the whole world. (Vt. St., sec.
2336; Fosters Exrs. vs. Dickerson, 64 Vt., 233.)"
The probate of a will in this jurisdiction is a proceeding in rem. The provision of notice by
Publication as a prerequisite to the allowance of a will is constructive notice to the whole
world, and when probate is granted, the judgment of the court is binding upon everybody,
even against the State. This court held in the case of Manalo vs. Paredes and Philippine
Food Co. (47 Phil., 938):
The proceeding for the probate of a will is one in rem (40 Cyc., 1265), and the court
acquires jurisdiction over all the persons interested, through the publication of the notice
prescribed by section 630 of the Code of Civil Procedure, and any order that may be
entered therein is binding against all of them.
Through the publication of the petition for the probate of the will, the court acquires
jurisdiction over all such persons as are interested in said will; and any judgment that may
be rendered after said proceeding is binding against the whole world.
In Everrett vs. Wing (103 Vt., 488, 492), the Supreme Court of Vermont held.
In this State the probate of a will is a proceeding in rem being in form and substance upon
the will itself to determine its validity. The judgment determines the status of the
instrument, whether it is or is not the will of the testator. When the proper steps required by
law have been taken the judgment is binding upon everybody, and makes the instrument

84 | S U C C E S S I O N

as to all the world just what the judgment declares it to be. (Woodruff vs. Taylor, 20 Vt., 65,
73; Burbeck vs. Little, 50 Vt., 713, 715; Missionary Society vs. Eells, 68 Vt., 497, 504; 35
Atl., 463.) The proceedings before the probate court are statutory and are not governed by
common law rules as to parties or causes of action. (Holdrige vs. Holdriges Estate, 53 Vt.,
546, 550; Purdy vs. Estate of Purdy, 67 Vt. 50, 55; 30 Atl., 695.) No process is issued
against anyone in such proceedings, but all persons interested in determining the state or
conditions of the instrument are constructively notified by the publication of notice as
required by G. L. 3219. (Woodruff vs. Taylor, supra; In re Warners Estate 98 Vt., 254; 271;
127 Atl., 362.)
Section 333, paragraph 4, of the Code of Civil Procedure establishes an incontrovertible
presumption in favor of judgments declared by it to be conclusive.
SEC. 333. Conclusive Presumptions. The following presumptions or deductions, which
the law expressly directs to be made from particular facts, are deemed conclusive.
xxx

xxx

xxx

4. The judgment or order of a court, when declared by this code to be conclusive.


Conclusive presumptions are inferences which the law makes so peremptory that it will not
allow them to be overturned by any contrary proof however strong. (Brant vs.Morning
Journal Assn., 80 N.Y.S., 1002, 1004; 81 App. Div., 183; see, also, Joslyn vs.Puloer, 59
Hun., 129, 140, 13 N.Y.S., 311.) The will in question having been probated by a competent
court, the law will not admit any proof to overthrow the legal presumption that it is genuine
and not a forgery.
The majority decision of the Court of Appeals cites English decisions to bolster up its
conclusion that "the judgment admitting the will to probate is binding upon the whole world
as to the due execution and genuineness of the will insofar as civil rights and liabilities are
concerned, but not for the purpose of punishment of a crime." The cases of Dominus
Rex vs. Vincent, 93 English Reports, Full Reprint, 795, the first case being decided in
1721, were cited to illustrate the earlier English decisions to the effect that upon indictment
for forging a will, the probating of the same is conclusive evidence in the defendants favor
of its genuine character. Reference is made, however, to the cases of Rex vs. Gibson, 168
English Reports, Full Reprint, 836, footnote (a), decided in 1802, and Rex vs. Buttery and
Macnamarra, 168 English Reports, Full Reprint, 836, decided in 1818, which establish a
contrary rule. Citing these later cases, we find the following quotation from Black on
Judgments, Vol. II, page 764.
A judgment admitting a will to probate cannot be attacked collaterally although the will was
forged; and a payment to the executor named therein of a debt due the decedent will
discharge the same, notwithstanding the spurious character of the instrument probated. It

85 | S U C C E S S I O N

has also been held that, upon an indictment for forging a will, the probate of the paper in
question is conclusive evidence in the defendants favor of its genuine character. But this
particular point has lately been ruled otherwise.
It was the case of Rex vs. Buttery, supra, which induced the Supreme Court of
Massachussetts in the case of Waters vs. Stickney (12 Allen 1; 90 Am. Dec., 122) also
cited by the majority opinion, to hold that "according to later and sounder decisions, the
probate, though conclusive until set aside of the disposition of the property, does not
protect the forger from punishment." This was reproduced in 28 R.C.L., p. 376, and quoted
in Barry vs. Walker (103 Fla., 533; 137 So., 711, 715), and Thompson vs. Freeman (149
So., 740, 742), also cited in support of the majority opinion of the Court of Appeals. The
dissenting opinion of the Court of Appeals in the instant case under review makes a
cursory study of the statutes obtaining in England, Massachussetts and Florida, and
comes to the conclusion that the decisions cited in the majority opinion do not appear to
"have been promulgated in the face of statutes similar to ours." The dissenting opinion
cites Whartons Criminal Evidence (11th ed., sec. 831), to show that the probate of a will in
England is only prima facie proof of the validity of the will (Op. Cit. quoting
Marriot vs. Marriot, 93 English Reprint, 770); and 21 L.R.A. (pp. 686689 and note), to show
that in Massachussetts there is no statute making the probate of a will conclusive, and that
in Florida the statute(sec. 1810, Revised Statutes) makes the probate conclusive evidence
as to the validity of the will with regard to personal, andprima facie as to real estate. The
cases decided by the Supreme Court of Florida cited by the majority opinion, supra, refer
to wills of both personal and real estate.
The petitioner cites the case of State vs. McGlynn (20 Cal., 233, decided in 1862), in which
Justice Norton of the Supreme Court of California, makes the following review of the nature
of probate proceedings in England with respect to wills personal and real property.
In England, the probate of wills of personal estate belongs to the Ecclesiastical Courts. No
probate of a will relating to real estate is there necessary. The real estate, upon the death
of the party seized, passes immediately to the devisee under the will if there be one; or if
there be no will, to the heir at law. The person who thus becomes entitled takes
possession. If one person claims to be the owner under a will, and another denies the
validity of the will and claims to be the owner as heir at law, an action of ejectment is
brought against the party who may be in possession by the adverse claimant; and on the
trial of such an action, the validity of the will is contested, and evidence may be given by
the respective parties as to the capacity of the testator to make a will, or as to any fraud
practiced upon him, or as to the actual execution of it, or as to any other circumstance
affecting its character as a valid devise of the real estate in dispute. The decision upon the
validity of the will in such action becomes res adjudicata, and is binding and conclusive
upon the parties to that action and upon any person who may subsequently acquire the
title from either of those parties; but the decision has no effect upon other parties, and
does not settle what may be called the status or character of the will, leaving it subject to
be enforced as a valid will, or defeated as invalid, whenever other parties may have a

contest depending upon it. A probate of a will of personal property, on the contrary, is a
judicial determination of the character of the will itself. It does not necessarily or ordinarily
arise from any controversy between adverse claimants, but is necessary in order to
authorize a disposition of the personal estate in pursuance of its provisions. In case of any
controversy between adverse claimants of the personal estate, the probate is given in
evidence and is binding upon the parties, who are not at liberty to introduce any other
evidence as to the validity of the will.
The intervenors, on the other hand, attempt to show that the English law on wills is
different from that stated in the case of State vs. McGlynn, supra, citing the following
statutes.
1. The Wills Act, 1837 (7 Will. 4 E 1 Vict. c. 26).
2. The Court of Probate Act, 1857 (20 and 21 Vict. c. 77).
3. The Judicature Act, 1873 (36 and 37 Vict. c. 66).
The Wills Act of 1837 provides that probate may be granted of "every instrumental
purporting to be testamentary and executed in accordance with the statutory
requirements . . . if it disposes of property, whether personal or real." The Ecclesiastical
Courts which took charge of testamentary causes (Ewells Blackstone [1910], p. 460), were
determined by the Court of Probate Act of 1857, and the Court of Probate in turn was,
together with other courts, incorporated into the Supreme Court of Judicature, and
transformed into the Probate Division thereof, by the Judicature Act of 1873. (Lord
Halsbury, The Laws of England[1910], pp. 151156.) The intervenors overlook the fact,
however, that the case of Rex vs. Buttery and Macnamarra, supra, upon which they rely in
support of their theory that the probate of a forged will does not protect the forger from
punishment, was decided long before the foregoing amendatory statutes to the English law
on wills were enacted. The case of State vs.McGlynn may be considered, therefore, as
more or less authoritative on the law of England at the time of the promulgation of the
decision in the case of Rex vs. Buttery and Macnamarra.
In the case of State vs. McGlynn, the Attorney General of California filed an information to
set aside the probate of the will of one Broderick, after the lapse of one year provided by
the law of California for the review of an order probating a will, in order that the estate may
be escheated to the State of California for the review of an probated will was forged and
that Broderick therefore died intestate, leaving no heirs, representatives or devisees
capable of inheriting his estate. Upon these facts, the Supreme Court of California held.
The fact that a will purporting to be genuine will of Broderick, devising his estate to a
devisee capable of inheriting and holding it, has been admitted to probate and established
as a genuine will by the decree of a Probate Court having jurisdiction of the case, renders

86 | S U C C E S S I O N

it necessary to decide whether that decree, and the will established by it, or either of them,
can be set aside and vacated by the judgment of any other court. If it shall be found that
the decree of the Probate Court, not reversed by the appellate court, is final and
conclusive, and not liable to be vacated or questioned by any other court, either
incidentally or by any direct proceeding, for the purpose of impeaching it, and that so long
as the probate stands the will must be recognized and admitted in all courts to be valid,
then it will be immaterial and useless to inquire whether the will in question was in fact
genuine or forged. (Statevs. McGlynn, 20 Cal., 233; 81 Am. Dec., 118, 121.).
Although in the foregoing case the information filed by the State was to set aside the
decree of probate on the ground that the will was forged, we see no difference in principle
between that case and the case at bar. A subtle distinction could perhaps be drawn
between setting aside a decree of probate, and declaring a probated will to be a forgery. It
is clear, however, that a duly probated will cannot be declared to be a forgery without
disturbing in a way the decree allowing said will to probate. It is at least anomalous that a
will should be regarded as genuine for one purpose and spurious for another.
The American and English cases show a conflict of authorities on the question as to
whether or not the probate of a will bars criminal prosecution of the alleged forger of the
probate will. We have examined some important cases and have come to the conclusion
that no fixed standard maybe adopted or drawn therefrom, in view of the conflict no less
than of diversity of statutory provisions obtaining in different jurisdictions. It behooves us,
therefore, as the court of last resort, to choose that rule most consistent with our statutory
law, having in view the needed stability of property rights and the public interest in general.
To be sure, we have seriously reflected upon the dangers of evasion from punishment of
culprits deserving of the severity of the law in cases where, as here, forgery is discovered
after the probate of the will and the prosecution is had before the prescription of the
offense. By and large, however, the balance seems inclined in favor of the view that we
have taken. Not only does the law surround the execution of the will with the necessary
formalities and require probate to be made after an elaborate judicial proceeding, but
section 113, not to speak of section 513, of our Code of Civil Procedure provides for an
adequate remedy to any party who might have been adversely affected by the probate of a
forged will, much in the same way as other parties against whom a judgment is rendered
under the same or similar circumstances. (Pecson vs. Coronel, 43 Phil., 358.)The
aggrieved party may file an application for relief with the proper court within a reasonable
time, but in no case exceeding six months after said court has rendered the judgment of
probate, on the ground of mistake, inadvertence, surprise or excusable neglect. An appeal
lies to review the action of a court of first instance when that court refuses to grant relief.
(Banco Espaol Filipino vs. Palanca, 37 Phil., 921; Philippine Manufacturing
Co. vs.Imperial, 47 Phil., 810; Samia vs. Medina, 56 Phil., 613.) After a judgment allowing
a will to be probated has become final and unappealable, and after the period fixed by
section 113 of the Code of Civil Procedure has expired, the law as an expression of the
legislative wisdom goes no further and the case ends there.

. . . The court of chancery has no capacity, as the authorities have settled, to judge or
decide whether a will is or is not a forgery; and hence there would be an incongruity in its
assuming to set aside a probate decree establishing a will, on the ground that the decree
was procured by fraud, when it can only arrive at the fact of such fraud by first deciding
that the will was a forgery. There seems, therefore, to be a substantial reason, so long as a
court of chancery is not allowed to judge of the validity of a will, except as shown by the
probate, for the exception of probate decrees from the jurisdiction which courts of chancery
exercise in setting aside other judgments obtained by fraud. But whether the exception be
founded in good reason or otherwise, it has become too firmly established to be
disregarded. At the present day, it would not be a greater assumption to deny the general
rule that courts of chancery may set aside judgments procured by fraud, than to deny the
exception to that rule in the case of probate decrees. We must acquiesce in the principle
established by the authorities, if we are unable to approve of the reason. Judge Story was
a staunch advocate for the most enlarged jurisdiction of courts of chancery, and was
compelled to yield to the weight of authority. He says "No other excepted case is known to
exist; and it is not easy to discover the grounds upon which this exception stands, in point
of reason or principle, although it is clearly settled by authority. (1 Storys Eq. Jur. sec.
440.)" (State vs. McGlynn, 20 Cal., 233; 81 Am. Dec., 118, 129. See, also, Tracy vs. Muir,
121 American State Reports, 118, 125.)
We hold, therefore, that in view of the provisions of sections 306, 333 and 625 of ourCode
of Civil Procedure, criminal action will not lie in this jurisdiction against the forger of a will
which had been duly admitted to probate by a court of competent jurisdiction.
The resolution of the foregoing legal question is sufficient to dispose of the case. However,
the other legal question with reference to the denial to the accused of his right to a speedy
trial having been squarely raised and submitted, we shall proceed to consider the same in
the light of cases already adjudicated by this court.
2. The Constitution of the Philippines provides that "In all criminal prosecutions the
accused . . . shall enjoy the right . . . to have a speedy . . . trial. . . . (Art. III, sec. 1, par.
17.See, also, G.O. No. 58, sec. 15, No. 7.) Similar provisions are to be found in the
Presidents Instructions to the Second Philippine Commission (par. 11), the Philippine Bill of
July 1, 1902 (sec. 5, par. 2) and the Jones Act of August 29, 1916 (sec. 3, par. 2). The
provisions in the foregoing organic acts appear to have been taken from similar provisions
in the Constitution of the United States (6th Amendment) and those of the various states of
the American Union. A similar injunction is contained in the Malolos Constitution (art. 8,
Title IV), not to speak of other constitutions. More than once this court had occasion to set
aside the proceedings in criminal cases to give effect to the constitutional injunction of
speedy trial. (Conde vs. Judge of First Instance and Fiscal of Tayabas [1923], 45 Phil.,
173; Conde vs. Rivera and Unson[1924], 45 Phil., 650; People vs.Castaeda and
Fernandez[1936]), 35 Off. Gaz., 1269; Kalaw vs. Apostol, Oct. 15, 1937,G.R. No. 45591;
Esguerra vs. De la Costa, Aug. 30, 1938, G.R. No. 46039.).

87 | S U C C E S S I O N

In Conde vs. Rivera and Unson, supra, decided before the adoption of our Constitution, we
said.
Philippine organic and statutory law expressly guarantee that in all criminal prosecutions
the accused shall enjoy the right to have a speedy trial. Aurelia Conde, like all other
accused persons, has a right to a speedy trial in order that if innocent she may go free, and
she has been deprived of that right in defiance of law. Dismissed from her humble position,
and compelled to dance attendance on courts while investigations and trials are arbitrarily
postponed without her consent, is palpably and openly unjust to her and a detriment to the
public. By the use of reasonable diligence, the prosecution could have settled upon the
appropriate information, could have attended to the formal preliminary examination, and
could have prepared the case for a trial free from vexatious, capricious, and oppressive
delays.
In People vs. Castaeda and Fernandez, supra, this court found that the accused had not
been given a fair and impartial trial. The case was to have been remanded to the court a
quo for a new trial before an impartial judge. This step, however, was found unnecessary. A
review of the evidence convinced this court that a judgment of conviction for theft, as
charged, could not be sustained and, having in view the right to a speedy trial guaranteed
by the Constitution to every person accused of crime, entered a judgment acquitting the
accused, with costs de oficio. We said.
. . . The Constitution, Article III, section 1, paragraph 17, guarantees to every accused
person the right to a speedy trial. This criminal proceeding has been dragging on for
almost five years now. The accused have twice appealed to this court for redress from the
wrong that they have suffered at the hands of the trial court. At least one of them, namely
Pedro Fernandez alias Piro, had been con-fined in prison from July 20, 1932 to November
27, 1934, for inability to post the required bond of P3,000 which was finally reduced to
P300. The Government should be the last to set an example of delay and oppression in
the administration of justice and it is the moral and legal obligation of this court to see that
the criminal proceedings against the accused come to an end and that they be immediately
dis-charged from the custody of the law. (Conde vs. Rivera and Unson, 45 Phil., 651.)
In Kalaw vs. Apostol, supra, the petitioner invoked and this court applied and gave effect to
the doctrines stated in the second Conde case, supra. In granting the writs prayed for, this
court, after referring to the constitutional and statutory provisions guaranteeing to persons
accused of crime the right to a speedy trial, said:
Se infiere de los preceptos legales transcritos que todo acusado en causa criminal tiene
derecho a ser juzgado pronta y publicamente. Juicio rapido significa un juicioque se
celebra de acuerdo con la ley de procedimiento criminal y los reglamentos, libre de
dilaciones vejatorias, caprichosas y opersivas (Burnett vs.State, 76 Ark., 295; 88S. W.,
956; 113 AMSR, 94; Stewart vs. State, 13 Ark., 720; Peo.vs. Shufelt, 61 Mich., 237; 28 N.

W., 79; Nixon vs. State, 10 Miss., 497; 41 AMD., 601; State vs. Cole, 4 Okl. Cr., 25; 109 P.,
736; State vs. Caruthers, 1 Okl. Cr., 428; 98 P., 474; State vs. Keefe, 17 Wyo., 227, 98 p.,
122;22 IRANS, 896; 17 Ann. Cas., 161). Segun los hechos admitidos resulta que al
recurrente se le concedio vista parcial del asunto, en el Juzgado de Primera Instancia de
Samar, solo despues de haber transcurrido ya mas de un ao y medio desde la
presentacion de la primera querella y desde la recepcion de la causa en dicho Juzgado, y
despues de haberse transferido dos veces la vista delasunto sin su consentimiento. A esto
debe aadirse que laprimera transferencia de vista era claramente injustificadaporque el
motivo que se alego consistio unicamente en laconveniencia personal del ofendido y su
abogado, no habiendose probado suficientemente la alegacion del primero de quese
hallaba enfermo. Es cierto que el recurrente habia pedido que, en vez de sealarse a vista
el asunto para el mayo de 1936, lo fuera para el noviembre del mismo ao; pero,aparte de
que la razon que alego era bastante fuerte porquesu abogado se oponia a comparecer por
compromisos urgentes contraidos con anterioridad y en tal circunstancia hubiera quedado
indefenso si hubiese sido obligado a entraren juicio, aparece que la vista se pospuso por
el Juzgado amotu proprio, por haber cancelado todo el calendario judicial preparado por el
Escribano para el mes de junio. Declaramos, con visto de estos hechos, que al recurrents
se leprivo de su derecho fundamental de ser juzgado prontamente.
Esguerra vs. De la Costa, supra, was a petition for mandamus to compel the respondent
judge of the Court of First Instance of Rizal to dismiss the complaint filed in a criminal case
against the petitioner, to cancel the bond put up by the said petitioner and to declare the
costs de oficio. In accepting the contention that the petitioner had been denied speedy trial,
this court said:

Consta que en menos de un ao el recurrente fue procesado criminalmente por el alegado


delito de abusos deshonestos, en el Juzgado de Paz del Municipio de Cainta, Rizal. Como
consecuencia de las denuncias que contra el se presentaron fue arrestado tres veces y
para gozar de libertad provisional, en espera de los juicios, se vio obligado a prestartres
fianzas por la suma de P1,000 cada una. Si no se da fin al proceso que ultimamente se ha
incoado contra el recurrente la incertidumbre continuara cerniendose sobre el y las
consiguientes molestias y preocupaciones continuaran igualmente abrumandole. El Titulo
III, articulo 1, No. 17,de la Constitucion preceptua que en todo proceso criminalel acusado
tiene derecho de ser juzgado pronta y publicamente. El Articulo 15, No. 7, de la Orden
General No. 58 dispone asimismo que en las causas criminales el acusado tendra
derecho a ser juzgado pronta y publicamente. Si el recurrente era realmente culpable del
delito que se le imputo, tenia de todos modos derechos a que fuera juzgado pronta y
publicamente y sin dilaciones arbitrarias y vejatorias. Hemos declarado reiteradamente
que existe un remedio positivo para los casos en que se viola el derecho constitucional del
acusado de ser juzgado prontamente. El acusado que esprivado de su derecho
fundomental de ser enjuiciado rapidamente tiene derecho a pedir que se le ponga en
libertad, si estuviese detenido, o a que la causa que pende contra el sea sobreseida
definitivamente. (Conde contra Rivera y Unson, 45 Jur. Fil., 682; In the matter of Ford
[1911], 160 Cal., 334; U. S. vs. Fox [1880], 3 Mont., 512; Kalaw contra Apostol, R. G. No.
45591, Oct. 15, 1937; Pueblo contra Castaeda y Fernandez, 35 Gac. Of., 1357.)
We are again called upon to vindicate the fundamental right to a speedy trial. The facts of
the present case may be at variance with those of the cases hereinabove referred to.
Nevertheless, we are of the opinion that, under the circumstances, we should consider the
substance of the right instead of indulging in more or less academic or undue factual
differentiations. The petitioner herein has been arrested four times, has put up a bond in
the sum of P4,000 and has engaged the services of counsel to undertake his defense an
equal number of times. The first arrest was made upon a complaint filed by one of the
intervenors herein for alleged falsification of a will which, sixteen months before, had been
probated in court. This complaint, after investigation, was dismissed at the complainant's
own request. The second arrest was made upon a complaint charging the same offense
and this complaint, too, was dismissed at the behest of the complainant herself who
alleged the quite startling ground that the petitioner was in poor health. The third arrest
was made following the filing of an information by the provincial fiscal of Pampanga, which
information was dismissed, after due investigation, because of insufficiency of the
evidence. The fourth arrest was made when the provincial fiscal secured a reinvestigation
of the case against the petitioner on the pretext that he had additional evidence to present,
although such evidence does not appear to have ever been presented.
It is true that the provincial fiscal did not intervene in the case until February 2, 1934, when
he presented an information charging the petitioner, for the third time, of the offense of
falsification. This, however, does not matter. The prosecution of offenses is a matter of
public interest and it is the duty of the government or those acting in its behalf to prosecute
all cases to their termination without oppressive, capricious and vexatious delay. The

88 | S U C C E S S I O N

Constitution does not say that the right to a speedy trial may be availed of only where the
prosecution for crime is commenced and undertaken by the fiscal. It does not exclude from
its operation cases commenced by private individuals. Where once a person is prosecuted
criminally, he is entitled to a speedy trial, irrespective of the nature of the offense or the
manner in which it is authorized to be commenced. In any event, even the actuations of the
fiscal himself in this case is not entirely free from criticism. From October 27, 1932, when
the first complaint was filed in the justice of the peace court of San Fernando, to February
2, 1934, when the provincial fiscal filed his information with the justice of the peace of
Mexico, one year, three months and six days transpired; and from April 27, 1933, when the
second criminal complaint was dismissed by the justice of the peace of Mexico, to
February 2, 1934, nine months and six days elapsed. The investigation following the fourth
arrest, made after the fiscal had secured a reinvestigation of the case, appears also to
have dragged on for about a year. There obviously has been a delay, and considering the
antecedent facts and circumstances within the knowledge of the fiscal, the delay may not
at all be regarded as permissible. In Kalaw vs. Apostol, supra, we observed that the
prosecuting officer all prosecutions for public offenses (secs. 1681 and 2465 of the Rev.
Adm. Code), and that it is his duty to see that criminal cases are heard without vexatious,
capricious and oppressive delays so that the courts of justice may dispose of them on the
merits and determine whether the accused is guilty or not. This is as clear an admonition
as could be made. An accused person is entitled to a trial at the earliest opportunity.
(Sutherland on the Constitution, p. 664; United States vs. Fox, 3 Mont., 512.) He cannot be
oppressed by delaying he commencement of trial for an unreasonable length of time. If the
proceedings pending trial are deferred, the trial itself is necessarily delayed. It is not to be
supposed, of course, that the Constitution intends to remove from the prosecution every
reasonable opportunity to prepare for trial. Impossibilities cannot be expected or
extraordinary efforts required on the part of the prosecutor or the court. As stated by the
Supreme Court of the United States, "The right of a speedy trial is necessarily relative. It is
consistent with delays and depends upon circumstances. It secures rights to a defendant.
It does not preclude the rights of public justice." (Beavers vs. Haubert [1905], 198 U. S.,
86; 25 S. Ct., 573; 49 Law. ed., 950, 954.).
It may be true, as seems admitted by counsel for the intervenors, in paragraph 8, page 3 of
his brief, that the delay was due to "the efforts towards reaching an amicable extrajudicial
compromise," but this fact, we think, casts doubt instead upon the motive which led the
intervenors to bring criminal action against the petitioner. The petitioner claims that the
intention of the intervenors was to press upon settlement, with the continuous threat of
criminal prosecution, notwithstanding the probate of the will alleged to have been falsified.
Argument of counsel for the petitioner in this regard is not without justification. Thus after
the filing of the second complaint with the justice of the peace court of Mexico, complainant
herself, as we have seen, asked for dismissal of the complaint, on the ground that "el
acusado tenia la salud bastante delicada," and, apparently because of failure to arrive at
any settlement, she decided to renew her complaint.

Counsel for the intervenors contend and the contention is sustained by the Court of
Appeals that the petitioner did not complain heretofore of the denial of his constitutional
right to a speedy trial. This is a mistake. When the petitioner, for the fourth time, was
ordered arrested by the Court of First Instance of Pampanga, he moved for reconsideration
of the order of arrest, alleging, among other things, "Que por estas continuas acusaciones
e investigaciones, el acusado compareciente no obstante su mal estado de salud desde el
ao 1932 en que tuvo que ser operado por padecer de tuberculosis ha tenido que
sostener litigios y ha sufrido la mar de humiliaciones y zozobras y ha incudo en enormes
gastos y molestias y ha desatendido su quebrantada salud." The foregoing allegation was
inserted on page 6 of the amended petition for certiorari presented to the Court of Appeals.
The constitutional issue also appears to have been actually raised and considered in the
Court of Appeals. In the majority opinion of that court, it is stated:
Upon the foregoing facts, counsel for the petitioner submits for the consideration of this
court the following questions of law: First, that the respondent court acted arbitrarily and
with abuse of its authority, with serious damage and prejudice to the rights and interests of
the petitioner, in allowing that the latter be prosecuted and arrested for the fourth time, and
that he be subjected, also for the fourth time, to a preliminary investigation for the same
offense, hereby converting the court into an instrument of oppression and vengeance on
the part of the alleged offended parties, Rosario Basa et al.; . . . .
And in the dissenting opinion, we find the following opening paragraph:
We cannot join in a decision declining to stop a prosecution that has dragged for about five
years and caused the arrest on four different occasions of a law abiding citizen for the
alleged offense of falsifying a will that years be competent jurisdiction.
From the view we take of the instant case, the petitioner is entitled to have the criminal
proceedings against him quashed. The judgment of the Court of Appeals is hereby
reversed, without pronouncement regarding costs. So ordered.
Avancea, C.J., Villa-Real, Imperial, Diaz and Concepcion, JJ., concur.

G.R. No. L-48840 December 29, 1943


ERNESTO M. GUEVARA, petitioner-appellant,
vs.
.
ROSARIO GUEVARA and her husband PEDRO BUISON, respondent-appellees
OZAETA, J.:

89 | S U C C E S S I O N

Ernesto M. Guevarra and Rosario Guevara, ligitimate son and natural daughter,
respectively, of the deceased Victorino L. Guevara, are litigating here over their inheritance
from the latter. The action was commenced on November 12, 1937, by Rosario Guevara to
recover from Ernesto Guevara what she claims to be her strict ligitime as an acknowledged
natural daughter of the deceased to wit, a portion of 423,492 square meters of a large
parcel of land described in original certificate of title No. 51691 of the province of
Pangasinan, issued in the name of Ernesto M. Guervara and to order the latter to pay
her P6,000 plus P2,000 a year as damages for withholding such legitime from her. The
defendant answered the complaint contending that whatever right or rights the plaintiff
might have had, had been barred by the operation of law.
It appears that on August 26, 1931, Victorino L. Guevara executed a will (exhibit A),
apparently with all the formalities of the law, wherein he made the following bequests: To
his stepdaughter Candida Guevara, a pair of earrings worth P150 and a gold chain worth
P40; to his son Ernesto M. Guevara, a gold ring worth P180 and all the furniture, pictures,
statues, and other religious objects found in the residence of the testator in Poblacion Sur,
Bayambang, Pangasinan; "a mi hija Rosario Guevara," a pair of earrings worth P120; to
his stepson Piuo Guevara, a ring worth P120; and to his wife by second marriage, Angustia
Posadas, various pieces of jewelry worth P1,020.
He also made the following devises: "A mis hijos Rosario Guevara y Ernesto M. Guevara y
a mis hijastros, Vivencio, Eduviges, Dionisia, Candida y Pio, apellidados Guevara," a
residential lot with its improvements situate in the town of Bayambang, Pangasinan, having
an area of 960 square meters and assessed at P540; to his wife Angustia Posadas he
confirmed the donation propter nuptias theretofore made by him to her of a portion of 25
hectares of the large parcel of land of 259-odd hectares described in plan Psu-66618. He
also devised to her a portion of 5 hectares of the same parcel of land by way of complete
settlement of her usufructurary right.1awphil.net
He set aside 100 hectares of the same parcel of land to be disposed of either by him
during his lifetime or by his attorney-in-fact Ernesto M. Guevara in order to pay all his
pending debts and to degray his expenses and those of his family us to the time of his
death.

las cien (100) hectareas referidas en el inciso (a) de este parrafo del testamento,
como su propiedad absoluta y exclusiva, en la cual extension superficial estan
incluidas cuarenta y tres (43) hectareas, veintitres (23) areas y cuarenta y dos
(42) centiareas que le doy en concepto de mejora.
A mi hija natural reconocida, Rosario Guevara, veintiun (21) hectareas, sesenta y
un (61) areas y setenta y un (71) centiareas, que es la parte restante.
Duodecimo. Nombro por la presente como Albacea Testamentario a mi hijo
Ernesto M. Guevara, con relevacion de fianza. Y una vez legalizado este
testamento, y en cuanto sea posible, es mi deseo, que los herederos y legatarios
aqui nombrados se repartan extrajudicialmente mis bienes de conformidad con
mis disposiciones arriba consignadas.
Subsequently, and on July 12, 1933, Victorino L. Guevarra executed whereby he conveyed
to him the southern half of the large parcel of land of which he had theretofore disposed by
the will above mentioned, inconsideration of the sum of P1 and other valuable
considerations, among which were the payment of all his debts and obligations amounting
to not less than P16,500, his maintenance up to his death, and the expenses of his last
illness and funeral expenses. As to the northern half of the same parcel of land, he
declared: "Hago constar tambien que reconozco a mi referido hijo Ernesto M. guevara
como dueo de la mitad norte de la totalidad y conjunto de los referidos terrenos por
haberlos comprado de su propio peculio del Sr. Rafael T. Puzon a quien habia vendido con
anterioridad."
On September 27, 1933, final decree of registration was issued in land registration case
No. 15174 of the Court of First Instance of Pangasinan, and pursuant thereto original
certificate of title No. 51691 of the same province was issued on October 12 of the same
year in favor of Ernesto M. Guevara over the whole parcel of land described in the deed of
sale above referred to. The registration proceeding had been commenced on November 1,
1932, by Victorino L. Guevara and Ernesto M. Guevara as applicants, with Rosario, among
others, as oppositor; but before the trial of the case Victorino L. Guevara withdrew as
applicant and Rosario Guevara and her co-oppositors also withdrew their opposition,
thereby facilitating the issuance of the title in the name of Ernesto M. Guevara alone.

The remainder of said parcel of land his disposed of in the following manner:
(d). Toda la porcion restante de mi terreno arriba descrito, de la extension
superficial aproximada de ciento veintinueve (129) hectareas setenta (70) areas,
y veiticinco (25) centiares, con todas sus mejoras existentes en la misma, dejo y
distribuyo, pro-indiviso, a mis siguientes herederos como sigue:
A mi hijo legitimo, Ernesto M. Guevara, ciento ocho (108) hectareas, ocho (8)
areas y cincuenta y cuatro (54) centiareas, hacia la parte que colinda al Oeste de

90 | S U C C E S S I O N

On September 27, 1933, Victorino L. Guevarra died. His last will and testament, however,
was never presented to the court for probate, nor has any administration proceeding ever
been instituted for the settlement of his estate. Whether the various legatees mentioned in
the will have received their respective legacies or have even been given due notice of the
execution of said will and of the dispositions therein made in their favor, does not
affirmatively appear from the record of this case. Ever since the death of Victorino L.
Guevara, his only legitimate son Ernesto M. Guevara appears to have possessed the land

adjudicated to him in the registration proceeding and to have disposed of various portions
thereof for the purpose of paying the debts left by his father.
In the meantime Rosario Guevara, who appears to have had her father's last will and
testament in her custody, did nothing judicially to invoke the testamentary dispositions
made therein in her favor, whereby the testator acknowledged her as his natural daughter
and, aside from certain legacies and bequests, devised to her a portion of 21.6171
hectares of the large parcel of land described in the will. But a little over four years after the
testor's demise, she (assisted by her husband) commenced the present action against
Ernesto M. Guevara alone for the purpose hereinbefore indicated; and it was only during
the trial of this case that she presented the will to the court, not for the purpose of having it
probated but only to prove that the deceased Victirino L. Guevara had acknowledged her
as his natural daughter. Upon that proof of acknowledgment she claimed her share of the
inheritance from him, but on the theory or assumption that he died intestate, because the
will had not been probated, for which reason, she asserted, the betterment therein made
by the testator in favor of his legitimate son Ernesto M. Guevara should be disregarded.
Both the trial court and the Court of appeals sustained that theory.
Two principal questions are before us for determination: (1) the legality of the procedure
adopted by the plaintiff (respondent herein) Rosario Guevara; and (2) the efficacy of the
deed of sale exhibit 2 and the effect of the certificate of title issued to the defendant
(petitioner herein) Ernesto M. Guevara.
I
We cannot sanction the procedure adopted by the respondent Rosario Guevara, it being in
our opinion in violation of procedural law and an attempt to circumvent and disregard the
last will and testament of the decedent. The Code of Civil Procedure, which was in force up
to the time this case was decided by the trial court, contains the following pertinent
provisions:
Sec. 625. Allowance Necessary, and Conclusive as to Execution. No will shall
pass either the real or personal estate, unless it is proved and allowed in the
Court of First Instance, or by appeal to the Supreme Court; and the allowance by
the court of a will of real and personal estate shall be conclusive as to its due
execution.
Sec. 626. Custodian of Will to Deliver. The person who has the custody of a
will shall, within thirty days after he knows of the death of the testator, deliver the
will into the court which has jurisdiction, or to the executor named in the will.
Sec. 627. Executor to Present Will and Accept or Refuse Trust. A person
named as executor in a will, shall within thirty days after he knows of the death of

91 | S U C C E S S I O N

the testor, or within thirty days after he knows that he is named executor, if he
obtained such knowledge after knowing of the death of the testor, present such
will to the court which has jurisdiction, unless the will has been otherwise
returned to said court, and shall, within such period, signify to the court his
acceptance of the trust, or make known in writing his refusal to accept it.
Sec. 628. Penalty. A person who neglects any of the duties required in the two
proceeding sections, unless he gives a satisfactory excuse to the court, shall be
subject to a fine not exceeding one thousand dollars.
Sec. 629. Person Retaining Will may be Committed. If a person having
custody of a will after the death of the testator neglects without reasonable cause
to deliver the same to the court having jurisdiction, after notice by the court so to
do, he may be committed to the prison of the province by a warrant issued by the
court, and there kept in close confinement until he delivers the will.
The foregoing provisions are now embodied in Rule 76 of the new Rules of Court, which
took effect on July 1, 1940.
The proceeding for the probate of a will is one in rem, with notice by publication to the
whole world and with personal notice to each of the known heirs, legatees, and devisees of
the testator (section 630, C. c. P., and sections 3 and 4, Rule 77). Altho not contested
(section 5, Rule 77), the due execution of the will and the fact that the testator at the time
of its execution was of sound and disposing mind and not acting under duress, menace,
and undue influence or fraud, must be proved to the satisfaction of the court, and only then
may the will be legalized and given effect by means of a certificate of its allowance, signed
by the judge and attested by the seal of the court; and when the will devises real property,
attested copies thereof and of the certificate of allowance must be recorded in the register
of deeds of the province in which the land lies. (Section 12, Rule 77, and section 624, C. C.
P.)
It will readily be seen from the above provisions of the law that the presentation of a will to
the court for probate is mandatory and its allowance by the court is essential and
indispensable to its efficacy. To assure and compel the probate of will, the law punishes a
person who neglects his duty to present it to the court with a fine not exceeding P2,000,
and if he should persist in not presenting it, he may be committed to prision and kept there
until he delivers the will.
The Court of Appeals took express notice of these requirements of the law and held that a
will, unless probated, is ineffective. Nevertheless it sanctioned the procedure adopted by
the respondent for the following reasons:

The majority of the Court is of the opinion that if this case is dismissed ordering
the filing of testate proceedings, it would cause injustice, incovenience, delay,
and much expense to the parties, and that therefore, it is preferable to leave
them in the very status which they themselves have chosen, and to decide their
controversy once and for all, since, in a similar case, the Supreme Court applied
that same criterion (Leao vs. Leao, supra), which is now sanctioned by section
1 of Rule 74 of the Rules of Court. Besides, section 6 of Rule 124 provides that, if
the procedure which the court ought to follow in the exercise of its jurisdiction is
not specifically pointed out by the Rules of Court, any suitable process or mode
of procedure may be adopted which appears most consistent to the spirit of the
said Rules. Hence, we declare the action instituted by the plaintiff to be in
accordance with law.
Let us look into the validity of these considerations. Section 1 of Rule 74 provides as
follows:
Section 1. Extrajudicial settlement by agreement between heirs. If the
decedent left no debts and the heirs and legatees are all of age, or the minors
are represented by their judicial guardians, the parties may, without securing
letters of administration, divide the estate among themselves as they see fit by
means of a public instrument filed in the office of the register of deeds, and
should they disagree, they may do so in an ordinary action of partition. If there is
only one heir or one legatee, he may adjudicate to himself the entire estate by
means of an affidavit filed in the office of the register of deeds. It shall be
presumed that the decedent left no debts if no creditor files a petition for letters of
administration within two years after the death of the decedent.
That is a modification of section 596 of the Code of Civil Procedure, which reads as
follows:
Sec. 596. Settlement of Certain Intestates Without Legal Proceedings.
Whenever all the heirs of a person who died intestate are of lawful age and legal
capacity and there are no debts due from the estate, or all the debts have been
paid the heirs may, by agreement duly executed in writing by all of them, and not
otherwise, apportion and divide the estate among themselves, as they may see
fit, without proceedings in court.
The implication is that by the omission of the word "intestate" and the use of the word
"legatees" in section 1 of Rule 74, a summary extrajudicial settlement of a deceased
person's estate, whether he died testate or intestate, may be made under the conditions
specified. Even if we give retroactive effect to section 1 of Rule 74 and apply it here, as the
Court of Appeals did, we do not believe it sanctions the nonpresentation of a will for
probate and much less the nullification of such will thru the failure of its custodian to

92 | S U C C E S S I O N

present it to the court for probate; for such a result is precisely what Rule 76 sedulously
provides against. Section 1 of Rule 74 merely authorizes the extrajudicial or judicial
partition of the estate of a decedent "without securing letter of administration." It does not
say that in case the decedent left a will the heirs and legatees may divide the estate
among themselves without the necessity of presenting the will to the court for probate. The
petition to probate a will and the petition to issue letters of administration are two different
things, altho both may be made in the same case. the allowance of a will precedes the
issuance of letters testamentary or of administration (section 4, Rule 78). One can have a
will probated without necessarily securing letters testamentary or of administration. We
hold that under section 1 of Rule 74, in relation to Rule 76, if the decedent left a will and no
debts and the heirs and legatees desire to make an extrajudicial partition of the estate,
they must first present that will to the court for probate and divide the estate in accordance
with the will. They may not disregard the provisions of the will unless those provisions are
contrary to law. Neither may they so away with the presentation of the will to the court for
probate, because such suppression of the will is contrary to law and public policy. The law
enjoins the probate of the will and public policy requires it, because unless the will is
probated and notice thereof given to the whole world, the right of a person to dispose of his
property by will may be rendered nugatory, as is attempted to be done in the instant case.
Absent legatees and devisees, or such of them as may have no knowledge of the will,
could be cheated of their inheritance thru the collusion of some of the heirs who might
agree to the partition of the estate among themselves to the exclusion of others.
In the instant case there is no showing that the various legatees other than the present
litigants had received their respective legacies or that they had knowledge of the existence
and of the provisions of the will. Their right under the will cannot be disregarded, nor may
those rights be obliterated on account of the failure or refusal of the custodian of the will to
present it to the court for probate.
Even if the decedent left no debts and nobdy raises any question as to the authenticity and
due execution of the will, none of the heirs may sue for the partition of the estate in
accordance with that will without first securing its allowance or probate by the court, first,
because the law expressly provides that "no will shall pass either real or personal estate
unless it is proved and allowed in the proper court"; and, second, because the probate of a
will, which is a proceeding in rem, cannot be dispensed with the substituted by any other
proceeding, judicial or extrajudicial, without offending against public policy designed to
effectuate the testator's right to dispose of his property by will in accordance with law and
to protect the rights of the heirs and legatees under the will thru the means provided by
law, among which are the publication and the personal notices to each and all of said heirs
and legatees. Nor may the court approve and allow the will presented in evidence in such
an action for partition, which is one in personam, any more than it could decree the
registration under the Torrens system of the land involved in an ordinary action
for reinvindicacion or partition.

We therefore believe and so hold that section 1 of Rule 74, relied upon by the Court of
Appeals, does not sanction the procedure adopted by the respondent.
The case of Leao vs. Leao (25 Phil., 180), cited by the Court of Appeals, like section 1 of
Rule 74, sanctions the extrajudicial partition by the heirs of the properties left by a
decedent, but not the nonpresentation of a will for probate. In that case one Paulina Ver
executed a will on October 11, 1902, and died on November 1, 1902. Her will was
presented for probate on November 10, 1902, and was approved and allowed by the Court
on August 16, 1904. In the meantime, and on November 10, 1902, the heirs went ahead
and divided the properties among themselves and some of them subsequently sold and
disposed of their shares to third persons. It does not affirmatively appear in the decision in
that case that the partition made by the heirs was not in accordance with the will or that
they in any way disregarded the will. In closing the case by its order dated September 1,
1911, the trial court validated the partition, and one of the heirs, Cunegunda Leao,
appealed. In deciding the appeal this Court said:
The principal assignment of error is that the lower court committed an error in
deciding that the heirs and legatees of the estate of Da. Paulina Ver had
voluntarily divided the estate among themselves.
In resolving that question this Court said:
In view of the positive finding of the judge of the lower court that there had been
a voluntary partition of the estate among the heirs and legatees, and in the
absence of positive proof to the contrary, we must conclude that the lower court
had some evidence to support its conclusion.
Thus it will be seen that as a matter of fact no question of law was raised and decided in
that case. That decision cannot be relied upon as an authority for the unprecedented and
unheard of procedure adopted by the respondent whereby she seeks to prove her status
as an acknowledged natural child of the decedent by his will and attempts to nullify and
circumvent the testamentary dispositions made by him by not presenting the will to the
court for probate and by claiming her legitime as an acknowledged natural child on the
basis of intestacy; and that in the face of express mandatory provisions of the law requiring
her to present the will to the court for probate.
In the subsequent case of Riosa vs. Rocha (1926), 48 Phil. 737, this Court departed from
the procedure sanctioned by the trial court and impliedly approved by this Court in the
Leao case, by holding that an extrajudicial partition is not proper in testate succession. In
the Riosa case the Court, speaking thru Chief Justice Avancea, held:
1. EXTRAJUDICIAL PARTITION; NOT PROPER IN TESTATE SUCCESSION.
Section 596 of the Code of Civil Procedure, authorizing the heirs of a person who

93 | S U C C E S S I O N

dies intestate to make extrajudicial partition of the property of the deceased,


without going into any court of justice, makes express reference to intestate
succession, and therefore excludes testate succession.
2. ID.; EFFECTS OF; TESTATE SUCCESSION. In the instant case, which is a
testate succession, the heirs made an extrajudicial partition of the estate and at
the same time instituted proceeding for the probate of the will and the
administration of the estate. When the time came for making the partition, they
submitted to the court the extrajudicial partition previously made by them, which
the court approved. Held: That for the purposes of the reservation and the rights
and obligations created thereby, in connection with the relatives benefited, the
property must not be deemed transmitted to the heirs from the time the
extrajudicial partition was made, but from the time said partition was approved by
the court. (Syllabus.)
The Court of Appeals also cites section 6 of Rule 124, which provides that if the procedure
which the court ought to follow in the exercise of its jurisdiction is not specifically pointed
out by the Rules of Court, any suitable process for mode of proceeding may be adopted
which appears most conformable to the spirit of the said Rules. That provision is not
applicable here for the simple reason that the procedure which the court ought to follow in
the exercise of its jurisdiction is specifically pointed out and prescribed in detail by Rules
74, 76, and 77 of the Rules of Court.
The Court of Appeals also said "that if this case is dismissed, ordering the filing of testate
proceedings, it would cause injustice, inconvenience, delay, and much expense to the
parties." We see no injustice in requiring the plaintiff not to violate but to comply with the
law. On the contrary, an injustice might be committed against the other heirs and legatees
mentioned in the will if the attempt of the plaintiff to nullify said will by not presenting it to
the court for probate should be sanctioned. As to the inconvenience, delay, and expense,
the plaintiff herself is to blame because she was the custodian of the will and she violated
the duty imposed upon her by sections 2, 4, and 5 of Rule 76, which command her to
deliver said will to the court on pain of a fine not exceeding P2,000 and of imprisonment for
contempt of court. As for the defendant, he is not complaining of inconvenience, delay, and
expense, but on the contrary he is insisting that the procedure prescribed by law be
followed by the plaintiff.
Our conclusion is that the Court of Appeals erred in declaring the action instituted by the
plaintiff to be in accordance with law. It also erred in awarding relief to the plaintiff in this
action on the basis of intestacy of the decedent notwithstanding the proven existence of a
will left by him and solely because said will has not been probated due to the failure of the
plaintiff as custodian thereof to comply with the duty imposed upon her by the law.

It is apparent that the defendant Ernesto M. Guevara, who was named executor in said
will, did not take any step to have it presented to the court for probate and did not signify
his acceptance of the trust or refusal to accept it as required by section 3 of Rule 76
(formerly section 627 of the Code of Civil Procedure), because his contention is that said
will, insofar as the large parcel of land in litigation is concerned, has been superseded by
the deed of sale exhibit 2 and by the subsequent issuance of the Torrens certificate of title
in his favor.
II
This brings us to the consideration of the second question, referring to the efficacy of the
deed of sale exhibit 2 and the effect of the certificate of titled issued to the defendant
Ernesto M. Guevara. So that the parties may not have litigated here in vain insofar as that
question is concerned, we deem it proper to decide it now and obviate the necessity of a
new action.
The deed of sale exhibit 2 executed by and between Victorino L. Guevara and Ernesto M.
Guevara before a notary public on July 12, 1933, may be divided into two parts: (a) insofar
as it disposes of and conveys to Ernesto M. Guevara the southern half of Victorino L.
Guevara's hacienda of 259-odd hectares in consideration of P1 and other valuable
considerations therein mentioned; and (b) insofar as it declares that Ernesto M. Guevara
became the owner of the northern half of the same hacienda by repurchasing it with his
own money from Rafael T. Puzon.
A. As to the conveyance of the southern half of the hacienda to Ernesto M. Guevara in
consideration of the latter's assumption of the obligation to pay all the debts of the
deceased, the Court of Appeals found it to be valid and efficacious because: "(a) it has not
been proven that the charges imposed as a condition is [are] less than the value of the
property; and (b) neither has it been proven that the defendant did not comply with the
conditions imposed upon him in the deed of transfer." As a matter of fact the Court of
Appeals found" "It appears that the defendant has been paying the debts left by his father.
To accomplish this, he had to alienate considerable portions of the above-mentioned land.
And we cannot brand such alienation as anomalous unless it is proven that they have
exceeded the value of what he has acquired by virtue of the deed of July 12, 1933, and
that of his corresponding share in the inheritance." The finding of the Court of Appeals on
this aspect of the case is final and conclusive upon the respondent, who did not appeal
therefrom.
B. With regard to the northern half of the hacienda, the findings of fact and of law made by
the Court of Appeals are as follows:
The defendant has tried to prove that with his own money, he bought from Rafael
Puzon one-half of the land in question, but the Court a quo, after considering the

94 | S U C C E S S I O N

evidence, found it not proven; we hold that such conclusion is well founded. The
acknowledgment by the deceased, Victorino L. Guevara, of the said transactions,
which was inserted incidentally in the document of July 12, 1933, is clearly belied
by the fact that the money paid to Rafael Puzon came from Silvestre P. Coquia,
to whom Victorino L. Guevara had sold a parcel of land with the right of
repurchase. The defendant, acting for his father, received the money and
delivered it to Rafael Puzon to redeem the land in question, and instead of
executing a deed of redemption in favor of Victorino L. Guevara, the latter
executed a deed of sale in favor of the defendant.
The plaintiff avers that she withdrew her opposition to the registration of the land
in the name of the defendant, because of the latter's promise that after paying all
the debt of their father, he would deliver to her and to the widow their
corresponding shares. As their father then was still alive, there was no reason to
require the delivery of her share and that was why she did not insist on her
opposition, trusting on the reliability and sincerity of her brother's promise. The
evidence shows that such promise was really made. The registration of land
under the Torrens system does not have the effect of altering the laws of
succession, or the rights of partition between coparceners, joint tenants, and
other cotenants nor does it change or affect in any other way any other rights and
liabilities created by law and applicable to unregistered land (sec. 70, Land
Registration Law). The plaintiff is not, then, in estoppel, nor can the doctrine
of res judicata be invoked against her claim. Under these circumstances, she has
the right to compel the defendant to deliver her corresponding share in the estate
left by the deceased, Victorino L. Guevara.
In his tenth to fourteenth assignments of error the petitioner assails the foregoing findings
of the Court of Appeals. But the findings of fact made by said court are final and not
reviewable by us on certiorari. The Court of Appeals found that the money with which the
petitioner repurchased the northern half of the land in question from Rafael Puzon was not
his own but his father's, it being the proceeds of the sale of a parcel of land made by the
latter to Silvestre P. Coquia. Said court also found that the respondent withdrew her
opposition to the registration of the land in the name of the petitioner upon the latter's
promise that after paying all the debts of their father he would deliver to her and to the
widow their corresponding shares. From these facts, it results that the interested parties
consented to the registration of the land in question in the name of Ernesto M. Guevara
alone subject to the implied trust on account of which he is under obligation to deliver and
convey to them their corresponding shares after all the debts of the original owner of said
land had been paid. Such finding does not constitute a reversal of the decision and decree
of registration, which merely confirmed the petitioner's title; and in the absence of any
intervening innocent third party, the petitioner may be compelled to fulfill the promise by
virtue of which he acquired his title. That is authorized by section 70 of the Land
Registration Act, cited by the Court of Appeals, and by the decision of this Court
in Severino vs. Severino, 44 Phil., 343, and the cases therein cited.

Upon this phase of the litigation, we affirm the finding of the Court of Appeals that the
northern half of the land described in the will exhibit A and in original certificate of title No.
51691 still belongs to the estate of the deceased Victorino L. Guevara. In the event the
petitioner Ernesto M. Guevara has alienated any portion thereof, he is under obligation to
compensate the estate with an equivalent portion from the southern half of said land that
has not yet been sold. In other words, to the estate of Victorino L. Guevara still belongs
one half of the total area of the land described in said original certificate of title, to be taken
from such portions as have not yet been sold by the petitioner, the other half having been
lawfully acquired by the latter in consideration of his assuming the obligation to pay all the
debts of the deceased.

"WHEREFORE, premises considered, judgment is hereby rendered:

Wherefore, that part of the decision of the Court of Appeals which declares in effect that
notwithstanding exhibit 2 and the issuance of original certificate of title No. 51691 in the
name of Ernesto M. Guevara, one half of the land described in said certificate of title
belongs to the estate of Victorino L. Guevara and the other half to Ernesto M. Guevara in
consideration of the latter's assumption of the obligation to pay all the debts of the
deceased, is hereby affirmed; but the judgment of said court insofar as it awards any relief
to the respondent Rosario Guevara in this action is hereby reversed and set aside, and the
parties herein are hereby ordered to present the document exhibit A to the proper court for
probate in accordance with law, without prejudice to such action as the provincial fiscal of
Pangasinan may take against the responsible party or parties under section 4 of Rule 76.
After the said document is approved and allowed by the court as the last will and testament
of the deceased Victorino L. Guevara, the heirs and legatees therein named may take
such action, judicial or extrajudicial, as may be necessary to partition the estate of the
testator, taking into consideration the pronouncements made in part II of this opinion. No
finding as to costs in any of the three instances.

5. Ordering the defendants to jointly and severally pay unto the plaintiff the sum
of P2,000.00 as attorney's fees and P10,000.00 as moral and exemplary
damages.

Yulo, C.J., and Hontiveros, 1 J., concur.

[G.R. No. 78778 : December 3, 1990.]


191 SCRA 814
LEONIDA CORONADO, FELIX BUENO, MELANIA RETIZOS, BERNARDINO
BUENASEDA and JOVITA MONTEFALCON, Petitioners, vs. THE COURT OF
APPEALS and JUANA BUENO ALBOVIAS, Respondents.
PARAS, J.:

This is a petition for review on certiorari seeking to reverse the decision* of the respondent
appellate court dated March 3, 1987 CA-G.R. CV No. 06911 entitled "Juana (Bueno)
Albovias et al., v. Leonida Coronado, et al.," affirming the decision of the lower court, the
decretal portion of which reads:: nad

95 | S U C C E S S I O N

1. Declaring Leonida Coronado to have no title or interest over the property in


question, hence, has no authority to dispose of the same in favor of her codefendants;
2. Declaring the sales executed by Coronado and subsequent transactions
involving the same property null and void ab initio;
3. Declaring the plaintiff to be the true and legal owner of the subject parcel of
land;
4. Ordering the defendants to vacate the subject premises and to surrender
possession thereof unto the plaintiff;

Costs against the defendants." (Rollo, p. 17)


As found by the respondent appellate court, the property subject of this case is a parcel of
land situated in Nagcarlan, Laguna, containing 277 square meters, more particularly
described as follows:: nad
"A parcel of land situated in the Poblacion, Municipality of Nagcarlan, province of
Laguna. Bounded on the North, by property of Epifania Irlandez (formerly
Bonifacio Formentera); on the East, by that of Julio Lopez; on the South, by that
of Dalmacio Monterola (formerly Domingo Bueno); and on the West, by C. Lirio
Street. Containing an area of two hundred seventy seven (277) square meters,
more or less. Assessed at P3,320.00 under tax declaration No. 241." (Ibid., p. 15)
Said parcel of land is being contested by Juana Albovias, herein private respondent, on the
one hand, and Leonida-Coronado, Felix Bueno, Melania Retizos, Bernardino Buenseda
and Jovita Montefalcon, herein petitioners, on the other hand.
Juana Albovias (JUANA, for brevity) claims that the property in question is a portion of a
bigger lot referred to as Parcel G in the last will and testament executed in 1918 by
Melecio Artiaga, grandfather of JUANA. This bigger lot was inherited under that will by
JUANA, her brother Domingo Bueno, and two other grandchildren, namely Bonifacio and
Herminigildo, both surnamed Formentera. Parcel G is described as follows:
"Isang lagay na lupa na ang bahagi ay walang tanim na halaman at ang bahagi naman ay
may tanim na saguing, tumatayo sa gawin Canloran ng Calle Avenida Rizal nitong
Nagcarlan, at humahangan sa Ibaba; sa ari cong Testador; sa Silangan, sa cay Enrique
Jovellano; sa Ilaya, sa namatay na Perfecto Nanagas, at sa Canloran, tubig na
pinamamagatang San Cido." (Ibid., p. 16)
JUANA further claims that sometime in 1925 or 1926, C. Lirio Street was created by the
Municipality of Nagcarla traversing said Parcel G and thus dividing it into two portions, one
on the west of C. Lirio St. and the other to the east of said street. Parcel G was divided by
the heirs in the following manner; the land was divided into two portions, the northern
portion of which was adjudicated in favor of the Formenteras and the southern portion was
given to JUANA and Doming Bueno. The southern portion in turn was partitioned between
JUANA and Domingo Bueno, the former getting the northern part adjoining the lot of the
Formenteras, and the latter the southern part which adjoins the lot of Perfecto Nanagas

(not owned by Dalmacio Monterola). The part allocated to Domingo was later sold by him
to Dalmacio Monterola, owner of the adjoining property (Ibid.).: nad

SAME IN THE ESTATE PROCEEDING IN THE TRIAL COURT AND EVEN ON APPEAL.
(Ibid., p. 119)

Moreover, JUANA claims that her property was included together with the two parcels of
land owned by Dalmacio Monterola, which were sold by Monterola's successor-in-interest
Leonida Coronado (now married to Felix Bueno) to Melania Retizos on April 18, 1970.
Melania Retizos in turn sold the lots, including that one being claimed by JUANA, to the
spouse Bernardino Buenaseda and Jovita Montefalcon, now the present possessors
thereof, sometime in 1974 (Ibid., pp. 16-17).

IV

On the other hand, Leonida Coronado and her co-petitioners (CORONADO, for brevity)
claim that the property in question was bequeathed to Leonida Coronado under a Will
executed by Dr. Dalmacio Monterola, who was allegedly in possession thereof even before
the outbreak of World War II (Ibid., p. 107).
Parenthetically, said will was probated under Sp. Proc. No. SC-283, entitled "Testate
Estate of the Deceased Monterola Leonida F. Coronado, petitioner (Ibid., p. 105). JUANA,
together with her husband, opposed the said probate. Despite their opposition, however,
the Will was allowed by the then Court of First Instance of Laguna, Sta. Cruz Branch (Ibid.,
p. 106). On appeal, said decision was affirmed by the Court of Appeals in CA-G.R. No.
40353, entitled "Leonida F. Coronado, petitioner-appellee v. Heirs of Dr. Dalmacio
Monterola, oppositors-appellants" (Ibid.). It is not apparent, however, from the record
whether or not said decision has already become final and executory.
As a result of the conflicting claims over the property in question, JUANA filed an action for
quieting of title, declaratory relief and damages against CORONADO in the Regional Trial
Court of the Fourth Judicial Region, Branch XXVI, Sta. Cruz, Laguna, docketed as Civil
Case No. 7345 (Ibid., p. 4).
As adverted to above (first par.), the lower court rendered judgment in favor of JUANA.
Not satisfied with the decision of the lower court, CORONADO elevated the case to the
Court of Appeals, which affirmed the decision appealed from (Ibid., p. 20). Hence, this
petition.:-cralaw
CORONADO raised the following assigned errors:
I
THE RESPONDENT COURT OF APPEALS SERIOUSLY ERRED IN ARRIVING AT A
CONCLUSION WHICH IS CONTRARY TO THE FACTS AND CIRCUMSTANCES OF THE
CASE AND IN NOT APPLYING THE APPLICABLE PROVISION OF LAW AND
JURISPRUDENCE LAID DOWN BY THIS HONORABLE COURT. (Ibid., p. 108)
II
THERE IS NO EVIDENCE PRESENTED TO SHOW THAT THE LAND IN QUESTION
CLAIMED BY PRIVATE RESPONDENT IS THE SAME PROPERTY ADJUDICATED TO
JUANA BUENO UNDER THE WILL OF THE DECEASED MELECIO ARTIAGA; NEITHER
IS THERE EVIDENCE TO SHOW THAT SAID WILL HAD BEEN PROBATED. (Ibid., p.
114)
III
PRIVATE RESPONDENT IS IN ESTOPPEL FROM QUESTIONING THE OWNERSHIP OF
THE PETITIONER OVER THE LAND IN QUESTION HAVING FAILED TO RAISE THE

96 | S U C C E S S I O N

THE RESPONDENT COURT OF APPEALS MISAPPRECIATED THE EVIDENCE


SUBMITTED AND FACTS ADMITTED ON RECORD. IT THEREFORE COMMITTED
GRAVE AND SERIOUS ERROR. (Ibid., p. 121)
As required by this Court, CORONADO filed their memorandum on May 8, 1989 (Ibid., p.
105); while that of JUANA was filed on October 13, 1989 (Ibid., p. 139).
The petition is devoid of merit.
Under the first assigned error, CORONADO assails the respondent appellate court's
finding that Dr. Dalmacio Monterola could not have acquired the subject land by acquisitive
prescription. Citing Art. 1116 of the New Civil Code in relation to Section 41 of the Code of
Civil Procedure, CORONADO claims that JUANA had already foreclosed whatever right or
legal title she had over the property in question, the reason being that Monterola's
continued possession of the said property for over ten years since 1934 ripened into full
and absolute ownership (Ibid., p. 112).
The argument has no factual basis.
Time and again, it has been ruled that the jurisdiction of the Supreme Court in cases
brought to it from the Court of Appeals is limited to reviewing and revising the errors of law
imputed to it, its findings of fact being conclusive. It is not the function of the Supreme
Court to analyze or weigh such evidence all over again, its jurisdiction being limited to
reviewing errors of law that might have been committed. Absent, therefore, a showing that
the findings complained of are totally devoid of support in the record, so that they are so
glaringly erroneous as to constitute serious abuse of discretion, such findings must stand,
for the Supreme Court is not expected or required to examine or contrast the oral and
documentary evidence submitted by the parties (Andres v. Manufacturers Hanover & Trust
Corporation, G.R. 82670, September 15, 1989). There are no convincing reasons in the
instant case to depart from this rule.
As found by the respondent appellate court, Monterola never claimed ownership over the
property in question. As a matter of fact, one of the deeds of donation executed by
Monterola in favor of Leonida Coronado acknowledged that the boundary owner on the
property conveyed to her is JUANA. This is precisely the reason why during the lifetime of
the late Dalmacio Monterola, JUANA had always been allowed to enter and reap the
benefits or produce of the said property. It was only after the death of said Monterola in
1970 that Leonida Coronado prohibited JUANA from entering it (Ibid., p. 18).:- nad
Even assuming arguendo that Monterola was indeed in continued possession of the said
property for over ten years since 1934, said possession is insufficient to constitute the
fundamental basis of the prescription. Possession, under the Civil Code, to constitute the
foundation of a prescriptive right, must be possession under claim of title (en concepto de
dueno), or to use the common law equivalent of the term, it must be adverse. Acts of
possessory character performed by one who holds by mere tolerance of the owner are
clearly not en concepto de dueno, and such possessory acts, no matter how long so
continued, do not start the running of the period of prescription (Manila Electric Company v.
Intermediate Appellate Court, G.R. 71393, June 28, 1989).

In this case, Monterola, as found by the respondent appellate court and the lower court,
never categorically claimed ownership over the property in question, much less his
possession thereof en concepto de dueno. Accordingly, he could not have acquired said
property by acquisitive prescription.
Anent the contention of CORONADO that Leonida Coronado could tack her possession to
that of Monterola, so that claim of legal title or ownership over the subject property, even
against the petitioners, the Buenasesas, who are purchasers for value and in good faith, is
a foregone or settled issue, the respondent appellate court aptly answered the same in this
wise:
"It follows that Leonida Coronado could not have derived ownership of the land in question
from her predecessor-in-interest Dalmacio Monterola, whether by prescription or by some
other title. Neither can she claim acquisitive prescription in her own name. It was only in
1970 after the death of Dalmacio Monterola that she asserted her claim of ownership
adverse to that of plaintiff-appellee. Having knowledge that she had no title over the land in
question, she must be deemed to have claimed it in bad faith. Under Article 1137 of the
Civil Code, ownership and other real rights over immovables prescribe through
uninterrupted adverse possession thereof for thirty years, without need of title or good
faith. And even granting that she had no notice or defect in her title and was, therefore, in
good faith, a period of ten years of possession is necessary for her to acquire the land by
ordinary prescription. (Article 1134, Civil Code). But she can claim to have possessed the
land only in 1968, the year the Monterola lots were donated to her. The period, however,
was interrupted in 1975, or 7 years after, when the complaint below was filed." (Rollo, pp.
18-19)
Under the second assigned error, CORONADO claims that the will under which JUANA
inherited the property in question from her grandfather, Melecio Artiaga, was never
probated; hence, said transfer for ownership was ineffectual considering that under Rule
75, Sec. 1 of the Rules of Court (formerly Sec. 125 of Act No. 190, no will shall pass either
real or personal property unless it is proved and allowed in the proper court (Ibid., p. 115).

Normally, the probate of a will does not look into its intrinsic validity. The authentication of a
will decides no other questions than such as touch upon the capacity of the testator and
the compliance with those requisites or solemnities which the law prescribes for the validity
of the wills. It does not determine nor even by implication prejudge the validity or efficiency
of the provisions of the will, thus may be impugned as being vicious or null,
notwithstanding its authentication. The question relating to these points remain entirely
unaffected, and may be raised even after the will has been authenticated (Maninang, et al.,
v. Court of Appeals, 114 SCRA 473 [1982]). Consequently, JUANA is not estopped from
questioning the ownership of the property in question, notwithstanding her having objected
to the probate of the will executed by Monterola under which Leonida Coronado is claiming
title to the said property.:-cralaw
Under the fourth assigned error, it is alleged by CORONADO that JUANA's petition is weak
for want of factual and legal support; the weakness of JUANA's position lies in the fact that
she did not only fail to identify the subject land, but also failed to explain the discrepancy in
the boundary of the property she is claiming to be hers (Rollo, p. 125).
The contention is unavailing.
The fact that JUANA failed to identify the property in question and to explain the
discrepancy in the boundary of said property, assuming they are true, is immaterial, in view
of the findings of the lower court as to the identity of the property in question. Moreover, the
lower court found sufficient evidence to support the conclusion that the property in question
is the same property adjudicated to JUANA under the will of Melecio Artiaga, and that
CORONADO has no right whatsoever to said property (Ibid., p. 20). Such findings are
conclusive upon this Court (Reynolds Philippine Corporation v. Court of Appeals, 169
SCRA 220 [1989]).
PREMISES CONSIDERED, the decision appealed from is hereby AFFIRMED.

G.R. No. 115925

August 15, 2003

The contention is without merit.chanrobles virtual law library


While it is true that no will shall pass either real or personal property unless it is proved and
allowed in the proper court (Art. 838, Civil Code), the questioned will, however, may be
sustained on the basis of Article 1056 of the Civil Code of 1899, which was in force at the
time said document was executed by Melecio Artiaga in 1918. The said article read as
follows:
"Article 1056. If the testator should make a partition of his properties by an act inter vivos,
or by will, such partition shall stand in so far as it does not prejudice the legitime of the
forced heir." (Mang-Oy v. Court of Appeals, 144 SCRA 33 [1986])
In this case, nowhere was it alleged nor shown that Leonida Coronado is entitled to
legitime from Melecio Artiaga. The truth of the matter is that the record is bereft of any
showing that Leonida Coronado and the late Melecio Artiaga were related to each other.
Under the third assigned error, CORONADO claims that JUANA is estopped from
questioning the ownership of Leonida Coronado over the land in question having failed to
raise the same in the estate proceedings in the trial court and even on appeal (Rollo, p.
119).
The contention is likewise without merit.

97 | S U C C E S S I O N

SPOUSES RICARDO PASCUAL and CONSOLACION SIOSON, petitioners,


vs.
COURT OF APPEALS and REMEDIOS S. EUGENIO-GINO, respondents.
CARPIO, J.:
The Case
This is a petition for review of the Decision 1 dated 31 January 1994 of the Court of Appeals
ordering the Register of Deeds of Metro Manila, District III, to place TCT No. (232252)
1321 in the name of respondent Remedios S. Eugenio-Gino. The Decision ordered the
Register of Deeds to cancel the names of petitioners Ricardo Pascual and Consolacion
Sioson ("petitioners") in TCT No. (232252) 1321. The Decision also directed petitioners to
pay respondent moral and exemplary damages and attorneys fees.
The Facts

Petitioner Consolacion Sioson ("CONSOLACION") and respondent Remedios S. EugenioGino ("REMEDIOS") are the niece and granddaughter, respectively, of the late Canuto
Sioson ("CANUTO"). CANUTO and 11 other individuals, including his sister Catalina
Sioson ("CATALINA") and his brother Victoriano Sioson ("VICTORIANO"), were co-owners
of a parcel of land in Tanza, Navotas, Metro Manila. The property, known as Lot 2 of Plan
Psu 13245, had an area of 9,347 square meters and was covered by Original Certificate of
Title No. 4207 issued by the Register of Deeds of Rizal. CATALINA, CANUTO, and
VICTORIANO each owned an aliquot 10/70 share or 1,335 square meters of Lot 2. 2
On 20 November 1951, CANUTO had Lot 2 surveyed and subdivided into eight lots (Lot
Nos. 2-A to 2-H) through Subdivision Plan Psd 34713 which the Director of Lands
approved on 30 May 1952. Lot No. 2-A, with an area of 670 square meters, and Lot No. 2E, with an area of 2,000 square meters, were placed under CANUTOs name. Three other
individuals took the remaining lots.3
On 26 September 1956, CANUTO and CONSOLACION executed a Kasulatan ng Bilihang
Tuluyan4("KASULATAN"). Under the KASULATAN, CANUTO sold his 10/70 share in Lot 2
in favor of CONSOLACION forP2,250.00. The KASULATAN, notarized by Notary Public
Jose T. de los Santos of Navotas, provides:
Na ako, CANUTO SIOSON, mamamayang Pilipino, may katampatang gulang,
kasal kay Raymunda San Diego, at naninirahan sa Tanza, Navotas, Rizal, sa
bisa at pamamagitan ng kasulatang ito ay nagpapatunay at nagpapatibay:
1. Na ako ang lubos at tunay na may-ari ng 10/70 bahaging hindi hati
(10/70 porcion pro-indiviso) ng isang lagay na lupa (Lote No. 2, Plano
Psu-13245), na nasa sa nayon ng Tanza, Municipio ng Navotas,
Provincia ng Rizal, at ang descripcion o pagkakakilanlan ng nasabing
lote ay nakasaad sa Certificado Original, de Titulo No. 4207 ng Oficina
ng Registrador de Titulos ng Rizal, gaya ng sumusunod:
xxxx

2. Na dahil at alang-alang sa halagang Dalawang Libo Dalawang Daan


at Limampung Piso (P2,250.00), salaping Pilipino, na sa akin ay
ibinayad ni CONSOLACION SIOSON, kasal kay Ricardo S. Pascual,
may sapat na gulang, mamamayang Pilipino, at naninirahan sa
Dampalit, Malabon, Rizal at ang pagkakatanggap ng nasabing halaga
ay aking inaamin at pinatutunayan, ay aking ipinagbili, inilipat at isinalin,
sa pamamagitan ng bilihang tuluyan at walang pasubali a favor [sic] sa
nasabing si CONSOLACION SIOSON, sa kanyang tagapagmana at
mapaglilipatan ang lahat ng aking titulo, karapatan at kaparti na
binubuo ng 10/70 bahaging hindi hati (10/70 porcion pro-indiviso) ng
loteng descrito or tinutukoy sa itaas nito. (Emphasis supplied)
CONSOLACION immediately took possession of Lot Nos. 2-A and 2-E. She later
declared the land for taxation purposes and paid the corresponding real estate
taxes.5
On 23 October 1968, the surviving children of CANUTO, namely, Felicidad and Beatriz,
executed a joint affidavit6("JOINT AFFIDAVIT") affirming the KASULATAN in favor of
CONSOLACION. They also attested that the lots their father had sold to CONSOLACION
were Lot Nos. 2-A and 2-E of Subdivision Plan Psd 34713. The JOINT AFFIDAVIT reads:
KAMING sina FELICIDAD SIOSON at BEATRIZ SIOSON, pawang mga Pilipino,
kapuwa may sapat na gulang at naninirahan, ang una sa Tanza, Navotas at ang
ikalawa sa Concepcion, Malabon, lalawigan ng Rizal, sa ilalim ng isang ganap na
panunumpa alinsunod sa batas, ay malayang nagsasalaysay ng mga
sumusunod:
Na kami ang mga buhay na anak na naiwan ni CANUTO SIOSON na
nagmamay-ari ng 10/70 bahaging hindi hati (10/70 porcion pro-indiviso)
ng isang lagay na lupa (Lote No. 2, plano Psu-13245), na nasa Nayon
ng Tanza, Navotas, Rizal, at ang mga palatandaan nito ay nasasaad sa
Certificado Original de Titulo No. 4207 ng Tanggapan ng Registrador de
Titulos ng Rizal;
Na sa lubos naming kaalaman, ay ipinagbili ng aming Ama na si
Canuto Sioson ang kaniyang buong bahagi na 10/70 sa nasabing Lote
No. 2, kay CONSOLACION SIOSON, may-bahay ni Ricardo S.
Pascual, na taga Dampalit, Malabon, Rizal, sa halagang P2,250.00,
salaping pilipino, noong ika 16 [sic] ng Septiembre, 1956, sa
pamamagitan ng isang KASULATAN NG BILIHANG TULUYAN na
pinagtibay sa harap ng Notario Publico Jose T. de los Santos nang
pechang nabanggit, sa Navotas, Rizal, (Doc. No. 194, Page No. 84;
Book No. IV; Series of 1956);

98 | S U C C E S S I O N

Na ang nasabing lupa na ipinagbili ng aming Ama kay Consolacion


Sioson ni Pascual, ay nakikilala ngayong mga Lote No. 2-A at Lote 2-E
ng Plano de Subdivision Psd-34713; na pinagtibay ng Assistant
Director of Lands noong Mayo 30, 1952;
Na aming ngayong pinatitibayan ang pagka-pagbili ng bahagi ng aming
Ama kay Consolacion Sioson ni Pascual ng ngayoy nakikilalang Lote
No. 2-A at Lote No. 2-E ng Plano de Subdivision Psd-34713. (Emphasis
supplied)
On 28 October 1968, CONSOLACION registered the KASULATAN and the JOINT
AFFIDAVIT with the Office of the Register of Deeds of Rizal ("Register of Deeds"). Based
on these documents, the Register of Deeds issued to CONSOLACION Transfer Certificate
of Title No. (232252) 1321 covering Lot Nos. 2-A and 2-E of Subdivision Plan Psd 34713
with a total area of 2,670 square meters.
On 4 February 1988, REMEDIOS filed a complaint against CONSOLACION and her
spouse Ricardo Pascual in the Regional Trial Court of Malabon, Branch 165, for
"Annulment or Cancellation of Transfer Certificate [of Title] and Damages." REMEDIOS
claimed that she is the owner of Lot Nos. 2-A and 2-E because CATALINA devised these
lots to her in CATALINAs last will and testament 7 ("LAST WILL") dated 29 May 1964.
REMEDIOS added that CONSOLACION obtained title to these lots through fraudulent
means since the area covered by TCT (232252) 1321 is twice the size of CANUTOs share
in Lot 2. REMEDIOS prayed for the cancellation of CONSOLACIONs title, the issuance of
another title in her name, and the payment to her of damages.
Petitioners sought to dismiss the complaint on the ground of prescription. Petitioners
claimed that the basis of the action is fraud, and REMEDIOS should have filed the action
within four years from the registration of CONSOLACIONs title on 28 October 1968 and
not some 19 years later on 4 February 1988. REMEDIOS opposed the motion, claiming
that she became aware of CONSOLACIONs adverse title only in February 1987.
CONSOLACION maintained that she had timely filed her complaint within the four-year
prescriptive on 4 February 1988.
In its order of 28 April 1988, the trial court denied petitioners motion to dismiss. The trial
court held that the reckoning of the prescriptive period for filing REMEDIOS complaint is
evidentiary in nature and must await the presentation of the parties evidence during the
trial. During the pre-trial stage, REMEDIOS clarified that she was claiming only
CATALINAs 10/70 share in Lot 2, or 1,335 square meters, which constitute of the area
of Lot Nos. 2-A and 2-E.8 The trial of the case then ensued.
The Ruling of the Trial Court

99 | S U C C E S S I O N

On 26 November 1990, the trial court rendered judgment dismissing the case and ordering
REMEDIOS to pay petitioners P10,000 as attorneys fees and the cost of suit. The trial
court held that the action filed by REMEDIOS is based on fraud, covered by the four-year
prescriptive period. The trial court also held that REMEDIOS knew of petitioners adverse
title on 19 November 1982 when REMEDIOS testified against petitioners in an ejectment
suit petitioners had filed against their tenants in Lot Nos. 2-A and 2-E. Thus, the complaint
of REMEDIOS had already prescribed when she filed it on 4 February 1988.
The trial court further ruled that REMEDIOS has no right of action against petitioners
because CATALINAs LAST WILL from which REMEDIOS claims to derive her title has not
been admitted to probate. Under Article 838 of the Civil Code, no will passes real or
personal property unless it is allowed in probate in accordance with the Rules of Court.
The dispositive portion of the trial courts decision provides:
WHEREFORE, judgment is hereby rendered in favor of the defendants and
against plaintiff, ordering:
1. The dismissal of this case;
2. The plaintiff to pay the defendants the sum of Ten Thousand
(P10,000.00) Pesos as and for attorneys fees; and
3. The plaintiff to pay the costs of suit.9
REMEDIOS appealed to the Court of Appeals.
The Ruling of the Court of Appeals
On 31 January 1994, the Court of Appeals rendered judgment reversing the decision of the
trial court. The appellate court held that what REMEDIOS filed was a suit to enforce an
implied trust allegedly created in her favor when CONSOLACION fraudulently registered
her title over Lot Nos. 2-A and 2-E. Consequently, the prescriptive period for filing the
complaint is ten years, not four. The Court of Appeals counted this ten-year period from 19
November 1982. Thus, when REMEDIOS filed her complaint on 4 February 1988, the tenyear prescriptive period had not yet expired.
The appellate court held that CATALINAs unprobated LAST WILL does not preclude
REMEDIOS from seeking reconveyance of Lot Nos. 2-A and 2-E as the LAST WILL may
subsequently be admitted to probate. The dispositive portion of the appellate courts ruling
provides:

WHEREFORE, the decision appealed from is REVERSED and SET ASIDE. The
Registry of Deeds of Rizal or Metro Manila, District III, is ordered to place
Transfer Certificate of Title No. (232252) 1321 under the name of Remedios S.
Eugenio-Gino as executor of the will of Catalina Sioson and cancel the names of
the Spouses Ricardo Pascual and Consolacion Sioson inscribed over said title as
owners of the covered lot. Defendants-appellees spouses Ricardo Pascual and
Consolacion Sioson are ordered to pay plaintiff-appellant Remedios S. EugenioGino moral damages in the amount of P50,000.00, exemplary damages
ofP20,000[.00] and attorneys fees of P20,000.00 and P500.00 per appearance.10
Petitioners sought reconsideration of the ruling. However, the Court of Appeals denied their
motion in its order dated 15 June 1994.
Hence, this petition.
The Issues
Petitioners allege the following assignment of errors:
I. THE COURT OF APPEALS ERRED IN HOLDING THAT PRIVATE
RESPONDENTS CAUSE OF ACTION IS NOT BARRED BY PRESCRIPTION
WHICH FINDING IS MANIFESTLY CONTRARY TO LAW AND THE
APPLICABLE DECISIONS OF THIS HONORABLE COURT.
II. THE COURT OF APPEALS ERRED IN NOT HOLDING THAT PRIVATE
RESPONDENT DOES NOT HAVE ANY TITLE AND HAS UTTERLY FAILED TO
PROVE ANY TITLE TO THE LOTS INVOLVED IN THIS CASE, AND IN
ORDERING THE CANCELLATION OF THE CERTIFICATE OF TITLE OF
PETITIONERS.
III. THE COURT OF APPEALS ACTED WITH GRAVE ABUSE OF DISCRETION
AMOUNTING TO LACK OF JURISDICTION AND IN GROSS VIOLATION OF
THE RULES OF COURT IN ORDERING THE ENTIRE PROPERTY COVERED
BY TRANSFER CERTIFICATE OF TITLE NO. (232252) 1321 TO BE PLACED IN
THE NAME OF PRIVATE RESPONDENT, BECAUSE THE CLAIM OF PRIVATE
RESPONDENT IS LIMITED ONLY TO ONE-HALF (1/2) PORTION OF THE
PROPERTY, AND THE OTHER HALF THEREOF UNQUESTIONABLY
BELONGS TO PETITIONERS.

100 | S U C C E S S I O N

IV. THE COURT OF APPEALS ERRED IN HOLDING THAT PETITIONERS


ACTED FRAUDULENTLY AND IN BAD FAITH IN SECURING THEIR
CERTIFICATE OF TITLE TO THE PROPERTY INVOLVED IN THIS CASE, AND
IN ORDERING PETITIONERS TO PAY PRIVATE RESPONDENTS MORAL
DAMAGES, EXEMPLARY DAMAGES AND ATTORNEYS FEES.11
The pivotal questions are: (1) whether prescription bars the action filed by REMEDIOS,
and (2) whether REMEDIOS is a real party-in-interest.
The Ruling of the Court
The petition has merit.
The Action is Barred by Prescription
The trial court held that the action filed by REMEDIOS is one based on fraud. REMEDIOS
action seeks to recover real property that petitioners allegedly acquired through fraud.
Consequently, the trial court held that the action prescribes in four years counted from
REMEDIOS actual discovery of petitioners adverse title. The trial court concluded that
REMEDIOS belatedly filed her suit on 4 February 1988 because she actually knew of
petitioners adverse title since 19 November 1982.
On the other hand, the Court of Appeals held that what REMEDIOS filed was a suit to
enforce an implied trust. REMEDIOS had ten years counted from actual notice of the
breach of trust, that is, the assertion of adverse title, within which to bring her action. The
appellate court held that REMEDIOS seasonably filed her complaint on 4 February 1988
because she allegedly discovered petitioners adverse title only on 19 November 1982.
What REMEDIOS filed was an action to enforce an implied trust but the same is already
barred by prescription.
Prescriptive Period is 10 Years Counted
From Registration of Adverse Title
The four-year prescriptive period relied upon by the trial court applies only if the fraud does
not give rise to an implied trust, and the action is to annul a voidable contract under Article
139012 of the Civil Code. In such a case, the four-year prescriptive period under Article
139113 begins to run from the time of discovery of the mistake, violence, intimidation,
undue influence or fraud.

In the present case, REMEDIOS does not seek to annul the KASULATAN. REMEDIOS
does not assail the KASULATAN as a voidable contract. In fact, REMEDIOS admits the
validity of the sale of 1,335 square meters of land under the KASULATAN. However,
REMEDIOS alleges that the excess area of 1,335 meters is not part of the sale under the
KASULATAN. REMEDIOS seeks the removal of this excess area from TCT No. (232252)
1321 that was issued to CONSOLACION. Consequently, REMEDIOS action is for
"Annulment or Cancellation of Transfer Certificate [of Title] and Damages." 14

[W]hile actions to enforce a constructive trust prescribes (sic) in ten years, reckoned from
the date of the registration of the property, we x x x are not prepared to count the period
from such date in this case. We note the petitioners sub rosa efforts to get hold of the
property exclusively for himself beginning with his fraudulent misrepresentation in his
unilateral affidavit of extrajudicial settlement that he is "the only heir and child of his mother
Feliza["] with the consequence that he was able to secure title in his name also . (Emphasis
supplied)

REMEDIOS action is based on an implied trust under Article 1456 since she claims that
the inclusion of the additional 1,335 square meters in TCT No. (232252) 1321 was without
basis. In effect, REMEDIOS asserts that CONSOLACION acquired the additional 1,335
square meters through mistake or fraud and thus CONSOLACION should be considered a
trustee of an implied trust for the benefit of the rightful owner of the property. Clearly, the
applicable prescriptive period is ten years under Article 1144 and not four years under
Articles 1389 and 1391.

Such commission of specific fraudulent conduct is absent in the present case. Other than
asserting that petitioners are guilty of fraud because they secured title to Lot Nos. 2-A and
2-E with an area twice bigger than what CANUTO allegedly sold to CONSOLACION,
REMEDIOS did not present any other proof of petitioners fraudulent conduct akin
to Adille.

It is now well-settled that the prescriptive period to recover property obtained by fraud or
mistake, giving rise to an implied trust under Article 1456 15 of the Civil Code, is ten years
pursuant to Article 1144.16 This ten-year prescriptive period begins to run from the date the
adverse party repudiates the implied trust, which repudiation takes place when the adverse
party registers the land.17
REMEDIOS filed her complaint on 4 February 1988 or more than 19 years after
CONSOLACION registered her title over Lot Nos. 2-A and 2-E on 28 October 1968.
Unquestionably, REMEDIOS filed the complaint late thus warranting its dismissal. As the
Court recently declared in Spouses Alfredo v. Spouses Borras,18
Following Caro,19 we have consistently held that an action for reconveyance based on an
implied trust prescribes in ten years. We went further by specifying the reference point of
the ten-year prescriptive period as the date of the registration of the deed or the issuance
of the title.
The Court of Appeals Reckoning of
Prescriptive Period from Actual Notice
of Adverse Title Not Justified
In holding that the action filed by REMEDIOS has not prescribed, the Court of Appeals
invoked this Courts ruling in Adille v. Court of Appeals. 20 In Adille, the Court reckoned the
ten-year prescriptive period for enforcing implied trusts not from registration of the adverse
title but from actual notice of the adverse title by the cestui que trust. However, the Court,
in justifying its deviation from the general rule, explained:

101 | S U C C E S S I O N

CONSOLACION obtained title to Lot Nos. 2-A and 2-E through the KASULATAN executed
by CANUTO and the JOINT AFFIDAVIT executed by his surviving children, one of whom,
Felicidad, is the mother of REMEDIOS. The KASULATAN referred to the sale of
CANUTOs 10/70 share in Lot 2 without specifying the area of the lot sold. The JOINT
AFFIDAVIT referred to the "Plano de Subdivision Psd-34713" without also specifying the
area of the lot sold. However, Subdivision Plan Psd 34713, as certified by the Assistant
Director of Lands on 30 May 1952, showed an area of 2,670 square meters in the name of
CANUTO. Based on these documents, the Register of Deeds issued TCT No. (232252)
1321 to CONSOLACION covering an area of 2,670 square meters.
REMEDIOS does not assail the KASULATAN or the JOINT AFFIDAVIT as fictitious or
forged. REMEDIOS even admits the authenticity of Subdivision Plan Psd 34713 as
certified by the Assistant Director of Lands. 21 Moreover, REMEDIOS has not contested
petitioners claim that CANUTO doubled his share in Lot 2 by acquiring VICTORIANOs
share.22
Plainly, the increase in the area sold from 1,335 square meters to 2,670 square meters is a
glaring mistake. There is, however, no proof whatsoever that this increase in area was the
result of fraud. Allegations of fraud in actions to enforce implied trusts must be proved by
clear and convincing evidence.23 Adille, which is anchored on fraud, 24 cannot apply to the
present case.
At any rate, even if we apply Adille to this case, prescription still bars REMEDIOS
complaint. As executrix of CATALINAs LAST WILL, REMEDIOS submitted to the then
Court of First Instance of Caloocan in Special Proceedings Case No. C-208 the inventory
of all the property comprising CATALINAs estate, which included Lot Nos. 2-A and 2-E. In
a motion dated 7 November 1977, CONSOLACION sought the exclusion of these lots from
the inventory, invoking her title over them. REMEDIOS was served a copy of the motion on
8 November 1977 against which she filed an opposition. Nevertheless, the trial court

overruled REMEDIOS objection. In its order of 3 January 1978, the trial court granted
CONSOLACIONs motion and ordered the exclusion of Lot Nos. 2-A and 2-E from the
estate of CATALINA. REMEDIOS did not appeal from this ruling.
REMEDIOS thus had actual notice of petitioners adverse title on 8 November 1977. Even
if, for the sake of argument, the ten-year prescriptive period begins to run upon actual
notice of the adverse title, still REMEDIOS right to file this suit has prescribed. REMEDIOS
had until 11 November 1987 within which to file her complaint. When she did so on 4
February 1988, the prescriptive period had already lapsed.
Respondent is Not a Real Party-in-Interest
Not only does prescription bar REMEDIOS complaint. REMEDIOS is also not a real partyin-interest who can file the complaint, as the trial court correctly ruled.
The 1997 Rules of Civil Procedure require that every action must be prosecuted or
defended in the name of the real party-in-interest who is the party who stands to benefit or
suffer from the judgment in the suit. 25 If one who is not a real party-in-interest brings the
action, the suit is dismissible for lack of cause of action. 26
REMEDIOS anchored her claim over Lot Nos. 2-A and 2-E (or over its one-half portion) on
the devise of these lots to her under CATALINAs LAST WILL. However, the trial court
found that the probate court did not issue any order admitting the LAST WILL to probate.
REMEDIOS does not contest this finding. Indeed, during the trial, REMEDIOS admitted
that Special Proceedings Case No. C-208 is still pending.27
Article 838 of the Civil Code states that "[N]o will shall pass either real or personal property
unless it is proved and allowed in accordance with the Rules of Court." This Court has
interpreted this provision to mean, "until admitted to probate, [a will] has no effect whatever
and no right can be claimed thereunder." 28 REMEDIOS anchors her right in filing this suit
on her being a devisee of CATALINAs LAST WILL. However, since the probate court has
not admitted CATALINAs LAST WILL, REMEDIOS has not acquired any right under the
LAST WILL. REMEDIOS is thus without any cause of action either to seek reconveyance
of Lot Nos. 2-A and 2-E or to enforce an implied trust over these lots.
The appellate court tried to go around this deficiency by ordering the reconveyance of Lot
Nos. 2-A and 2-E to REMEDIOS in her capacity as executrix of CATALINAs LAST WILL.
This is inappropriate because REMEDIOS sued petitioners not in such capacity but as the
alleged owner of the disputed lots. Thus, REMEDIOS alleged in her complaint:
3. The plaintiff is a niece and compulsory heir of the late CATALINA SIOSON who died
single and without any child of her own and who, during her lifetime, was the owner of
those two (2) parcels of land located at Tanza, Navotas, Rizal (now Metro Manila), formerly

102 | S U C C E S S I O N

covered by Original Certificate of Title No. 4207 of the Registry of Deeds for the Province
of Rizal, x x x.
4. The plaintiff, aside from being the compulsory heir of the deceased CATALINA SIOSON,
has sole and exclusive claim of ownership over the above-mentioned two (2) parcels of
land by virtue of a will or "Huling Habilin at Pagpapasiya" executed by Catalina Sioson on
May 19, 1964 before Notary Public Efren Y. Angeles at Navotas, Rizal, in which document
the deceased Catalina Sioson specifically and exclusively bequeathed to the plaintiff the
above-mentioned Lots 2-A and 2-E of Psd-34713 approved by the Bureau of Lands on
May 30, 1952. Copy of the "Huling Habilin at Pagpapasiya" consisting of four (4) pages is
hereto attached and forms an integral part hereof as Annex "A;"
5. Sometime on or about February, 1987, plaintiff discovered that the above-mentioned
Lots 2-A and 2-E of subdivision plan Psd-34713 are now registered or titled in the name of
the defendants under Transfer Certificate of Title No. (232252) 1321 of the Registry of
Deeds of Rizal, now Metro-Manila District III. Copy of the title is hereto attached and forms
an integral part hereof as Annex "B;"
6. Upon further inquiry and investigation, plaintiff discovered that the defendants were able
to obtain title in their name of the said parcels of land by virtue of a "Kasulatan ng Bilihang
Tuluyan" allegedly executed by Canuto Sioson on September 26, 1956 before Notary
Public Jose [T.] de los Santos of Navotas, Metro-Manila. Copy of the said document is
hereto attached and forms an integral part hereof as Annex "C;"
7. The plaintiff also discovered that although x x x the original sale did not specify the
parcels of land sold by Canuto Sioson, the defendants submitted an alleged Affidavit
executed by Felicidad Sioson and Beatriz Sioson identifying the lots sold by Canuto Sioson
to the defendants as Lots 2-A and 2-E of subdivision plan Psd-34713. Copy of the Affidavit
dated October 3, 1968 on the basis of which the present Transfer Certificate of Title No.
(232252) 1321 was issued to the defendants is hereto attached and forms an integral part
hereof as Annex "D;"
8. The defendants are clearly guilty of fraud in presenting the aforementioned Affidavit
(Annex "D") to the Register of Deeds as the basis of their claim to Lots 2-A and 2-E in view
of the fact that the parcels sold to them by Canuto Sioson, assuming there was such a
sale, were different parcels of land, Lots 2-A and 2-E being the properties of the late
Catalina Sioson who bequeathed the same to the plaintiff.
xxxx
12. Because of the defendants fraudulent actuations on this matter, plaintiff suffered and
continious [sic] to suffer moral damages arising from anxiety, shock and wounded feelings.
Defendants should also be assessed exemplary damages by way of a lesson to deter

them from again committing the fraudulent acts, or acts of similar nature, by virtue of which
they were able to obtain title to the parcels of land involved in this case x x x. 29(Emphasis
supplied)
Indeed, all throughout the proceedings below and even in her Comment to this petition,
REMEDIOS continued to pursue her claim as the alleged owner of one-half of the disputed
lots.

IKALIMA. - Aking inihahayag at ginagawa na tagapagmana, sa aking kusang loob,


ang pinalaki kong APO na si ANSELMO P. MANGULABNAN, may sapat na gulang,
kasal kay Flora Umagap, at naninirahan sa San Lorenzo, Gapan, Nueva Ecija, at anak ng
aking anak na si SIMPLICIA, at sa aking APO na si ANSELMO ay aking ipinagkakaloob
at ipinamamana, sa aking pagkamatay, ang mga sumusunod kong pagaari:
LOT NO. TITLE NO. KINALALAGYAN NABANGGIT SA

Other Matters Raised in the Petition

288-A NT-47089 Sta. Cruz (1) p. 2

The Court deems it unnecessary to pass upon the other errors petitioners assigned
concerning the award of damages and attorneys fees to REMEDIOS. Such award
assumes that REMEDIOS is a real party-in-interest and that she timely filed her complaint.
As earlier shown, this is not the case.

3348-A 100629 Poblacion (2) p. 2

WHEREFORE, we GRANT the petition. The Decision of the Court of Appeals dated 31
January 1994 and its Resolution dated 15 June 1994 are SET ASIDE. The complaint filed
by respondent Remedios Eugenio-Gino, dated 2 February 1988 is DISMISSED.
SO ORDERED.
Davide, Jr., C.J., Vitug, Ynares-Santiago and Azcuna, JJ., concur.

3349-B 100630 Poblacion (3) p. 2


xxx[1] (Underscoring in the original; emphasis supplied)
The testatrixs son Bernardo Patulandong (Patulandong), respondent herein, was in
the will appointed as the executor.
During her lifetime, the testatrix herself filed a petition for the probate of her will
before the then Court of First Instance (CFI) of Nueva Ecija where it was docketed as Sp.
Pro. No. 128.
By Order[2] of January 11, 1973, the CFI admitted the will to probate.

[G.R. No. 144915. February 23, 2004]


CAROLINA CAMAYA, FERDINAND CAMAYA, EDGARDO CAMAYA and
CARPIO-MORALES, J.:
Before this Court is a petition for review on certiorari under Rule 45 of the 1997
Revised Rules of Court seeking the reversal of the Court of Appeals Decision dated June
19, 2000 in CA-G.R. CV No. 53757, In re: Petition for the Probate of the Codicil (Will) of
Rufina Reyes; Bernardo Patulandong v. Anselmo Mangulabnan v. Carolina G. Camaya,
Ferdinand Camaya and Edgardo Camaya.
On November 17, 1972, Rufina Reyes (testatrix) executed a notarized will wherein
she devised, among others, Lot No. 288-A to her grandson Anselmo Mangulabnan
(Mangulabnan). The pertinent portion of her will reads:

On June 27, 1973, the testatrix executed a codicil modifying above-quoted paragraph
five of her will in this wise:
UNA. - Ang Lote No. 288-A na nakalagay sa barrio ng Sta. Cruz, Gapan, Nueva Ecija,
magsukat 36,384 metro cuadrados, at nagtataglay ng TCT No. NT-47089, na aking
ipinamana sa aking apong si ANSELMO P. MANGULABNAN, sangayon sa Pangkat
IKA-LIMA, pp. 5-6, ng aking HULING HABILIN (Testamento), ay ipinasiya kong
ipagkaloob at ipamana sa aking mga anak na sina BERNARDO, SIMPLICIA,
GUILLERMA at JUAN nagaapellidong PATULANDONG, at sa aking apong si
ANSELMO P. MANGULABNAN, sa magkakaparehong bahagi na tig-ikalimang bahagi
bawat isa sa kanila.
IKALAWA. - Na maliban sa pagbabagong ito, ang lahat ng mga tadhana ng aking HULING
HABILIN ay aking pinagtitibay na muli.
x x x[3] (Underscoring in the original; emphasis supplied) On May 14, 1988, the testatrix
died.

103 | S U C C E S S I O N

Mangulabnan later sought the delivery to him by executor Patulandong of the title to
Lot 288-A. Patulandong refused to heed the request, however, in view of the codicil which
modified the testators will.
Mangulabnan thus filed an action for partition against Patulandong with the Regional
Trial Court of Gapan, Nueva Ecija, docketed as Civil Case No. 552 (the partition case).
On June 8, 1989, the trial court rendered a decision in the partition case, [4] the
dispositive portion of which reads:
WHEREFORE, the court orders the partitioning of the properties and the defendant to
deliver the copy of the Transfer Certificate of Title No. NT-47089.
However, in view of the case cited by the plaintiff himself, the court holds that the
partition is without prejudice [to]... the probate of the codicil in accordance with the
Rules of Court, [P]alacios vs. Catimbang Palacios cited by the plaintiff:
After a will has been probated during the lifetime of the testator, it does not necessarily
mean that he cannot alter or revoke the same before his death. Should he make a new
will, it would also be allowable of his petition and if he should die before he had a chance
to present such petition, the ordinary probate proceedings after the testators death would
be in order.
The Court also orders that the right of the tenants of the agricultural land in question
should be protected meaning to say that the tenants should not be ejected. (Emphasis and
underscoring supplied)

On January 16, 1996, the trial rendered a decision [10] in Sp. Proc. No. 218 admitting
the codicil to probate and disposing as follows:
WHEREFORE, in view of all the foregoing, judgment is hereby rendered in the following
manner:
1. Declaring Transfer Certificate of Title No. NT-215750 issued by the Register
of Deeds of Nueva Ecija in the name of Anselmo Mangulabnan dated
February 7, 1991 and the Deed of Absolute Sale executed by him in
favor of the intervenors Carolina, Ferdinand and Edgardo, all surnamed
Camaya on February 19, 1991 and Transfer Certificate of Title No. NT216446 under date March 18, 1991 issued in the names of the abovenamed intervenors as NULL and VOID and of no force and effect; and,
2. Ordering the Register of Deeds of Nueva Ecija to cancel Transfer of
Certificate of Title Nos. NT-215750 and NT-216446 and reissue the
corresponding Certificate of Titles to Bernardo R. Patulandong, Filipino,
married to Gorgonia Mariano residing at San Vicente, Gapan, Nueva
Ecija, Juan R. Patulandong, Filipino, widower and residing at San
Lorenzo, Gapan, Nueva Ecija; Guillerma R. Patulandong Linsangan of
legal age, Filipino, widow and residing at San Vicente, Gapan, Nueva
Ecija, Simplicia R. Patulandong Mangulabnan, of legal age, widow, and
residing at San Lorenzo, Gapan, Nueva Ecija and her grandson,
Anselmo Mangulabnan with full personal circumstances stated herein
to the extent of one fifth (1/5) each pursuant to the approved codicil
(will) of Rufina Reyes dated June 27, 1973.[11]

On July 17, 1989 Patulandong filed before the Regional Trial Court of Nueva Ecija a
petition[5] for probate of the codicil of the testatrix, docketed as Sp. Proc. No. 218.

The Camayas who had been allowed to intervene in Sp. Proc. No. 218, and
Mangulabnan, filed a Motion for Reconsideration of the above-said decision but it was
denied by Order[12] of February 28,1996.

On December 28, 1989, the probate court issued an Order [6] setting the petition for
hearing and ordering the publication of said order.

On appeal to the Court of Appeals, the Camayas and Mangulabnan (hereinafter


referred to as petitioners) raised the following errors:

On February 7, 1991, by virtue of the decision in the partition case, Mangulabnan


caused the cancellation of the title of the testatrix over Lot No. 288-A and TCT No. NT215750[7] was issued in his name.

1. THERE WERE SERIOUS SUBSTANTIAL DEPARTURES FROM THE


FORMALITIES REQUIRED BY THE RULES, THE LAW, AND THE
AUTHORITY OF THE REGIONAL TRIAL COURT SETTING AS A
PROBATE COURT.

Mangulabnan later sold to herein petitioners Camayas Lot No. 288-A by a Deed of
Sale dated February 19, 1991. [8] TCT No. NT-215750 was thus cancelled and TCT No. NT216446[9]was issued in the name of the Camayas.

104 | S U C C E S S I O N

2. THE OPPOSITOR DID NOT ONLY ACQUIRE LOT NO. 288-A BY WILL BUT
HE ALSO ACQUIRED THE SAME BY PARTITION IN A CIVIL CASE
WHERE THE DECISION HAS ALREADY REACHED ITS FINALITY AND

THEREFORE CAN NO LONGER BE NEGATED BY A QUESTIONABLE


CODICIL.
3. THAT THE SUBJECT LOT 288-A IS NO LONGER WITHIN THE REACHED
(sic) OF THE PETITIONER CONSIDERING THAT THE OPPOSITOR
VENDOR HAD A CLEAN TITLE AND THAT THE INTERVENORSVENDEED HAD ACQUIRED THE SAME BY WAY OF SALE AS
INNOCENT PURCHASER IN GOOD FAITH AND FOR VALUE.[13]
By Decision[14] of June 19, 2000, the Court of Appeals affirmed that of the trial court.
Hence, the present petition for Review on Certiorari proffering the following issues:
1. Whether the probate court exceeded its jurisdiction when it declared null and
void and ordered the cancellation of the TCTs of petitioners and the deed of
sale; and
2. Whether the final judgment in Civil Case No. 552 bars the allowance of the
codicil.
As to the first issue, petitioners contend that the under the law, the probate court has
no power, authority, and jurisdiction to declare null and void the sale and titles of
petitioners;[15] and that the probate court can only resolve the following issues:
1. Whether or not the instrument which is offered for probate is the last will and
testament of the decedent; in other words, the question is one of identity[;]
2. Whether or not the will has been executed in accordance with the formalities
prescribed by law; in other words, the question is one of due execution[;
and]
3. Whether the testator had testamentary capacity at the time of the execution
of the will; in other words, the question is one of capacity.[16]

administrator. If there is no dispute, well and good; but if there is, then the parties, the
administrator, and the opposing parties have to resort to an ordinary action for a
final determination of the conflicting claims of title because the probate court
cannot do so.
xxx
Having been apprised of the fact that the property in question was in the possession of
third parties and more important, covered by a transfer certificate of title issued in the
name of such third parties, the respondent court should have denied the motion of the
respondent administrator and excluded the property in question from the inventory of the
property of the estate. It had no authority to deprive such third persons of their
possession and ownership of the property. x x x (Emphasis and underscoring supplied)
Following Cuizon, the probate court exceeded its jurisdiction when it further declared
the deed of sale and the titles of petitioners null and void, it having had the effect of
depriving them possession and ownership of the property.
Moreover, following Section 48 of the Property Registry Decree which reads:
SECTION 48. Certificate not subject to collateral attack. - A certificate of title shall not be
subject to collateral attack. It cannot be altered, modified, or cancelled except in a direct
proceeding in accordance with law,
petitioners titles cannot, under probate proceedings, be declared null and void.
As to the second issue, petitioners argue that by allowing the codicil to probate, it in
effect amended the final judgment in the partition case which is not allowed by law; [18] and
that petitioner Camayas are innocent purchasers for value and enjoy the legal presumption
that the transfer was lawful.[19]
Petitioners first argument does not persuade.

this Court elucidated on the limited jurisdiction of a probate

Though the judgment in the partition case had become final and executory as it was
not appealed, it specifically provided in its dispositive portion that the decision was without
prejudice [to] ... the probate of the codicil. The rights of the prevailing parties in said
case were thus subject to the outcome of the probate of the codicil.

It is well-settled rule that a probate court or one in charge of proceedings whether


testate or intestate cannot adjudicate or determine title to properties claimed to be a
part of the estate and which are equally claimed to belong to outside parties. All that
said court could do as regards said properties is to determine whether they should
or should not be included in the inventory or list of properties to be administered by the

The probate court being bereft of authority to rule upon the validity of petitioners
titles, there is no longer any necessity to dwell on the merits of petitioners Camayas claim
that they are innocent purchasers for value and enjoy the legal presumption that the
transfer was lawful.

In Cuizon v. Ramolete,
court, to wit:

[17]

105 | S U C C E S S I O N

WHEREFORE, the petition is GRANTED IN PART.


The Decision of the Court of Appeals dated June 19, 2000 in CA-G.R. CV No. 53757
affirming the January 16, 1996 Decision of Regional Trial Court, Branch 35, of Gapan,
Nueva Ecija, is hereby AFFIRMED with MODIFICATION.
The decision allowing the codicil is AFFIRMED, but the 1) declaration as null and
void of Transfer Certificate of Title No. NT-215750 issued on February 7, 1991 by the
Register of Deeds of Nueva Ecija in the name of Anselmo Mangulabnan, the February 19,
1991 Deed of Absolute Sale executed by him in favor of the intervenors - herein petitioners
Carolina, Ferdinand and Edgardo Camaya, and Transfer Certificate of Title No. NT-216446
issued on March 18, 1991 in favor of the petitioners Camayas, and 2) the order for the
Register of Deeds of Nueva Ecija to cancel Transfer of Certificate of Title Nos. NT-215750
and NT-216446 and reissue the corresponding Certificate of Titles to Bernardo R.
Patulandong, Juan R. Patulandong, Guillerma R. Patulandong Linsangan, Simplicia R.
Patulandong Mangulabnan, and Anselmo Mangulabnan to the extent of one-fifth (1/5) each
pursuant to the approved codicil are SET ASIDE, without prejudice to respondent and his
co-heirs ventilation of their right in an appropriate action.
SO ORDERED.
Vitug, (Chairman), Sandoval-Gutierrez, and Corona, JJ., concur.

G.R. No. L-46078

May 25, 1939

In the matter of the will of the deceased Mauro Salvacion.


GREGORIA REYNOSO, administratrix-appellant,
vs.
JOSE E. TOLENTINO, guardian ad litem of the minors Maurito and Remedios
Aguila,
JOAQUIN CAMPOSANO, guardian ad litem of the minor Corazon Camposano,
VALERIO SALVACION, NUMERIANO SALVACION, AMADEO SALVACION, and
MARTINA ALLA, legatees-appellants.
AVANCEA, C.J.:
Mauro Salvacion died on June 30, 1932 in the municipality of Lucena, Province of
Tayabas, without leaving any descendant or ascendant. His widow, Gregoria Reynoso,
who survived him, is now the administratrix appointed in this testate proceeding.

The properties left by the deceased are conjugal in nature because they were acquired
during his marriage with his widow. He left a will and a codicil upon his death, wherein he
made a partition of the conjugal properties between him and his widow, and disposed by
way of legacy of the half corresponding to him.
The attorney of the administration of this testate thereafter prepared the partition of the
properties left by the deceased between the widow and the legatees.
The widow opposed the approval of this partition as to the coconut trees, alleging that it is
unequal not only as to the number of trees but also as to the quality thereof. Over this
opposition of the widow, the court, without affording her an opportunity to substantiate her
opposition and present evidence in support thereof, approved the partition. To this
resolution the widow expected.
The legatees, on the other hand, also opposed the approval of the partition in so far as it
casts the burden of the widow's usufruct upon one-half of what corresponds to each one.
Moreover, these legatees contend that the allowance received by the widow during the
liquidation of the conjugal properties should be charged against her in so far as it exceeds
the products of the properties allotted to her. The court also overruled this opposition and
approved the partition in this respect.
In so far as it refers to the appeal of the widow, we are of the opinion that the resolution of
the court, approving the partition, is erroneous. The court should have substantiated the
opposition of the widow and should have given her an opportunity to adduce evidence in
its support. However, the court, relying only upon the fact that the partition was made in
accordance with the will of the deceased, approved it. The will, in so far as the the testator
alone made therein a partition of the conjugal properties by assigning to himself those
which he liked and to the wife those which she did not like, is illegal. The conjugal property
is one between husband and wife wherein each one, except as to the administration
thereof, has equal rights. Each one has a right to one-half of these properties and each
one occupies the same position as to its ownership. It is an encroachment upon these
rights of each of the spouses if one of them could designate which and how much these
properties should correspond to him. Any of this spouses is entitled to be heard in the
partition of the conjugal properties in order to defend his or her equal share.
As to the appeal of the legatees, the theory upon which it is based in plainly erroneous.
The usufruct which article 837 of the Civil Code gives to the widow is upon one-half of the
properties of the deceased spouse and not upon the properties of the widow herself, such
as the half of the conjugal properties corresponding to her.
The contention that the allowance received by the widow should be charged against her
share in the conjugal properties in so far as it exceeds the fruits of the properties
corresponding to her, is perfectly legal. But we are precluded from ruling upon this point,

106 | S U C C E S S I O N

because there is neither showing nor allegation as to the amount of the fruits of the
properties during the liquidation. Without this, we are not in a position to decide whether or
not the widow received by way of allowance more than that corresponding to her from the
fruits of the properties.
In view of the foregoing, the appealed judgment is modified in the sense that the court
should permit the widow to substantiate her opposition and to present evidence in support
thereof, and is affirmed in all other respects, with the costs to the defendants as appellants
and legatees. So ordered.
Villa-Real, Imperial, Diaz, Laurel, Concepcion, and Moran, JJ., concur.

G.R. No. L-12207

December 24, 1959

JUAN PALACIOS, petitioner-appellant,


vs.
. MARIA CATIMBANG PALACIOS, oppositor-appellee
BAUTISTA ANGELO, J.:
Juan Palacios executed his last will and testament on June 25, 1946 and availing himself
of the provisions of the new Civil Code, he filed on May 23, 1956 before the Court of First
Instance of Batangas a petition for its approval. In said will, he instituted as his sole heirs
his natural children Antonio C. Palacios and Andrea C. Palacios.
On June 21, 1956, Maria Catimbang filed a opposition to the probate of the will alleging
that she is the acknowledged natural daughter of petitioner but that she was completely
ignored in said will thus impairing here legitime.
After the presentation of petitioner's evidence relative to the essential requisites and
formalities provided by law for the validity of a will, the court on July 6, 1956 issued an
order admitting the will to probate. The court, however, set a date for the hearing of the
opposition relative to the intrinsic validity of the will and, after proper hearing concerning
this incident, the court issued another order declaring oppositor to be the natural child of
petitioner and annulling the will insofar as it impairs her legitime, with costs against
petitioner.
From this last order, petitioner gave notice of his intention to appeal directly to the
Supreme Court, and accordingly, the record was elavated to this Court.

107 | S U C C E S S I O N

It should be noted that petition instituted the present proceeding in order to secure the
probate of his will availing himself of the provisions of Article 838, paragraph 2, of the new
Civil Code, which permit a testator to petition the proper court during his lifetime for the
allowance of his will, but to such petition on Maria Catimbang filed an opposition alleging
that she is the acknowledged natural daughter of petitioner but that she was completely
ignored in the will thus impairing her object to the probate of the will insofar as it due
execution is concerned or on the ground that it has not complied with the formalities
prescribed by law; rather she objects to its intrinsic validity or to the legality of the
provisions of the will.
We hold that such opposition cannot be entertained in this proceeding because its only
purpose is merely to determine if the will has been executed in accordance with the
requirements of the law, much less if the purpose of the opposition is to show that the
oppositor is an acknowledged natural child who allegedly has been ignored in the will for
issue cannot be raised here but in a separate action. This is especially so when the
testator, as in the present case, is still alive and has merely filed a petition for the
allowance of his will leaving the effects thereof after his death.lawphi1.net
This is in line with our ruling in Montaano vs. Suesa, 14 Phil., 676, wherein we said: "The
authentication of the will decides no other questions than such as touch upon the capacity
of the testator and the compliance with those requisites or solemnities which the law
prescribes for the validity of a will. It does not determine nor even by implication prejudge
the validity or efficiency of the provisions; that may be impugned as being vicious or null,
notwithstanding its authentication. The questions relating to these points remain entirely
un-affected, and may be raised even after the will has been authenticated."
On the other hand, "after a will has been probated during the lifetime of a testator, it does
not necessarily mean that he cannot alter or revoke the same before he has had a chance
to present such petition, the ordinary probate proceedings after the testator's death would
be in order" (Report of the Code Commission, pp. 53-54).The reason for this comment is
that the rights to the succession are transmitted from the moment of the death of the
decedent (Article 777, new Civil Code.).
It is clear that the trial court erred in entertaining the opposition and in annulling the portion
of the will which allegedly impairs the legitime of the oppositor on the ground that, as it has
found, she is an extraneous matter which should be treshed out in a separate action.
Wherefore, the order appealed from is set aside, without pronouncement as to costs.
Paras, C.J., Bengzon, Padilla, Labrador, Concepcion, Endencia, Barrera and Gutierrez
David., JJ., concur.

G.R. No. L-23445

June 23, 1966

REMEDIOS NUGUID, petitioner and appellant,


vs.
FELIX NUGUID and PAZ SALONGA NUGUID, oppositors and appellees.

the intrinsic validity or efficacy of the provisions of the will, the legality of any devise or
legacy therein.1

SANCHEZ, J.:
Rosario Nuguid, a resident of Quezon City, died on December 30, 1962, single, without
descendants, legitimate or illegitimate. Surviving her were her legitimate parents, Felix
Nuguid and Paz Salonga Nuguid, and six (6) brothers and sisters, namely: Alfredo,
Federico, Remedios, Conrado, Lourdes and Alberto, all surnamed Nuguid.
On May 18, 1963, petitioner Remedios Nuguid filed in the Court of First Instance of Rizal a
holographic will allegedly executed by Rosario Nuguid on November 17, 1951, some 11
years before her demise. Petitioner prayed that said will be admitted to probate and that
letters of administration with the will annexed be issued to her.
On June 25, 1963, Felix Nuguid and Paz Salonga Nuguid, concededly the legitimate father
and mother of the deceased Rosario Nuguid, entered their opposition to the probate of her
will. Ground therefor, inter alia, is that by the institution of petitioner Remedios Nuguid as
universal heir of the deceased, oppositors who are compulsory heirs of the deceased in
the direct ascending line were illegally preterited and that in consequence the institution
is void.
On August 29, 1963, before a hearing was had on the petition for probate and objection
thereto, oppositors moved to dismiss on the ground of absolute preterition.
On September 6, 1963, petitioner registered her opposition to the motion to
dismiss.1wph1.t
The court's order of November 8, 1963, held that "the will in question is a complete nullity
and will perforce create intestacy of the estate of the deceased Rosario Nuguid" and
dismissed the petition without costs.
A motion to reconsider having been thwarted below, petitioner came to this Court on
appeal.
1. Right at the outset, a procedural aspect has engaged our attention. The case is for the
probate of a will. The court's area of inquiry is limited to an examination of, and
resolution on, the extrinsic validity of the will. The due execution thereof, the testatrix's
testamentary capacity, and the compliance with the requisites or solemnities by law
prescribed, are the questions solely to be presented, and to be acted upon, by the court.
Said court at this stage of the proceedings is not called upon to rule on

108 | S U C C E S S I O N

A peculiar situation is here thrust upon us. The parties shunted aside the question of
whether or not the will should be allowed probate. For them, the meat of the case is the
intrinsic validity of the will. Normally, this comes only after the court has declared that the
will has been duly authenticated. 2 But petitioner and oppositors, in the court below and
here on appeal, travelled on the issue of law, to wit: Is the will intrinsically a nullity?
We pause to reflect. If the case were to be remanded for probate of the will, nothing will be
gained. On the contrary, this litigation will be protracted. And for aught that appears in the
record, in the event of probate or if the court rejects the will, probability exists that the case
will come up once again before us on the same issue of the intrinsic validity or nullity of the
will. Result: waste of time, effort, expense, plus added anxiety. These are the practical
considerations that induce us to a belief that we might as well meet head-on the issue of
the validity of the provisions of the will in question. 3 After all, there exists a justiciable
controversy crying for solution.
2. Petitioner's sole assignment of error challenges the correctness of the conclusion below
that the will is a complete nullity. This exacts from us a study of the disputed will and the
applicable statute.
Reproduced hereunder is the will:
Nov. 17, 1951
I, ROSARIO NUGUID, being of sound and disposing mind and memory, having amassed a
certain amount of property, do hereby give, devise, and bequeath all of the property which I
may have when I die to my beloved sister Remedios Nuguid, age 34, residing with me at
38-B Iriga, Q.C. In witness whereof, I have signed my name this seventh day of November,
nineteen hundred and fifty-one.
(Sgd.) Illegible
T/ ROSARIO NUGUID
The statute we are called upon to apply in Article 854 of the Civil Code which, in part,
provides:
ART. 854. The preterition or omission of one, some, or all of the compulsory heirs
in the direct line, whether living at the time of the execution of the will or born

after the death of the testator, shall annul the institution of heir; but the devises
and legacies shall be valid insofar as they are not inofficious. ...
Except for inconsequential variation in terms, the foregoing is a reproduction of Article 814
of the Civil Code of Spain of 1889, which is similarly herein copied, thus
Art. 814. The preterition of one or all of the forced heirs in the direct line, whether
living at the time of the execution of the will or born after the death of the testator,
shall void the institution of heir; but the legacies and betterments 4 shall be valid,
in so far as they are not inofficious. ...
A comprehensive understanding of the term preterition employed in the law becomes a
necessity. On this point Manresa comments:
La pretericion consiste en omitar al heredero en el testamento. O no se le
nombra siquiera o aun nombrandole como padre, hijo, etc., no se le instituya
heredero ni se le deshereda expresamente ni se le asigna parte alguna de los
bienes, resultando privado de un modo tacito de su derecho a legitima.
Para que exista pretericion, con arreglo al articulo 814, basta que en el
testamento omita el testador a uno cualquiera de aquellos a quienes por su
muerte corresponda la herencia forzosa.
Se necesita, pues, a) Que la omision se refiera a un heredero forzoso. b) Que la
omision sea completa; que el heredero forzoso nada reciba en el testamento.
It may now appear trite bat nonetheless helpful in giving us a clear perspective of the
problem before us, to have on hand a clear-cut definition of the word annul:
To "annul" means to abrogate, to make void ... In re Morrow's Estate, 54 A. 342,
343, 204 Pa. 484.6
The word "annul" as used in statute requiring court to annul alimony provisions of
divorce decree upon wife's remarriage means to reduce to nothing; to annihilate;
obliterate; blot out; to make void or of no effect; to nullify; to abolish. N.J.S.A.
2:50 38 (now N.J.S. 2A:34-35). Madden vs. Madden, 40 A. 2d 611, 614, 136
N..J Eq. 132.7
ANNUL. To reduce to nothing; annihilate; obliterate; to make void or of no effect;
to nullify; to abolish; to do away with. Ex parte Mitchell, 123 W. Va. 283, 14 S.E.
2d. 771, 774.8

109 | S U C C E S S I O N

And now, back to the facts and the law. The deceased Rosario Nuguid left no
descendants, legitimate or illegitimate. But she left forced heirs in the direct ascending line
her parents, now oppositors Felix Nuguid and Paz Salonga Nuguid. And, the will
completely omits both of them: They thus received nothing by the testament; tacitly, they
were deprived of their legitime; neither were they expressly disinherited. This is a clear
case of preterition. Such preterition in the words of Manresa "anulara siempre la institucion
de heredero, dando caracter absoluto a este ordenamiento referring to the mandate of
Article 814, now 854 of the Civil Code. 9 The one-sentence will here institutes petitioner as
the sole, universal heir nothing more. No specific legacies or bequests are therein
provided for. It is in this posture that we say that the nullity is complete. Perforce, Rosario
Nuguid died intestate. Says Manresa:
En cuanto a la institucion de heredero, se anula. Lo que se anula deja de existir,
en todo o en parte? No se aade limitacion alguna, como en el articulo 851, en el
que se expresa que se anulara la institucion de heredero en cuanto prejudique a
la legitima del deseheredado Debe, pues, entenderse que la anulacion es
completa o total, y que este articulo como especial en el caso que le motiva rige
con preferencia al 817. 10
The same view is expressed by Sanchez Roman:
La consecuencia de la anulacion o nulidad de la institucion de heredero por
pretericion de uno, varios o todos los forzosos en linea recta, es la apertura de la
sucesion intestada total o parcial. Sera total, cuando el testador que comete la
pretericion, hubiese dispuesto de todos los bienes por titulo universal de
herencia en favor de los herederos instituidos, cuya institucion se anula, porque
asi lo exige la generalidad del precepto legal del art. 814, al determinar, como
efecto de la pretericion, el de que "anulara la institucion de heredero." ... 11
Really, as we analyze the word annul employed in the statute, there is no escaping the
conclusion that the universal institution of petitioner to the entire inheritance results
in totally abrogating the will. Because, the nullification of such institution of universal heir
without any other testamentary disposition in the will amounts to a declaration that
nothing at all was written. Carefully worded and in clear terms, Article 854 offers no leeway
for inferential interpretation. Giving it an expansive meaning will tear up by the roots the
fabric of the statute. On this point, Sanchez Roman cites the "Memoria annual del Tribunal
Supreme, correspondiente a 1908", which in our opinion expresses the rule of
interpretation, viz:
... El art. 814, que preceptua en tales casos de pretericion la nulidad de la
institucion de heredero, no consiente interpretacion alguna favorable a la
persona instituida en el sentido antes expuesto aun cuando parezca, y en algun
caso pudiera ser, mas o menos equitativa, porque una nulidad no significa en

Derecho sino la suposicion de que el hecho o el acto no se ha realizado,


debiendo por lo tanto procederse sobre tal base o supuesto, y
consiguientemente, en un testamento donde falte la institucion, es obligado
llamar a los herederos forzosos en todo caso, como habria que llamar a los de
otra clase, cuando el testador no hubiese distribudo todos sus bienes en
legados, siendo tanto mas obligada esta consecuencia legal cuanto que, en
materia de testamentos, sabido es, segun tiene declarado la jurisprudencia, con
repeticion, que no basta que sea conocida la voluntad de quien testa si esta
voluntad no aparece en la forma y en las condiciones que la ley ha exigido para
que sea valido y eficaz, por lo que constituiria una interpretacion arbitraria,
dentro del derecho positivo, reputar como legatario a un heredero cuya
institucion fuese anulada con pretexto de que esto se acomodaba mejor a la
voluntad del testador, pues aun cuando asi fuese, sera esto razon para modificar
la ley, pero no autoriza a una interpretacion contraria a sus terminos y a los
principios que informan la testamentifaccion, pues no porque parezca mejor una
cosa en el terreno del Derecho constituyente, hay razon para convereste juicio
en regla de interpretacion, desvirtuando y anulando por este procedimiento lo
que el legislador quiere establecer. 12
3. We should not be led astray by the statement in Article 854 that, annullment
notwithstanding, "the devises and legacies shall be valid insofar as they are not
inofficious". Legacies and devises merit consideration only when they are so expressly
given as such in a will. Nothing in Article 854 suggests that the mere institution of a
universal heir in a will void because of preterition would give the heir so instituted a
share in the inheritance. As to him, the will is inexistent. There must be, in addition to such
institution, a testamentary disposition granting him bequests or legacies apart and
separate from the nullified institution of heir. Sanchez Roman, speaking of the two
component parts of Article 814, now 854, states that preterition annuls the institution of the
heir "totalmente por la pretericion"; but added (in reference to legacies and bequests)
"pero subsistiendo ... todas aquellas otras disposiciones que no se refieren a la institucion
de heredero ... . 13 As Manresa puts it, annulment throws open to intestate succession the
entire inheritance including "la porcion libre (que) no hubiese dispuesto en virtud de
legado, mejora o donacion. 14
As aforesaid, there is no other provision in the will before us except the institution of
petitioner as universal heir. That institution, by itself, is null and void. And, intestate
succession ensues.
4. Petitioner's mainstay is that the present is "a case of ineffective disinheritance rather
than one of preterition". 15From this, petitioner draws the conclusion that Article 854 "does
not apply to the case at bar". This argument fails to appreciate the distinction between
pretention and disinheritance.

110 | S U C C E S S I O N

Preterition "consists in the omission in the testator's will of the forced heirs or anyone of
them, either because they are not mentioned therein, or, though mentioned, they are
neither instituted as heirs nor are expressly disinherited." 16 Disinheritance, in turn, "is
a testamentary disposition depriving any compulsory heir of his share in the legitime for a
cause authorized by law. " 17 In Manresa's own words: "La privacion expresa de la legitima
constituye
la desheredacion. La
privacion
tacita
de
la
misma
se
denomina pretericion." 18 Sanchez Roman emphasizes the distinction by stating that
disinheritance "es siempre voluntaria"; preterition, upon the other hand, is presumed to be
"involuntaria". 19 Express as disinheritance should be, the same must be supported by a
legal cause specified in the will itself. 20
The will here does not explicitly disinherit the testatrix's parents, the forced heirs. It simply
omits their names altogether. Said will rather than be labeled ineffective disinheritance is
clearly one in which the said forced heirs suffer from preterition.
On top of this is the fact that the effects flowing from preterition are totally different from
those of disinheritance. Preterition under Article 854 of the Civil Code, we repeat, "shall
annul the institution of heir". This annulment is in toto, unless in the will there are, in
addition, testamentary dispositions in the form of devises or legacies. In ineffective
disinheritance under Article 918 of the same Code, such disinheritance shall also "annul
the institution of heirs", put only "insofar as it may prejudice the person disinherited", which
last phrase was omitted in the case of preterition. 21 Better stated yet, in disinheritance the
nullity is limited to that portion of the estate of which the disinherited heirs have been
illegally deprived. Manresa's expressive language, in commenting on the rights of the
preterited heirs in the case of preterition on the one hand and legal disinheritance on the
other, runs thus: "Preteridos, adquiren el derecho a todo; desheredados, solo les
corresponde un tercio o dos tercios, 22 el caso.23
5. Petitioner insists that the compulsory heirs ineffectively disinherited are entitled to
receive their legitimes, but that the institution of heir "is not invalidated," although the
inheritance of the heir so instituted is reduced to the extent of said legitimes. 24
This is best answered by a reference to the opinion of Mr. Chief Justice Moran in
the Neri case heretofore cited,viz:
But the theory is advanced that the bequest made by universal title in favor of the
children by the second marriage should be treated as legado and mejora and,
accordingly, it must not be entirely annulled but merely reduced. This theory, if
adopted, will result in a complete abrogation of Articles 814 and 851 of the Civil
Code. If every case of institution of heirs may be made to fall into the concept of
legacies and betterments reducing the bequest accordingly, then the provisions
of Articles 814 and 851 regarding total or partial nullity of the institution, would. be
absolutely meaningless and will never have any application at all. And the

remaining provisions contained in said article concerning the reduction of


inofficious legacies or betterments would be a surplusage because they would be
absorbed by Article 817. Thus, instead of construing, we would be destroying
integral provisions of the Civil Code.

Leodegaria Julian, a native of Sta. Maria, Ilocos Sur, died on February 12, 1973 in Davao
City at the age of sixty-seven. She was survived by her husband, Felix Balanay, Sr., and by
their six legitimate children named Felix Balanay, Jr., Avelina B. Antonio, Beatriz B.
Solamo, Carolina B. Manguiob, Delia B. Lanaban and Emilia B. Pabaonon.

The destructive effect of the theory thus advanced is due mainly to a failure to
distinguish institution of heirs from legacies and betterments, and a general from
a special provision. With reference to article 814, which is the only provision
material to the disposition of this case, it must be observed that the institution of
heirs is therein dealt with as a thing separate and distinct from legacies or
betterments. And they are separate and distinct not only because they are
distinctly and separately treated in said article but because they are in
themselves different. Institution of heirs is a bequest by universal title of property
that is undetermined. Legacy refers to specific property bequeathed by a
particular or special title. ... But again an institution of heirs cannot be taken as a
legacy. 25

Felix J. Balanay, Jr. filed in the lower court a petition dated February 27, 1973 for the
probate of his mother's notarial will dated September 5, 1970 which is written in English. In
that will Leodegaria Julian declared (a) that she was the owner of the "southern half of nine
conjugal lots (par. II); (b) that she was the absolute owner of two parcels of land which she
inherited from her father (par. III), and (c) that it was her desire that her properties should
not be divided among her heirs during her husband's lifetime and that their legitimes
should be satisfied out of the fruits of her properties (Par. IV).

The disputed order, we observe, declares the will in question "a complete nullity". Article
854 of the Civil Code in turn merely nullifies "the institution of heir". Considering, however,
that the will before us solely provides for the institution of petitioner as universal heir, and
nothing more, the result is the same. The entire will is null.
Upon the view we take of this case, the order of November 8, 1963 under review is hereby
affirmed. No costs allowed. So ordered.

G.R. No. L-39247 June 27, 1975


In the Matter of the Petition to Approve the Will of Leodegaria Julian. FELIX
BALANAY,
JR., petitioner,
vs.
HON. ANTONIO M. MARTINEZ, Judge of the Court of First Instance of
Davao,
Branch
VI;
AVELINA
B.
ANTONIO
and
DELIA
B.
LANABAN, respondents.

Then, in paragraph V of the will she stated that after her husband's death (he was eightytwo years old in 1973) her paraphernal lands and all the conjugal lands (which she
described as "my properties") should be divided and distributed in the manner set forth in
that part of her will. She devised and partitioned the conjugal lands as if they were all
owned by her. She disposed of in the will her husband's one half share of the conjugal
assets. *
Felix Balanay, Sr. and Avelina B. Antonio opposed the probate of the will on the grounds of
lack of testamentary capacity, undue influence, preterition of the husband and alleged
improper partition of the conjugal estate. The oppositors claimed that Felix Balanay, Jr.
should collate certain properties which he had received from the testatrix.
Felix Balanay, Jr., in his reply to the opposition, attached thereto an affidavit of Felix
Balanay, Sr. dated April 18, 1973 wherein he withdrew his opposition to the probate of the
will and affirmed that he was interested in its probate. On the same date Felix Balanay, Sr.
signed an instrument captioned "Conformation (sic) of Division and Renunciation of
Hereditary Rights" wherein he manifested that out of respect for his wife's will he "waived
and renounced' his hereditary rights in her estate in favor of their six children. In that same
instrument he confirmed the agreement, which he and his wife had perfected before her
death, that their conjugal properties would be partitioned in the manner indicated in her
will.

AQUINO, J.:
Felix Balanay, Jr. appealed by certiorari from the order of the Court of First Instance of
Davao dated February 28, 1974, declaring illegal and void the will of his mother,
Leodegaria Julian, converting the testate proceeding into an intestate proceeding and
ordering the issuance of the corresponding notice to creditors (Special Case No. 1808).
The antecedents of the appeal are as follows:

Avelina B. Antonio, an oppositor, in her rejoinder contended that the affidavit and
"conformation" of Felix Balanay, Sr. were void. The lower court in its order of June 18, 1973
"denied" the opposition and reset for hearing the probate of the will. It gave effect to the
affidavit and conformity of Felix Balanay, Sr. In an order dated August 28, 1973 it appointed
its branch clerk of court as special administrator of the decedent's estate.
Mrs. Antonio moved for the reconsideration of the lower court's order of June 18, 1973 on
the grounds (a) that the testatrix illegally claimed that she was the owner of the southern

111 | S U C C E S S I O N

half of the conjugal lots and (b) that she could not partition the conjugal estate by allocating
portions of the nine lots to her children. Felix Balanay, Jr., through his counsel,
Hermenegildo Cabreros, opposed that motion. The lower court denied it in its order of
October 15, 1973.

Avelina B. Antonio and Delia B. Lanaban opposed the motion for reconsideration. The
lower court denied the motion in its order of June 29, 1974. It clarified that it declared the
will void on the basis of its own independent assessment of its provisions and not because
of Atty. Montaa's arguments.

In the meanwhile, another lawyer appeared in the case. David O. Montaa, Sr., claiming to
be the lawyer of petitioner Felix Balanay, Jr. (his counsel of record was Atty. Cabreros),
filed a motion dated September 25, 1973 for "leave of court to withdraw probate of alleged
will of Leodegaria Julian and requesting authority to proceed by intestate estate
proceeding." In that motion Montaa claimed to be the lawyer not only of the petitioner but
also of Felix Balanay, Sr., Beatriz B. Solamo, Carolina B. Manguiob and Emilia B.
Pabaonon.

The basic issue is whether the probate court erred in passing upon the intrinsic validity of
the will, before ruling on its allowance or formal validity, and in declaring it void.

Montaa in his motion assailed the provision of the will which partitioned the conjugal
assets or allegedly effected a compromise of future legitimes. He prayed that the probate
of the will be withdrawn and that the proceeding be converted into an intestate proceeding.
In another motion of the same date he asked that the corresponding notice to creditors be
issued.
Avelina B. Antonio and Delia B. Lanaban, through Atty. Jose B. Guyo, in their comments
dated October 15, 1973 manifested their conformity with the motion for the issuance of a
notice to creditors. They prayed that the will be declared void for being contrary to law and
that an intestacy be declared.
The lower court, acting on the motions of Atty. Montaa, assumed that the issuance of a
notice to creditors was in order since the parties had agreed on that point. It adopted the
view of Attys. Montaa and Guyo that the will was void. So, in its order of February 28,
1974 it dismissed the petition for the probate, converted the testate proceeding into an
intestate proceeding, ordered the issuance of a notice to creditors and set the intestate
proceeding for hearing on April 1 and 2, 1974. The lower court did not abrogate its prior
orders of June 18 and October 15, 1973. The notice to creditors was issued on April 1,
1974 and published on May 2, 9 and 16 in the Davao Star in spite of petitioner's motion of
April 17, 1974 that its publication be held in abeyance.
Felix Balanay, Jr., through a new counsel, Roberto M. Sarenas, in a verified motion dated
April 15, 1974, asked for the reconsideration of the lower court's order of February 28,
1974 on the ground that Atty. Montaa had no authority to withdraw the petition for the
allowance of the will. Attached to the motion was a copy of a letter dated March 27, 1974
addressed to Atty. Montaa and signed by Felix Balanay, Jr., Beatriz V. Solamo, Carolina
B. Manguiob and Emilia B. Pabaonon, wherein they terminated Montaa's services and
informed him that his withdrawal of the petition for the probate of the will was without their
consent and was contrary to their repeated reminder to him that their mother's will was
"very sacred" to them.

112 | S U C C E S S I O N

We are of the opinion that in view of certain unusual provisions of the will, which are of
dubious legality, and because of the motion to withdraw the petition for probate (which the
lower court assumed to have been filed with the petitioner's authorization), the trial court
acted correctly in passing upon the will's intrinsic validity even before its formal validity had
been established. The probate of a will might become an idle ceremony if on its face it
appears to be intrinsically void. Where practical considerations demand that the intrinsic
validity of the will be passed upon, even before it is probated, the court should meet the
issue (Nuguid vs. Nuguid, 64 O.G. 1527, 17 SCRA 449. Compare with Sumilang vs.
Ramagosa, L-23135, December 26, 1967, 21 SCRA 1369; Cacho vs. Udan, L-19996, April
30, 1965, 13 SCRA 693).1wph1.t
But the probate court erred in declaring, in its order of February 28, 1974 that the will was
void and in converting the testate proceeding into an intestate proceeding notwithstanding
the fact that in its order of June 18, 1973 , it gave effect to the surviving husband's
conformity to the will and to his renunciation of his hereditary rights which presumably
included his one-half share of the conjugal estate.
The rule is that "the invalidity of one of several dispositions contained in a will does not
result in the invalidity of the other dispositions, unless it is to be presumed that the testator
would not have made such other dispositions if the first invalid disposition had not been
made" (Art. 792, Civil Code). "Where some of the provisions of a will are valid and others
invalid, the valid parts will be upheld if they can be separated from the invalid without
defeating the intention of the testator or interfering with the general testamentary scheme,
or doing injustice to the beneficiaries" (95 C.J.S. 873).
The statement of the testatrix that she owned the "southern half of the conjugal lands is
contrary to law because, although she was a coowner thereof, her share was inchoate
and proindiviso (Art. 143, Civil Code; Madrigal and Paterno vs. Rafferty and Concepcion,
38 Phil. 414). But That illegal declaration does not nullify the entire will. It may be
disregarded.
The provision of the will that the properties of the testatrix should not be divided among her
heirs during her husband's lifetime but should be kept intact and that the legitimes should
be paid in cash is contrary to article 1080 of the Civil Code which reads:

ART. 1080. Should a person make a partition of his estate by an


act inter vivos, or by will, such partition shall be respected, insofar as it
does not prejudice the legitime of the compulsory heirs.
A parent who, in the interest of his or her family, to keep any
agricultural, industrial, or manufacturing enterprise intact, may avail
himself of the right granted him in this article, by ordering that the
legitime of the other children to whom the property is not assigned be
paid in cash. (1056a)
The testatrix in her will made a partition of the entire conjugal estate among her six
children (her husband had renounced his hereditary rights and his one-half conjugal
share). She did not assign the whole estate to one or more children as envisaged in article
1080. Hence, she had no right to require that the legitimes be paid in cash. On the other
hand, her estate may remain undivided only for a period of twenty years. So, the provision
that the estate should not be divided during her husband's lifetime would at most be
effective only for twenty years from the date of her death unless there are compelling
reasons for terminating the coownership (Art. 1083, Civil Code).
Felix Balanay, Sr. could validly renounce his hereditary rights and his one-half share of the
conjugal partnership (Arts. 179[1] and 1041, Civil Code) but insofar as said renunciation
partakes of a donation of his hereditary rights and his one-half share in the conjugal estate
(Art. 1060[1] Civil Code), it should be subject to the limitations prescribed in articles 750
and 752 of the Civil Code. A portion of the estate should be adjudicated to the widower for
his support and maintenance. Or at least his legitime should be respected.
Subject to the foregoing observations and the rules on collation, the will is intrinsically valid
and the partition therein may be given effect if it does not prejudice the creditors and impair
the legitimes. The distribution and partition would become effective upon the death of Felix
Balanay, Sr. In the meantime, the net income should be equitably divided among the
children and the surviving spouse.
It should be stressed that by reason of the surviving husband's conformity to his wife's will
and his renunciation of his hereditary rights, his one-half conjugal share became a part of
his deceased wife's estate. His conformity had the effect of validating the partition made in
paragraph V of the will without prejudice, of course, to the rights of the creditors and the
legitimes of the compulsory heirs.

113 | S U C C E S S I O N

Article 793 of the Civil Code provides that "property acquired after the making of a will shall
only pass thereby, as if the testator had it at the time of making the will, should it expressly
appear by the will that such was his intention". Under article 930 of the Civil Code "the
legacy or devise of a thing belonging to another person is void, if the testator erroneously
believed that the thing pertained to him. But if the thing bequeathed, though not belonging
to the testator when he made the will, afterwards becomes his, by whatever title, the
disposition shall take effect."
In the instant case there is no doubt that the testatrix and her husband intended to partition
the conjugal estate in the manner set forth in paragraph V of her will. It is true that she
could dispose of by will only her half of the conjugal estate (Art. 170, Civil Code) but since
the husband, after the dissolution of the conjugal partnership, had assented to her
testamentary partition of the conjugal estate, such partition has become valid, assuming
that the will may be probated.
The instant case is different from the Nuguid case, supra, where the testatrix instituted as
heir her sister and preterited her parents. Her will was intrinsically void because it
preterited her compulsory heirs in the direct line. Article 854 of the Civil Code provides that
"the preterition or omission of one, some, or all of the compulsory heirs in the direct line,
whether living at the time of the execution of the will or born after the death of the testator,
shall annul the institution of heir; but the devises and legacies, shall be valid insofar as
they are not inofficious." Since the preterition of the parents annulled the institution of the
sister of the testatrix and there were no legacies and devises, total intestacy resulted (.Art.
960[2], Civil Code).1wph1.t
In the instant case, the preterited heir was the surviving spouse. His preterition did not
produce intestacy. Moreover, he signified his conformity to his wife's will and renounced his
hereditary rights. .
It results that the lower court erred in not proceeding with the probate of the will as
contemplated in its uncancelled order of June 18, 1973. Save in an extreme case where
the will on its face is intrinsically void, it is the probate court's duty to pass first upon the
formal validity of the will. Generally, the probate of the will is mandatory (Art. 838, Civil
Code; Guevara vs. Guevara, 74 Phil. 479 and 98 Phil. 249; Fernandez vs. Dimagiba, L23638, October 12, 1967, 21 SCRA 428).
As aptly stated by Mr. Justice Barredo, "the very existence of a purported testament is in
itself prima facie proof that the supposed testator has willed that his estate should be
distributed in the manner therein provided, and it is incumbent upon the state that, if legally
tenable, such desire be given effect independent of the attitude of the parties affected
thereby" (Resolution, Vda. de Precilla vs. Narciso, L-27200, August 18, 1972, 46 SCRA
538, 565).

To give effect to the intention and wishes of the testatrix is the first and principal law in the
matter of testaments (Dizon-Rivera vs. Dizon, L-24561, June 30, 1970, 33 SCRA 554,
561). Testacy is preferable to intestacy. An interpretation that will render a testamentary
disposition operative takes precedence over a construction that will nullify a provision of
the will (Arts. 788 and 791, Civil Code).

WHEREFORE, the lower court's orders of February 28, and June 29, 1974 are set aside
and its order of June 18, 1973, setting for hearing the petition for probate, is affirmed. The
lower court is directed to conduct further proceedings in Special Case No. 1808 in
consonance with this opinion. Costs, against the private respondents.
SO ORDERED.

Testacy is favored. Doubts are resolved in favor of testacy especially where the will
evinces an intention on the part of the testator to dispose of practically his whole estate. So
compelling is the principle that intestacy should be avoided and that the wishes of the
testator should prevail that sometimes the language of the will can be varied for the
purpose of giving it effect (Austria vs. Reyes, L-23079, February 27, 1970, 31 SCRA 754,
762).
As far as is legally possible, the expressed desire of the testator must be followed and the
dispositions of the properties in his will should be upheld (Estorque vs. Estorque, L-19573,
June 30, 1970, 33 SCRA 540, 546).

G.R. No. L-62952 October 9, 1985


SOFIA J. NEPOMUCENO, petitioner,
vs.
THE HONORABLE COURT OF APPEALS, RUFINA GOMEZ, OSCAR JUGO
ANG, CARMELITA JUGO,respondents.

The law has a tender regard for the wishes of the testator as expressed in his will because
any disposition therein is better than that which the law can make (Castro vs. Bustos, L25913, February 28, 1969, 27 SCRA 327, 341).

This is a petition for certiorari to set aside that portion of the decision of the respondent
Court of Appeals (now intermediate Appellate Court) dated June 3, 1982, as amended by
the resolution dated August 10, 1982, declaring as null and void the devise in favor of the
petitioner and the resolution dated December 28, 1982 denying petitioner's motion for
reconsideration.

Two other errors of the lower court may be noticed. It erred in issuing a notice to creditors
although no executor or regular administrator has been appointed. The record reveals that
it appointed a special administrator. A notice to creditors is not in order if only a special
administrator has been appointed. Section 1, Rule 86 of the Rules of Court, in providing
that "immediately after granting letters of testamentary or of administration, the court shall
issue a notice requiring all persons having money claims against the decedent to file them
in the office of the clerk of said court" clearly contemplates the appointment of an executor
or regular administrator and not that of a special administrator.

Martin Jugo died on July 16, 1974 in Malabon, Rizal. He left a last Will and Testament duly
signed by him at the end of the Will on page three and on the left margin of pages 1, 2 and
4 thereof in the presence of Celestina Alejandro, Myrna C. Cortez, and Leandro Leano,
who in turn, affixed their signatures below the attestation clause and on the left margin of
pages 1, 2 and 4 of the Will in the presence of the testator and of each other and the
Notary Public. The Will was acknowledged before the Notary Public Romeo Escareal by
the testator and his three attesting witnesses.

It is the executor or regular administrator who is supposed to oppose the claims against
the estate and to pay such claims when duly allowed (See. 10, Rule 86 and sec. 1, Rule
88, Rules of Court).
We also take this occasion to point out that the probate court's appointment of its branch
clerk of court as special administrator (p. 30, Rollo) is not a salutary practice because it
might engender the suspicion that the probate Judge and his clerk of court are in cahoots
in milking the decedent's estate. Should the branch clerk of court commit any abuse or
devastavit in the course of his administration, the probate Judge might find it difficult to
hold him to a strict accountability. A court employee should devote his official time to his
official duties and should not have as a sideline the administration of a decedent's estate.

114 | S U C C E S S I O N

In the said Will, the testator named and appointed herein petitioner Sofia J. Nepomuceno
as his sole and only executor of his estate. It is clearly stated in the Will that the testator
was legally married to a certain Rufina Gomez by whom he had two legitimate children,
Oscar and Carmelita, but since 1952, he had been estranged from his lawfully wedded
wife and had been living with petitioner as husband and wife. In fact, on December 5,
1952, the testator Martin Jugo and the petitioner herein, Sofia J. Nepomuceno were
married in Victoria, Tarlac before the Justice of the Peace. The testator devised to his
forced heirs, namely, his legal wife Rufina Gomez and his children Oscar and Carmelita his
entire estate and the free portion thereof to herein petitioner. The Will reads in part:
Art. III. That I have the following legal heirs, namely: my
aforementioned legal wife, Rufina Gomez, and our son, Oscar, and
daughter Carmelita, both surnamed Jugo, whom I declare and admit to

be legally and properly entitled to inherit from me; that while I have
been estranged from my above-named wife for so many years, I cannot
deny that I was legally married to her or that we have been separated
up to the present for reasons and justifications known fully well by
them:

WHEREFORE, the decision a quo is hereby


question declared valid except the devise in
which is declared null and void. The properties
passed on in intestacy to the appellant in
pronouncement as to cost.

set aside, the will in


favor of the appellant
so devised are instead
equal shares, without

Art. IV. That since 1952, 1 have been living, as man and wife with one
Sofia J. Nepomuceno, whom I declare and avow to be entitled to my
love and affection, for all the things which she has done for me, now
and in the past; that while Sofia J. Nepomuceno has with my full
knowledge and consent, did comport and represent myself as her own
husband, in truth and in fact, as well as in the eyes of the law, I could
not bind her to me in the holy bonds of matrimony because of my
aforementioned previous marriage;

On June 15, 1982, oppositors Rufina Gomez and her children filed a "Motion for Correction
of Clerical Error" praying that the word "appellant" in the last sentence of the dispositive
portion of the decision be changed to "appellees" so as to read: "The properties so devised
are instead passed on intestacy to the appellees in equal shares, without pronouncement
as to costs." The motion was granted by the respondent court on August 10, 1982.

On August 21, 1974, the petitioner filed a petition for the probate of the last Will and
Testament of the deceased Martin Jugo in the Court of First Instance of Rizal, Branch
XXXIV, Caloocan City and asked for the issuance to her of letters testamentary.

The main issue raised by the petitioner is whether or not the respondent court acted in
excess of its jurisdiction when after declaring the last Will and Testament of the deceased
Martin Jugo validly drawn, it went on to pass upon the intrinsic validity of the testamentary
provision in favor of herein petitioner.

On May 13, 1975, the legal wife of the testator, Rufina Gomez and her children filed an
opposition alleging inter alia that the execution of the Will was procured by undue and
improper influence on the part of the petitioner; that at the time of the execution of the Will,
the testator was already very sick and that petitioner having admitted her living in
concubinage with the testator, she is wanting in integrity and thus, letters testamentary
should not be issued to her.
On January 6, 1976, the lower court denied the probate of the Will on the ground that as
the testator admitted in his Will to cohabiting with the petitioner from December 1952 until
his death on July 16, 1974, the Will's admission to probate will be an Idle exercise because
on the face of the Will, the invalidity of its intrinsic provisions is evident.
The petitioner appealed to the respondent-appellate court.
On June 2, 1982, the respondent court set aside the decision of the Court of First Instance
of Rizal denying the probate of the will. The respondent court declared the Will to be valid
except that the devise in favor of the petitioner is null and void pursuant to Article 739 in
relation with Article 1028 of the Civil Code of the Philippines. The dispositive portion of the
decision reads:

115 | S U C C E S S I O N

On August 23, 1982, the petitioner filed a motion for reconsideration. This was denied by
the respondent court in a resolution dated December 28, 1982.

The petitioner submits that the validity of the testamentary provision in her favor cannot be
passed upon and decided in the probate proceedings but in some other proceedings
because the only purpose of the probate of a Will is to establish conclusively as against
everyone that a Will was executed with the formalities required by law and that the testator
has the mental capacity to execute the same. The petitioner further contends that even if
the provisions of paragraph 1 of Article 739 of the Civil Code of the Philippines were
applicable, the declaration of its nullity could only be made by the proper court in a
separate action brought by the legal wife for the specific purpose of obtaining a declaration
of the nullity of the testamentary provision in the Will in favor of the person with whom the
testator was allegedly guilty of adultery or concubinage.
The respondents on the other hand contend that the fact that the last Will and Testament
itself expressly admits indubitably on its face the meretricious relationship between the
testator and the petitioner and the fact that petitioner herself initiated the presentation of
evidence on her alleged ignorance of the true civil status of the testator, which led private
respondents to present contrary evidence, merits the application of the doctrine enunciated
in Nuguid v. Felix Nuguid, et al. (17 SCRA 449) and Felix Balanay, Jr. v. Hon. Antonio
Martinez, et al.(G.R. No. L- 39247, June 27, 1975). Respondents also submit that the
admission of the testator of the illicit relationship between him and the petitioner put in
issue the legality of the devise. We agree with the respondents.

The respondent court acted within its jurisdiction when after declaring the Will to be validly
drawn, it went on to pass upon the intrinsic validity of the Will and declared the devise in
favor of the petitioner null and void.

The rule, however, is not inflexible and absolute. Given exceptional circumstances, the
probate court is not powerless to do what the situation constrains it to do and pass upon
certain provisions of the Will.

The general rule is that in probate proceedings, the court's area of inquiry is limited to an
examination and resolution of the extrinsic validity of the Will. The rule is expressed thus:

In Nuguid v. Nuguid (17 SCRA 449) cited by the trial court, the testator instituted the
petitioner as universal heir and completely preterited her surviving forced heirs. A will of
this nature, no matter how valid it may appear extrinsically, would be null and void.
Separate or latter proceedings to determine the intrinsic validity of the testamentary
provisions would be superfluous.

xxx xxx xxx


... It is elementary that a probate decree finally and definitively settles
all questions concerning capacity of the testator and the proper
execution and witnessing of his last Will and testament, irrespective of
whether
its
provisions
are
valid
and
enforceable
or
otherwise. (Fernandez v. Dimagiba, 21 SCRA 428)
The petition below being for the probate of a Will, the court's area of
inquiry is limited to the extrinsic validity thereof. The testators
testamentary capacity and the compliance with the formal requisites or
solemnities prescribed by law are the only questions presented for the
resolution of the court. Any inquiry into the intrinsic validity or efficacy of
the provisions of the will or the legality of any devise or legacy is
premature.
xxx xxx xxx
True or not, the alleged sale is no ground for the dismissal of the
petition for probate. Probate is one thing; the validity of the
testamentary provisions is another. The first decides the execution of
the document and the testamentary capacity of the testator; the second
relates to descent and distribution (Sumilang v. Ramagosa, 21 SCRA
1369)
xxx xxx xxx
To establish conclusively as against everyone, and once for all, the
facts that a will was executed with the formalities required by law and
that the testator was in a condition to make a will, is the only purpose of
the proceedings under the new code for the probate of a will. (Sec.
625). The judgment in such proceedings determines and can determine
nothing more. In them the court has no power to pass upon the validity
of any provisions made in the will. It can not decide, for example, that a
certain legacy is void and another one valid. ... (Castaneda v.
Alemany, 3 Phil. 426)

116 | S U C C E S S I O N

Even before establishing the formal validity of the will, the Court in Balanay .Jr. v.
Martinez (64 SCRA 452) passed upon the validity of its intrinsic provisions.
Invoking "practical considerations", we stated:
The basic issue is whether the probate court erred in passing upon the
intrinsic validity of the will, before ruling on its allowance or formal
validity, and in declaring it void.
We are of the opinion that in view of certain unusual provisions of the
will, which are of dubious legality, and because of the motion to
withdraw the petition for probate (which the lower court assumed to
have been filed with the petitioner's authorization) the trial court acted
correctly in passing upon the will's intrinsic validity even before its
formal validity had been established. The probate of a will might
become an Idle ceremony if on its face it appears to be intrinsically
void. Where practical considerations demand that the intrinsic validity of
the will be passed upon, even before it is probated, the court should
meet the issue (Nuguid v. Nuguid, 64 O.G. 1527, 17 SCRA 449.
Compare with Sumilang vs. Ramagosa L-23135, December 26, 1967,
21 SCRA 1369; Cacho v. Udan L-19996, April 30, 1965, 13 SCRA 693).
There appears to be no more dispute at this time over the extrinsic validity of the Will. Both
parties are agreed that the Will of Martin Jugo was executed with all the formalities
required by law and that the testator had the mental capacity to execute his Will. The
petitioner states that she completely agrees with the respondent court when in resolving
the question of whether or not the probate court correctly denied the probate of Martin
Jugo's last Will and Testament, it ruled:
This being so, the will is declared validly drawn. (Page 4, Decision,
Annex A of Petition.)

On the other hand the respondents pray for the affirmance of the Court of Appeals'
decision in toto.
The only issue, therefore, is the jurisdiction of the respondent court to declare the
testamentary provision in favor of the petitioner as null and void.
We sustain the respondent court's jurisdiction. As stated in Nuguid v. Nuguid, (supra):
We pause to reflect. If the case were to be remanded for probate of the
will, nothing will be gained. On the contrary, this litigation will be
protracted. And for aught that appears in the record, in the record, in
the event of probate or if the court rejects the will, probability exists that
the case will come up once again before us on the same issue of the
intrinsic validity or nullity of the will. Result, waste of time, effort,
expense, plus added anxiety. These are the practical considerations
that induce us to a belief that we might as well meet head-on the issue
of the validity of the provisions of the will in question. (Section 2, Rule 1,
Rules of Court. Case, et al. v. Jugo, et al., 77 Phil. 517, 522). After all,
there exists a justiciable controversy crying for solution.
We see no useful purpose that would be served if we remand the nullified provision to the
proper court in a separate action for that purpose simply because, in the probate of a will,
the court does not ordinarily look into the intrinsic validity of its provisions.
Article 739 of the Civil Code provides:

Article 1028 of the Civil Code provides:


The prohibitions mentioned in Article 739, concerning donations inter
vivos shall apply to testamentary provisions.
In Article III of the disputed Will, executed on August 15, 1968, or almost six years before
the testator's death on July 16, 1974, Martin Jugo stated that respondent Rufina Gomez
was his legal wife from whom he had been estranged "for so many years." He also
declared that respondents Carmelita Jugo and Oscar Jugo were his legitimate children. In
Article IV, he stated that he had been living as man and wife with the petitioner since 1952.
Testator Jugo declared that the petitioner was entitled to his love and affection. He stated
that Nepomuceno represented Jugo as her own husband but "in truth and in fact, as well
as in the eyes of the law, I could not bind her to me in the holy bonds of matrimony
because of my aforementioned previous marriage.
There is no question from the records about the fact of a prior existing marriage when
Martin Jugo executed his Will. There is also no dispute that the petitioner and Mr. Jugo
lived together in an ostensible marital relationship for 22 years until his death.
It is also a fact that on December 2, 1952, Martin Jugo and Sofia J. Nepomuceno
contracted a marriage before the Justice of the Peace of Victoria, Tarlac. The man was
then 51 years old while the woman was 48. Nepomuceno now contends that she acted in
good faith for 22 years in the belief that she was legally married to the testator.
The records do not sustain a finding of innocence or good faith. As argued by the private
respondents:

The following donations shall be void:


(1) Those made between persons who were guilty of adultery or
concubinage at the time of the donation;
(2) Those made between persons found guilty of the same criminal
offense, in consideration thereof;
(3) Those made to a public officer or his wife, descendants and
ascendants, by reason of his office.
In the case referred to in No. 1, the action for declaration of nullity may
be brought by the spouse of the donor or donee; and the guilt of the
donor and donee may be proved by preponderance of evidence in the
same action.

117 | S U C C E S S I O N

First. The last will and testament itself expressly admits indubitably on
its face the meretricious relationship between the testator and
petitioner, the devisee.
Second. Petitioner herself initiated the presentation of evidence on her
alleged ignorance of the true civil status of the testator, which led
private respondents to present contrary evidence.
In short, the parties themselves dueled on the intrinsic validity of the
legacy given in the will to petitioner by the deceased testator at the start
of the proceedings.
Whether or not petitioner knew that testator Martin Jugo, the man he
had lived with as man and wife, as already married, was an important

and specific issue brought by the parties before the trial court, and
passed upon by the Court of Appeals.
Instead of limiting herself to proving the extrinsic validity of the will, it
was petitioner who opted to present evidence on her alleged good faith
in marrying the testator. (Testimony of Petitioner, TSN of August 1,
1982, pp. 56-57 and pp. 62-64).
Private respondents, naturally, presented evidence that would refute
the testimony of petitioner on the point.
Sebastian Jugo, younger brother of the deceased testator, testified at
length on the meretricious relationship of his brother and petitioner.
(TSN of August 18,1975).
Clearly, the good faith of petitioner was by option of the parties made a
decisive issue right at the inception of the case.
Confronted by the situation, the trial court had to make a ruling on the
question.
When the court a quo held that the testator Martin Jugo and petitioner
'were deemed guilty of adultery or concubinage', it was a finding that
petitioner was not the innocent woman she pretended to be.
xxx xxx xxx
3. If a review of the evidence must be made nonetheless, then private
respondents respectfully offer the following analysis:
FIRST: The secrecy of the marriage of petitioner with the deceased
testator in a town in Tarlac where neither she nor the testator ever
resided. If there was nothing to hide from, why the concealment' ? Of
course, it maybe argued that the marriage of the deceased with private
respondent Rufina Gomez was likewise done in secrecy. But it should
be remembered that Rufina Gomez was already in the family way at
that time and it would seem that the parents of Martin Jugo were not in
favor of the marriage so much so that an action in court was brought
concerning the marriage. (Testimony of Sebastian Jugo, TSN of August
18, 1975, pp. 29-30)

118 | S U C C E S S I O N

SECOND: Petitioner was a sweetheart of the deceased testator when


they were still both single. That would be in 1922 as Martin Jugo
married respondent Rufina Gomez on November 29, 1923 (Exh. 3).
Petitioner married the testator only on December 5, 1952. There was a
space of about 30 years in between. During those 30 years, could it be
believed that she did not even wonder why Martin Jugo did not marry
her nor contact her anymore after November, 1923 - facts that should
impel her to ask her groom before she married him in secrecy,
especially so when she was already about 50 years old at the time of
marriage.
THIRD: The fact that petitioner broke off from Martin Jugo in 1923 is by
itself conclusive demonstration that she new that the man she had
openly lived for 22 years as man and wife was a married man with
already two children.
FOURTH: Having admitted that she knew the children of respondent
Rufina Gomez, is it possible that she would not have asked Martin Jugo
whether or not they were his illegitimate or legitimate children and by
whom? That is un-Filipino.
FIFTH: Having often gone to Pasig to the residence of the parents of
the deceased testator, is it possible that she would not have known that
the mother of private respondent Oscar Jugo and Carmelita Jugo was
respondent Rufina Gomez, considering that the houses of the parents
of Martin Jugo (where he had lived for many years) and that of
respondent Rufina Gomez were just a few meters away?
Such pretentions of petitioner Sofia Nepomuceno are unbelievable.
They are, to say the least, inherently improbable, for they are against
the experience in common life and the ordinary instincts and
promptings of human nature that a woman would not bother at all to
ask the man she was going to marry whether or not he was already
married to another, knowing that her groom had children. It would be a
story that would strain human credulity to the limit if petitioner did not
know that Martin Jugo was already a married man in view of the
irrefutable fact that it was precisely his marriage to respondent Rufina
Gomez that led petitioner to break off with the deceased during their
younger years.
Moreover, the prohibition in Article 739 of the Civil Code is against the making of a
donation between persons who are living in adultery or concubinage. It is
the donation which becomes void. The giver cannot give even assuming that the recipient

may receive. The very wordings of the Will invalidate the legacy because the testator
admitted he was disposing the properties to a person with whom he had been living in
concubinage.
WHEREFORE, the petition is DISMISSED for lack of merit. The decision of the Court of
Appeals, now Intermediate Appellate Court, is AFFIRMED. No costs.
SO ORDERED.

[G.R. No. 16763. December 22, 1921. ]


PASCUAL COSO, Petitioner-Appellant, v. FERMINA FERNANDEZ DEZA ET
AL., objectors-appellees.
SYLLABUS
1. WILLS; UNDUE INFLUENCE. In the absence of fraud or imposition, mere affection,
even if illegitimate, is not undue influence and does not invalidate a will.
OSTRAND, J. :

This is an appeal from a decision of the Court of First Instance of Manila setting aside a will
on the ground of undue influence alleged to have been exerted over the mind of a testator
by one Rosario Lopez. The will gives the tercio de libre disposicion to an illegitimate son
had by the testator with said Rosario Lopez, and also provides for the payment to her of
nineteen hundred Spanish duros by way of reimbursement for expenses incurred by her in
taking care of the testator in Barcelona during the years 1909 to 1916, when he is alleged
to have suffered from severe illness.
The evidence shows that the testator, a married man and resident of the Philippine Islands,
became acquainted with Rosario Lopez in Spain in 1898 and that he had illicit relations
with her for many years thereafter. After his return to the Philippines she followed him,
arriving in Manila in February, 1918, and remained in close communication with him until
his death in February, 1919. There is no doubt that she exercised some influence over him
and the only question for our determination is whether this influence was of such a
character as to vitiate the will
The English and American rule in regard to undue influence is thus stated in 40 Cyc., 11441149.
"Mere general or reasonable influence over a testator is not sufficient to invalidate a will; to
have that effect the influence must be undue. The rule as to what constitutes undue
influence has been variously stated, but the substance of the different statements is that,
to be sufficient to avoid a will, the influence exerted must be of a kind that so overpowers

119 | S U C C E S S I O N

and subjugates the mind of the testator as to destroy his free agency and make him
express the will of another, rather than his own.
". . . such influence must be actually exerted on the mind of the testator in regard to the
execution of the will in question, either at the time of the execution of the will,, or so near
thereto as to be still operative, with the object of procuring a will in favor of particular
parties, and it must result in the making of testamentary dispositions which the testator
would not otherwise have made . . .
". . . and while the same amount of influence may become undue when exercised by one
occupying an improper and adulterous relation to testator, the mere fact that some
influence is exercised by a person sustaining that relation does not invalidate a will, unless
it is further shown that the influence destroys the testators free agency."cralaw virtua1aw
library
The burden is upon the parties challenging the will to show that undue influence, in the
sense above expressed, existed at the time of its execution and we do not think that this
burden has been carried in the present case. While it is shown that the testator entertained
strong affections for Rosario Lopez, it does not appear that her influence so overpowered
and subjugated his mind as to "destroy his free agency and make him express the will of
another rather than his own." He was an intelligent man, a lawyer by profession, appears
to have known his own mind, and may well have been actuated only by a legitimate sense
of duty in making provisions for the welfare of his illegitimate son and by a proper feeling of
gratitude in repaying Rosario Lopez for the sacrifices she had made for him. Mere
affection, even if illegitimate, is not undue influence and does not invalidate a will. No
imposition or fraud has been shown in the present case.
"Influence gained by kindness and affection will not be regarded as undue, if no
imposition or fraud be practiced, even though it induces the testator to make an unequal
and unjust disposition of his property in favor of those who have contributed to his comfort
and ministered to his wants, if such disposition is voluntarily made." (Mackall v. Mackall,
135 U. S., 167.)
It may be further observed that under the Civil Law the right of a person with legal heirs to
dispose of his property by will is limited to only a portion of his estate, and that under the
law in force in these Islands before the enactment of the Code of Civil Procedure, the only
outside influences affecting the validity of a will were duress, deceit, and fraud. The
present doctrine of undue influence originated in a legal system where the right of the
testator to dispose of his property by will was nearly unlimited. Manifestly. greater
safeguards in. regard to execution of wills may be warranted when the right to so dispose
of property is unlimited than when it is restricted to the extent it is in this jurisdiction There
is, therefore, certainly no reason for giving the doctrine of undue influence a wider scope
here than it enjoys in the United States.
For the reasons stated, the decision of the lower court disallowing the will of Federico
Gimenez Zoboli is hereby reversed and it is ordered that the will be admitted to probate.
No costs will be allowed. So ordered.
Johnson, Street, Malcolm, Avancea, Villamor, Johns and Romualdez, JJ., concur.

You might also like